2nd Plastic Surgery Case Review - Oral Board Study Guide, 2nd Edition-Thieme (2020)

You might also like

Download as pdf or txt
Download as pdf or txt
You are on page 1of 298

TPS 23 x 31 - 2 | 31.10.

20 - 14:00
TPS 23 x 31 - 2 | 31.10.20 - 14:00
TPS 23 x 31 - 2 | 31.10.20 - 14:00

Plastic Surgery Case Review

Oral Board Study Guide

Second Edition

Albert S. Woo, MD, FACS


Associate Professor of Surgery, Pediatrics and Neurosurgery
Chief, Pediatric Plastic Surgery
Director, Cleft and Craniofacial Center
Division of Plastic and Reconstructive Surgery
The Warren Alpert Medical School of Brown University
Providence, Rhode Island, USA

Reena A. Bhatt, MD
Clinical Assistant Professor of Surgery
Division of Plastic and Reconstructive Surgery
The Warren Alpert Medical School of Brown University
Providence, Rhode Island, USA

223 Illustrations

Thieme
New York • Stuttgart • Delhi • Rio de Janeiro
TPS 23 x 31 - 2 | 31.10.20 - 14:00

Library of Congress Cataloging-in-Publication Data Important note: Medicine is an ever-changing science undergo-
is available from the publisher. ing continual development. Research and clinical experience are
continually expanding our knowledge, in particular our knowl-
edge of proper treatment and drug therapy. Insofar as this book
mentions any dosage or application, readers may rest assured that
the authors, editors, and publishers have made every effort to
ensure that such references are in accordance with the state of
knowledge at the time of production of the book.
Nevertheless, this does not involve, imply, or express any
guarantee or responsibility on the part of the publishers in respect
to any dosage instructions and forms of applications stated in the
book. Every user is requested to examine carefully the manufac-
turers’ leaflets accompanying each drug and to check, if necessary
in consultation with a physician or specialist, whether the dosage
schedules mentioned therein or the contraindications stated by the
manufacturers differ from the statements made in the present
book. Such examination is particularly important with drugs that
are either rarely used or have been newly released on the market.
Every dosage schedule or every form of application used is entirely
at the user’s own risk and responsibility. The authors and publishers
request every user to report to the publishers any discrepancies or
inaccuracies noticed. If errors in this work are found after publi-
cation, errata will be posted at www.thieme.com on the product
description page.
Some of the product names, patents, and registered designs
referred to in this book are in fact registered trademarks or pro-
prietary names even though specific reference to this fact is not
always made in the text. Therefore, the appearance of a name
without designation as proprietary is not to be construed as a
representation by the publisher that it is in the public domain.

© 2021. Thieme. All rights reserved.

Thieme Medical Publishers, Inc.


333 Seventh Avenue, 18th Floor
New York, NY 10001, USA
www.thieme.com
+1 800 782 3488, customerservice@thieme.com

Cover design: Thieme Publishing Group


Typesetting by DiTech Process Solutions, India
This book, including all parts thereof, is legally protected by copy-
Printed in USA by King Printing Company, Inc. 54321 right. Any use, exploitation, or commercialization outside the
narrow limits set by copyright legislation without the publisher’s
ISBN 978-1-68420-062-7 consent is illegal and liable to prosecution. This applies in particular
to photostat reproduction, copying, mimeographing or duplication
Also available as an e-book: of any kind, translating, preparation of microfilms, and electronic
eISBN 978-1-68420-063-4 data processing and storage.
TPS 23 x 31 - 2 | 31.10.20 - 14:00

To my children, Nathan and Emi, who provide me with unending lessons on love, joy, patience, and understanding.

Albert S. Woo, MD, FACS


TPS 23 x 31 - 2 | 31.10.20 - 14:00
TPS 23 x 31 - 2 | 31.10.20 - 14:00

Contents

Preface . . . . . . . . . . . . . . . . . . . . . . . . . . . . . . . . . . . . . . . . . . . . . . . . . . . . . . . . . . . . . . . . . . . . . . . . . . . . . . . . . . . . . . . . . . . . . . . . . . . . . xiii
Contributors . . . . . . . . . . . . . . . . . . . . . . . . . . . . . . . . . . . . . . . . . . . . . . . . . . . . . . . . . . . . . . . . . . . . . . . . . . . . . . . . . . . . . . . . . . . . . . . xiv
How To Use This Book: Tips from a Recent Board Examinee . . . . . . . . . . . . . . . . . . . . . . . . . . . . . . . . . . . . . . . . . xvii
Practical Tips for the Oral Board Collection and Oral Board Preparation. . . . . . . . . . . . . . . . . . . . . . . . . . . . . xix

Section I: Facial Fractures

1. Nasal Fractures . . . . . . . . . . . . . . . . . . . . . . . . . . . . . . . . . . . . . . . . . . . . . . . . . . . . . . . . . . . . . . . . . . . . . . . . . . . . . . . . . . . . . . . . 3
Charles C. Jehle and Albert S. Woo

2. Zygomatic Fractures . . . . . . . . . . . . . . . . . . . . . . . . . . . . . . . . . . . . . . . . . . . . . . . . . . . . . . . . . . . . . . . . . . . . . . . . . . . . . . . . . . 5
Vinay Rao and Albert S. Woo

3. Mandibular Fractures . . . . . . . . . . . . . . . . . . . . . . . . . . . . . . . . . . . . . . . . . . . . . . . . . . . . . . . . . . . . . . . . . . . . . . . . . . . . . . . . . 9
Vinay Rao and Albert S. Woo

4. Frontal Sinus Fractures . . . . . . . . . . . . . . . . . . . . . . . . . . . . . . . . . . . . . . . . . . . . . . . . . . . . . . . . . . . . . . . . . . . . . . . . . . . . . . 13


Dardan Beqiri, Lauren O. Roussel, and Albert S. Woo

5. Le Fort Fractures . . . . . . . . . . . . . . . . . . . . . . . . . . . . . . . . . . . . . . . . . . . . . . . . . . . . . . . . . . . . . . . . . . . . . . . . . . . . . . . . . . . . . 17
Sara A. Neimanis and Clinton S. Morrison

6. Pediatric Mandible Fractures . . . . . . . . . . . . . . . . . . . . . . . . . . . . . . . . . . . . . . . . . . . . . . . . . . . . . . . . . . . . . . . . . . . . . . . . 21


Lauren O. Roussel and Albert S. Woo

Section II: Facial Reconstruction

7. Malignant Skin Lesion . . . . . . . . . . . . . . . . . . . . . . . . . . . . . . . . . . . . . . . . . . . . . . . . . . . . . . . . . . . . . . . . . . . . . . . . . . . . . . . 27


W. Kelsey Snapp, Albert S. Woo, and Antonio Cruz

8. Lip Reconstruction . . . . . . . . . . . . . . . . . . . . . . . . . . . . . . . . . . . . . . . . . . . . . . . . . . . . . . . . . . . . . . . . . . . . . . . . . . . . . . . . . . . 31
Dardan Beqiri and Albert S. Woo

9. Nose Reconstruction . . . . . . . . . . . . . . . . . . . . . . . . . . . . . . . . . . . . . . . . . . . . . . . . . . . . . . . . . . . . . . . . . . . . . . . . . . . . . . . . . 35
Albert S. Woo

10. Eyelid Reconstruction . . . . . . . . . . . . . . . . . . . . . . . . . . . . . . . . . . . . . . . . . . . . . . . . . . . . . . . . . . . . . . . . . . . . . . . . . . . . . . . . 39


Raman Mehrzad, Antonio Cruz, and Daniel Kwan

vii
TPS 23 x 31 - 2 | 31.10.20 - 14:00

Contents

11. Ear Reconstruction . . . . . . . . . . . . . . . . . . . . . . . . . . . . . . . . . . . . . . . . . . . . . . . . . . . . . . . . . . . . . . . . . . . . . . . . . . . . . . . . . . . 43


Dardan Beqiri and Albert S. Woo

12. Cheek Reconstruction . . . . . . . . . . . . . . . . . . . . . . . . . . . . . . . . . . . . . . . . . . . . . . . . . . . . . . . . . . . . . . . . . . . . . . . . . . . . . . . 47


Albert S. Woo

13. Acquired Facial Paralysis. . . . . . . . . . . . . . . . . . . . . . . . . . . . . . . . . . . . . . . . . . . . . . . . . . . . . . . . . . . . . . . . . . . . . . . . . . . . . 51


Raman Mehrzad, Albert S. Woo, and Daniel Kwan

14. Congenital Facial Paralysis . . . . . . . . . . . . . . . . . . . . . . . . . . . . . . . . . . . . . . . . . . . . . . . . . . . . . . . . . . . . . . . . . . . . . . . . . . 55


Craig B. Birgfeld

Section III: Face, Congenital

15. Unilateral Cleft Lip Deformity . . . . . . . . . . . . . . . . . . . . . . . . . . . . . . . . . . . . . . . . . . . . . . . . . . . . . . . . . . . . . . . . . . . . . . . 61


Albert S. Woo

16. Bilateral Cleft Lip Deformity . . . . . . . . . . . . . . . . . . . . . . . . . . . . . . . . . . . . . . . . . . . . . . . . . . . . . . . . . . . . . . . . . . . . . . . . 65


Vinay Rao and Albert S. Woo

17. Cleft Palate . . . . . . . . . . . . . . . . . . . . . . . . . . . . . . . . . . . . . . . . . . . . . . . . . . . . . . . . . . . . . . . . . . . . . . . . . . . . . . . . . . . . . . . . . . . 69


Vinay Rao and Albert S. Woo

18. Pierre Robin Sequence . . . . . . . . . . . . . . . . . . . . . . . . . . . . . . . . . . . . . . . . . . . . . . . . . . . . . . . . . . . . . . . . . . . . . . . . . . . . . . . 73


Rajiv J. Iyengar, Karl Bruckman, and Derek M. Steinbacher

19. Prominent Ear Deformity. . . . . . . . . . . . . . . . . . . . . . . . . . . . . . . . . . . . . . . . . . . . . . . . . . . . . . . . . . . . . . . . . . . . . . . . . . . . 77


Lauren O. Roussel and Patrick K. Sullivan

20. Microtia . . . . . . . . . . . . . . . . . . . . . . . . . . . . . . . . . . . . . . . . . . . . . . . . . . . . . . . . . . . . . . . . . . . . . . . . . . . . . . . . . . . . . . . . . . . . . . . 81
Kristopher M. Day and Raymond J. Harshbarger

21. Giant Hairy Nevus . . . . . . . . . . . . . . . . . . . . . . . . . . . . . . . . . . . . . . . . . . . . . . . . . . . . . . . . . . . . . . . . . . . . . . . . . . . . . . . . . . . . 87


Clinton S. Morrison and Sara A. Neimanis

Section IV: Face, Cosmetic

22. Nonsurgical Rejuvenation of the Face . . . . . . . . . . . . . . . . . . . . . . . . . . . . . . . . . . . . . . . . . . . . . . . . . . . . . . . . . . . . . . 91


J. Thomas Paliga and Ivona Percec

23. Aging Face and Neck . . . . . . . . . . . . . . . . . . . . . . . . . . . . . . . . . . . . . . . . . . . . . . . . . . . . . . . . . . . . . . . . . . . . . . . . . . . . . . . . . 95


Jonathan P. Brower and Patrick K. Sullivan

viii
TPS 23 x 31 - 2 | 31.10.20 - 14:00

Contents

24. Aging Upper Face . . . . . . . . . . . . . . . . . . . . . . . . . . . . . . . . . . . . . . . . . . . . . . . . . . . . . . . . . . . . . . . . . . . . . . . . . . . . . . . . . . . . 99


Karen Leong

25. Lower Eyelid Ectropion (Cicatricial) . . . . . . . . . . . . . . . . . . . . . . . . . . . . . . . . . . . . . . . . . . . . . . . . . . . . . . . . . . . . . . 103


Jason Chow and Michael E. Migliori

26. Lower Eyelid Ectropion (Involutional and Paralytic) . . . . . . . . . . . . . . . . . . . . . . . . . . . . . . . . . . . . . . . . . . . . 107


Jason Chow and Michael E. Migliori

27. Rhinoplasty . . . . . . . . . . . . . . . . . . . . . . . . . . . . . . . . . . . . . . . . . . . . . . . . . . . . . . . . . . . . . . . . . . . . . . . . . . . . . . . . . . . . . . . . . 109


Raman Mehrzad and Albert S. Woo

28. Gender Transition (Male-to-Female). . . . . . . . . . . . . . . . . . . . . . . . . . . . . . . . . . . . . . . . . . . . . . . . . . . . . . . . . . . . . . 113


Angie M. Paik and Daniel Kwan

29. Gender Transition (Female-to-Male). . . . . . . . . . . . . . . . . . . . . . . . . . . . . . . . . . . . . . . . . . . . . . . . . . . . . . . . . . . . . . 117


Angie M. Paik and Daniel Kwan

Section V: Foot and Lower Extremity Reconstruction

30. Open Wound: Upper Third of Leg. . . . . . . . . . . . . . . . . . . . . . . . . . . . . . . . . . . . . . . . . . . . . . . . . . . . . . . . . . . . . . . . . 123


Reena A. Bhatt

31. Open Wound: Middle Third of Lower Leg . . . . . . . . . . . . . . . . . . . . . . . . . . . . . . . . . . . . . . . . . . . . . . . . . . . . . . . . 127


Marten N. Basta and Daniel Kwan

32. Open Wound: Lower Third of Lower Leg . . . . . . . . . . . . . . . . . . . . . . . . . . . . . . . . . . . . . . . . . . . . . . . . . . . . . . . . . 131


Marten N. Basta and Daniel Kwan

33. Foot and Ankle Reconstruction . . . . . . . . . . . . . . . . . . . . . . . . . . . . . . . . . . . . . . . . . . . . . . . . . . . . . . . . . . . . . . . . . . . 135


Reena A. Bhatt

Section VI: Breast

34. Breast Cancer Reconstruction . . . . . . . . . . . . . . . . . . . . . . . . . . . . . . . . . . . . . . . . . . . . . . . . . . . . . . . . . . . . . . . . . . . . . 141


Victor A. King and Glyn E. Jones

35. Breast Augmentation . . . . . . . . . . . . . . . . . . . . . . . . . . . . . . . . . . . . . . . . . . . . . . . . . . . . . . . . . . . . . . . . . . . . . . . . . . . . . . 145


Zachary Okhah and Richard Zienowicz

36. Secondary Breast Deformities (Reconstructive and Aesthetic) . . . . . . . . . . . . . . . . . . . . . . . . . . . . . . . . 149


Elizabeth Kiwanuka and Karl H. Breuing

ix
TPS 23 x 31 - 2 | 31.10.20 - 14:00

Contents

37. Tuberous Breast Deformity. . . . . . . . . . . . . . . . . . . . . . . . . . . . . . . . . . . . . . . . . . . . . . . . . . . . . . . . . . . . . . . . . . . . . . . . 153


Lauren O. Roussel and Karl H. Breuing

38. Mastopexy/Augmentation . . . . . . . . . . . . . . . . . . . . . . . . . . . . . . . . . . . . . . . . . . . . . . . . . . . . . . . . . . . . . . . . . . . . . . . . 157


Rachel R. Sullivan

39. Breast Reduction . . . . . . . . . . . . . . . . . . . . . . . . . . . . . . . . . . . . . . . . . . . . . . . . . . . . . . . . . . . . . . . . . . . . . . . . . . . . . . . . . . . 161


Victor A. King and Glyn E. Jones

40. Gynecomastia . . . . . . . . . . . . . . . . . . . . . . . . . . . . . . . . . . . . . . . . . . . . . . . . . . . . . . . . . . . . . . . . . . . . . . . . . . . . . . . . . . . . . . 165


Lauren O. Roussel and Rachel R. Sullivan

Section VII: Trunk

41. Ischial Pressure Sores . . . . . . . . . . . . . . . . . . . . . . . . . . . . . . . . . . . . . . . . . . . . . . . . . . . . . . . . . . . . . . . . . . . . . . . . . . . . . . 171


Elizabeth Kiwanuka, Albert S. Woo, and Paul Y. Liu

42. Body Contouring after Massive Weight Loss . . . . . . . . . . . . . . . . . . . . . . . . . . . . . . . . . . . . . . . . . . . . . . . . . . . . 175


Jonathan P. Brower and Rachel R. Sullivan

43. Major Liposuction . . . . . . . . . . . . . . . . . . . . . . . . . . . . . . . . . . . . . . . . . . . . . . . . . . . . . . . . . . . . . . . . . . . . . . . . . . . . . . . . . . 179


Dardan Beqiri and Rachel R. Sullivan

44. Abdominal Wall Defect . . . . . . . . . . . . . . . . . . . . . . . . . . . . . . . . . . . . . . . . . . . . . . . . . . . . . . . . . . . . . . . . . . . . . . . . . . . . 183


Marten N. Basta and Karl H. Breuing

45. Sternal Wound Infection . . . . . . . . . . . . . . . . . . . . . . . . . . . . . . . . . . . . . . . . . . . . . . . . . . . . . . . . . . . . . . . . . . . . . . . . . . 187


Marten N. Basta and Karl H. Breuing

46. Chest Wall Reconstruction . . . . . . . . . . . . . . . . . . . . . . . . . . . . . . . . . . . . . . . . . . . . . . . . . . . . . . . . . . . . . . . . . . . . . . . . 191


Marten N. Basta, Albert S. Woo, and Karl H. Breuing

47. Perineal Reconstruction . . . . . . . . . . . . . . . . . . . . . . . . . . . . . . . . . . . . . . . . . . . . . . . . . . . . . . . . . . . . . . . . . . . . . . . . . . . 195


Lauren O. Roussel and Rachel R. Sullivan

48. Abdominoplasty. . . . . . . . . . . . . . . . . . . . . . . . . . . . . . . . . . . . . . . . . . . . . . . . . . . . . . . . . . . . . . . . . . . . . . . . . . . . . . . . . . . . 199


Angie M. Paik and Karl H. Breuing

Section VIII: Burns

49. Acute Burn Injury . . . . . . . . . . . . . . . . . . . . . . . . . . . . . . . . . . . . . . . . . . . . . . . . . . . . . . . . . . . . . . . . . . . . . . . . . . . . . . . . . . 205


Charles C. Jehle and Albert S. Woo

x
TPS 23 x 31 - 2 | 31.10.20 - 14:00

Contents

50. Electrical Injuries . . . . . . . . . . . . . . . . . . . . . . . . . . . . . . . . . . . . . . . . . . . . . . . . . . . . . . . . . . . . . . . . . . . . . . . . . . . . . . . . . . . 209


Ean Saberski, David Tsai, and Adnan Prsic

51. Upper Extremity Burns . . . . . . . . . . . . . . . . . . . . . . . . . . . . . . . . . . . . . . . . . . . . . . . . . . . . . . . . . . . . . . . . . . . . . . . . . . . . 213


W. Kelsey Snapp, Albert S. Woo, and Adnan Prsic

52. Scalp Burn Reconstruction . . . . . . . . . . . . . . . . . . . . . . . . . . . . . . . . . . . . . . . . . . . . . . . . . . . . . . . . . . . . . . . . . . . . . . . . 217


Lauren O. Roussel and Albert S. Woo

53. Neck Burn Contracture . . . . . . . . . . . . . . . . . . . . . . . . . . . . . . . . . . . . . . . . . . . . . . . . . . . . . . . . . . . . . . . . . . . . . . . . . . . . 221


Sarah A. Frommer and Renata S. Maricevich

Section IX: Hand

54. Flexor Tendon Laceration . . . . . . . . . . . . . . . . . . . . . . . . . . . . . . . . . . . . . . . . . . . . . . . . . . . . . . . . . . . . . . . . . . . . . . . . . 227


Reena A. Bhatt

55. Degloving Injury . . . . . . . . . . . . . . . . . . . . . . . . . . . . . . . . . . . . . . . . . . . . . . . . . . . . . . . . . . . . . . . . . . . . . . . . . . . . . . . . . . . 231


Charles C. Jehle and Adnan Prsic

56. Traumatic Amputation . . . . . . . . . . . . . . . . . . . . . . . . . . . . . . . . . . . . . . . . . . . . . . . . . . . . . . . . . . . . . . . . . . . . . . . . . . . . 235


W. Kelsey Snapp and Adnan Prsic

57. Peripheral Nerve Injury . . . . . . . . . . . . . . . . . . . . . . . . . . . . . . . . . . . . . . . . . . . . . . . . . . . . . . . . . . . . . . . . . . . . . . . . . . . . 239


W. Kelsey Snapp and Reena A. Bhatt

58. Dupuytren's Contracture . . . . . . . . . . . . . . . . . . . . . . . . . . . . . . . . . . . . . . . . . . . . . . . . . . . . . . . . . . . . . . . . . . . . . . . . . . 243


Charles C. Jehle and Reena A. Bhatt

59. Syndactyly . . . . . . . . . . . . . . . . . . . . . . . . . . . . . . . . . . . . . . . . . . . . . . . . . . . . . . . . . . . . . . . . . . . . . . . . . . . . . . . . . . . . . . . . . . 247


Angie M. Paik and Reena A. Bhatt

60. Metacarpal and Phalangeal Fractures . . . . . . . . . . . . . . . . . . . . . . . . . . . . . . . . . . . . . . . . . . . . . . . . . . . . . . . . . . . . 251


W. Kelsey Snapp and Reena A. Bhatt

61. Carpal Tunnel Syndrome . . . . . . . . . . . . . . . . . . . . . . . . . . . . . . . . . . . . . . . . . . . . . . . . . . . . . . . . . . . . . . . . . . . . . . . . . . 255


W. Kelsey Snapp and Reena A. Bhatt

62. Adult Brachial Plexus Injury . . . . . . . . . . . . . . . . . . . . . . . . . . . . . . . . . . . . . . . . . . . . . . . . . . . . . . . . . . . . . . . . . . . . . . . 259


Timothy Fei and Kyle Chepla

xi
TPS 23 x 31 - 2 | 31.10.20 - 14:00

Contents

Section X: Ethics

63. Ethics . . . . . . . . . . . . . . . . . . . . . . . . . . . . . . . . . . . . . . . . . . . . . . . . . . . . . . . . . . . . . . . . . . . . . . . . . . . . . . . . . . . . . . . . . . . . . . . . 265


Karel-Bart Celie, Sabrina Khalil, and Michael A. Harrington

Index . . . . . . . . . . . . . . . . . . . . . . . . . . . . . . . . . . . . . . . . . . . . . . . . . . . . . . . . . . . . . . . . . . . . . . . . . . . . . . . . . . . . . . . . . . . . . . . . . . . . . . 269

xii
TPS 23 x 31 - 2 | 31.10.20 - 14:00

Preface
The field of plastic surgery is as wide as it is deep. It is a each case includes a photograph of a representative patient
specialty unique in its ability to defy definition. Contrary to and a short description, such as one might find in a board
the paradigm of other surgical disciplines, plastic surgery examination setting. Readers are encouraged to examine
has no organ system to call its own. Therefore, those in our each scenario and thoroughly explore how the patient
field will regularly operate on patients from head to toe, might be approached clinically. What critical elements of
venturing from skin to bone and sometimes delving even the patient’s history and physical examination are nec-
deeper into the vital organs of the body. Possibly the best essary? Is any further work-up needed before a surgical
means of description comes from the original Greek term plan of action is determined? What key components of
plastikos, derived in turn from plassein, meaning “to form, treatment should be identified and discussed in a test
mold, or sculpt.” Indeed, this is the moniker by which we are environment? Should complications occur, how will they
known. We are plastic surgeons, who use specialized tech- be managed? What critical mistakes should the practi-
niques and principles to remold the body, replacing what tioner be conscious to avoid? What ethical dilemmas
has been lost or reshaping what has become malformed. might present themselves? Each of these questions should
Appropriately, this concept was expressed best by our be asked and answered before the subsequent body of the
founding father, Gaspare Tagliacozzi, who in 1597 stated, text is read.
“We restore, repair, and make whole those parts … which The text was inspired by the “mock oral examination”
fortune has taken away, not so much that they may delight that is regularly conducted by myriad plastic surgery pro-
the eye, but that they may buoy up the spirit and help the grams across the country for the benefit of our trainees.
mind of the afflicted.” Although the examination is invariably a difficult under-
Given the vastness of the field and the unusual difficulty taking for the residents, trainees and faculty alike have
in defining its boundaries, the task that academic plastic always uniformly agreed that the testing is a tremendously
surgeons face in teaching the specialty to ensuing genera- worthwhile learning endeavor. We hope that this work may
tions becomes especially daunting. Despite this enormous prove in some way useful to other plastic surgeons, at all
undertaking, training programs have done an excellent job levels of experience.
in working to define a curriculum and provide an expansive Residents might use this resource as a quick review
array of surgical experiences for residents and fellows. guide, which may highlight areas of further study or point
Because it is nearly impossible to have every trainee learn out details in decision-making that are possibly not readily
every single operative procedure, the specialty has been obvious in didactic texts. Older surgeons may find the book
distilled into numerous critical areas of learning—with a useful as a review, reminding each of us of the small details
number of standard procedures and foundational principles that we may have forgotten. Our students might find this
that all plastic surgeons are expected to master. volume useful for its clinical relevance, as they begin to
The purpose of this book is to serve as an educational explore the incredible offerings and challenges presented to
resource for those pursuing a career in plastic surgery. In our specialty.
particular, it is especially geared toward preparing indivi- This work is a labor of love, as all academic endeavors
duals for their oral board examinations. The cases are there- tend to be, so it is our sincere hope that you will find this
fore designed to be relatively short and straightforward, book useful in your clinical and educational journey. As the
focusing upon the critical elements of knowledge and deci- next generation of plastic surgeons, may you mold and
sion-making. They are not necessarily intended to present sculpt the future of our specialty, build a brighter tomorrow
the latest “cutting-edge” procedures or to be exhaustive, but not just for our profession but for all in the society as a
rather to discuss safe and proven methods of patient care. whole. It is our deepest desire that you might continue this
Given the popularity of the original text, we are pleased mission of inspiring others to “buoy up the spirit and help
to present this second edition. The book has been expanded the mind of the afflicted.”
to 10 sections with over 60 new cases, each of which Albert S. Woo, MD, FACS
explores a fundamental topic of study. The first page of Reena A. Bhatt, MD

xiii
TPS 23 x 31 - 2 | 31.10.20 - 14:00

Contributors
Marten N. Basta, MD Karel-Bart Celie, MD
Resident in Plastic Surgery Resident Physician
Division of Plastic and Reconstructive Surgery Division of Plastic and Reconstructive Surgery
The Warren Alpert Medical School of Brown University; Keck School of Medicine
Resident in Plastic Surgery University of Southern California
Rhode Island Hospital Los Angeles, California, USA
Providence, Rhode Island, USA
Kyle Chepla, MD
Dardan Beqiri, MD Associate Professor of Surgery
Resident in Plastic Surgery Division of Plastic and Reconstructive Surgery
Division of Plastic and Reconstructive Surgery Metrohealth Medical Center
The Warren Alpert Medical School of Brown University Cleveland, Ohio, USA
Providence, Rhode Island, USA
Jason Chow, MD
Reena A. Bhatt, MD Resident Physician
Clinical Assistant Professor of Surgery Division of Ophthalmology
Division of Plastic and Reconstructive Surgery The Warren Alpert Medical School of Brown University
The Warren Alpert Medical School of Brown University Providence, Rhode Island, USA
Providence, Rhode Island, USA
Antonio Cruz, MD
Craig B. Birgfeld, MD, FACS Clinical Assistant Professor of Dermatology
Associate Professor of Surgery Department of Dermatology
Division of Plastic Surgery The Warren Alpert Medical School of Brown University
University of Washington Providence, Rhode Island, USA
Seattle Children’s Hospital
Seattle, Washington, USA Kristopher M. Day, MD
Fellow
Karl H. Breuing, MD Craniofacial and Pediatric Plastic Surgery
Residency Program Director University of Texas
Division of Plastic and Reconstructive Surgery Austin, Texas, USA
The Warren Alpert Medical School of Brown University
Providence, Rhode Island, USA Timothy Fei, MD
Cleveland Combined Hand and Upper Extremity Surgery
Jonathan P. Brower, MD Fellow
Resident in Plastic Surgery Metrohealth Medical Center
Division of Plastic Surgery Cleveland, Ohio, USA
The Warren Alpert Medical School of Brown University
Providence, Rhode Island, USA Sarah A. Frommer, MD, PhD
Clinical Assistant Professor of Surgery
Karl Bruckman, MD, DDS Division of Plastic Surgery
Clinical Assistant Professor of Surgery Baylor College of Medicine
Division of Plastic and Reconstructive Surgery Texas Children’s Hospital
Stanford University School of Medicine Houston, Texas, USA
Stanford University Medical Center and Lucile Packard
Children’s Hospital
Palo Alto, California, USA

xiv
TPS 23 x 31 - 2 | 31.10.20 - 14:00

Contributors

Michael A. Harrington, MD, MPH Daniel Kwan, MD


Associate Professor of Plastic Surgery Assistant Professor of Surgery
Residency Program Director Chief, Adult Reconstructive Surgery
Department of Plastic Surgery Division of Plastic and Reconstructive Surgery
University of South Florida The Warren Alpert Medical School of Brown University
Tampa, Florida, USA Providence, Rhode Island, USA

Raymond J. Harshbarger, MD, FACS, FAAP Karen Leong, MD


Associate Professor of Surgery Physician
Fellowship Director Private Practice
Craniofacial and Pediatric Plastic Surgery Newport Beach, California, USA
University of Texas
Austin, Texas, USA Paul Y. Liu, MD
Chairman of Plastic Surgery
Rajiv J. Iyengar, MD Professor of Surgery
Resident Physician Division of Plastic Surgery
Division of Plastic and Reconstructive Surgery The Warren Alpert School of Medicine of Brown University
Department of Surgery Providence, Rhode Island, USA
Yale School of Medicine
New Haven, Connecticut, USA Renata S. Maricevich, MD
Assistant Professor of Surgery
Charles C. Jehle, MD Division of Plastic Surgery
Resident Physician Baylor College of Medicine
Division of Plastic and Reconstructive Surgery Texas Children’s Hospital
The Warren Alpert Medical School of Brown University Houston, Texas, USA
Providence, Rhode Island, USA
Raman Mehrzad, MD, MHL, MBA
Glyn E. Jones, MD, FACS Resident Physician
Professor of Surgery Division of Plastic and Reconstructive Surgery
University of Illinois College of Medicine at Peoria The Warren Alpert Medical School of Brown University
Peoria, Illinois, USA Providence, Rhode Island, USA

Sabrina Khalil, BS Michael E. Migliori, MD


Medical Student Chairman of Ophthalmology
Morsani College of Medicine Professor of Surgery
University of South Florida Division of Ophthalmology
Tampa, Florida, USA The Warren Alpert Medical School of Brown University
Providence, Rhode Island, USA
Victor A. King, MD
Resident Physician Clinton S. Morrison, MD, FACS
Division of Plastic and Reconstructive Surgery Assistant Professor of Surgery
The Warren Alpert Medical School of Brown University Division of Plastic Surgery
Providence, Rhode Island, USA University of Rochester Medical Center
Rochester, New York, USA
Elizabeth Kiwanuka, MD, PhD
Resident Physician Sara A. Neimanis, MD
Division of Plastic and Reconstructive Surgery Resident Physician
The Warren Alpert Medical School of Brown University Division of Plastic and Reconstructive Surgery
Providence, Rhode Island, USA University of Rochester Medical Center
Rochester, New York, USA

xv
TPS 23 x 31 - 2 | 31.10.20 - 14:00

Contributors

Zachary Okhah, MD W. Kelsey Snapp, MD


Resident Physician Resident Physician
Division of Plastic and Reconstructive Surgery Division of Plastic and Reconstructive Surgery
The Warren Alpert Medical School of Brown University The Warren Alpert Medical School of Brown University
Providence, Rhode Island, USA Providence, Rhode Island, USA

Angie M. Paik, MD Derek M. Steinbacher MD, DMD, FACS


Resident Physician Professor of Surgery
Division of Plastic and Reconstructive Surgery Director of Craniomaxillofacial Surgery
The Warren Alpert Medical School of Brown University Division of Plastic and Reconstructive Surgery
Providence, Rhode Island, USA Yale School of Medicine
New Haven, Connecticut, USA
J. Thomas Paliga, MD
Resident Physician Patrick K. Sullivan, MD
Division of Plastic and Reconstructive Surgery Associate Professor of Surgery
University of Pennsylvania School of Medicine Chief, Aesthetic Surgery
Philadelphia, Pennsylvania, USA Division of Plastic and Reconstructive Surgery
The Warren Alpert Medical School of Brown University
Ivona Percec, MD, PhD Providence, Rhode Island, USA
Associate Director of Cosmetic Surgery
Director, Basic Science Research Rachel R. Sullivan, MD
Medical Director, Skin Care Program Clinical Assistant Professor of Surgery
Assistant Professor of Surgery Division of Plastic and Reconstructive Surgery
Division of Plastic and Reconstructive Surgery The Warren Alpert Medical School of Brown University
University of Pennsylvania School of Medicine Providence, Rhode Island, USA
Philadelphia, Pennsylvania, USA
David Tsai, MD
Adnan Prsic, MD Resident Physician
Assistant Professor of Surgery Division of Plastic and Reconstructive Surgery
Division of Plastic and Reconstructive Surgery Yale School of Medicine
Yale School of Medicine New Haven, Connecticut, USA
New Haven, Connecticut, USA
Albert S. Woo, MD, FACS
Vinay Rao, MD, MPH Associate Professor of Surgery, Pediatrics and Neurosurgery
Resident Physician Chief, Pediatric Plastic Surgery
Division of Plastic and Reconstructive Surgery Director, Cleft and Craniofacial Center
The Warren Alpert Medical School of Brown University Division of Plastic and Reconstructive Surgery
Providence, Rhode Island, USA The Warren Alpert Medical School of Brown University
Providence, Rhode Island, USA
Lauren O. Roussel, MD
Resident Physician Richard Zienowicz, MD, FACS
Division of Plastic and Reconstructive Surgery Associate Professor of Surgery
The Warren Alpert Medical School of Brown University Division of Plastic and Reconstructive Surgery
Providence, Rhode Island, USA The Warren Alpert Medical School of Brown University
Providence, Rhode Island, USA
Ean Saberski, MD
Resident Physician
Division of Plastic and Reconstructive Surgery
Yale School of Medicine
New Haven, Connecticut, USA

xvi
TPS 23 x 31 - 2 | 31.10.20 - 14:00

How To Use This Book: Tips from a Recent Board


Examinee
The American Board of Plastic Surgery (ABPS) Oral Exami- 4. Use this text as an outline for your studies.
nation is a culmination of your training thus far. Although you a. Each of the book’s sections is divided into chapters
may be familiar with most of the topics in this book, you that cover the most high-yield topics. Using this text
should nonetheless leave ample time for preparation (i.e., 2–3 as an outline to refer to other sources that can be
months). By the end of your study, you should be familiar helpful as you study and explore topics in depth. You
with all of the material in this text and feel comfortable on can expect any question related to clinical
being examined on any common plastic surgery issue. management.
This book is not an exhaustive encyclopedia of facts
b. The goal of the examination is to test surgical
about the topics tested in the oral examination. Instead, it
judgment, rather than knowledge (which was the
is designed to serve as a high-yield summary of the most
focus of the written boards). As a result, there are no
likely subjects to be tested during the “unknown” compo-
“pimp” questions on specific facts. Therefore, do not
nents of the examination. The book is purposefully kept
spend your time memorizing trivia. The examiners
short, to keep the material manageable. Certain subjects in
plastic surgery (e.g., craniosynostosis, Kienböck’s disease) instead want to know whether you will make safe
are relatively low yield and are not covered by design. surgical decisions.
Remember that there are two “unknown” sections dur- c. Use the margins in this book to add additional
ing the board examination. Each section covers six cases information.
with an allotted time of 45 minutes. The examinee has 10 d. For topics not found in this book, make flash cards.
minutes before each testing session to review the cases 5. Pictures/Techniques:
before meeting the examiners. One should plan on averag- a. Pay particular attention to the techniques/pictures in
ing 7.5 minutes per case. the chapters as you may be asked to draw markings
Here is a five-step approach to the use of this book: of incisions and other preoperative planning
1. Practice out loud. This is an oral examination. (z-plasty, syndactyly, etc.).
2. Analyze and describe the photograph: b. Learn one technique well (e.g., Rotation-
a. Defect/problem advancement Millard cleft lip repair).
b. Missing layers (skin, bone, mucosa, etc.) c. Safety is the key: You need not know the latest and
c. Anatomy greatest surgical technique. Rather, you need to know
d. Have a diagnosis by the end of your description (or a a safe, reasonable technique for treatment.
plan to establish the diagnosis) i. Whatever procedure you describe, know the
e. You will have 10 minutes to review your unknown complications and solutions.
cases prior to the examination ii. Do not introduce procedures or plans that you are
i. Become used to organizing your thoughts, not comfortable elaborating on.
diagnosis, work-up, consultations, treatment, 6. Be efficient with the use of your time.
complications, plan B, plan C…etc. a. Remember that you should average roughly
3. Follow the sequence: 7.5 minutes per case. Do not dilly-dally or overuse
a. The ABPS Oral Board Examination is a well- your time elaborating on the history and physical
structured and organized oral examination that is examination.
fairly uniform in its flow across examiners. b. The examiner has a list of boxes that need
Examiners are instructed on how and in which to be checked for you to receive full credit for the
sequence to examine you. Thus, becoming familiar case. Using delay tactics simply hurts your
with the “flow” of the oral examination is to your chances of meeting all of the milestones by
advantage. The chapters are designed in a way that moving on.
replicates the course of the actual examination. c. Upon completion of the examination, you will be
b. As you begin your studies, keep this“flow” in mind given an opportunity to go back to previous cases
and compartmentalize the information in the and elaborate on or change previous answers to
manner outlined in the chapters. questions.

xvii
TPS 23 x 31 - 2 | 31.10.20 - 14:00

How To Use This Book: Tips from a Recent Board Examinee

7. Practice makes perfect. be no nods of agreement during the examination. Get


a. Get used to being “exothermic” and explaining what used to it and don’t let it psych you out.
you see and thinking aloud your thought processes.
You cannot receive points if you do not state your Adnan Prsic, MD
observations. Assistant Professor of Surgery
b. Do not look to the examiners for verification. They Division of Plastic and Reconstructive Surgery
are trained to maintain a poker face throughout the Yale School of Medicine
test, which may unnerve some examinees. There will New Haven, Connecticut, USA

xviii
TPS 23 x 31 - 2 | 31.10.20 - 14:00

Practical Tips for the Oral Board Collection and Oral


Board Preparation
This introductory text is meant to help guide your prepa- to the original codes submitted need to be submitted to the
ration for the oral board examination. It is critical to read the ABPS. For example, if you accidentally bill a patient twice for
American Board of Plastic Surgery Booklet of Information for a procedure and remit the overbilled amount, this will still
the year that you will take the examination. Although this is be reviewed by the ABPS.
up to date at the time of printing, the American Board of Advertisement in the first year of practice is also some-
Plastic Surgery (ABPS) continues to revise the booklet of what challenging. It is imperative that your advertising
information on a yearly basis. If possible, review the booklet does not include any implication or suggestion that you are
a few times before you start practice, and certainly before already board-certified before passing the oral examina-
you submit your cases to the ABPS. tion. For those of you in group practice or academic set-
tings, advertising for your partners may indicate that
Your First Year of Practice everyone in the practice is board-certified. You must be
careful to avoid misrepresentation in both print and elec-
The first year of practice can be both daunting and exciting.
tronic advertising. You should personally review adver-
You have finally made it! Ultimately, your goal is to take
tising mailers, websites, videos, social media, and other
excellent care of your patients in your practice as a plastic
formats to make sure that there is no mention of board
surgeon by making wise and safe clinical decisions. You will
certification during this first year of practice.
have complications—all surgeons do. However, the key is to
It is also helpful to keep an electronic file with PDF
learn from your mistakes and avoid making them in the
versions of important documents—medical school diplo-
future.
mas, residency diplomas, state medical licenses, hospital
Cases are traditionally collected between July 1 and
privileging letters, surgery site credentialing letters, etc.—so
March 31, and a minimum of 50 major cases are required
that they are easily gathered at the time of submission for
for submission.
your oral board examination.
Make it a habit to take photographs of all preoperative
patients (even when there is nothing to see—e.g., preoper-
ative carpal tunnel release). You should also obtain a pho- What to Collect for Each Patient
tographic consent from each patient that you see at the It may be helpful to keep the relevant data for all of your
initial consultation or interaction. It is very important to patients in an electronic spreadsheet and update this spread-
take your own pictures so that each patient has good quality sheet accordingly every 2 to 4 weeks during your first year of
preoperative, intraoperative, and postoperative images. practice. Do not wait until the last minute to enter your cases
These images should reliably tell the story of every case. If into the ABPS clinical case log system. You must have a system
possible, try to standardize your photographs and minimize in place to capture the critical data elements for each case.
blood, instrumentation, and sponges during the intraopera- Ideally, you should upload your cases from your spreadsheet
tive photos. to the ABPS clinical case log system every week or every
It is essential to stay on top of the timeline set for oral 2 weeks. Attempting to do this at the last minute will be time-
board collection and submission. The key dates should be consuming and overwhelming!
marked on your calendar and reminders set several weeks At a minimum, you will need the following information
in advance, as the actual submission is challenging to for each patient:
complete at the last minute. For example, there is now a ● Name or initials

peer evaluation section due in the early spring of the year ● Medical record number

that you sit for the examination—this includes evaluations ● Surgery date
by the chief of surgery, chief of staff, chief of anesthesiology, ● Length of surgery
operating room nursing supervisor, former fellowship ● Type of anesthesia
director, and two ABPS-certified plastic surgeons. ● Length of anesthesia
Similarly, stay on top of your medical documentation and ● Surgery location
your coding/billing during your first year of practice. It may ● Type of admission
be helpful to take a coding course or to work with your
● Diagnosis
hospital/practice billers to understand the codes and
● ICD-10 code
charges for each surgery. Ultimately, you are responsible
● Anatomy and category
for what is put on the patient’s bill, and it is important to
● Procedure
avoid up-coding or down-coding. Additionally, any changes

xix
TPS 23 x 31 - 2 | 31.10.20 - 14:00

Practical Tips for the Oral Board Collection and Oral Board Preparation

● CPT code(s)
redact the protected health information or editing software
● Complication to cover the information.
● Mortality When you are analyzing your own case books before the
● Photo consent examination, it is important to understand the following
● Photographs—preoperative, intraoperative, and questions regarding your decision-making:
postoperative ● Did you make an accurate diagnosis?
● History and physical ● Why did you choose this treatment option?
● Relevant imaging ● What other surgical options did you consider?
● Operative report ● Would you choose this treatment option again?
● Pathology report ● What are the known complications of this operation?
● Postoperative notes ● What can you do to prevent the known
● Consultations complications?
● Billing form ● How would you manage the complications?

● Is your result less than average?

● Were there any ethical issues with your coding?


Known Case Books and the Digital ● How do you think you did overall with your case?

Format ● What would you do differently next time?

There is one section of “known” cases on the examination.


There is no paper copy of your case books at the exam-
This section lasts for 45 minutes and covers five of your own
ination. The known case section is now reviewed on
cases, which have been preselected by the ABPS. There is a
tablets with the examiner and examinee. You should
possibility that you will be asked to prepare additional
familiarize yourself with the relevant sections in your
cases, if documentation is missing from your initial five
personal copy of the PDF well before the test, so that you
procedures selected. Some candidates may be asked to
are not wasting time looking for sections during the actual
provide 8 or 11 cases in total to the examiner to choose
examination. At a minimum, the following details should
from. The best means of avoiding this is by ensuring metic-
be memorized for your cases:
ulous documentation and follow-up.
● Operative time
This section has undergone the most change in recent
● Microsurgical ischemia time
iterations of the examination. The hardbound “blue books”
● Estimated blood loss and need for transfusion
have been replaced with a digital copy. All of the informa-
● Tumescent volumes
tion required and the preparation of these case reports are
● Significant differences in anesthesia time versus surgical
the same, but they are now uploaded electronically several
months (typically 3 months) before the examination. This time
means that examiners can review your books ahead of time, ● Unexpected items in the perioperative record

rather than simply the night before your examination. This ● Hypothermia

also means that you have an end-date for when you must ● Return to the operating room

stop working on your books, several months before the ● Details of the imaging

actual examination. ● Consultations

With the advent of the electronic medical record, there is


quite a lot of redundancy built into the documentation,
which necessarily increases the number of pages you will Unknown Cases
submit for each patient. The current ABPS guidelines now
There are two sections of “unknown” cases in the exami-
put page limits on certain sections, such as progress notes
nation, which are the focus of the remainder of this book.
and lab values, so it is important to provide a succinct copy
Each section lasts for 45 minutes and covers six unknown
of the documentation in your casebook for these sections,
cases, averaging 7.5 minutes for each case. These sections
rather than submitting an exhaustive record. The selected
will test the full breadth of plastic surgery, so any topic is fair
documents should tell the story clearly, with extraneous
game.
information removed.
Each case can be broken down into five parts:
Gone are the days of marking out the patient’s name and
● Identify the clinical diagnosis and assess the problem
medical record number on every page with a permanent
● Preoperative considerations and work-up
black marker. Given that the case books now number
● Treatment plan—details of the operation, acceptable
hundreds of pages for each patient, there are more efficient
alternatives, why you chose this option, risks and
ways to remove identifying information from the docu-
ments. You should either use a professional PDF reader to benefits of your choice

xx
TPS 23 x 31 - 2 | 31.10.20 - 14:00

Practical Tips for the Oral Board Collection and Oral Board Preparation

● Understanding possible complications and managing – Free flap landmarks (rectus abdominis, latissimus
them dorsi, fibula, radial forearm, anterolateral thigh, etc.)
● Safety/coding/ethics – Pedicled flap landmarks

It is very common to be expected to draw normal anat-


omy or relevant details of a surgery during the assessment. Preparing for the Oral Board
The following list should serve as a guide for possible
diagrams and techniques that you may be asked to illustrate Examination
at the examination: This is an ORAL examination, so you must practice orally.
● Craniofacial: You may also be asked to draw your operation, so this
– External ear anatomy should be part of your practice sessions. You should be able
– Cleft lip repair (unilateral and bilateral) to draw the details and verbally explain them at the same
– Cleft palate repair time. Practice before the examination is critical to your
success.
– Ear reconstruction with local flaps
To facilitate study, you will notice that the beginning of
– Cheek reconstruction with local flaps
every chapter of this book begins with a case image and a
– Eyelid reconstruction with local flaps
short caption. This is a rough recapitulation of the testing
– Lip reconstruction with local flaps (Abbe flap,
environment. Before proceeding to the next page, imagine
Estlander flap, etc.)
yourself as the examinee. Think through what key points
● Aesthetic:
you would describe during the examination. Ask yourself
– Facelift incisions whether you feel prepared to discuss the case in its entirety.
– Browlift incisions Only after going through this mental exercise, should you
– Blepharoplasty incisions proceed to read the body of the chapter, which is designed
– Necklift incisions as a concise refresher of key elements for discussion and
– Rhinoplasty surgical planning elaboration.
– Injection patterns for fillers and botulinum toxin You may find that you study the content best when
● Hand: alone, but you should practice case scenarios ideally in
– Hand or foot syndactyly release (volar and dorsal groups. If possible, practice with other test-takers who
surfaces) come from diverse practice backgrounds—if you are trained
– Brachial plexus in craniofacial surgery, you should consider partnering with
– Extensor tendon mechanism another plastic surgeon trained in hand surgery. You should
– Flexor tendon mechanism meet on a regular basis several months before the exam-
– Extensor tendon repair ination to review your known case books and to practice
– Flexor tendon repair unknown cases. Try to provide constructive feedback, as
you cannot change the decisions that were made in the past.
– Carpal tunnel incision
You should also simulate at least one mock examination,
– Cubital tunnel incision
being the examinee and the examiner, alternatively. Try to
– Compartment syndrome release
complete these mock examinations in the allocated time, to
● General:
replicate the actual examination experience. Consider ask-
– Rhomboid flap
ing your colleagues, especially recent exam takers, to prac-
– Bilobed flap
tice with you and review your selected known cases.
– Z-plasty Above all, good luck for successful completion of the
– W-plasty exam!
– Keystone flap Noopur Gangopadhyay, MD, FAAP
– Breast reduction markings Assistant Professor of Plastic Surgery
– Mastopexy markings Ann & Robert H. Lurie Children’s Hospital of Chicago
– Cross-sectional anatomy of the upper arm, upper leg, Northwestern University Feinberg School of Medicine
forearm, and lower leg Chicago, Illinois, USA

xxi
TPS 23 x 31 - 2 | 31.10.20 - 14:00
1 Nasal Fractures 3
Section I
2 Zygomatic Fractures 5
Facial Fractures 3 Mandibular Fractures 9

4 Frontal Sinus Fractures 13

5 Le Fort Fractures 17

6 Pediatric Mandible Fractures 21

I
Case 1 Nasal Fractures
Charles C. Jehle and Albert S. Woo

Case 1 (a, b) A 63-year-old man presents to the emergency department complaining of nasal pain after falling on his face.

3
Facial Fractures

1.1 Description 1.4 Treatment


● Obvious nasal deformity with deviation of the nasal dorsum ● If septal hematoma is present, it must be drained
to the left immediately. Mucous membranes are compressed against
● Overlying soft tissue injury and swelling septum with nasal packings/tampon to prevent recurrence.
● Periorbital ecchymoses, primarily on the right side ● If displaced and patient desires correction, a closed nasal
● Presumed nasal airway obstruction, particularly on the right side reduction is the first-line treatment.
– Preparation of nasal cavity: Oxymetazoline (Afrin) pledgets
for hemostasis; lidocaine injection for anesthesia. Ideally
1.2 Work-Up performed with sedation.
– Bones reduced with boies elevator. Nasal packing for 3 days
1.2.1 History and dorsal splint placed for protection.
● Mechanism of injury: Helpful in determining angle and – Septal deviation may be improved with Asch forceps. Doyle
magnitude of force splints sutured in place to maintain reduction.
● Previous fractures or injuries to the nose ● Open (septo) rhinoplasty may be indicated if closed nasal
● History of facial surgery, particularly nasal or septal surgery. reduction is unsuccessful.
Septoplasty, in particular, would raise concerns about adequate
septal support. This may also eliminate the possibility of
cartilage graft harvest from the nose, if necessary 1.5 Ethical Considerations
● Premorbid nasal airway patency
● Patients may encourage a physician to include information
● Medications, particularly anti-platelets or blood thinners
which may not be true to obtain approval for surgery. This
must be avoided.
1.2.2 Physical Examination ● When decision for a rhinoplasty has been made, patient may
● Advanced Trauma Life Support (ATLS) protocol: Identify any request additional work to improve baseline nasal
potentially life-threatening conditions. appearance (e.g., remove a hump). While this may be done at
● Detailed examination of the face, including evaluation for sensory additional cost, it must not be charged to insurance.
and motor changes, and eye, midface, and intraoral injury
● Nasal examination: Evaluate for tenderness, crepitus, or step-
off. Assessment of nasal dorsum for straightness, position to
1.6 Complications
midline, and lateral profile. Speculum examination to assess ● Inadequate reduction
septum for deviation and septal hematoma. ● Unstable reduction often results from improper packing
and/or splinting
1.2.3 Pertinent Imaging or Diagnostic ● Residual nasal deformity: Especially in very comminuted
fractures, the nasal bones may deviate over time even after
Studies perfect reduction
● Isolated nasal fractures can be evaluated by clinical – To minimize deformity, patients should be taught to
examination alone. manipulate the nose regularly to a straight position for
● Consider maxillofacial computed tomography (CT) scan in the several weeks after reduction
setting of high energy mechanism of injury, presence of ● Significant epistaxis
distracting injuries, or if patient is unable to cooperate with – Nasal packing with gauze or nasal tampons. If unsuccessful,
examination. (1) surgery for direct ligation of sphenopalatine or anterior/
– Evaluate for additional fractures of the face, including posterior ethmoidal vessels or (2) interventional radiology
orbital floor fracture, which may be difficult to assess in for embolization
acute swelling. ● Injury to cribriform plate: Occurs when the surgeon does not
measure distance to nasal bones and elevator is too deep
during closed reduction
1.2.4 Consultations
● Trauma evaluation based on mechanism of injury and if other
injuries are suspected. 1.7 Critical Errors
● Ophthalmology consultation should be considered if any ● Failure to rule out other injuries, especially in high energy
injury to the globe is suspected or if orbital fracture is found. trauma.
● Missing or not treating septal hematoma immediately,
1.3 Patient Counseling resulting in septal necrosis and perforation.

● Patient should be counseled that closed nasal reduction has


no guarantee of success. Secondary surgical correction with
(septo) rhinoplasty may be necessary.
● Closed nasal reduction does not correct baseline nasal
deformities present before injury.

4
Case 2 Zygomatic Fractures
Vinay Rao and Albert S. Woo

Case 2 (a, b) A 34-year-old male presents to the emergency department complaining of right cheek pain, numbness, and swelling after an assault.

5
Facial Fractures

2.1 Description 2.4 Treatment


● Right mid-facial and periorbital edema with malar depression 2.4.1 Initial Management (in the
and mild right hypoglobus, with presumed enophthalmos
● Computed tomography (CT) demonstrates comminuted, Emergency Department)
displaced right zygomaticomaxillary complex (ZMC) fracture, ● Always start with ABCs of trauma. All emergent injuries must
and fracture of coronoid process of mandible be managed first.
● By definition, the right orbital floor is fractured in a displaced ● Ophthalmology consultation in all orbital fractures to rule out
zygomatic injury injury of globe must be performed prior to operative
interventions as manipulation may exacerbate eye injury.
2.2 Work-Up ● Definitive treatment of facial fractures may be delayed for up
to 2 weeks without compromising results.
2.2.1 History
2.4.2 Nondisplaced Fractures
● Mechanism of injury: Helpful in determining angle of force
and severity of injury ● Simple, nondisplaced fractures do not need surgery and may
● Change in vision, loss of vision, or diplopia be managed conservatively
– Must rule out orbital injury prior to operative intervention ● Recommend soft diet to minimize activation of the masseter
● Trismus can occur with medial displacement of the zygomatic
arch impinging on the temporalis muscle 2.4.3 Isolated Zygomatic Arch Fracture
● Relevant medical history (previous facial injuries or
● Temporal (Gillies) approach (▶ Fig. 2.1)
fractures), surgical history (previous facial surgeries), and
– A 2-cm transverse incision is made in the hairline. Dissect
social history (alcohol, smoking, drug use)
through the superficial and deep temporal fascia layers
until temporalis muscle is visible. An elevator is advanced
2.2.2 Physical Examination behind the displaced arch.
● Signs of ZMC fractures are malar depression (masked by soft
● Intra-oral approach (Keen) may also be used for reduction.
tissue swelling early on), periorbital ecchymoses,
enophthalmos, and/or hypoglobus (usually masked by orbital 2.4.4 Displaced Fractures
swelling), inferior slant of the palpebral fissure, and ● Timing of repair: 1 to 2 weeks for adults and within 1 week for
tenderness at infraorbital rim and along zygomaticofrontal pediatric patients
(ZF) suture – Addressing fractures at a later time may require
● Numbness of the cheek, nose, upper lip, and teeth: Typical of osteotomies to allow for adequate reduction. Coronal access
V2 distribution may be needed to osteotomize the zygomatic arch.
● Associated eye examination: Look for visual changes, ● Fractures that are significantly displaced or comminuted
diplopia, or extra-ocular muscle entrapment require open reduction/internal fixation. Plates should be
positioned at facial buttresses (▶ Fig. 2.2).
2.2.3 Pertinent Imaging or Diagnostic
Studies
● High resolution maxillofacial CT scan
– Evaluate the five articulations of the zygoma: (1) lateral
orbital rim (zygomaticofrontal), (2) inferior orbital rim, (3)
zygomaticomaxillary buttress, (4) zygomatic arch, and (5)
lateral orbital wall (zygomaticosphenoid)
– Evaluate orbital floor defect on coronal cut images

2.3 Patient Counseling


● Older patients, in particular, may consider nonoperative
management despite displaced fractures. Surgeons must help
weigh risks against benefits of surgery.
● Patients must be informed of possible development of
enophthalmos/hypoglobus, malar asymmetry/depression, or
appreciable bony step-offs. These clinical examination findings
may become more prominent as facial swelling subsides.
● With operative management, adverse events/complications
must be discussed, including asymmetric malar positioning, Fig. 2.1 Temporal (Gillies) approach for elevation of zygomatic arch
fractures. (Source: Treatment. In: Janis J, ed. Essentials of Plastic
development of possible entropion/ectropion and a remote
Surgery. 2nd Edition. Thieme; 2014.)
possibility of vision loss.

6
Zygomatic Fractures

Fig. 2.2 Buttresses of the face. Vertical


buttresses include the zygomaticomaxillary
(ZM) and nasomaxillary (NM). The
infraorbital rim, maxillary alveolus, and
mandible contribute to transverse
buttresses of the face.

that the best means of confirming three-dimensional


reduction of the ZMC fracture is at the lateral orbital wall,
which is accessed through this approach.
– Lower eyelid incision (transconjunctival, subciliary, or
subtarsal) for inferior orbital rim and orbital floor
– Upper buccal sulcus incision for access to maxillary
buttresses
● Orbital floor evaluation
– Zygomatic reduction may cause orbital floor blowouts to
become more prominent.
– If a sizeable defect is present, the floor should be
reconstructed with an implant (e.g., porous polyethylene or
titanium) or bone graft after the zygoma has been reduced.

2.5 Complications
● Retrobulbar hematoma
Fig. 2.3 Reconstruction of zygomaticomaxillary complex (ZMC) – It can occur at time of injury or postoperatively.
fracture with three-point fixation and orbital floor reconstruction. With
– Signs are severe eye pain, proptosis, afferent pupillary
standard anterior approach, the zygomaticofrontal (ZF) suture,
infraorbital rim, and zygomaticomaxillary (ZM) buttress are plated. defect, change in visual acuity, and ultimately blindness.
Orbital floor is addressed with a titanium plate. (Source: Operative – Surgical emergency: Requires immediate lateral canthotomy
Technique and Exemplary Repair. In: Pollock R, ed. Craniomaxillofacial with inferior cantholysis for drainage of the hematoma.
Buttresses. Anatomy and Operative Repair. Thieme; 2012.) – Mannitol, acetazolamide, and ophthalmology consult are
supplementary measures.
– Critical points of fixation include: (1) Zygomaticofrontal ● Diplopia
region or lateral orbital rim, (2) infraorbital rim, and (3) – Commonly seen after surgery due to edema
zygomaticomaxillary buttress. At least three points of – Differential diagnosis following zygomatic repair includes
fixation are necessary to guarantee three-dimensional extraocular muscle entrapment, muscle contusion,
stability. When indicated, the zygomatic arch may be periorbital edema, enophthalmos, or motor nerve palsies
stabilized as a fourth point of fixation (▶ Fig. 2.3). – Muscle entrapment is ruled out if forced duction procedure
● The operative approach is determined by the status of the was performed
zygomatic arch. If the arch is comminuted or otherwise – If no structural abnormality is suspected, the patient can
irreducible, a coronal incision will be needed for reduction follow-up for monitoring
and fixation of the arch. Otherwise, the zygomatic fracture ● Inadequate reduction resulting in malposition or
can be addressed with an anterior approach. enophthalmos
● The standard anterior approach consists of three incisions: ● V2 distribution (infraorbital nerve) anesthesia/paresthesias
– Lateral part of upper blepharoplasty (or lateral brow) – Most commonly due to nerve contusion and generally
incision for access to the lateral orbital rim and wall. Note resolves within 6 months

7
Facial Fractures

Lower lid ectropion (external incision) or entropion


(transconjunctival incision)
2.6 Critical Errors
– This usually responds to eyelid massage but may require ● Failure to assess ABCs in acute trauma
surgical correction. ● Missing other facial injuries on examination or CT. Watch out
– The subciliary incision has the highest risk of ectropion for naso-orbito-ethmoid (NOE) fractures, which may occur
when compared to transconjunctival or subtarsal concomitantly.
approaches. ● Failure to identify orbital injury, which can be worsened with
● Infection requires antibiotics and possible hardware removal surgery
● Inadequately addressing the orbital floor at the time of
zygomatic reduction
● Inability to recognize and treat a retrobulbar hematoma

8
Case 3 Mandibular Fractures
Vinay Rao and Albert S. Woo

Case 3 (a–c) A 21-year-old female presents to the emergency department complaining of facial pain after being shot in the face, with bullet entering
from right cheek and exiting left jaw.

9
Facial Fractures

When CT is unavailable, X-ray studies may be useful.


3.1 Description ●

– Option 1: Panorex. Visualization of entire mandible and


● Bullet hole in right cheek, exit site presumed to be in left jaw dentition. Limited evaluation at symphysis and condyles.
● Displaced, comminuted body and angle fracture of the Additional Towne's view improves evaluation of
mandible with large butterfly fragment and malocclusion on subcondylar regions.
examination – Option 2: Mandible series (anteroposterior. lateral, and
● Unclear facial nerve function or evidence of parotid injury bilateral oblique views)

3.2 Work-Up 3.3 Patient Counseling


3.2.1 Initial Assessment ● Patients must be informed that despite the surgeon's best
efforts, they may have long-term sequelae from injury. This
● ABCs: Attention to patency of the airway given multiple
includes the possibility of malocclusion, temporomandibular
mandibular fractures. Intubation may be necessary if the
joint (TMJ) problems, arthritis, dental malalignment, and
patient cannot protect the airway.
permanent nerve injury.
● Cervical spine precautions: This is mandatory in the setting of
high energy injuries until the spine is cleared both clinically


and radiographically.
Concomitant injuries: Manage any potentially life-threatening
3.4 Treatment
injuries first. The repair of mandibular fractures is not 3.4.1 Initial Management (in the
emergent and can be performed on an elective basis
(generally within 14 days of injury).
Emergency Department)
● Treatment of any associated soft tissue injuries
3.2.2 History ● Bridle wire (optional): Stainless steel wire typically placed
two teeth away on either side of a fracture line to help with
● Mechanism of injury, presence/location of pain, loss of
temporary stability. Useful to increase patient comfort in the
sensation, presence of loose or missing teeth, use of dentures,
setting of unstable fractures.
assessment of occlusion
● Relevant medical history (prior mandibular or facial
fractures), surgical history (prior facial surgery), social history 3.4.2 Nondisplaced Fractures
(alcohol, smoking, drug use)
● If stable: Conservative management with a soft, non-chew
diet for roughly 4 weeks. Subsequent instability or
3.2.3 Physical Examination displacement necessitates operative treatment.
● Palpate bony structures in a systematic fashion to identify ● Mild instability or displacement: Treat with
areas of tenderness, deformity, step-off, and instability. maxillomandibular fixation (MMF).
● Assess mobility (ability to open/close mouth, deviation of – Arch bars (or hybrid MMF) versus intermaxillary fixation
mandible on movement) and occlusion (may be evaluated (IMF) screws
based on wear facets of teeth) – MMF is effective only in patients when appropriate
● State of dentition: Edentulous mandibles will require more dentition is present
aggressive procedures to rigidly fixate bone segments due to – Rule of thumb
○ Subcondylar: 2 weeks with early range of motion (ROM)
decreased bone stock.
● Evidence of intraoral lacerations (exposed bone), loose or using guiding elastics
○ Body/Angle: 4 weeks
absent teeth, identification and removal of prosthetics
○ (Para) symphyseal: 6 weeks
(dentures), sublingual hematoma, or foreign bodies
● Neurologic examination: The mental/inferior alveolar nerve
provides sensation to the lower lip. The marginal mandibular 3.4.3 Displaced Fractures
branch of the facial nerve innervates the depressors of the
lower lip and is rarely injured. ● Open Reduction/Internal Fixation (ORIF)
● In gunshot wounds, must consider all structures in path of – (1) Wide exposure of fractures, (2) Establish occlusion (MMF
bullet, including parotid, facial nerve, muscle, tongue, and may help), (3) Reduction and plate fixation, (4) Release MMF
other intraoral injuries. and confirm normal occlusion with condyles seated in the
● Assess presence of concomitant midfacial fractures (may alter TMJ, and (5) Re-establish MMF if indicated.
occlusion). ● Transfacial approaches are preferred for ORIF of comminuted
fractures and gunshot wounds: Increased visualization, access
to all mandibular surfaces; also used in edentulous patients
3.2.4 Pertinent Imaging or Diagnostic
for improved reduction.
Studies ● Plating technique
● High-resolution maxillofacial computed tomography (CT): – Stronger plates (i.e., fracture or reconstruction instead of
Gold standard for imaging. Three-dimensional mini-plates) necessary to establish rigid fixation of the
reconstructions may assist in further evaluating injury. inferior border of the mandible.

10
Mandibular Fractures

● (Para) symphyseal fractures: Inherently unstable and require


ORIF
– Intraoral approach for most fractures: Easy approach with
excellent exposure
– Bigonial compression needed in reduction of mandible, as it
tends to splay
– Extraoral (submental) approach may be needed for
comminuted fractures
● Prior to completion of the case, the patient must be examined
with MMF released.
– The condyles must be firmly seated in the TMJ fossa, and
normal occlusion must be confirmed. If this does not result
in good occlusion, the plates must be released and the
fracture re-reduced.
– Once occlusion has been confirmed, MMF may be re-
established as necessary.
● Edentulous mandibles: Prone to malunion due to limited
bone stock
– Edentulous mandibles require more aggressive treatment,
generally with an open approach and fixation with large
reconstruction plates.
– In limited circumstances, one may consider using a
Fig. 3.1 Zones of the mandible.
Gunning splint or wiring in patient's dentures. It provides
less stability than open approach.
● Gunshot wounds: These are high energy injuries with severe
– A tension band may be placed superiorly to avoid soft tissue injury. They should almost always be accessed
splaying of the fracture line. It can be a mini-plate just with an external incision.
below the tooth roots or an arch bar anchored to the
dentition.
● Subcondylar/Ramus fractures (▶ Fig. 3.1)
3.5 Ethical Considerations
– Mildly displaced: Preferentially treat with closed methods ● Patients may enter with complaints of malocclusion or
(MMF with early release and elastic guidance). nonunion after management elsewhere. One should avoid the
– ORIF necessary if deformity precludes proper occlusion temptation to suggest another surgeon's treatment was
(i.e., condylar head displacement into middle cranial fossa; incorrect or inadequate without complete information.
foreign body lodged in TMJ; bilateral subcondylar fractures
which will result in an anterior open bite).
– Preferred approaches: Extraoral, via retromandibular 3.6 Complications
incision (more direct access) or submandibular (Risdon)
● Malocclusion: Adequate occlusion must be confirmed
approach from inferiorly.
intraoperatively.
● Angle fractures: Require ORIF. If the 3rd molar is involved, it
● Malunion/Nonunion: These may require debridement and
should be removed if it interferes with reduction.
bone grafting.
– Champy technique is acceptable in straightforward,
● Infection: Avoid the urge to remove plates until fracture
noncomminuted fractures of the angle. This entails
healing has been achieved. Treat with long-term antibiotics
placement of a tension band plate at the internal angle
until plates are removed.
without use of an inferior border plate.
● Damage to inferior alveolar nerve: Avoid injury during plating.
– Two plates: Intraoral approach with plating using tension
● Ankylosis: Avoid MMF longer than 2 weeks in the setting of
band (along internal border) and inferior border plates.
condylar fractures.
Percutaneous access may be necessary for screw fixation
along the inferior border.
– External Risdon (submandibular) approach for comminuted
fractures.
3.7 Critical Errors
● Body fractures: Require ORIF ● Inadequate treatment of significantly displaced fractures
– Intraoral (vestibular) approach is preferred for most (i.e., with MMF)
fractures. ● Forgetting to confirm normal occlusion upon completion of
– Risdon approach is used for comminuted fractures and surgery
gunshot wounds. It is useful for wide access to posterior ● Failure to remove plates and re-reduce the mandible if the
body fractures. patient does not have normal occlusion after MMF is released

11
Case 4 Frontal Sinus Fractures
Dardan Beqiri, Lauren O. Roussel, and Albert S. Woo

Case 4 (a, b) A 21-year-old male unrestrained passenger presents to the emergency department following a motor vehicle crash against a tree.

13
Facial Fractures

4.1 Description 4.3 Patient Counseling


● Large avulsion of the forehead involving the brow ● Patients should be informed of anticipated significant
● Computed tomography (CT) demonstrates depression of the deformities to the forehead without intervention. Edema
anterior table of the frontal sinus from trauma frequently masks this deformity early on.
● Status of the posterior table of the frontal sinus and ● The use of a coronal incision versus “open sky” approach via
nasofrontal outflow tract is unknown and must be elucidated an open laceration should also be discussed with patients.

4.2 Work-Up 4.4 Treatment


4.2.1 History ● Management is guided by the pattern of injury (▶ Fig. 4.1).
● Antibiotics are recommended due to contamination of the
● Mechanism of injury operative site by nasal mucosa from the frontal sinus.
– Blunt versus sharp ● Older patients may choose to accept a deformity and forgo
– High energy of impact raises concern for additional injuries the risks of surgery.
● Change in vision, loss of vision, or double vision
– Must rule out orbital injuries prior to considering operative
intervention 4.4.1 Nondisplaced Anterior Table
● Numbness of forehead: Injury to V1 distribution Fracture
● Rhinorrhea: Concern for dural injury and cerebrospinal fluid
(CSF) leak
● Nondisplaced or minimally displaced fractures of the frontal
sinus often do not require operative repair and may be
managed conservatively.
4.2.2 Physical Examination
● ABCs and primary survey: Identify any potentially 4.4.2 Isolated, Displaced Anterior Table
life-threatening conditions
– Greater suspicion depending on mechanism and greater
Fracture
force of injury ● Open reduction/internal fixation (ORIF) through laceration or
● Identify lacerations over the forehead, glabella, or coronal incision with preservation of sinus
supraorbital ridge – Coronal incision is often superior to lacerations, which may
– May be utilized for direct access in repair limit access.
● Palpate for step-offs and/or depressions in the frontal area ● Anterior table fractures with associated nasofrontal duct
– Edema often masks contour deformities injury but no significant posterior table involvement
● Evaluate for sensory changes in supraorbital/supratrochlear – Frontal sinus obliteration recommended to remove the sinus
nerves mucosa and block off the outflow tract with pericranial
● Test function of frontalis and corrugator muscles of forehead flap. The sinus is then packed with bone or fat.
● Examine for nasal drainage: Must rule out CSF rhinorrhea ○ Pericranial flap is a flap of periosteum and loose areolar

– Ring test at the bedside: CSF will separate on filter paper tissue separated from the frontalis muscle above and
and create a double ring or halo sign based on supratrochlear or supraorbital vessels.
– Beta-2 transferrin is the definitive test for CSF, but it may – The anterior table is elevated for access. Complete removal
take several days to obtain the results of frontal sinus mucosa is performed with diamond bur on
a high speed drill.
– The nasofrontal outflow tract is then obliterated with
4.2.3 Pertinent Imaging or Diagnostic pericranial flap, fat, fascia, or bone chips to prevent mucosal
Studies ingrowth from the ethmoids.
○ There is no significant advantage in any one particular
● High resolution maxillofacial CT scan of both axial and
technique over another.
coronal planes
– The anterior table is replaced, reduced, and plated. A small
– Evaluate for injury to anterior and posterior tables;
gap is left in the bone to allow for the pericranial flap, if
determine degree of comminution/displacement
necessary.
– Evaluate nasofrontal outflow tract for ability to drain the
– Obliteration of the sinus is performed only if there is
frontal sinus
minimal or no posterior table displacement and CSF leak.
– Identify intracranial injuries (e.g., pneumocephalus, etc.)
and other facial fractures
4.4.3 Combined Anterior/Posterior
4.2.4 Consultations Table Fractures
● Neurosurgical consultation is necessary if intracranial injury ● Involvement of the posterior table raises concern for dural
is suspected (e.g., significant displacement of posterior table, injury and must be addressed by a neurosurgeon prior to
pneumocephalus, CSF rhinorrhea). repair of the fracture.

14
Frontal Sinus Fractures

Fig. 4.1 Algorithm for management of frontal


sinus fractures. (Reproduced with permission
from Yavuzer R, Sari A, Kelly C, et al. Management
of frontal sinus fractures. Plast Reconstr Surg
2005;115:79e–93e.)

– Injuries with less than one table-width displacement of the ● Mucocele/mucopyocele: This may present many years
posterior table are frequently observed when there is no following trauma due to inadequate removal of sinus mucosa.
clear evidence of dural tear. – Treat with drainage of the mucocele and obliteration of the
● When the posterior table is minimally involved but the frontal sinus.
nasofrontal ducts are injured, frontal sinus obliteration is ● Cavernous sinus thrombosis
indicated (▶ Fig. 4.1). ● Post-traumatic deformity from inadequate anterior table
● When the posterior table is significantly displaced or reconstruction
comminuted, cranialization of the frontal sinus with
occlusion of the frontal sinus outflow tracts should be
performed. This is a combined procedure with the 4.6 Critical Errors
neurosurgery team. ● Ignoring clear drainage from the nose, thereby missing a CSF
– The frontal sinus is “cranialized” by removal of the posterior
leak
table, allowing the brain to extend forward into the ● Failure to obtain neurosurgical consultation when there is
previous sinus space.
involvement of the posterior table or evidence of violation of
– Pericranial flap is placed along the floor of the anterior
the dura (CSF leak)
cranial fossa to separate the nasal cavity from the brain. The ● Not recognizing injury to nasofrontal outflow tract,
nasofrontal outflow tract is also obliterated in this process.
necessitating plugging of the tract with obliteration versus
cranialization of the frontal sinus
4.5 Complications ●


Failure to remove all of the mucosa during sinus obliteration
Not separating the nasal cavity from the anterior cranial fossa
● Frontal sinusitis, meningitis/encephalitis, brain/epidural during cranialization
abscess: The nasofrontal outflow tract must be obliterated to
prevent bacterial contamination of intracranial contents.

15
Case 5 Le Fort Fractures
Sara A. Neimanis and Clinton S. Morrison

Case 5 (a, b) A 38-year-old male presents complaining of malocclusion and upper jaw pain following a motor vehicle collision.

17
Facial Fractures

5.1 Description
● Anterior open bite on physical examination
● Transverse fracture across the maxilla involving the
zygomaticomaxillary (ZM) and nasomaxillary (NM)
buttresses on both sides, consistent with a Le Fort I fracture
● Fracture of the pterygoid plates

5.2 Work-Up
5.2.1 History
● Mechanism of injury: Helpful for determining severity of
impact and trajectory of force
● Changes in vision, occlusion, sensation, breathing, or hearing
● Previous facial trauma

5.2.2 Physical Examination


● Identify any potentially life-threatening conditions. Always
take spinal precautions and rule out cervical injury.
● Perform a detailed examination of the face including:
– Inspection for swelling/edema
Fig. 5.1 Le Fort fracture patterns: (I) Transverse fracture through the
– Palpation for tenderness, crepitus, or step-off zygomaticomaxillary (ZM) and nasomaxillary (NM) buttresses; (II)
– Examination for sensory and motor changes pyramidal fracture through ZM buttresses, infraorbital rims, medial
– Eye examination: Visual acuity, diplopia, entrapment orbit, and nasofrontal (NF) junction; (III) complete craniofacial
– Intranasal and intraoral examination of ears and tympanic disjunction with separation of the cranium from the face at
membrane zygomaticofrontal (ZF) sutures, orbital, and NF junction.
● State of dentition: Fractured, missing, or decayed teeth and
occlusal pattern
● Assess for midface instability: Stabilize the face at the nasal
root (left hand) and grasp the upper anterior alveolar arch
5.2.4 Consultations
(right hand) and pull forward/down. ● Ophthalmology consultation in all orbital fractures to rule out
– If the midface is mobile with stability at nasal root, it is injury of globe. It must be performed prior to operative
indicative of a Le Fort I fracture. intervention as intraoperative manipulation may exacerbate
– If there is also mobility at the nasofrontal (NF) suture, it is a eye injury.
Le Fort II fracture.
– If there is also mobility at the zygomaticofrontal (ZF) suture,
it is a Le Fort III fracture (see ▶ Fig. 5.1). 5.3 Patient Counseling
● Regardless of any need for surgical repair, patients are placed
5.2.3 Pertinent Imaging or Diagnostic on a soft diet for several weeks to allow for bony healing.
They should be educated on how to maintain appropriate
Studies protein and calorie intake during this time.
● High resolution maxillofacial computed tomography (CT) ● Operative reconstruction may require intermaxillary fixation,
scan which would necessitate a liquid diet.
– Fracture of the pterygoid plates is the sine qua non of Le
Fort fractures
– Le Fort I is a transverse fracture of the maxilla involving the 5.4 Treatment
ZM and NM buttresses. ● Advanced Trauma Life Support (ATLS) protocol: All emergent
– Le Fort II is a pyramidal fracture involving the ZM
injuries must be managed first.
buttresses, inferior orbital rims, inferior and medial orbital
– Definitive treatment of facial fractures may be delayed for
walls, and nasofrontal (NF) region.
up to 2 weeks without compromising results. Longer delays
– Le Fort III denotes craniofacial disjunction and involves the
in treatment increase risk of infection and need for
zygomatic arch, lateral orbital rim, lateral orbital wall,
osteotomies as bone healing takes place.
orbital floor, medial orbital wall, and NF region.

18
Le Fort Fractures

5.4.1 Nondisplaced, Stable Fractures – Le Fort II: Fixation of ZM buttresses and infraorbital rims.
Uncommonly, the NF junction is necessary if this region is
● Nonoperative management is an option with a soft, nonchew significantly displaced.
diet for 4–6 weeks – Le Fort III: Plating at zygomaticofrontal (lateral orbital wall)
● Close follow-up to ensure that patient maintains good regions and NF junction
occlusion ● MMF is then released and occlusion is checked with
● If occlusion is disrupted, maxillomandibular fixation is mandibular condyles seated in the glenoid fossa.
indicated

5.4.2 Displaced, Unstable Fractures 5.5 Ethical Considerations


● These injuries may occur in the setting of severe, life-
● Require open reduction/internal fixation
threatening trauma. While facial fracture repair may not
● The establishment of normal occlusion is critical, as this is
seem like a priority, it should be performed if the prognosis is
most noticeable to the patient
reasonable as malocclusion could be detrimental to the
– Maxillomandibular fixation (MMF) can be established by
rehabilitation process and delayed repair is significantly more
using dental wear facets as guides. This may be
complicated.
accomplished with arch bars or intermaxillary fixation
(IMF) screws.
● Bone gaps at buttresses, especially those > 5 mm, will not heal
and will require bone grafting
5.6 Complications
● In easily reducible, minimally displaced fractures of the ● Malocclusion: It may be secondary to incomplete initial
maxilla and/or mandible, patients may be treated with 4–6 reduction of the fractures or lack of stabilization with MMF.
weeks of MMF ● Nonunion, malunion, or fibrous union: They require
debridement of the fracture site, possible bone grafting, and
re-fixation.
5.4.3 Surgical Technique ● Infection: If infection is early prior to bony healing and
● Nasal intubation hardware is stable, wash out the wounds and attempt to
– If there are skull base fractures, intracranial injury is salvage the hardware. Plates may be removed after several
possible with blind intubations. Alternatives include months have elapsed and bony union is achieved. In the
submental intubation or tracheostomy. interim, the patient may be treated with long-term,
● Surgical approach to midface fractures is accomplished suppressive antibiotics.
through a gingivobuccal sulcus incision. Orbital fractures can
be approached through lower eyelid (e.g., transconjunctival,
subtarsal) incisions and brow or upper blepharoplasty 5.7 Critical Errors
incisions. A coronal approach may be necessary to access the ● Missed injuries and failure to perform a trauma work-up
NF region. ● Failure to diagnose and treat concomitant facial fractures.
● Occlusion is established with MMF. Watch out for naso-orbito-ethmoid (NOE) fractures which
● Reduction of the fracture: Impacted fractures or difficult require additional reduction and stabilization.
reductions due to early fibrous union may require the use ● Failure to ensure centric occlusion at the time of fracture
Rowe disimpaction forceps. repair
● Fracture stabilization ● Removing hardware too early in face of infection, leading to
– Le Fort I: Plating is performed at the ZM buttress and the loss of reduction
NM buttresses.

19
Case 6 Pediatric Mandible Fractures
Lauren O. Roussel and Albert S. Woo

Case 6 (a–c) A 3-year-old boy presents to the emergency department after falling out of a second story window. He complains of pain in his jaw and
occlusal abnormalities.

21
Facial Fractures

Plain radiography (Mandible Series: anteroposterior (AP),


6.1 Description ●

lateral, oblique, and open mouth Towne's views): This study


● Clinical photo reveals intraoral injury including dental injury is of limited benefit in younger patients whose skeletons are
and crossbite of the mandible with evidence of mandibular less calcified than adults.
widening
● Computed tomography (CT) scan reveals left parasymphyseal
fracture with mild displacement 6.3 Patient Counseling
● Bilateral subcondylar fractures: right side with loss of vertical ● Families must be counseled on progressive issues with facial
height and left with minimal angulation without loss of growth: Vertical growth of the mandible occurs in the
height condylar region and may be disrupted in injuries to this area.
Patients must therefore be followed in the long term to assess
facial growth.
6.2 Work-Up ● Other complications particularly associated with condylar
6.2.1 History and Physical Examination injuries include ankylosis of the mandible and arthritis of the
temporomandibular region.
● Complete trauma evaluation ● Dental work and orthodontics may be necessary even after
– Evaluate airway, breathing, and circulation appropriate management of fractures.
– Rarely, intubation may be necessary for airway protection
– Cervical spine evaluation
– Assessment of associated injuries 6.4 Treatment
● History and examination are more challenging in children
● Treat pediatric fractures within 7 days due to rapid healing.
due to limited maturity and inability to articulate subjective
● Perfect occlusion is less important due to remodeling and
complaints
functional adaptation of the mandible with continued
● May obtain history from parent or guardian
growth. Maxillomandibular fixation (MMF) is frequently a
– Beware of child abuse/neglect. Concern for these issues
good option.
must be reported to child protective services.
● Avoid injury to permanent tooth roots, which may not be
● Inspect face for asymmetry, areas of tenderness, swelling, or
erupted yet. It is critically important to visualize dental roots
ecchymosis
on radiographs prior to surgical intervention.
– Occlusion may be difficult to determine in younger patients
as teeth are widely spaced and naturally mobile
– Chin laceration may indicate superiorly directed force 6.4.1 Acrylic Dental Splints (Gunning
consistent with condylar fractures
– Deviation of jaw opening or limited mobility
Splints)
– Intraoral examination to evaluate for lacerations or ● If age < 2 years, deciduous teeth have not completely erupted
hematomas and cannot tolerate arch bars. Open reduction/internal
● State of dentition fixation with plates also risks injury to tooth roots.
– Children aged 6–12 years will present with various states of ● Acrylic splint may be fixed in place to both occlusal surfaces
mixed dentition. Younger children will have permanent with circum-mandibular, circum-piriform, and other types of
tooth roots deep to their primary dentition. These factors fixation wires (see ▶ Fig. 6.1).
will critically influence a surgeon's options for reconstruction
of injuries.
– Presence of mixed dentition with teeth in varying stages of
6.4.2 Erich Arch Bars for
eruption makes evaluation of malocclusion challenging in Maxillomandibular Fixation
pediatric patients.
● Preferred treatment for pediatric mandible fractures, if
– Assess for fracture, stability, tooth root exposure, and dental
possible
caries.
● If adult molars are present or deciduous molars are
reasonably solid, they may be used to stabilize arch bars.
6.2.2 Pertinent Imaging or Diagnostic (Children aged 2–5 years have reasonably solid deciduous
molars.)
Studies – Circum-mandibular wires may be used to additionally
● High-resolution maxillofacial CT: Gold standard for stabilize lower arch bar
evaluation of facial trauma. Three-dimensional – Maxillary arch bar may be fixated anteriorly to piriform
reconstructions may assist in evaluating injury. It may require aperture or anterior nasal spine to prevent injury to
sedation in young children. developing tooth buds
● Panorex: Panorex requires patient's cooperation and the ● In older children with adult dentition, standard MMF may be
patient must be upright for the study. It permits visualization established with arch bars wired to the teeth or
of entire mandible and localization of developing permanent intermaxillary fixation (IMF) screws anchored to bone
dentition. Towne's view is used to improve evaluation of
condyles, if necessary.

22
Pediatric Mandible Fractures

● Choice of osteosynthesis material


– Titanium plates remain gold standard for ORIF. For younger
children, one must consider removal after fractures are
healed to prevent growth disturbance over time.
– Bioresorbable may be considered but is not FDA approved
for load-bearing surfaces.
● In bilateral subcondylar fractures, the vertical height of the
mandible is lost
– At least one of the fractures must be repaired to re-
establish the height of the mandible and prevent eventual
collapse resulting in a permanent open bite
● Surgical access (see Chapter 3)
– Intraoral approach through a gingivolabial sulcus incision
for symphyseal and body fractures
– Intraoral approach with percutaneous pinning versus
Risdon (external) approach for angle fractures

6.5 Complications
● Abnormalities of occlusion or dentition: Suggestive of
inadequate initial reduction or stabilization
● Ankylosis of the TMJ joint: Especially notable when the
condyle is fractured and the patient is treated with long-term
immobilization
– Must remove MMF within 2 weeks to minimize risk of this
complication
– Guiding elastics may be used thereafter
Fig. 6.1 Options for wiring techniques, including circum-mandibular,
● Growth disturbance: May be unavoidable due to severity of
circum-piriform, circum-orbital, and circum-zygomatic wires. At least injury, especially in the subcondylar region
three points of fixation should be utilized to optimize stability of the
fixation.
6.6 Critical Errors
● Failure to carefully consider the possibility of child abuse in
6.4.3 Timing of Immobilization unusual situations and appropriately report this to child
protective services
● Children heal faster due to increased ossification capability of
● Inadequately addressing subcondylar fractures and
periosteum.
mandibular height, resulting in permanent open bite and
● (Sub) condylar fractures: When fractures to the condyle are
malocclusion
present, immobilization should be minimized (roughly 2
● Plating the mandibular fractures without regard to
weeks) to decrease the risk of ankylosis of the
permanent tooth roots or inferior alveolar nerve injury
temporomandibular joint (TMJ).
● Neglecting to discuss with family the possibility of growth
● Body and angle fractures: 3–4 weeks
disturbances related to subcondylar fractures
● Parasymphyseal fractures: 4–6 weeks
● Failure to release MMF and confirm the presence of normal
occlusion with condyles seated appropriately in the glenoid
6.4.4 Open Reduction and Internal fossa upon completion of the case
Fixation (ORIF)
● Presence of developing tooth roots is critical in operative
planning. Screws must be placed carefully to avoid injury to
permanent tooth roots.

23
7 Malignant Skin Lesion 27
Section II
8 Lip Reconstruction 31
Facial Reconstruction 9 Nose Reconstruction 35

10 Eyelid Reconstruction 39

11 Ear Reconstruction 43

12 Cheek Reconstruction 47

13 Acquired Facial Paralysis 51

14 Congenital Facial Paralysis 55

II
Case 7 Malignant Skin Lesion
W. Kelsey Snapp, Albert S. Woo, and Antonio Cruz

Case 7 A 68-year-old male presents with a lesion on the left side of his face. He reports that the growth has been present for over a year, is painless,
and occasionally bleeds.

27
Facial Reconstruction

particularly in melanoma, where the depth of a tumor is


7.1 Description critical to prognosis
● Large, eroded, pearly pink plaque with telangiectasia located – Shave biopsy is an acceptable method of assessment if the
on the left temple, involving the temporal hairline, and concern is for a nonmelanoma skin cancer (basal or
demonstrating evidence of ulceration. squamous cell carcinomas)
● There is significant concern for malignancy. ● Imaging may be necessary after an initial diagnosis is
established, especially if deeper tissue invasion is a concern or
in lesions of the scalp where one is suspicious of possible
7.2 Work-Up bony penetration
– Magnetic resonance imaging (MRI): Useful adjunct to
7.2.1 History determine extent of tumor and lymph node status in cases
● Length of time the lesion has been present of aggressive tumor histology (e.g., perineural invasion)
● Associated symptoms: Pain, itching, bleeding, and
hyperkeratosis
● Sun exposure history, tanning bed use
7.3 Patient Counseling
● History of facial surgery, especially surrounding the nose ● Counsel patient about the diagnosis, prognosis, and
– Previous surgery may affect reconstructive options reconstructive options
– Primary tumor versus recurrent tumor ● Patients must understand that the size of the final defect
○ Previous treatment with cryotherapy, topical
cannot be anticipated in skin cancer until resection has been
medications, electrodesiccation and/or curettage completed
○ Scar in area of malignancy ● Though the goal is to achieve the most aesthetic results, it is
● Complicating comorbidities: Cardiopulmonary/peripheral reasonable for some patients (e.g., medically complicated,
vascular disease, diabetes, tobacco product use, steroid use, elderly) to opt for simpler skin graft reconstruction or no
anticoagulation, and chemotherapy reconstruction at all
● Supplementation with vitamin E, fish oil, krill oil, omega, and ● Skin graft or skin substitute may be a reasonable temporizing
garlic option until definitive pathology confirms negative margins
● History of radiation, immunosuppression (organ transplant
recipient, AIDS)
● Personal or family history of skin cancer 7.4 Treatment
● Genetic conditions: Xerodermapigmentosum, Gorlin's
(nevoid basal cell) syndrome, albinism 7.4.1 Nonsurgical Treatment
● Topical 5-fluorouracil or imiquimod, cryosurgery,
7.2.2 Physical Examination electrodesiccation and curettage, and radiation therapy are all
treatment options for low-risk basal cell carcinoma (BCC) and
● Detailed evaluation of affected area and surrounding face/
squamous cell carcinoma (SCC)
neck to assess the lesion with or without dermatoscopic
– All associated with higher recurrence rates than surgical
analysis/magnification
excision
– Characteristics of affected area (e.g., hair bearing, adjacent
– Primarily for patients in which excision is contraindicated
skin laxity, and pigmentation)
or impractical
● Skin lesion findings
– Size, color, shape of lesion, skin irregularity, hyperkeratosis,
and ulceration
7.4.2 Excision
– Confirm absence of involvement of deeper structures
(e.g., parotid and facial nerve) ● Consider Mohs micrographic surgery, if available and meets
● Lymph node examination to assess for signs of metastatic the Mohs Appropriate Use Criteria (▶ Table 7.1)
disease – Now recommended by the American Academy of
● Full body integument examination Dermatology for all high-risk BCC and SCC
– Allows examination of ~100% of surgical margins; highest
cure rates
7.2.3 Diagnostic Studies – Board examiner may require that you excise the lesion
● Establish a diagnosis: If patient presents without previous yourself
treatment, a biopsy should be performed. ● BCC (▶ Table 7.2)
● Full-thickness incisional versus excisional biopsies may be – Most common form of skin cancer
performed – 4 mm margins recommended for low-risk lesions
– If the concern is for an atypical pigmented lesion, an – 7 mm for more aggressive subtypes (morpheaform, infiltrative)
excisional or shave biopsy should be performed to evaluate – Curettage can help assist in defining the subclinical margin.
as much of the lesion as possible to minimize sampling Surgical margin should be taken beyond that.
error ● SCC (▶ Table 7.2)
– Avoid shave biopsies of a portion of a pigmented lesion as – 4 mm margin if low-risk lesion (< 2 cm, well-differentiated,
they may lead to incomplete assessment of the lesion, not invasive)

28
Malignant Skin Lesion

Table 7.1 Indications for Mohs micrographic surgery


BCC or SCC with indistinct borders
Recurrent or incompletely excised BCC or SCC
Lesions located in high-risk areas/involving embryonic fusion planes: Examples: eyelids, nose, ear, nasolabial folds, upper lip, vermilion border, columella,
periorbital region, temple, preauricular and postauricular areas, scalp
Lesions located in areas of cosmetic or functional importance: Examples: genital, anal, perianal, hand, foot, and nail units
Rapidly growing or aggressive lesions (> 2 cm diameter)
Lesions with aggressive histologic appearance: Examples: Morpheaform or metatypical BCC, undifferentiated or poorly differentiated SCC
Lesions with perivascular or perineural invasion
Infiltrating, adenoidal, or multicentric lesions
Tumors located in previously irradiated location
Lesions in immunosuppressed patients
Lesions in patients with basal cell nevus syndrome

Abbreviations: BCC, basal cell carcinoma; SCC, squamous cell carcinoma.


Adapted from Shang BJ, Meyers A, Rosen CA, et al. 2010. Mohs surgery. Emedicine. https://emedicine.medscape.com/article/2212475-overview

Table 7.2 Standard margin recommendations for different skin cancer types
Basal cell cancer Squamous cell cancer Melanoma
(Breslow thickness)

Standard: 4 mm < 2 cm, well-differentiated: 4 mm In situ: 5 mm


Aggressive subtypes: 7 mm > 2 cm, invasive to fat, high-risk location: 6 mm < 1 mm: 1 cm
1–2 mm: 1–2 cm
> 2 mm: 2 cm

– 6 mm margin if high-risk lesion (> 2 cm, invasive into fat, or 7.4.3 Reconstruction
located on the central face, ears, scalp, hands, feet, or
genitalia) (See Chapters 8 to 12)
○ Further high-risk criteria indicating need for adjuvant ● Goals: Restoration of function (oral competence, speech) and

radiation therapy: Size > 2 cm, perineural invasion, optimization of aesthetics


● Should be delayed until negative margins are confirmed on
invasion into deep tissues (fat, muscle, bone), and
aggressive histologic subtype (poorly differentiated) final pathology
● Local wound care, Integra, or temporizing skin graft in
● Melanoma: Margins determined by Breslow thickness
(▶ Table 7.2) interim
● Fresh frozen pathologic evaluation cannot ensure negative
– In situ: 5 mm margins
– < 1 mm: 1 cm margin margins
● Consider facial subunits (see Chapter 12, ▶ Fig. 12.1)
– 1–2 mm: 1–2 cm margin
● Surgical pearls
– > 2.1 mm: 2 cm
– Sentinel lymph node biopsy: Consider for 0.8–1 mm – Mark or tattoo landmarks prior to injecting local anesthetic
thickness or < 0.8 mm with ulceration. Recommended (e.g., vermilion border)
for > 1 mm thickness (Surgical Oncology consultation) – Replace “like with like”: Need to reconstruct all appropriate
– Stage III melanoma (positive lymph nodes) may require layers (i.e., skin, mucosa, and cartilage/bone)
immunotherapy (Medical Oncology consultation)
● Final excision margins
– Following excision, additional margins may be sent as fresh 7.5 Complications
specimens to confirm absence of residual disease. However,
fresh frozen pathologic evaluation is notoriously unreliable ● Recurrent cancer or positive margins: Re-excision is
and cannot ensure negative margins. necessary
– Most reliable method of confirming negative margins is – Reconstruct only after negative margin is achieved
with permanent sections. Unfortunately, these sections may – Mohs surgery is indicated in all recurrent tumors
take over a week for final results. ● Wound dehiscence, partial flap necrosis: Treat with local
○ Some laboratories are equipped for rapid overnight wound care or readvance flap if adequate laxity is present
preparation that may assist in staged excision with ● Infection: Antibiotics and conservative debridement, if
margin control (i.e., “slow Mohs”). appropriate

29
Facial Reconstruction

7.6 Ethical Considerations 7.7 Critical Errors


● The patient's age, quality of life, and medical comorbidities ● Failure to perform appropriate biopsy when there is concern
should all be weighed heavily when considering treatment of for skin cancer
malignant skin lesions ● Inadequate resection of malignancy
● BCC is rarely fatal. However, it can cause significant ● Reconstruction prior to ensuring negative margins
disfigurement over time if not properly treated

30
Case 8 Lip Reconstruction
Dardan Beqiri and Albert S. Woo

Case 8 A 53-year-old female seeks treatment following Mohs excision of basal cell carcinoma involving the left upper lip.

31
Facial Reconstruction

8.1 Description 8.4.1 Excision


● Partial thickness central upper lip defect involving roughly (See Chapter 7, ▶ Table 7.1)
● Basal cell carcinoma
1/4 of the upper lip
– Several tissues are involved: vermilion, white roll, and – 2–5 mm margin standard
cutaneous lip – 7 mm for more aggressive subtypes
● Squamous cell carcinoma
– Multiple key anatomic structures affected: Cupid's bow,
philtral dimple, and left philtral column – Most common skin cancer of lip, > 90% occur on lower lip
● Orbicularis oris muscle appears intact – 4 mm margin if < 2 cm lesion, well-differentiated, not
invasive
– 6 mm margin if > 2 cm, invasive into fat, or in high-risk
8.2 Work-Up location (central face, ears, scalp, hands, feet,
and genitalia)
8.2.1 History ● Melanoma: Margins determined by Breslow thickness

– In situ: 5 mm margins
● Sun exposure history
– < 1 mm: 1 cm margin
● Personal and family history of skin cancer
– 1–2 mm: 1 cm
● Genetic conditions: Xerodermapigmentosum, Gorlin's
– > 2.1 mm: 2 cm
(nevoid basal cell) syndrome, albinism, and vitiligo
● Final margins

– Following excision, additional margins may be sent as fresh


8.2.2 Physical Examination specimens to confirm absence of residual disease. However,
fresh frozen pathologic evaluation is notoriously unreliable
● Detailed examination of lips and oral cavity to assess the
and cannot ensure negative margins.
lesion or defect
– Most reliable method of confirming negative margins is
● Characterize findings associated with skin lesion (if present):
with permanent sections. Unfortunately, these sections may
Size, color, shape of lesion, skin irregularity, and
take several days for final results.
hyperkeratosis
● Lymph node examination to assess for signs of metastatic
disease 8.4.2 Reconstruction
● Goals: Restoration of function (oral competence, speech) and
8.2.3 Diagnostic Studies optimization of aesthetics
● Reconstruction should be delayed until negative margins are
● If patient presents with a lesion prior to a resection, a biopsy
confirmed on final pathology
should be performed at the time of evaluation to establish a
– Local wound care with dressing changes can be used in the
diagnosis
interim
● Full-thickness incisional versus excisional biopsies may be
– Integra or skin graft may also be employed for temporary
performed.
coverage
– Avoid shave biopsies as they may lead to incomplete
● Surgical pearls
assessment of the lesion, particularly in melanoma, where
– Mark or tattoo landmarks prior to injecting local anesthetic
the depth of a tumor is critical to prognosis.
(e.g., vermilion border)
– Primary closure: Upper lip ≤ 1/4 defect, Lower lip ≤ 1/3
8.3 Patient Counseling defect
– Larger defects may require Abbe (lip switch) flap or other
● Patients must understand that the size of the final defect reconstructive procedures
cannot be anticipated in skin cancer until resection has been ● Mucosal/vermilion defects: Replace “like with like”
completed. – Mucosal advancement: Ideal for partial thickness defects
● While flap reconstructions may yield the most aesthetic – Vermilion advancement: Musculo-vermilion flaps for small
results, it is reasonable for some patients (e.g., medically full-thickness defects
complicated, elderly) to opt for simpler skin graft – Facial artery musculomucosal (FAMM) flap
reconstructions or even no reconstruction at all. ○ Flap of intraoral mucosa useful for correction of long

vermilion-only defects
○ Based on the facial artery: Superior or inferior pedicle
8.4 Treatment – Vermilion lip switch (Abbe flap)
○ Utilized for larger defects, primarily of the upper lip
● Consider Mohs surgery referral, if available
○ Requires second stage (in 2–3 weeks) for division of flaps
– Allows examination of ~100% of surgical margins, resulting
in highest cure rates – Tongue flap
○ Anteriorly based, from the ventral surface
– Board examiner may require that you excise the lesion
○ Requires second stage division
yourself

32
Lip Reconstruction

Fig. 8.1 Abbe flap. (a) The Abbe flap is elevated from the central lower lip. For central upper defects, it is elevated to the labiomental fold. For lateral
defects, it continues through the central chin pad. (b) It is inset onto the columella, above the columellar base, with the extensions to the nasal sill.
(c) The flap is divided and inset at 2 weeks.

● Upper lip full-thickness defects: Focus on restoration of


landmarks
– Primary closure, if defect ≤ 1/4 of lip
– Full-thickness skin graft: When muscle layer is not involved
○ Reconstruction of full aesthetic subunit preferred

– Abbe flap (▶ Fig. 8.1)


○ Lip switch from lower to upper lip, designed as half the

width of the defect


○ Can correct defects involving 1/3–1/2 of the lip

○ Second stage division at 2–3 weeks

– Karapandzic flap (▶ Fig. 8.2)


○ Rotational flap primarily for lower lip defect involving

1/3–2/3 of lip; utilized for upper lip as well


○ Restores oral competence (preserves neurovascular

pedicle), but may result in microstomia (especially in


larger defects)
● Lower lip full-thickness defects
– Primary closure if defect ≤ 1/3 of lip
– Reverse Abbe flap: Upper to lower lip switch
– Schuchardt procedure (similar to Bernard, Webster
procedures): For 1/3 to 2/3 lip defects
○ Performed with barrel-shaped incisions along

labiomental crease
○ May be combined with bilateral lip switch to prevent

microstomia
– Karapandzic flap: As noted above (see upper lip full-
thickness defects)
● Commissure defects
– Estlander flap (▶ Fig. 8.3): Lip switch involving the
commissure
○ Useful for full-thickness defects 1/2–2/3 of the lip

● Total lip reconstruction


– Free radial forearm flap with palmaris longus sling for
support

Fig. 8.2 (a, b) Karapandzic flap.

33
Facial Reconstruction

8.5 Complications
● Recurrent cancer: Re-excision is necessary
● Wound dehiscence, partial flap necrosis: Treat with local
wound care
● Microstomia: May be preventable with postoperative
splinting
– Abbe flap (single or bilateral) may be useful adjuncts to
primary flap procedure
● Oral incompetence: Orbicularis reconstruction is important
for prevention

8.6 Critical Errors


● Failure to perform appropriate biopsy when there is concern
for skin cancer
● Inadequate resection
● Reconstruction prior to ensuring negative margins
● Failure to repair orbicularis oris musculature
● Failure to address all three layers of missing tissue (mucosa,
muscle, and skin) or ignoring critical structures (philtrum,
Cupid's bow, and white roll)

Fig. 8.3 Estlander flap.

34
Case 9 Nose Reconstruction
Albert S. Woo

Case 9 A 65-year-old male presents to the clinic with dissatisfaction with his appearance following Mohs excision of a basal cell carcinoma on the tip of
the nose and reconstruction with skin graft.

35
Facial Reconstruction

9.1 Description 9.4 Treatment


● Evidence of amputation of the nasal tip, involving skin, ● Consider Mohs surgery consultation, if available
cartilage, and mucosa – The technique allows examination of entire surgical margin
● The structural framework and nasal lining have been violated and cure rates of up to 98%
● Defect primarily involves one nasal subunit, the tip, but also – Board examiner may require that you excise the lesion
looks to have some extension into the columella yourself
● In melanoma, surgical oncology consultation is
recommended
9.2 Work-Up
9.2.1 History 9.4.1 Excision
● Sun exposure history (See Chapter 7, ▶ Table 7.2)
● History of facial surgery, especially surrounding the nose ● Basal cell carcinoma: 2–5 mm margin

– Previous surgery may affect reconstructive options ● Squamous cell carcinoma

● Medical issues that may affect healing: History of radiation, – 4 mm margin if low risk: < 2 cm lesion, well-differentiated,
chemotherapy, immunosuppression, smoking, and diabetes not invasive
● Personal and family history of skin cancer – 6 mm margin if high risk: > 2 cm, poorly differentiated,
invasive into fat
● Melanoma: Excision margins determined by Breslow
9.2.2 Physical Examination thickness
● Detailed evaluation of nose and surrounding face to assess – In situ: 5 mm margin
the lesion or defect – ≤ 1 mm: 1 cm margin
● Characterize findings associated with skin lesion (if present): – > 1–2 mm: 1–2 cm margin
Size, color, shape of lesion, skin irregularity, and – > 2 mm: 2 cm margin
hyperkeratosis – Sentinel lymph node biopsy: Consider for 0.8–1 mm
● If resected, evaluate for size and shape of the defect, nasal thickness or < 0.8 mm with ulceration. Recommended
subunits involved, depth of excision (cartilage or mucosal for > 1 mm thickness. (Surgical Oncology consultation)
involvement), presence of perichondrium on cartilage, laxity – Stage III melanoma (positive lymph nodes) may require
of surrounding skin, and involvement of nostril sill immunotherapy (Medical Oncology consultation)
● Final excision margins
– If there is cartilage loss, assess for donor sites: nasal
septum, ear, rib – Fresh frozen pathologic evaluation is notoriously unreliable
and cannot ensure negative margins
– Most reliable method of confirming negative margins is
9.2.3 Diagnostic Studies with permanent sections. Unfortunately, these sections may
● If patient presents initially without previous treatment, a take several days to obtain.
biopsy should be performed at the time of evaluation to
establish a diagnosis 9.4.2 Reconstruction
– Full-thickness incisional versus excisional biopsies may be
performed. ● Do not reconstruct until tumor-free margin is confirmed
– Avoid shave biopsies as they may lead to incomplete – Coverage with dressing, Integra placement, and temporary
assessment of the lesion, particularly in melanoma, where skin graft are reasonable temporizing measures
the depth of a tumor is critical to prognosis. ● There are nine nasal subunits (▶ Fig. 9.1)
– 3 midline subunits (dorsum, tip, and columella)
– 3 paired lateral subunits (sidewalls, alae, and soft triangles)
9.3 Patient Counseling – Scars best placed at borders of subunits for optimal
aesthetics
● The nose is a complex three-dimensional structure and some
deformity should be expected following excision, even with
the best reconstruction. 9.4.3 Reconstructive Options
● While flap reconstructions may yield the most aesthetic
● Primary closure
results, it is reasonable for some patients (e.g., medically
– Small defects < 0.5 cm in mobile skin areas
complicated, elderly) to opt for simpler skin graft
● Secondary intention: Coverage with moist dressing
reconstructions or even no reconstruction at all.
(e.g., petroleum jelly)
● Skin graft may be a reasonable temporizing option until
– Small, superficial defects over concave or planar surfaces in
definitive pathology confirms negative margins.
nonmobile areas not amenable to direct closure
(e.g., medial canthal region, nasal sidewalls, alar groove)

36
Nose Reconstruction

● Local flaps: Small (< 2 cm) superficial defects – Use non hair-bearing skin above the clavicles for best color
– Transposition (Banner) flap: Useful for smaller defects match (e.g., posterior auricular, pre-auricular,
(< 1.2 cm) supraclavicular, or forehead).
– Bilobed flap (▶ Fig. 9.2): May assist with slightly larger ● Composite chondrocutaneous graft from ear
defects (< 1.5 cm) – Small (< 1 cm) defects of alar rim and columella involving
– Dorsal nasal (Miter) flap (▶ Fig. 9.3): Dorsum and tip defects cartilage and mucosa or skin
(< 2 cm) ● Regional flaps
● Full-thickness skin graft – Nasolabial flap: Based on perforators from facial and
– Useful when local flaps are not a good option and patients angular arteries
opt out of more extensive procedures. – Forehead flap: Based on supratrochlear artery
– Skin grafts cannot cover cartilage without perichondrium or ○ Two or three stages, depending on severity of defect

bone without periosteum. Such defects must be covered by ○ Workhorse for larger (> 1.5 cm) defects

a vascularized flap. – Subunit principle: If a defect comprises > 50% of the tip or
alar subunit, the residual normal skin should be discarded
and the entire subunit resurfaced
● Full-thickness defects: Require replacement of outer skin,
framework, and lining
– Framework donor sites: Nasal septum, ear, and costal
cartilage; bone graft
– Nasal lining options: Advancement flaps from residual nasal
lining, turnover flap of adjacent skin, skin graft over
vascularized bed, septal mucoperichondrial flaps, folded
forehead flap, free flap (radial forearm, anterolateral thigh
(ALT), dorsalis pedis)

9.5 Ethical Considerations


● Patients may present with complaint of a poor result
following reconstruction by a different surgeon. Resist the
temptation to criticize the work of another surgeon unless it
is clearly outside accepted standards of treatment.
– Remind patient of complexity of the nose and difficulty to
guarantee an excellent reconstruction
Fig. 9.1 Aesthetic subunits of the nose.

Fig. 9.2 Bilobed flap.

37
Facial Reconstruction

9.6 Complications
● Recurrent cancer: Re-excise and reconstruct only after
negative margin is achieved
● Wound dehiscence, partial flap necrosis: Local wound care

9.7 Critical Errors


● Failure to biopsy suspicious skin lesion
● Inadequate resection
● Reconstruction prior to ensuring negative margins
● Inability to describe the defect (nasal subunits)
● Failure to reconstruct all necessary layers of the nose
(mucosa, cartilage framework, and skin)
● Poor flap design or inability to draw flap for reconstruction

Fig. 9.3 Dorsal nasal flap.

38
Case 10 Eyelid Reconstruction
Raman Mehrzad, Antonio Cruz, and Daniel Kwan

Case 10 An 87-year-old male presents with a defect of the medial aspect of the left lower eyelid following Mohs excision of basal cell carcinoma.

39
Facial Reconstruction

10.1 Description 10.2.3 Diagnostic Studies


● Partial-thickness defect of medial aspect of left lower eyelid ● Establish the diagnosis: If not done earlier, a biopsy should be
– < 25% of eye lid, but include eyelid margin and possibly the performed at initial visit
inferior lacrimal duct ● Confirm negative pathologic margins following resection
● Contiguous skin and muscle defect extending to cheek–eyelid prior to attempting significant reconstruction
junction ● Magnetic resonance imaging (MRI): Useful adjunct to
● Possibility of medial canthal tendons involvement determine extent of tumor and lymph node status in cases of
aggressive tumor histology (e.g., perineural invasion or
deeply invasive tumors)
10.2 Work-Up
10.2.1 History 10.2.4 Consultations
● History of ophthalmologic conditions, including dry eye, ● Ophthalmology: For visual acuity and field testing; Schirmer
excessive tearing, vision problems, thyroid eye disease, and test postoperatively
Bell's palsy
● Personal or family history of skin malignancy or significant
sun exposure 10.3 Patient Counseling
● History of previous periorbital trauma, surgery, or facial
● Counsel patient about the diagnosis, prognosis, and
radiation
reconstructive options.
● Ophthalmic medications
● Set reasonable expectations for reconstructive outcome.
● Discuss risk of ectropion following reconstructive procedures
10.2.2 Physical Examination and possible need ofcanthoplasty or canthopexy for eyelid
support.
● Divide the periocular region into “zones” (▶ Fig. 10.1)
● Determine layers that have been lost
– Full or partial thickness
– Partial thickness: Skin, muscle, tarsus, and mucosa 10.4 Treatment
– Full thickness: < 25%, 25–50%, or > 50% of the eyelid For treatment protocol, refer to ▶ Fig. 10.2
● Evaluate laxity of the eyelid, canthal support, and any
involvement of lacrimal system
● Identify viable elements available for reconstruction 10.4.1 Key Points
(e.g., skin, muscle, tarsus, conjunctiva)
● Vascularity, scarring, radiation damage, and severe actinic ● Need to reconstruct all missing layers
damage should be evaluated for the surrounding periorbital – Anterior lamella: Outer covering of eyelid comprising the
● Evaluate eyelid function skin and orbicularis muscle. Local or regional tissue flaps are
preferred for best color and texture match.
– Middle lamella: Structural support layer composed of tarsus
and septum. Proper anchoring of the eyelid along the
medial and lateral canthus is essential for proper function
and support.
– Posterior lamella: Inner lining of conjunctiva.
Reconstruction of the posterior lamella is best carried out
using grafts from the tarsus, mucosa, or hard palate.
● Full-thickness defects (i.e., all three lamellae) must use at
least one flap for reconstruction
– Cannot reconstruct full-thickness defects with grafts alone.
A flap is necessary to provide vascularity to the graft.

10.4.2 Zone I (Upper Eyelid)


● Partial thickness
– < 50% of eyelid width: Primary closure with local tissue
advancement
– > 50% of eyelid width: Full thickness skin graft (FTSG) from
contralateral upper eyelid
● Full thickness
Fig. 10.1 Zones of the eyelid. – < 25%: Primary closure with possible canthotomy and/or
cantholysis

40
Eyelid Reconstruction

Fig. 10.2 Treatment protocol. (Reproduced with permission from Spinelli H, Jelks GW. Periocular reconstruction: a systematic approach. Plast Reconstr
Surg 1993;91(6):1017–1024.)

– 25–75%: Reconstructive options ○ Tarsoconjunctival (Hughes) flap with FTSG

○ Tarsoconjunctival (Cutler-Beard) flap with FTSG – 50–75%: Tarsoconjunctival (Hughes) flap with FTSG
○ Composite graft (e.g., nasal septal cartilage–mucosa) and – > 75%: Composite graft (i.e., hard palate graft versus septal
local myocutaneous flap (e.g., Tripier, Fricke or Tenzel cartilage/mucosa) with cheek advancement flap
flaps)
– > 75%: Lower eyelid transposition (Mustarde) flap:
Commonly known as a lower eyelid reconstruction with 10.4.4 Zone III (Medial Canthus)
cheek advancement, an eyelid switch/share technique and ● Intubate the lacrimal system
cheek advancement for donor site closure ● Local flaps from upper eyelid or glabella
● Healing by secondary intention is acceptable (in areas of
10.4.3 Zone II (Lower Eyelid) concavity)
● If detached, medial canthus should be reconstructed posterior
● Partial thickness and superior to its original location
– < 50%: Primary closure with local tissue advancement
– > 50%: Reconstructive options
○ FTSG from contralateral upper eyelid

○ Upper eyelid transposition (Fricke, Tripier) flap


10.4.5 Zone IV (Lateral Canthus)
● Full thickness ● Cheek advancement flap or FTSG
– < 50%: Reconstructive options ● Lateral canthal supporting procedure: Canthopexy or
○ Primary closure with canthotomy and/or cantholysis canthoplasty

41
Facial Reconstruction

10.4.6 Zone V (Periorbital) or needed. Vertical vector will pull the eyelid down, increasing
the risk of ectropion.
MultipleZone Defect ● Entropion: Deficient posterior lamella from scarring
● Corneal protection is priority ● Corneal abrasion: Lubricate eye and protect with corneal
● Immediate coverage with myocutaneous flap or FTSG shield. Avoid contact of suture material with cornea.
● Definitive reconstruction may need to be staged ● Suboptimal aesthetic outcome, ptosis, infection, and
scarring

10.5 Ethical Considerations


10.7 Critical Errors
● Follow guidelines on reconstructive options based on the size
of defect and not based on reimbursement ● Failure to identify extent of defect (e.g., anterior and/or
● Consider the patient's age, past medical history, and other posterior lamella)
factors before offering surgery ● Reconstruction prior to confirming negative margins
● Inadequately reconstructing all lamellae appropriately
● Not providing adequate vasculature for graft healing
10.6 Complications – Cannot place graft on graft (FTSG + chondromucosal
composite graft)
Complications are seen in approximately 10% of cases and may ● Incomplete evaluation of lacrimal system and canthal
require secondary surgery to correct eyelid malposition
involvement
● Ectropion: All flaps should be designed with tension oriented

horizontally. Perform lateral canthal support procedure when

42
Case 11 Ear Reconstruction
Dardan Beqiri and Albert S. Woo

Case 11 A 70-year-old male presents with lesion of the left ear, which has been present for over a year.

43
Facial Reconstruction

Fresh frozen pathologic evaluation cannot ensure


11.1 Description ○

negative margins
● Fungating and ulcerative mass involving the upper two-thirds – Local wound care, Integra, or temporizing skin graft in interim
of the left ear, including the helix, scapha, and antihelix ● Antibiotic coverage
● Surrounding edema, induration, and discoloration of the skin – Sulfamylon (topical) and fluoroquinolones (systemic) have
● Lesion has high suspicion for malignancy excellent cartilaginous penetration

11.2 Work-Up 11.4.1 Reconstructive Ladder


● Primary closure + /– Tanzer's excision patterns (▶ Fig. 11.1)
11.2.1 History ● Wedge resection: Defect < 1.5 cm
● Length of time the lesion has been present ● Skin grafts may be used for superficial defects
● Associated symptoms: pain, scabbing, itching, bleeding, and – Usually harvested from contralateral posterior auricular ear
hyperkeratosis – Skin graft cannot be placed on raw cartilage without
● History of sun exposure overlying perichondrium
● Personal and family history of skin cancer – Exposed raw cartilage must be covered by flap or debrided
● Helical rim advancement (▶ Fig. 11.2)
– Useful for defects of helical rim
11.2.2 Physical Examination – Chondrocutaneous flaps are advanced into the defect
● Thorough examination of the ear, including size and shape of ● Antia-Buch flap (▶ Fig. 11.3)
lesion – Useful for upper 1/3 helical defects
● Characterize findings associated with skin lesion (if present): – The base of the helix is advanced in V-Y fashion and coupled
Size, color, shape of lesion, skin irregularity, ulceration, and with helical rim advancement from below
hyperkeratosis – May result in decrease in height of ear
– Depth of lesion and likelihood for cartilaginous or full- ● Posterior auricular flap: Large flap of skin from behind the
thickness involvement. ear can be advanced to cover the posterior ear
● Lymph node examination – Typically used in a staged fashion for initial coverage
● Full body integument examination followed by additional release and further reconstruction,
possibly with skin graft
● Tubed pedicle flap from postauricular skin
11.2.3 Diagnostic Studies – Useful to reconstruct longer sections of missing helical rim
● If patient presents initially without resection, a biopsy should – Must be divided in second stage
be performed at the time of evaluation to establish a diagnosis. ● Banner flap: Skin flap based on anterosuperior
● Full-thickness incisional versus excisional biopsies may be auriculocephalic sulcus
performed. Avoid shave biopsies. – Combine with contralateral auricular cartilage graft for
● When the likelihood of cancer is high, as in this case, take larger (> 2 cm) defects
multiple biopsies along the periphery. ● Chondrocutaneous transposition flaps: Often require grafting
of donor site
– Converse flap: Contralateral auricular cartilage graft
11.3 Patient Counseling tunneled under skin flap from mastoid, requires second
stage (3 weeks) for division and inset
● Attempts at reconstruction may result in ear deformities, – Orticochea procedure: Based laterally on helix
including a smaller or less projecting ear – Davis flap: Based anteriorly on crus helicis, from conchal bowl
● The poor blood supply of the auricular cartilage puts these ● Temporoparietal fascia (TPF) flap
cases at higher risk of infection, particularly chondritis – Thin flap of TPF may be harvested from scalp based on
● Patients must be made aware that with large malignant superficial temporal artery
lesions, amputation may be a possibility – Large flap (10 × 12 cm) may be elevated for complete
coverage of the ear, if necessary
● Total ear reconstruction (see Chapter 20)
11.4 Treatment – Autologous rib cartilage framework covered with posterior
● For skin malignancies, consider Mohs surgery, if available (see auricular skin or TPF flap and skin graft
▶ Fig. 36.1) – Medpor construct covered with TPF flap and skin graft
– Allows examination of complete surgical margins, resulting ● Complete amputation/avulsion: Replantation if viable blood
in highest cure rates supply is available (e.g., superficial temporal or posterior
– Board examiner may require that you excise it yourself auricular artery)
● Excision (see Chapter 7 for margins required for different ● External auditory canal
types of malignancies) – Maintenance of patency more important than choice of
● Reconstruction coverage
– Should be delayed until negative margins are confirmed on – May use skin graft over vascularized bed
final pathology – Requires use of stent or splint for 6 months

44
Ear Reconstruction

Fig. 11.1 (a–d) Wedge resection and primary


closure with excision of accessory triangles.
(Source: Unit Defects. In: Day T, Farrior E, Frodel J
et al., ed. Facial Plastic Surgery. The Essential
Guide. Thieme; 2005.)

Scar contracture leading to blockage of external auditory canal


11.5 Complications ●

– Reconstruction and long-term stenting necessary


● Recurrent cancer: Re-excise and await confirmation of ● Cartilaginous deformities
negative margins prior to considering definitive
reconstruction
● Wound dehiscence, partial flap necrosis: Local wound care 11.6 Critical Errors
● Infection/chondritis ● Failure to perform appropriate biopsy to establish diagnosis
– Antibiotics: Ciprofloxacin has excellent cartilage ● Inadequate resection of malignant lesion
penetration ● Reconstruction prior to ensuring negative margins
– Debridement if appropriate ● Skin grafting on top of raw cartilage without perichondrium
● Hematoma/cauliflower ear: Prompt incision, drainage, and ● Inadequate dressing: Bolster, suction drain to prevent hematoma
bolster application ● Failure to stent the external auditory canal

45
Facial Reconstruction

Fig. 11.2 (a–d) Helical rim advancement. Incision


is made in the scapha and a chondrocutaneous
flap is advanced into the defect in the helical rim.
(Source: Ears. In: Sherris D, Larrabee Jr W, ed.
Principles of Facial Reconstruction. A Subunit
Approach to Cutaneous Repair. 2nd Edition.
Thieme; 2009.)

a b

c d

Fig. 11.3 (a–c) Antia-Buch flap. The base of the helix is advanced posteriorly in a V-Y fashion.

46
Case 12 Cheek Reconstruction
Albert S. Woo

Case 12 A 63-year-old female presents to the operating room after Mohs excision of lentigo maligna melanoma of the right cheek.

47
Facial Reconstruction

Facial subunit involvement (▶ Fig. 12.1)


12.1 Description ●

– Reconstruction based on correction of facial subunits


● Following resection, roughly 2 cm full-thickness defect of the – Cheek may be considered one large subunit or divided into
right cheek, abutting the inferolateral border of the right zones
lower eyelid
● Does not appear to violate superficial musculoaponeurotic
system (SMAS) into deeper structures
12.2.3 Diagnostic Studies
● Moderate skin laxity on examination ● If patient presents initially without resection, a biopsy should
be performed to establish a diagnosis.
● Full-thickness incisional versus excisional biopsies may be
12.2 Work-Up performed. Avoid shave biopsies.

12.2.1 History
● Malignancy: Timeline of presentation 12.3 Patient Counseling
● Sun and environmental exposure history ● Patients must understand that some degree of asymmetry is
● Personal and family history of skin cancer to be expected following reconstruction as the surgeon must
● Genetic conditions: Xeroderma pigmentosum, Gorlin's somehow make up for the absence of tissue.
(nevoid basal cell) syndrome, albinism ● While flap reconstructions may yield the most aesthetic
● Complicating comorbidities: Cardiopulmonary/peripheral results, it is reasonable for some patients (e.g., medically
vascular disease, diabetes, tobacco/steroid use, previous complicated, elderly) to opt for simpler skin graft
surgery or irradiation, anticoagulation, transplant recipient reconstructions or even no reconstruction at all.

12.2.2 Physical Examination 12.4 Treatment


● Lymph node examination to check for metastatic disease
● Reconstruction should be delayed until negative margins are
● Wound characteristics (hair bearing areas, adjacent skin
confirmed on final pathology
laxity)
– Local wound care, Integra, or temporizing skin graft in
● Confirm absence of involvement of deeper structures
interim
(e.g., parotid, facial nerve)
– Fresh frozen pathologic evaluation cannot ensure negative
margins

12.4.1 Excision
See Chapter 7, ▶ Table 7.2

12.4.2 Reconstruction
● Eyelid support must be considered when operating along the
eyelid/cheek junction
– Consider canthoplasty/canthopexy for additional support of
lax eyelid
● Primary closure: Smaller defects where adequate skin laxity is
present
● Skin grafts: Less ideal color match
– If patient is a poor flap candidate due to comorbidities
– When there is high risk of recurrence or for temporary
coverage prior to definitive reconstruction
● Transposition flaps (e.g., banner, rhomboid): Useful for
smaller defects of the face
● Mustardé cheek rotation flap (▶ Fig. 12.2)
– Cheek rotation flap with wide base useful for defects of the
lower eyelid or infraorbital region
– Allows tissue to be advanced superiorly to minimize
retraction of the lower eyelid
Fig. 12.1 Facial subunits may be utilized to guide reconstruction. – Extends along eyelid margin transversely to the
preauricular region

48
Cheek Reconstruction

– Burow's triangle removed in the lateral cervical region ● Cervicopectoral flap (▶ Fig. 12.4)
– May be elevated in subcutaneous plane or deep to SMAS (to – Design similar to cervicofacial flap but dissection extends
increase blood supply) post-auricularly and down in front of hairline across neck to
● Cervicofacial advancement flap (▶ Fig. 12.3) allow additional movement
– Anteriorly or posteriorly-based flap which advances/rotates – Cervicofacial flap may be extended to cervicopectoral flap if
facial skin to fill defect inadequate release of the tissues is achieved with initial
– Similar to Mustardé flap, but without involvement of lower procedure
eyelid – Dissected deep to platysma, can incorporate pectoral and
– Designed below eyelid transversely to ear, extends deltoid fascia
inferiorly around earlobe ● Regional flaps
○ Dissected above SMAS, releasing zygomatic retaining – Useful for large defects
ligaments – Deltopectoral, cervicohumeral, pectoralis major, trapezius,
and latissimus flaps
– Less ideal skin and color match than local flaps using like
skin
● Tissue expansion
– May be performed when few reconstructive options exist
and reconstruction may be delayed
– Care must be taken to avoid compression overlying carotid
artery
● Microvascular reconstruction
– May be performed when loco-regional options are absent
or inadequate for reconstruction
– Helpful for reconstruction of large oral mucosal defects or
when composite tissue reconstruction (mucosa, bone, skin)
is necessary
– Disadvantage: Poor color and texture match

12.5 Complications
● Ectropion: Frequently results from excessive downward pull
of lower eyelid
● Partial flap loss: Can often be managed with local wound
care
● Contour abnormalities and unsightly incisions/color match
● Alteration of hair bearing region with advancement of hair
into previously hairless areas or vice versa
Fig. 12.2 Mustardé cheek rotation flap for reconstruction of defects
● Hematoma: Large flaps should have drains placed at initial
involving the lower eyelid.
procedure

Fig. 12.3 Design for cervicofacial flap.


(a) Inferiorly based. (b) Superiorly based.

49
Facial Reconstruction

Fig. 12.4 Design for cervicopectoral flap.


(a) Anteriorly based. (b) Posteriorly based.

– Err on the side of larger flaps


12.6 Critical Errors ● Failure to consider the importance of the lower eyelid,
● Failure to obtain negative margins on pathology prior to resulting in ectropion
reconstruction – Inadequate support of the eyelid (canthoplasty/canthopexy)
● Poor flap design which is inadequate to reconstruct a sizeable – Poor design of flap with downward vector along lower
facial defect eyelid, leading to high risk of ectropion

50
Case 13 Acquired Facial Paralysis
Raman Mehrzad, Albert S. Woo, and Daniel Kwan

Case 13 A 65-year-old female presents with complaint of facial asymmetry after acoustic neuroma resection 10 years ago. She is interested in
discussing treatment options to improve the appearance and function of her face.

51
Facial Reconstruction

High resolution contrast-enhanced computed tomography


13.1 Description ●

(CT) or gadolinium-enhanced magnetic resonance imaging


● Complete left-sided facial paralysis (MRI)
● Facial asymmetry: Left palpebral fissure widening, left – Should include the brain (temporal bone) and parotid gland
nasolabial fold effacement, right-sided deviation of Cupid's ● Possible biopsy: Facial nerve, lip (for salivary tumors), and
bow, and inferior malposition of the left oral commissure FNA of parotid mass
which demonstrates no movement on smiling ● Electrodiagnostic studies: Nerve conduction studies,
electromyography (EMG)
● Electroneurography (ENoG): Compares amplitude of
13.2 Work-Up summation potentials of paralyzed face to normal side

13.2.1 History
13.2.4 Etiology
● Onset of symptoms
– Etiology: Bell's palsy, surgery, and stroke ● Bell's palsy: Acute peripheral facial palsy of unknown cause
– Unilateral or bilateral; complete or incomplete – Herpes simplex virus activation (zoster) is the likely cause
– Duration and rate of progression: Acute, subacute, or chronic in most cases
● Associated symptoms: Headaches, blurred vision, dry eyes, ● Disease: Guillain–Barre syndrome, sarcoidosis, Sjögren’s
vertigo, hearing loss, otorrhea, oral incompetence, speech syndrome
difficulties, snoring, and nasal obstruction ● Infection: Otitis media, Lyme disease (neuroborreliosis), HIV
● History of trauma, infections (Bell's palsy, Ramsay-Hunt, ● Tumors or stroke
Lyme disease, TB), neuromuscular disease (Myasthenia
Gravis, Charcot–Marie–Tooth, Guillain–Barre), tumors
13.2.5 Consultations
(Neurofibromatosis Type II), diabetes, travel history,
pregnancy, family history, and surgical history (otologic ● Depend upon the case and associated symptoms: Possibilities
surgery, rhytidectomy, parotidectomy) include ophthalmology, otology, neurology, infectious
disease, speech pathology, physical/occupational therapy, and
psychiatry/psychology
13.2.2 Physical Examination
● Perform complete head, neck, and cranial nerve examination
● Examine all branches of facial nerve (complete or incomplete 13.3 Patient Counseling
involvement) ● Patients must be made aware that surgical intervention will
– Temporal (Frontal): Elevation of forehead never result in perfect facial symmetry. Procedures are
– Zygomatic: Closure of orbicularis oculi designed to improve symmetry and function.
– Buccal: Elevation of cheek and oral commissure ● Some procedures are static while others allow dynamic
– Marginal mandibular: Depression of oral commissure and movement of the face.
lower lip
– Cervical: Contraction of platysma
● Eyes: Evaluate eye closure, vision, corneal defects, and 13.4 Treatment
ectropion
– Schirmer's test (see Chapter 25) 13.4.1 Nonsurgical Management
– Bell's phenomenon (see Chapter 25): If absent, there is ● Steroid treatment for idiopathic, autoimmune, or certain
greater concern for corneal injury
traumatic injuries
● Evaluate facial movements at rest and in multiple different
– Bell's palsy: Steroid course and Valtrex within 10 days of
expressions
symptom onset
– Assess midline deviation, degree of excursion with movement ● Corneal protection
– Assess brow movement, nasal valve function, and oral
– Lubrication, especially at night with eye ointment to
competence
prevent injury and drying
– Synkinesis: Involuntary contraction of additional facial
– Eye patch when necessary
muscles with voluntary facial movement—due to aberrant ● Antibiotics or antivirals for specific infections, if identified
neuroregeneration ● Neuromuscular retraining
● Assess overall muscle status (hypertonic, normal, or atrophic),
– To facilitate symmetrical movements and minimize
voluntary and involuntary movements (synkinesis,
undesired gross motor activity (such as synkinesis)
dyskinesis, fasciculations)
– Mirror training, negative biofeedback, stretching exercises,
and massage
13.2.3 Pertinent Imaging or Diagnostic ● Botulinum toxin: Helps minimize undesired movements and
assists with facial symmetry
Studies – Useful to establish symmetry of smile when only marginal
● Hematologic work-up: Complete blood count (CBC) (evaluate mandibular nerve is involved on one side
for infection, leukemia) and Lyme titer

52
Acquired Facial Paralysis

○ Can enable spontaneous smile without muscle retraining


13.4.2 Surgical Management
– Single-stage free muscle transfer driven by ipsilateral
● Goals: Corneal protection, normal resting tone, oral masseteric branch of CN V
competence, symmetric smile ○ Patient must consciously bite down to smile.
● Determinants of treatment – Donor muscles: Gracilis, latissimus, pectoralis minor, and
– Duration of injury/presence of motor end units serratus
– Nature and extent of injury/insult – Access with facelift-type preauricular incisions extending
past earlobe along posterior border of mandible
● Other symmetry procedures
13.4.3 Acute Repair following Nerve
– Platinum weights put in upper eyelid to allow for eyelid
Injury closure
● Direct nerve repair: Acceptable if tension-free coaptation is – Canthopexy/plasty to improve paralytic lower eyelid
possible in the acute period (ensure the repair is outside of ectropion
any zone of injury) – Brow lift (endoscopic, open, or direct brow lift above
● Interpositional nerve grafting: Performed if tension-free eyebrow)
coaptation is not possible ● Postoperative care: Physical therapy/speech therapy,
– Donor site options: Sural (reversed to prevent branching), including muscle retraining, may optimize outcomes. Oral
split sural, great auricular commissure splints may be used to prevent descent in the
– Useful in planned oncologic facial nerve resections early postoperative period.
● Cross-facial nerve grafting from contralateral facial nerve
(VII) to one or more branches of injured nerve to reinnervate
acute unilateral palsy
13.5 Ethical Considerations
– Performed only in early reconstruction when contralateral ● Patients are likely to be frustrated from having suffered facial
side is completely normal paralysis and may be critical of previous care/question
– In delayed reconstruction, motor end units are not whether mistakes were made.
functional after 12–18 months and a free muscle transfer is ● Some patients may have unreasonable expectations of
required (see next section) outcomes from surgery.

13.4.4 Delayed Surgical Reconstruction 13.6 Complications


● Static slings: Provide resting symmetry only
● Asymmetry (static/dynamic) and poor muscle function are
– Tensor fascia lata, temporoparietal fascia, palmaris,
common findings after reconstruction
plantaris, dermal allograft
● Hematoma: As in facelift procedures, return immediately to
● Regional muscle transfers (temporalis muscle sling/
the operating room (OR) for evacuation
advancement or masseter muscle transfer)
● Synkinesis/dyskinesis is a common finding in nerve regrowth
– Avoids free tissue transfer and may allow for dynamic
and may not be avoidable
movement
● Failure of nerve graft or flap
– Patient must consciously bite down on affected side to
● Disinsertion of muscle
smile
● Free muscle transfer for dynamic movement (two main
options for innervation source)
– Cross-facial nerve graft with free muscle transfer (two-
13.7 Critical Errors
stage procedure) ● Failure to initiate steroid therapy as soon as possible, when
○ Most commonly performed free muscle transfer for facial indicated
reanimation ● Forming unrealistic expectations
○ Stage 1: Sural nerve graft sutured to redundant branches ● Overlooking issues of eyelid closure and prevention of corneal
of CN VII on unaffected side. Ends are banked in abrasions/xerophthalmia
contralateral upper lip. ● Inadequately educating patient regarding options and
○ Stage 2: Free muscle transfer driven by cross-facial nerve procedures
graft, performed approximately 9–12 months later (test ● Poor evaluation regarding etiology of paralysis
for Tinel's sign in nerve graft to determine readiness for ● Failure to promptly return to OR if hematoma develops or if
free muscle transfer). there is concern for free flap failure

53
Case 14 Congenital Facial Paralysis
Craig B. Birgfeld

Case 14 A 4-year-old female presents with inability to move the left side of her face since birth.

55
Facial Reconstruction

14.1 Description 14.4 Treatment


● Complete left-sided, unilateral facial nerve palsy 14.4.1 Two-Stage Facial Reanimation
– Inferior descent of left oral commissure
– Inability to completely close left eye with intact Bell's reflex (Cross-Facial Nerve Graft with Gracilis
– Left sclera white, without injection/irritation Free Flap)
● Benefit of spontaneous smile, but requires two surgeries
14.2 Work-Up ● Stage 1: Cross-facial sural nerve graft
– Sural nerve harvested behind lateral malleolus
14.2.1 History – Facelift incision with small extension to Risdon incision on
face (▶ Fig. 14.1)
● Family history of congenital palsy
– Identify two redundant buccal branches of intact facial
● History of birth trauma
nerve both of which elevate the commissure (using nerve
● Symptoms of dry or irritated eyes
stimulator)
● Feeding problems or speech difficulties
– Reversed graft from buccal branch (epineural suture) to
● Lack of lateral eye movement (Moebius syndrome)
contralateral upper lip
– May attempt to follow Tinel's sign along graft to confirm
14.2.2 Physical Examination axonal growth, but this is unreliable in children
● Stage 2: Functioning gracilis free flap (> 6 months after Stage 1)
● Observe facial movement during conversation/build rapport – Begin with frozen section biopsy of distal graft to confirm
with patient axons
● Sequentially test facial nerve function of all branches on both – Elevate facelift flap from preauricular to commissure
sides – Identify and dissect facial artery and vein
● Test for Bell's reflex : Hold eyelid open as patient actively tries – Remove buccal fat pad to identify facial vein and debulk
to close eye cheek
– This is a protective phenomenon – Bluntly tunnel from commissure to upper lip incision for
● Inspect sclera for signs of irritation/dryness nerve
● Look for lower eyelid malposition – Place sutures around commissure such that a nasolabial
● Donor nerves: Test for contralateral buccal branches of facial crease symmetric with the other side is created
nerve (smile), ipsilateral nerve to masseter (bite), spinal – Harvest gracilis from short medial thigh incision.
accessory nerve (shoulder shrug), and hypoglossal nerve Length harvested = distance from zygomatic arch to
(tongue movement) commissure
– May split gracilis length-wise to decrease bulkiness
14.2.3 Diagnostic Studies – Inset distal muscle at commissure and pass nerve through
tunnel to lip
● Obtain computed tomography (CT) scan, only if there is – Complete venous and arterial anastomosis
concern for concomitant craniofacial microsomia – Inset proximal muscle to zygomatic arch and superficial
musculoaponeurotic system (SMAS)
14.2.4 Consultation – Complete neurorrhaphy in mouth

● Best managed by a multidisciplinary team: Plastic surgery,


pediatric otolaryngology, neurosurgery, pediatrics, speech
pathology, child psychology, nutrition, social work, and nursing.

14.3 Patient Counseling


● Emphasize the need to maintain corneal lubrication to
prevent desiccation and ulceration.
● Discuss the pros and cons of single-stage surgery (nerve to
masseter) and two-stage surgery (cross-facial nerve graft).
● Discuss the timeline of two-stage facial reanimation
beginning with cross-facial nerve graft after age 4, followed
6 months later by gracilis free flap.
● Explain the additional time needed (6–12 months) for Fig. 14.1 Planned facelift incision with short neck extension.
movement to begin.

56
Congenital Facial Paralysis

14.4.2 One-Stage Facial Reanimation – Temporalis fascia is unfolded to reach the commissure
– Donor defect repaired with Alloderm or Medpor
(Free Gracilis to Nerve to Masseter) – May leave bulge as muscle passes over zygomatic arch
● Pros ● Temporalis advancement
– One surgery instead of two – Coronoidectomy
– Only one side of face used so no risk to contralateral facial – Distal insertion advanced to commissure
nerve – May need tendon graft weave to reach commissure
– Faster reinnervation
– More robust muscle strength
– Useful for bilateral facial nerve palsy (Moebius syndrome) 14.5 Ethical Considerations
● Cons ● Generally, procedures are not performed on children if they
– Smile created by firing masseter (biting teeth together) object, even if the family would like to proceed.
– Requires some learning ● It is important to explain to families that no reanimation
● Surgical technique procedure will be perfect and that extensive rehabilitation
– Similar to Stage 2 above, but utilize nerve to masseter may be necessary to achieve a reasonable result.
rather than cross-facial nerve graft
– Identify nerve to masseter as it passes through sigmoid
notch just inferior to zygomatic arch 14.6 Complications
● Thrombosis of arterial or venous anastomosis
14.4.3 One-Stage Temporalis Transfer ● Failure of cross-facial graft reinnervation
● Nerve injury to unaffected side
● Pros
● Infection
– One surgery instead of two
– No microsurgery required
– Less operative time, may be done as outpatient
● Cons
14.7 Critical Errors
– Limited control of smile vector of pull ● Inability to accurately diagnose the affected side and nerve
– Less strength branches involved
– Firing muscle requires biting teeth together ● Failure to address corneal dryness with lubricant or surgery
– Requires some learning ● Absence of backup for donor vessels (i.e., superficial
● Temporalis muscle turnover procedure (▶ Fig. 14.2) temporal)
– Hemi-coronal incision ● Lack of knowledge of techniques for addressing free flap
– Central strip of temporalis is taken failure

Fig. 14.2 Temporalis muscle turnover procedure


with fascia unfolded to reach orbicularis.

57
58
15 Unilateral Cleft Lip Deformity 61
Section III
16 Bilateral Cleft Lip Deformity 65
Face, Congenital 17 Cleft Palate 69

18 Pierre Robin Sequence 73

19 Prominent Ear Deformity 77

20 Microtia 81

21 Giant Hairy Nevus 87

III
Case 15 Unilateral Cleft Lip Deformity
Albert S. Woo

Case 15 (a, b) A 3-month-old male infant presents to clinic with a cleft deformity identified at birth.

61
Face, Congenital

15.1 Description Table 15.1 Timeline for management of a child with cleft lip and palate
deformity
● Complete left-sided, unilateral cleft lip deformity Age Treatment
– Cleft nasal deformity: Nostril is widened and slumped (alar
cartilage is inferiorly, posteriorly, and laterally). The nasal Newborn Feeding assessment, initial clinical evaluation,
possible genetics referral
tip is bulbous and shifted toward the cleft.
– Septal deformity: The septum and columella are shifted 0–3 Months Nasoalveolar molding therapy may be offered,
away from the cleft. possible cleft lip adhesion
● Alveolar cleft visible 3 Months (or after Definitive cleft lip repair
● Complete unilateral cleft palate inferred based on wide nasal molding)
alveolar defect
1 Year Cleft palate repair

3–4 Years Assessment of velopharyngeal competence


15.2 Work-Up 7–10 Years Alveolar bone grafting following presurgical
orthodontics (during period of mixed dentition)
15.2.1 History
Skeletal maturity Septorhinoplasty, final revisions as necessary;
● Family history of orofacial clefting orthognathic surgery, if there is evidence of
● Feeding difficulties, confirm appropriate weight gain midfacial growth disturbance
● Additional medical problems/congenital abnormalities and
associated syndromes ○ Passive molding appliance rapidly becoming the gold
standard for optimizing nasal shape
15.2.2 Physical Examination ○ Alveolar molding alone takes place until alveolar ridges

are 5 mm apart, then nasal prongs are attached to


● Evaluate involved structures (lip, alveolus, palate)
improve the shape of the nose
– Lower lip pits: Van der Woude syndrome (autosomal
– Latham appliance
dominant) ○ Active molding appliance which expands palate and
● Evaluate for associated birth anomalies consistent with a
retracts premaxilla
syndromic presentation ○ Less commonly used due to concerns regarding maxillary

growth
15.2.3 Diagnostic Studies ● Lip adhesion
– Performed surgically, in place of molding techniques
Only if there is concern for other systemic illness or syndrome – Preliminary repair of skin with or without muscle between
ages of 6 weeks and 3 months
15.2.4 Consultation – Goal: Minimize tension during the definitive cleft repair
performed around 3–6 months of age
● Best managed by a multidisciplinary team: Plastic surgery, ● Cleft lip repair: Approximately at the age of 3 months
pediatric otolaryngology, speech pathology, child psychology, – Rule of 10s: 10 lb of weight, 10 grams of hemoglobin (Hb),
audiology, genetics, pediatric dentistry, orthodontics, 10 weeks of age
maxillofacial surgery, social work, and nursing – May be delayed secondary to molding (NAM) or earlier lip
● Genetic evaluation if additional congenital abnormalities are adhesion
present ● Cleft palate repair: Approximately 1 year of age
– Earlier repairs favor speech but potentially compromise
maxillary growth
15.3 Patient Counseling – The opposite is true for palatoplasty after 18 months of age
● Cleft care is best managed via a multidisciplinary team ● Alveolar bone grafting
● Discuss the likelihood of several surgeries over the child's – Performed during period of mixed dentition (roughly 7–10
lifetime (see ▶ Table 15.1 for cleft management timeline) years old), after appropriate orthodontics
● Feeding: Critical aspect of cleft care ● Cleft nasal/septal reconstruction
– Specialized nipples/bottles: Dr. Brown bottle, Haberman – Optimally performed once the patient has reached skeletal
nipple (with a squeezable tip) or Pigeon nipple (with maturity. Can be combined with “touch up” procedures to
crosscut opening for faster flow) optimize appearance.
● Molding: Narrows cleft to optimize repair – Septoplasty is frequently deferred until this time.
– Not employing any molding technique is also a reasonable ● Elaborate on the need for long-term follow-up through the
option Cleft Team
– Lip taping: With steri-strips or commercially available – Assess for appropriate development and absence of negative
devices (such as DynaCleft) outcomes, such as velopharyngeal insufficiency (see Chapter
– Nasoalveolar molding (NAM) 17) or maxillary hypoplasia, requiring jaw surgery.

62
Unilateral Cleft Lip Deformity

Should you choose to use these, be prepared to elaborate


15.4 Treatment during questioning.
15.4.1 Cleft Lip Repair Technique ● Primary cleft nasal reconstruction: Performed at time of primary
lip repair, now widely accepted, but not mandatorily performed.
● Millard rotation-advancement repair – The nose may be accessed via cleft incisions or with
– It is the most commonly recognized technique for external incision along the rim of the affected side.
examination (▶ Fig. 15.1). ● Surgical markings
– The short medial lip element is rotated, and the lateral lip is – Essential component of examination. You must know how
advanced into the defect. to mark a cleft lip.
– You should become very familiar with the Millard – Critical points (▶ Fig. 15.3)
technique as this is the most common procedure known by ○ Center of Cupid's bow
examiners. ○ Peak of Cupid's bow on non-cleft side
– Other valid techniques include: Fisher repair (▶ Fig. 15.2), ○ Measure from Point 1 to Point 2 and mark point on
rotation-advancement with pennant flap (unilimb Z- opposite site
plasty), Mohler modification, and Noordhoff technique. ○ Edge of white roll on lateral lip element

Fig. 15.1 (a, b) Millard rotation-advancement flap for correction of unilateral cleft lip deformity.

Fig. 15.2 (a, b) Fisher anatomic subunit


technique. (Source: Surgical Technique. In: Losee
J, Kirschner R, ed. Comprehensive Cleft Care. 2nd
Edition. Thieme; 2015.)

63
Face, Congenital

15.5 Complications
● Cleft lip dehiscence
● Infection

15.6 Critical Errors


● Inability to draw a cleft lip repair and to identify where each
point goes with closure
● Unfamiliarity of timing of repair
● Discussing surgical techniques other than Millard rotation-
advancement repair without appropriate knowledge/
familiarity
● Advocating controversial procedures, such as
gingivoperiosteoplasty, primary septal reconstruction, or
other nonstandard procedures, may negatively impact
examination
● Discussing optional therapies without appropriate knowledge
(e.g., Latham appliance). If you mention it, be prepared to
Fig. 15.3 Definitive markings on patient case study with Millard elaborate upon it.
rotation-advancement.

– Critical flaps
○ Rotation flap: From Point 3 in an arc up to base of

columella and potential back-cut past midline to get


appropriate rotation
○ Advancement flap: From Point 4, skirting along lateral cleft

margin to nostril, then extending laterally to base of ala


○ C Flap: Used to reconstruct base of columella on cleft side

○ L Flap: Can be used to reconstruct nostril floor or advance

lateral nasal wall


○ M Flap: Can add additional mucosa for intraoral lining

64
Case 16 Bilateral Cleft Lip Deformity
Vinay Rao and Albert S. Woo

Case 16 (a, b) A 12-day-old infant boy born with a cleft lip.

65
Face, Congenital

16.1 Description 16.3.1 Feeding: Critical Aspect of Cleft


● Bilateral cleft lip deformity Care
– Left side is complete, right incomplete with presence of ● Specialized nipples/bottles: Dr. Brown bottle, Haberman
Simonart's band nipple (with a squeezable tip) or Pigeon nipple (with crosscut
– Flattening of nasal tip and widening of nose with shortened opening for faster flow)
columella ● Weight gain: After the first 2 weeks of life, the child should
– Nostrils are asymmetric and wider on left side due to gain half a pound every week
complete cleft – General principle is to hold infant at 45 degrees and drip
● Left alveolar cleft deformity with gap noted between formula into the mouth and allow the infant to swallow.
segments – Babies with cleft palate cannot create adequate suction on a
● Palate is not visualized bottle or breast.

16.2 Work-Up 16.3.2 Presurgical Molding


● Not mandatory, but narrows cleft and improves symmetry
16.2.1 History ● Lip taping: Started soon after birth
● Family history of orofacialclefting ● Nasoalveolar Molding (NAM): Facilitates lip repair by
● Evidence of feeding difficulties, appropriate weight gain bringing lip elements together, improving position of nasal
● Monitor for appropriate weight gain cartilages, and lengthening columella. It also aligns the
● Additional medical problems and associated syndromes alveolar segments. Rapidly becoming the gold standard for
preoperative molding prior to surgical intervention.
● Latham appliance (active molding appliance): Expands palate,
16.2.2 Physical Examination retracts premaxilla; less commonly used due to concerns
● The examination should focus on four features: Nose, lip, regarding maxillary growth
alveolus, and palate
– Cleft lip and palate classification
○ Unilateral versus bilateral
16.3.3 Cleft Lip Adhesion
○ Complete (involvement of nasal floor) versus incomplete ● Used in wide clefts to decrease tension
○ Isolated cleft lip (primary palate) versus cleft lip and ● Skin and mucosa, with or without orbicularis muscle, are
palate (primary and secondary palates) sutured together
● Additional facial dysmorphic features ● Performed at 6 weeks to 3 months of age
– Lower lip pits: Van der Woude syndrome (autosomal
dominant)
● Complete physical examination to look for any other
16.3.4 Cleft Lip Repair
anatomic abnormality ● Timing: Lip repair is typically performed around 3 months of
– Consider possibility of syndromic presentation age.
– May be delayed due to health concerns or additional time
needed for molding
16.2.3 Pertinent Imaging or Diagnostic – Rule of 10s: 10 weeks of age, 10 lbs of weight, 10 grams of
Studies hemoglobin
● Echocardiogram, renal ultrasound, skeletal X- rays, etc.,
● Various techniques have been described (e.g., McComb, Trott,
depending on physical findings Cutting, Millard, Fisher, and Mulliken methods; ▶ Fig. 16.1).
Due to only minor variations between procedures, any
reasonable technique may be used as long as key principles
16.2.4 Consultations are followed.
● Orbicularis reconstruction (▶ Fig. 16.2): Current methods
● Children with clefts are ideally cared for by multidisciplinary
bring in muscle from the lateral lip to the midline to
team: Plastic surgery, otolaryngology, speech pathology,
reconstruct the orbicularis sling.
audiology, child/developmental psychology, nursing,
● Philtral preservation: The prolabial skin is preserved to create
pediatric dentistry, orthodontics, and oral and maxillofacial
the philtrum. However, the vermilion (central red lip) is
surgery.
discarded and vermilion from the lateral lip is brought to the
midline.
16.3 Treatment ● Markings (▶ Fig. 16.3)
– Philtral flap: The width of the philtral flaps (Points 1, 2,
● Management via multidisciplinary team and 3) are marked 2 to 2.5 mm apart. The flap narrows
● Schedule of treatment (see ▶ Table 15.1 for cleft management slightly as it extends up to the columella, with its length
timeline) typically 6 to 8 mm.

66
Bilateral Cleft Lip Deformity

Fig. 16.1 Standard markings for bilateral lip repair.

Fig. 16.2 Bilateral cleft lip repair advocated by Mulliken. (a) Orbicularis
muscle and vermilion from lateral lip elements are brought to the
midline. The skin of the prolabium is preserved to create the philtrum.
(b) Result following repair of the cleft lip. External rim incisions shown
Fig. 16.3 Authors' suggested markings (modeled after Cutting for primary cleft nasal reconstruction.
technique).

16.4 Ethical Considerations


– White roll and vermilion: The white roll is marked (Points 4
and 5) on the lateral lip element where it begins to
● Bilateral cleft lip repair is a complex procedure requiring
diminish medially. Another point (6 and 7) is marked lateral multiple surgical procedures and multidisciplinary care. As a
to this point equal to the distance from points 1 to result, it is often in the best interest of the patient and family
2 (~2–2.5 mm wide). Mark the vermilion perpendicular to have the patient treated within a cleft center of excellence
to points 4 and 5. Draw a line from points 6 and 7, above by specialized cleft surgeons.
the white roll, up to the nasal sill and then transversely
under the ala.
– Nasal reconstruction: Optional procedure NOT performed by
16.5 Complications
all cleft surgeons. Exposure is obtained via separate rim ● Bleeding
incision (▶ Fig. 16.1 and ▶ Fig. 16.2b). The lower lateral ● Infection
nasal cartilages are dissected in the supraperichondrial ● Dehiscence
plane and interdomal sutures are placed to approximate the ● Prolabial ischemia: Remove nasal stent or any other
nasal tip. compressive structures, if placed

67
Face, Congenital

Advocating controversial procedures, such as


16.6 Critical Errors ●

gingivoperiosteoplasty, primary septal reconstruction, or


● Failure to monitor feeding/weight gain other non-standard procedures, may negatively impact
● Inability to draw a cleft lip repair and to identify where each examination
point goes with closure ● Discussing optional therapies without appropriate knowledge
● Unfamiliarity with timing of repair (e.g., Latham appliance). If you mention it, be prepared to
● Failure to evaluate for other congenital anomalies elaborate upon it.

68
Case 17 Cleft Palate
Vinay Rao and Albert S. Woo

Case 17 A 5-year-old boy with concerns over hypernasal speech.

69
Face, Congenital

17.1 Description 17.2.3 Pertinent Imaging or Diagnostic


● Photo reveals a submucous cleft palate Studies
– Bifid uvula ● Evaluate other organ systems (e.g., echocardiogram, renal
– Zona pellucida: Diastasis of levator muscle with notable ultrasound, X-rays of the spine) if there is suspicion for other
thinning of soft tissue at the midline, especially with congenital anomalies or syndrome.
elevation of the palate ● Perform genetic testing if syndrome is suspected, possible
– Patient is phonating with anterior displacement of levator through chromosomal microarray analysis.
muscles (inverted V)

17.2.4 Consultations
17.2 Work-Up ● Multidisciplinary team: Plastic surgery, pediatric
otolaryngology, speech pathology, audiology, genetics,
17.2.1 History pediatric dentistry, orthodontics, oral–maxillofacial surgery,
● Pregnancy, birth, and newborn history social work, and nursing
– Prenatal care and exposures (alcohol, smoking, ● Genetic evaluation, particularly if associated anomalies are
anticonvulsants, corticosteroids) present
– Gestational age of the newborn at birth (e.g., preterm, term,
and postterm)
● Family history of orofacial clefting 17.3 Patient Counseling
● Additional medical problems ● Cleft palate is best managed via a multidisciplinary team
– Cleft palate without cleft lip: 40% incidence of syndromic ● Discuss the likelihood of several surgeries over the child's
presentation
lifetime (see Chapter 15, ▶ Table 15.1)
● Airway concerns ● Feeding: Infants with cleft palates are at higher risk of being
– Consider Pierre Robin sequence (Chapter 18) if small jaw
underweight
and airway obstruction
– Inability to create effective suction force because of palatal
● Feeding and weight gain history
cleft. As a result, the infant tires out before achieving full
feeding and satiation.
17.2.2 Physical Examination – Elevate head and cradle infant at 45 degrees
– Specialized nipples/bottles: Haberman feeder (with a
● Classify the extent of cleft and structural involvement
squeezable tip), Pigeon nipple (with cross-cut opening for
– Complete (i.e., soft and hard palates) or incomplete (i.e., soft
faster flow), Dr. Brown's level 2 nipple with Pigeon valve
palate alone)
– Primary and/or secondary palate (dividing point is the
incisive foramen) 17.4 Treatment
– Unilateral or bilateral (vomer visible on one or both sides)
– Cleft lip involvement, if any 17.4.1 Cleft Palate Repair Technique
– Veau cleft palate classification system (▶ Table 17.1)
● Timing: Typically around 1 year of age. Earlier repair puts
● Evaluate for facial dysmorphic features and other congenital
child at increased risk of maxillary growth abnormalities;
anomalies
later repair delays language development.
– Cleft palate alone (without cleft lip): 40% incidence of
● Soft palate repair
syndromic presentation
– Intravelar veloplasty: Most commonly utilized technique
– Mandible evaluation: Pierre Robin sequence—micrognathia/
wherein levator veli palatini muscles are dissected out and
retrognathia, glossoptosis, and airway difficulties (see
reapproximated from their anterior malposition;
Chapter 18)
repositioned in the midline with a transverse orientation.
● Head-to-toe examination for other anatomic abnormalities
– Double-opposing Z-plasty (Furlow) (▶ Fig. 17.1):
Musculomucosal flaps are elevated with opposing
Table 17.1 Veau cleft classification system Z-plasties from the oral and nasal mucosa layers.
Veau classification Description ● Hard palate repair
system – Von Langenbeck palatoplasty: Relaxing incisions along the
lateral edge of the hard palate and incisions medially at the
Veau I Incomplete cleft of the palate: Cleft of the soft
palate only mucosa along the edges of the cleft. Raise bilateral
Veau II Complete cleft of the secondary palate: Cleft of bipedicled mucoperiosteal flaps that approximate oral
soft and hard palates surface of the cleft. Best used for incomplete clefts of the
Veau III Unilateral cleft lip and palate secondary palate without cleft lip or alveolus.
Veau IV Bilateral cleft lip and palate

70
Cleft Palate

Fig. 17.1 Furlow double-opposing Z-plasty.


(a) An oral Z-plasty is initially marked over the
soft palate mucosa. (b) A musculomucosal flap is
raised on the left and a mucosal flap is raised on
the right. (c) An “opposing” Z-plasty is then
performed on the nasal mucosa. Now, a
musculomucosal flap is raised on the right and
the left contains only nasal mucosa. (d) Both
Z-plasties are then transposed and the muscle
layers are notably overlapped on top of each
other during closure.

– Two-flap palatoplasty (Bardach) (▶ Fig. 17.2): Most


commonly utilized technique wherein anterior flaps are
17.5 Ethical Considerations
elevated based on greater palatine vessels. It is used to ● Cleft care should be performed in multidisciplinary centers
repair complete clefts of the primary and secondary that specialize in the care of these complex cases.
palates.
– Vomer flap: Important in the setting of bilateral, complete
cleft palate. Vomer flap is used to close any remaining gap 17.6 Complications
in the hard palate. Vomer is exposed in the cleft gap and
● Airway obstruction: Can occur due to bleeding, edema, or
septal mucosa flaps are raised and are sutured to adjacent
tongue swelling.
nasal mucosa laterally.
– May place nasopharyngeal airway. If tongue stitch is in
● All patients require postoperative airway monitoring with
place, it can be used to pull the tongue forward to open the
continuous pulse oximetry
posterior airway. If there is no response to the above
● Choose appropriate surgical technique
measure, endotracheal intubation may be needed.
– Incomplete soft palate cleft (Veau I): Soft palate repair with
● Bleeding: Usually minor and self-limited, requires airway
intravelar veloplasty or Furlow palatoplasty
monitoring. If severe, return to operating room for control.
– Complete cleft of the soft and hard palate (Veau II): von
● Palatal fistula: Usually manifests several weeks after surgery.
Langenbeck repair with intravelar veloplasty or Furlow
This is not an emergency and may be managed electively
palatoplasty
roughly 6 months after initial palatoplasty.
– Unilateral or bilateral cleft lip and palate (Veau III and IV):
● Velopharyngeal insufficiency (hypernasal speech) may
Two-flap palatoplasty technique with either intravelar
present in up to 20% of patients treated with cleft palate
veloplasty or Furlow palatoplasty
repair.
– Submucous cleft palate: Surgical repair is considered
– Mandates regular follow-up of patient for many years after
only after velopharyngeal insufficiency is clearly
cleft palate repair
identified (usually 3-5 years of age). Treated like Veau I
– Requires thorough evaluation with velopharyngeal imaging
cleft.
(video nasendoscopy, speech videofluoroscopy)

71
Face, Congenital

Fig. 17.2 Bardach two-flap palatoplasty.


(a) Anterior flaps are elevated based on the
greater palatine vessels. (b) The soft palate is
incised and the muscle is separated from the
nasal and oral mucosal layers. (c) The levator
musculature is dissected out and reapproximated
at the midline to establish the intravelar
veloplasty. (d) Appearance of the palate after
repair is complete.

– Multiple surgical options exist, including palatal re-repair, ● Inadequately addressing feeding issues associated with cleft
pharyngeal flap, and sphincter pharyngoplasty palate
– Treatment is tailored to specific requirements of the patient ● Not assessing for other congenital anomalies
● Inability to draw cleft palate repair
● Not familiar with timing of repair
17.7 Critical Errors ● Failure to monitor the airway in the postoperative period
● Failure to identify micrognathia and airway obstruction
(missing Pierre Robin sequence)

72
Case 18 Pierre Robin Sequence
Rajiv J. Iyengar, Karl Bruckman, and Derek M. Steinbacher

Case 18 A newborn infant is taken to the neonatal intensive-care unit (ICU) due to concerns over airway distress soon after delivery.

73
Face, Congenital

Overcome glossoptosis and facilitate airflow into larynx directly


18.1 Description ●

● Feeding with nasogastric tube until able to tolerate oral intake


● Infant identified to have Robin sequence
– Classic triad of micrognathia, glossoptosis, and airway
obstruction with or without cleft palate
18.4.2 Tongue–Lip Adhesion
● Mandible appears hypoplastic (see ▶ Fig. 18.2)
● Apparently intubated with nasal feeding tube in place ● Removal of rectangular patch of mucosa on underside of
tongue encompassing floor of mouth, alveolus, and lower lip
18.2 Work-Up ● Tongue advanced forward and lateral mucosal incisions
sutured to one another
● Multidisciplinary team approach of cleft/craniofacial ● Important to suture genioglossus and orbicularis muscle as
specialists to address mandibular hypoplasia, feeding part of repair
difficulties, and airway obstruction
● Genetic evaluation may be warranted
– Stickler syndrome, chromosome 22q11.2 deletion
18.4.3 Distraction Osteogenesis
(velocardiofacial syndrome), hemifacial microsomia, ● Principles
Treacher Collins syndrome, Nager syndrome, CHARGE – Latency: Device held in original position with no forces
syndrome applied
● Airway evaluation is most important ○ May begin distraction the following day in neonates

– Obstructive sleep apnea assessment with polysomnography ○ Delay for 5 days for older patients

(sleep study) – Activation: Distractor proceeds to separate bone segments


– Bronchoscopy and nasoendoscopy to identify site(s) of ○ Bone stimulated by tension and stress, with ossification

airway obstruction from either end of the central fibrous zone


○ Can provide crucial information regarding subglottic ○ 2 mm/day in neonates (1 mm 2 × /day)

pathology including laryngomalacia, tracheomalacia, and ○ 1 mm/day in older patients (0.5 mm 2 × /day)

subglottic webs – Consolidation: Distractors maintained in position for a


○ May be more than 1 level of obstruction period of 4 to 8 weeks
– Magnetic resonance imaging (MRI) and computed ○ Ossification may be confirmed by X-ray or CT scan

tomography (CT) scans can be used for three-dimensional ● Internal or external devices can be used for distraction
reconstructions to evaluate airway patency ● Substantial airway volume increases often result in
decannulation of tracheostomy dependent patients and
improvements in apnea-hypopnea indices
18.3 Patient Counseling ● Recent literature indicates long-term superiority of
Mandibular distraction osteogenesis (MDO) over Tongue lip
● Stratification of severity guides treatment strategies
adhesion (TLA)
– Above work-up should identify additional obstructive
– More effective in preventing tracheostomy and avoiding
pathology
gastrostomy
– Attempt prone positioning for mild cases (70% of cases)
– However, complications are more common and can be
– Severe airway obstruction necessitating tracheostomy
more severe (open-bite deformity, dental complications,
placement (10%) should undergo operative
facial nerve injuries)
intervention
● Three-dimensional planning and computer-aided design
– Management of intermediate severity (20%) is controversial
○ Nasopharyngeal airway
(CAD)/computer-aided manufacturing (CAM) may be used in
○ Tongue–lip adhesion
mandibular distraction
○ Distraction osteogenesis
● Technique
– Submandibular incision for access to angle of mandible
● Characterizing mandibular hypoplasia
– Bilateral bicortical osteotomies posterior to developing
– Smaller volume
tooth buds
– Shorter ramus
○ Beware of facial nerve branches and inferior alveolar nerve
– Obtuse symphyseal angle
– Devices placed parallel and co-linear to each other

18.4 Treatment (see ▶ Fig. 18.1) 18.5 Complications


18.4.1 Nasopharyngeal Airway ● Loosening of pins
● Minimally invasive, low complication rate ● Scarring
● Most successful in nonsyndromic cases ● Damage to tooth buds

74
Pierre Robin Sequence

Micrognathia

Feeding Airway Sleep study


evaluation assessment

Radiologic, Cannot
Cleft
Swallow bronch, AHI <5 AHI >5 be
palate?
DL extubated

Cleft Feeding, Mechanical Position


palate problem? TLA
teaching, OT change
algorithm Aspiration?

G-tube, NG Mandibular
Choanal repair NP trumpet
tube distraction
Bifacial
microsomia,
TCS, Lower airway
nager, eval/repair,
VACTERL ENT
Care and Tracheostomy
longitudinal
evaluation
Follow
ENT
those
algorithms
Genetics Audiology

Dental Pediatrics Ophtho

Pulm Social As
work needed

OT/PT Orthodontics

Other
specialists

Fig. 18.1 Algorithm for Robin sequence airway management.

75
Face, Congenital

18.6 Critical Errors


● Forgetting to incorporate a multidisciplinary team in the care
of this patient population
● Failure to perform endoscopic evaluation and
polysomnogram to rule out non retroglossal causes of airway
obstruction prior to initiating surgical management
● Moving directly to surgical intervention without attempting
noninvasive management to alleviate airway obstruction

Fig. 18.2 Tongue-lip adhesion.

76
Case 19 Prominent Ear Deformity
Lauren O. Roussel and Patrick K. Sullivan

Case 19 A 25-year-old female requests evaluation for correction of the protrusiveness of her ears.

77
Face, Congenital

19.1 Description 19.3 Patient Counseling


● Adult patient with evidence of ear asymmetry with the left ● As with all cosmetic/elective procedures, there is no
side more prominent than right guarantee to success with otoplasty. Patients must be made
● The antihelical fold (scaphoconchal angle) appears obtuse aware of the possibilities for imperfect results following
with increased projection of the scapha and helix surgery.
● Age at time of intervention varies among providers. Some
suggest operating as early as 4–6 years of age, prior to the
19.2 Work-Up start of school to minimize any potential psychosocial issues
associated with being teased or low self-esteem. Others wait
19.2.1 History until they feel that the child can be involved in the decision to
● Patient's age have the operation and is a willing participant. The authors
● Patient's goals and expectations generally wait until the child is 6 years of age and agrees to
● History of prior surgery to ears surgery.
● History of prior ear trauma ● Surgical intervention does not need to be delayed until
● History of poor wound healing (e.g., prior keloids) completion of ear growth.
● Older patients are at higher risk for recurrence due to the
stiffness of adult cartilage which can potentially overpower
19.2.2 Physical Examination suture correction of ear deformities.
● Presence/absence of preoperative asymmetry
● Overall size and shape of ear
● Helix–mastoid angle
19.4 Treatment
● Helix–mastoid distance (upper, middle, and lower thirds of
the ear)
19.4.1 Nonsurgical Management
● Cartilage consistency ● Serial molding of the ear
● Characteristics to be evaluated (▶ Fig. 19.1) – Newborns to < 6 weeks old have high levels of circulating
– Upper third of the ear maternal estrogen making ear cartilage soft and malleable
○ Underdeveloped/deficient antihelical fold – Custom molds help reshape the ear over time
○ Obtuse scaphoconchal angle (> 90 degrees)

– Middle third of the ear


○ Conchal hypertrophy (concha cavum > 1.5 cm deep)
19.4.2 Surgical Management
○ Obtuse conchal–mastoid angle (> 25 degrees) ● Goal: To create an ear with soft, natural contours and all
– Lower third of the ear aspects of the ear in appropriate position
○ Large, prominent lobule ● Addressing underdeveloped antihelical fold
○ Position of lobule – Mustarde sutures (▶ Fig. 19.2)
○ Horizontal mattress sutures used to recreate antihelical fold

○ Results in setting back the helical rim and scapha

– Cartilage scoring/rasping (▶ Fig. 19.3)


○ Anterior surface of antihelical fold cartilage is scored/

rasped and bent away from cut side toward side of intact
perichondrium
○ Desired amount of cartilage warping can be adjusted by

the extent of the scoring


○ May result in sharp edges on cartilage if improperly

performed
● Addressing deep concha cavum
– Cavum rotation and fixation
– Cartilage resection (which we have found to be rarely
needed)
○ Full-thickness crescent of cartilage excised from the

concha
○ Excision planned so that closure is at the union of

posterior wall of the concha and floor


● Prominent lobule
– Lobulopexy
Fig. 19.1 (a, b) Anatomic differences between normal and prominent
ears. (Source: Anatomy. In: Pu L, ed. Aesthetic Plastic Surgery in Asians: – Lobuloplasty
○ Skin excision from medial surface of the earlobe
Principles & Techniques. Thieme; 2015.)
– Posterior repositioning of the lobule

78
Prominent Ear Deformity

Fig. 19.2 Correction of prominent ears using


mustarde suture. (Source: Traditional Operative
Fossa triangularis
Techniques. In: Mesa J, Buchman S, Mackay D
et al., ed. Atlas of Operative Craniofacial Surgery.
Thieme; 2019.)

Scapha

Concha

● Some providers suggest waiting until patient is able to


understand the implications of surgery and can comply with
postoperative instructions.

19.6 Complications
● Asymmetry
● Persistent deformity, under- or overcorrection
● Hematoma
● Poor wound healing, hypertrophic scarring, and keloid
formation
● Infection and perichondritis
● Cartilage or skin necrosis

19.7 Critical Errors


● Poorly planned incisions resulting in prominent scarring
from surgery
● Failure to address parts of ear contributing to prominent
appearance
● Deformity to the external auditory canal resulting from
anterior displacement of conchal cartilage when fixated
posteriorly
● Overcorrection resulting in unnatural appearance
● Focal addressing of one aspect of the ear creating lack of
proportion in the other aspects of the ear.
– Telephone ear deformity: Middle third of the ear
aggressively set back relative to upper and lower thirds of
Fig. 19.3 Correction of prominent ears using cartilage scoring. the ear causing the helical rim to form a “C” when viewed
posteriorly.

19.5 Ethical Considerations


● Some patients (especially children) may disagree with
their parents and not want to have surgery. Surgeons should
avoid forcing children into surgery despite their parents'
wishes.

79
Case 20 Microtia
Kristopher M. Day and Raymond J. Harshbarger

Case 20 (a, b) A 32 year-old female enters to discuss interest in correcting the deformity of her right ear.

81
Face, Congenital

– Microtia: Small peanut-shaped remnant; “lobule-type,”


20.1 Description usually with aural atresia
● Adult female with grade 3 “lobular type” microtia of the right – Anotia: Complete absence of external ear
ear ● Characteristic associated conditions
● Absence of external auditory meatus suggesting right-sided – Hemifacial microsomia: Facial asymmetry
hearing loss ○ Goldenhar: Preauricular skin tags and epibulbar dermoids

● Examination shows small, malformed, hypoplastic residual – Treacher Collins syndrome: Malar hypoplasia and micrognathia
cartilage with abnormal lobule orientation and aural – Nager syndrome: High nasal bridge and thumb hypoplasia
atresia

20.2.3 Pertinent Imaging and


20.2 Work-Up Diagnostic Studies
20.2.1 History ● Audiometric testing
● Temporal bone and craniomaxillofacial computerized
● Hearing loss and previous hearing aid placement tomography protocols
– Possible use of bone anchored hearing aid (BAHA)
● Family history of ear abnormalities, facial clefts, congenital
hand conditions, or other craniofacial syndromes 20.3 Consultations
– Visual impairment (e.g., oculoauricovertebral syndrome)
● Audiologist
– Renal disease (e.g., branchio-oto-renal syndrome)
– Hearing evaluation
– Cardiac disease (e.g., CHARGE syndrome)
– Commonly have conductive hearing loss > sensorineural
hearing loss
○ Inner ear usually intact
20.2.2 Physical Examination ○ Middle ear atresia common

● Classify anomaly ● Otolaryngologist


– Unilateral or bilateral – Evaluate for BAHA candidacy:
– Severity by grade ○ Initial temporary hearing aid worn with head strap prior

– Presence or absence of external ear canal and/or hearing loss to ear reconstruction
● Grades of ear hypoplasia (see ▶ Fig. 20.1) ○ Critical to initiate early for speech development in

– Mild hypoplasia: Smaller ear, all elements present bilateral microtia


– Moderate hypoplasia: Some missing elements; “concha- ○ Permanent BAHA placed postoperatively to avoid injury

type,” possibly canal stenosis to skin envelope

Fig. 20.1 Grades of ear hypoplasia. (Source: Reinisch J, Tahiri Y, eds. Modern Microtia Reconstruction. In: Derderian CA, Microtia. Switzerland:
Springer; 2019:23–41.)

82
Microtia

– Coverage with TPF flap and FTSG; lobule rotation (see


20.4 Patient Counseling ▶ Fig. 20.3)
● The timing, expected number of surgical stages, approach ○ Requires a generous (10 × 13 cm) TPF flap for complete

options, and potential complications should be discussed vascularized coverage of implant


with the patients and their families ○ FTSG from abdomen and contralateral retroauricular

region (unilateral) or inner arm (bilateral)


– Tragal reconstruction, contouring, and repositioning may
20.5 Treatment (see ▶ Table 20.1) occur as a second stage 3 months later
● Risks of implant extrusion, infection, and fracture
20.5.1 Autogenous Reconstruction
● Four-stage Brent technique (losing favor due to number of
procedures) 20.5.3 Prosthetic
– Begins as early as 6 years of age
○ Framework construction and inset in postauricular pocket ● Anaplastologists design the prosthetic ear mirroring the
○ Earlobe transposition contralateral native ear
○ Ear elevation and full-thickness skin graft (FTSG) ● Attached by:
○ Tragus construction and conchal excavation – Adhesive: time-consuming and potentially
● Two-stage Nagata technique (gaining in popularity) unstable
– Does not begin until at least 10 years of age to enable – Osteointegrated framework: anchored to mastoid bone
sufficient cartilage harvest with metallic posts
○ Violates periauricular soft tissue, complicating
– Framework construction and placement, lobule
transposition (see ▶ Fig. 20.2) later autogenous or implant-based
○ Contralateral sixth to ninth ribs harvest reconstruction
○ Construct has two stacked layers of cartilage to improve

projection of the antihelix; also contains tragal component


– Ear elevation with banked cartilage graft, coverage with
temporoparietal fascia (TPF) flap and FTSG
20.5.4 External Auditory Atresia
(see ▶ Fig. 20.3) Reconstruction
● Performed by otolaryngologist
20.5.2 Implant-Based ● Higher complication rate
● Not necessary if hearing aids provide adequate
Reconstruction hearing
● Medpor® implant ● If pursued, middle ear reconstruction should be done after
– Single-stage reconstruction with porous polyethylene external ear reconstruction to minimize the impact of scar
(Stryker, Kalamazoo, MI) auricular implant tissue

Table 20.1 Summary of treatments


Pros Cons Indications

Autogenous Like tissue replacement, less risk of Chest wall donor site, more stages, Surgeon and patient preference in
construct exposure older age requirement, longer time primary reconstruction with
to completion, less projection adequate soft tissue

Implant-based Obviates need for donor site, faster Risk of construct exposure, infec- Surgeon and patient preference in
and earlier reconstruction age, pos- tion, or fracture primary reconstruction with
sibly longer-lasting, greater projec- adequate soft tissue
tion and definition

Prosthetic Avoids surgery or chest wall donor Daily placement and periodic Salvage revision or post-traumatic
site, customizable to contralateral replacement, not like tissue, pre- cases with compromised surround-
ear vents future reconstruction ing soft tissue

83
Face, Congenital

Fig. 20.2 (a–j) Nagata technique: Ear construct. (Source: Cartilage Sculpture. In: Firmin F, Dusseldorp J, Marchac A, ed. Auricular Reconstruction.
Thieme; 2016.)

– Evacuate air with red rubber catheter with U-stitch tied


20.6 Ethical Considerations upon removal during exhalation
● If a patient desires a reconstructive approach not performed – If there is evidence of pleural defect, then repair directly
by the surgeon, referral to a specialist should be offered ● Chest wall contour deformity
– Prevent by preserving perichondrium and replacing unused
cartilage
20.7 Complications ● Fracture of construct
– Limited: Suture repair
● Construct infection: Incision and debridement with irrigation
– Extensive: Construct replacement
and systemic antibiotics
● Cartilage resorption: May require revision of reconstruction,
● Hematoma: Immediate evacuation
if significant
● Pneumothorax: Tube thoracostomy is rarely indicated

84
Microtia

Fig. 20.3 Second stage Nagata reconstruction with cartilage grafting for ear elevation and coverage with temporoparietal fascia flap. (Source: Type A
Technique: Temporofascial Flap. In: Firmin F, Dusseldorp J, Marchac A, ed. Auricular Reconstruction. Thieme; 2016.)

Failing to promptly and accurately manage hematoma,


20.8 Critical Errors ●

cartilage, or infection
● Neglecting to assess for other craniofacial syndromes ● Performing middle ear reconstruction before external ear
● Failure to refer to an audiologist for hearing and hearing aid reconstruction
evaluation
● Forgetting to monitor unaffected ear to protect patient's
hearing status and speech development

85
Case 21 Giant Hairy Nevus
Clinton S. Morrison and Sara A. Neimanis

Case 21 A 10-month-old female child presents with skin lesion present since birth.

87
Face, Congenital

There may be aesthetic concerns depending on size and


21.1 Description ●

location of the nevus


● Large, pigmented, hair-bearing skin lesion on the posterior ● Laser and dermabrasion is not indicated
trunk of an otherwise healthy-appearing child – Superficial procedures may improve appearance but retain
● Satellite nevi are noted the risk of malignancy as the entire lesion is not removed
● Presence at birth suggests giant congenital melanocytic nevus

21.4 Treatment
21.2 Work-Up ● Goal is to excise as much of the nevus as possible in the
fewest number of operations
21.2.1 History ● Treatment may begin around 1 year of age to reduce
● Presence of lesion at birth or its development later malignant transformation which usually occurs early in life
● Changes in size, color, and other appearances. Has it grown ● If suspicious for malignancy, biopsy should be done
proportionally with the child? immediately. If positive for malignancy, should excise and
● Presence of additional satellite lesions treat based on current skin cancer guidelines.
● Presence of associated conditions (e.g., spina bifida, ● Reconstruction options should follow the reconstructive
neurofibromatosis) ladder, including local tissue rearrangement and skin grafting,
● Family history of congenital nevi of skin cancer if available
– Split-thickness grafts may produce suboptimal aesthetic
results and lead to issues with contractures
21.2.2 Physical Examination – Full-thickness grafts are often not possible given the large size
● Evaluate the focus of the lesion as well as the rest of the skin of the defects but may be useful in areas such as the eyelids
for presence of satellite lesions ● Serial excision
● Location of lesions – Consider if the lesion is small enough or oriented in such a
– Cephalic or midline neck/spine lesions may be associated way that it can be excised in two to three operations
with neurologic conditions ● Tissue expansion: Mainstay of reconstruction due to the large
– May occur in bathing trunk pattern or with stocking glove size of these lesions
distribution – Replace defect with appropriate tissue (e.g., expand
● Measure and document the size of the lesion unaffected scalp to replace hair-bearing skin in case of a
– Giant hairy nevus: At least 20 cm in diameter or comprising scalp nevus)
2% of the total body surface area – May use multiple expanders and/or multiple stages
● Take photos for comparison at future visits and monitoring – Tissue expanders will need to be inflated over time, which
● Assess for changes that suggest malignancy such as can be traumatic in children
ulceration, bleeding, or change from prior visits

21.5 Ethical Considerations


21.2.3 Pertinent Imaging or Diagnostic
● Surgery is clearly indicated due to the risk of malignant
Studies degeneration. In cases where families are not compliant with
● Magnetic resonance imaging (MRI) of the brain and spine care, they may need to be reported to child protective
recommended for midline head/neck/spine nevi to rule out services, for the welfare of the patient.
neurocutaneous melanosis
● Advanced imaging will often require anesthesia in this age
group
21.6 Complications
● Biopsy any portions of nevi suspected for malignancy ● Malignancy
● Graft or flap loss
● Tissue expansion: Infection, extrusion of implants,
21.2.4 Consultations hematoma/seroma, and expander rupture
● Dermatology for regular surveillance and monitoring – If exposed/infected, the expander must be removed. If
● Neurology for patients with midline head/neck/spine lesions expansion has already been done, a portion of the nevus
can be excised and expanded tissue may be advanced in the
same operation
21.3 Patient Counseling
● Risk of malignant transformation to melanoma is 5–10% for
giant hairy nevi
21.7 Critical Errors
– Most frequently in the first decade of life ● Neglecting to biopsy suspicious lesions
● Excision of giant congenital nevi is recommended to reduce ● Not referring patient with cephalic or midline neck/spine
the risk of malignant transformations nevi for MRI or neurology
– Treatment will likely require multiple surgical interventions ● Inadequate patient/family counseling on malignant
for over months to years transformation and multistage procedures

88
22 Nonsurgical Rejuvenation of the Face 91
Section IV
23 Aging Face and Neck 95
Face, Cosmetic 24 Aging Upper Face 99

25 Lower Eyelid Ectropion (Cicatricial) 103

26 Lower Eyelid Ectropion (Involutional and


Paralytic) 107

27 Rhinoplasty 109

28 Gender Transition
(Male-to-Female) 113

29 Gender Transition
(Female-to-Male) 117

IV
Case 22 Nonsurgical Rejuvenation of the Face
J. Thomas Paliga and Ivona Percec

Case 22 (a–c) A 57-year-old woman presents with concerns of facial aging and interest in nonsurgical rejuvenation.

91
Face, Cosmetic

22.1 Description 22.2 Work-Up


● Thin middle-aged woman with aged-related changes to the 22.2.1 History
face
● Fitzpatrick skin type II and Glogau class III wrinkle scale ● Identify primary concerns and create a hierarchy of problem
(▶ Table 22.1 and ▶ Table 22.2) areas
● Upper face: Static and dynamic transverse forehead and – If patient could address only one issue, what would it be?
vertical glabellar rhytids, mild brow ptosis, and temporal ● Prior surgical or nonsurgical procedures, how the patient
hollowing tolerated them and perceived improvement
● Periorbital region: Crow's feet, crepey lower eyelid skin, tear ● Smoking and ultraviolet (UV) exposure
trough deformity, no upper eyelid ptosis, and moderate lower ● Current skin care regimen, if any
eyelid tone ● History of autoimmune disorders, seasonal and medical
● Midface: Malar volume loss with superficial and deep fat pad allergy, and use of anticoagulants and other medications
atrophy, mild descent, and grade II nasolabial fold wrinkle
(see ▶ Table 22.3) 22.2.2 Physical Examination
● Perioral: Vertical static and dynamic perioral rhytids, upper
and lower lip fat pad atrophy with lengthening and flattening (Systematic Facial Analysis)
of lip contour ● Upper to lower facial regions
● Lower face: Marionette lines, mental strain and mental crease – Upper face: Temples, brows, and periorbital regions
deepening, minimal jowling, and weakened mandibular angle ○ Temporal narrowing, periorbital hollowing, and brow
● Neck: Medial platysmal bands, minimal skin excess and ptosis
lipoatrophy ○ Forehead and glabellarrhytids, and crow's feet

Table 22.2 Glogau photo aging classification


Table 22.1 Fitzpatrick classification for sun-reactive skin types
Class Wrinkle Age Clinical findings
Type Skin color Hair Eye Effect of sun exposure? characteristics range
color color
I No wrinkles 20s–30s Mild pigmentary changes
I Light Red Blue/ Always burns, never tans No keratoses
green Minimal to no makeup
II Light Blonde Blue Usually burns, tans with II Wrinkles in Late Early senile lentigines
difficulty motion 30s–40s Palpable keratoses
III Medium Brown Brown Sometimes mild burn, Parallel smile lines
tans average Some makeup foundation

IV Medium/dark Brown/ Dark Rarely burns, tans with III Wrinkles at 50 s or Obvious dyschromia
brown black brown ease rest older Visible keratoses
Telangiectasias
V Dark brown Black Dark Very rarely burns, tans Heavy makeup foundation
brown very easily
IV Only wrinkles 60s–70s Yellow-gray color of skin
VI Black Black Dark Never burns, tans very Skin malignancies
brown easily Makeup results in “cakes and cracks”

Table 22.3 Wrinkle severity rating scale (WSRS)


Grade Wrinkle severity NLF characteristics Anticipated effect of filler

I Absent Not visible None indicated


Continuous skin line

II Mild Shallow visible fold Slight improvement


Slight indentation

III Moderate Moderately deep fold Excellent correction


Visible at rest, disappears on stretch

IV Severe Very long and deep fold Significant improvement


Prominent facial feature
< 2 mm fold visible on stretch

V Extreme Extremely long and deep fold Unlikely to have satisfactory correction
Detrimental to facial appearance
2–4 mm visible fold on stretch

Abbreviation: NLF, nasolabial fold.

92
Nonsurgical Rejuvenation of the Face

– Midface: Malar, submalar, preauricular regions, nasolabial – Perioral: Upper and lower lip cutaneous fat pads, mucosal
folds, and nose lips, and vermilion border
○ Loss of malar projection and nasal support, tear trough – Lower face: Prejowl sulcus, mental prominence, marionette
deformity, and prominent nasolabial folds lines, and posterior jawline
– Lower face: Posterior, middle, and anterior jawline; perioral ● Lasers/Dermabrasion/Chemical peels
area; and lips – Improvement of skin photodamage, fine superficial rhytids,
○ Jowling, prejowl sulcus, perioral rhytids, loss of vermilion and loss of elasticity/turgor
and mucosal volume, upper lip lengthening and ● Chemical lipolysis (deoxycholic acid)
flattening, downturned oral commissures, marionette – Submental adiposity and adiposity of jowls
lines, and mentalis ptosis ● Biostimulators (polylactic acid, polymethyl methacrylate,
● Outer structures to midline structures calcium hydroxyapatite)
● Asymmetry: 100% of faces are asymmetric – Similar to HA fillers but delayed onset and difference in
● Tissue quality: Thin versus thick ability to manage complications
– Fitzpatrick skin type (see ▶ Table 22.1)
● Assess depth of aging at all levels: Bone, soft tissues, and skin
– Glogau photo aging classification (see ▶ Table 22.2) 22.5 Ethical Considerations
– Wrinkle severity rating scale (see ▶ Table 22.3) ● Adequate patient counseling to understand cost to
● Dynamic assessment with key facial expressions
improvement ratio and the need for maintenance treatments
● Ethnicity and goals
for appropriate correction
● Gender goals ● Need for procedure staging
● Patient education when patient is better served by surgical
22.2.3 Clinical Photography correction

● Standardized imaging with neutral background


● In repose: Anteroposterior (AP), mid-lateral, and true lateral 22.6 Complications
of both right and left face
● In animation: Open smile, closed smile, frown, grimace, and ● Vascular occlusion/Embolism
lip pursing – Hyaluronidase for HA fillers
– Hyaluronidase, steroid injections, and 5-FU for non-HA
products
22.3 Patient Counseling – Repeat hyaluronidase injections, frequent monitoring, and
timely diagnosis
● Discuss individual patient's anatomy and age-related changes – Local wound care
– Explain how neuromodulation, soft tissue fillers, and – Immediate recognition of retinal artery embolism
adjunct skin treatments work together to restore a youthful ○ Referral to experienced oculoplastic surgeon
appearance ○ Consider retro- or peribulbar hyaluronidase injections
● Discuss expectations of what can be accomplished with ● Upper eyelid ptosis
individual treatments – Apraclonidine drops: Patient reassurance, anatomy review,
– Reiterate need for consistent maintenance for optimal reassess patient's anatomy once neuromodulation has worn
results off
● Discuss expected longevity and estimated cost of treatment ● Foreign body/Allergic/Inflammatory reactions/Delayed
granulomas
– Massage, corticosteroids, antihistamines, and antibiotics for
22.4 Treatment biofilm
● Requires understanding of the unique properties of each – Avoidance of bacterial exposure (e.g., dental or other
injectable and adjunct procedures procedures, flights, etc.) for 2 weeks prior to and post
● Botulinumtoxin injections
– Upper face: Frontalis, corrugators, and procerus ● Excessive bleeding/Bruising
– Periorbital: Orbicularis oculi – Avoidance of anticoagulants (prescription or over-the-
– Lower face counter [OTC] supplements one week prior to procedure)
○ Masseter for increased lower facial width secondary to – Need for slow injection in small amounts and with the
masseter hypertrophy smallest needle size
○ Mentalis for mentalis strain

○ Depressor angulioris (DAO) for lowered oral commissures

○ Orbicularis oris for perioral wrinkles 22.7 Critical Errors


– Neck: Platysma for platysmal bands ● Poor understanding of anatomy and depth of injection
● Soft tissue fillers (HA: hyaluronic acid) ● Incorrect product use for application and depth of injection
– Upper face: Temples, superior orbital rim, and lateral brow ● Poor patient education
– Periorbital: Tear troughs ● Inability to manage complications
– Midface: Malar fat pads, nasolabial folds, and alar base

93
Case 23 Aging Face and Neck
Jonathan P. Brower and Patrick K. Sullivan

Case 23 (a, b) A 62-year-old female requests a consultation for rejuvenation of the midface and neck, complaining that she looks “old and tired” and
is self-conscious of the appearance of her neck.

95
Face, Cosmetic

Face and neck rejuvenation are contouring procedures


23.1 Description ●

– Nonsurgical treatments (neurotoxins, peels) may remain


● Elderly female with minimal actinic damage and glabellar necessary to treat rhytids
rhytids, crow's feet, perioral rhytids, and moderate tear ● Preoperative photos
trough deformity – Essential to document existing anatomy and asymmetry
● Midface: Soft tissue descent and moderate jowling with that may not be treated or even improved by surgery
prominence and asymmetry of nasolabial and labiomental folds
● Neck: Blunted cervicomental angle with marked skin laxity
and evidence of excess subcutaneous + /- subplatysmal fat; no
23.4 Treatment
evidence of platysmal bands
23.4.1 Facelift
● Incisions: The extent and position of incisions vary based on
23.2 Work-Up anatomy and hairline, but typically follow the general pattern
seen in ▶ Fig. 23.1.
23.2.1 History – Short scar technique: Preauricular incision extends
● What are the patient's primary aesthetic concerns? around the earlobe but not into hairline. This technique is
● Previous facial procedures and surgeries limited to younger patients with minimal excess skin.
● Medical history ● Surgical techniques: Multiple options are available, although
– Special attention is paid to cardiopulmonary diagnoses superficial musculoaponeurotic system (SMAS) and SMA
including hypertension, use of anticoagulants, and smoking sectomy procedures are most common.
history – SMAS facelift (▶ Fig. 23.2)
● Ophthalmologic history, if procedure is undertaken with ○ The SMAS is incised transversely below (traditional SMAS

periorbital rejuvenation dissection) or above (extended SMAS dissection) the


– Visual acuity, dry eye zygomatic arch, and then preauricularly down to the
anterior border of the sternocleidomastoid.
○ The SMAS flap is redraped and plicated in a
23.2.2 Physical Examination
superoposterior direction.
● Aesthetic analysis of facial aging ○ Facial nerve branches run in the sub-SMAS layer and can
– Upper third be injured with overly aggressive dissection.
○ Brow position and symmetry

○ Dermatochalasis; skin quality and presence of rhytides

○ Lower lid laxity, fat herniation, and tear trough deformity

– Middle third
○ Malar volume descent

○ Prominence of nasolabial folds

○ May choose to comment on nose (see Chapter 27)

– Lower third
○ Lip fullness

○ Marionette lines; jowls

○ Chin projection

– Neck
○ Skin laxity

○ Subcutaneous versus subplatysmal fat: Determined by a

pinch test in which the tissue is gently pinched while the


patient swallows. Tissue that pulls out of the examiner's
hand indicates subplatysmal fat.
○ Platysmal banding

○ Measurement of cervicomental angle (ideal 90–110 degrees)

● Skin type and quality


– Fitzpatrick Scale for skin type classifies response of skin to
UV light
○ Useful for determining response of skin to nonsurgical

adjuncts for rejuvenation (peels, lasers, etc.)


– Overall skin quality, including pre-existing photo damage

Fig. 23.1 Standard facelift incision. Typically extends from within the
23.3 Patient Counseling temporal hairline, anterior to the ear and behind the tragus, around
the lobule and into hairline of the posterior scalp. (Source: Primary
● It is critical to elucidate patient's expectations and establish
Superficial Musculoaponeurotic System (SMAS) Facelift and Neck Lift.
realistic outcomes from surgery In: Connell B, Sundine M, ed. Aesthetic Rejuvenation of the Face and
– Durability and longevity of result are not guaranteed Neck. Thieme; 2016.)

96
Aging Face and Neck

Fig. 23.2 Standard superficial musculoaponeurotic system (SMAS) Fig. 23.3 SMAsectomy procedure. Superficial musculoaponeurotic
dissection and vector of lift. (Source: Primary Superficial system (SMAS) resection takes place along malar eminence and lateral
Musculoaponeurotic System (SMAS) Facelift and Neck Lift. In: Connell edge of orbicularis muscle, over the parotid gland, and inferiorly into the
B, Sundine M, ed. Aesthetic Rejuvenation of the Face and Neck. neck to the posterior portion of the platysma muscle. (Source: Primary
Thieme; 2016.) Superficial Musculoaponeurotic System (SMAS) Facelift and Neck Lift. In:
Connell B, Sundine M, ed. Aesthetic Rejuvenation of the Face and Neck.
Thieme; 2016.)
– SMASectomy procedure (▶ Fig. 23.3)
○ Similar to SMAS procedure but rather than dissecting

under the SMAS layer, an ellipse of SMAS is excised from ● Be prepared to discuss management if the patient has had
the malar eminence to the posterior neck. The SMAS prior facial aesthetic surgery by another surgeon and is either
edges are then directly repaired to tighten the SMAS. dissatisfied or experiencing a complication
○ May decrease risk of facial nerve injury due to limited
● Have a revision policy prepared, including conditions for
dissection revision and expectations for payment
– Subcutaneous facelift
○ Undermining only in subcutaneous plane with skin flap

redraped in superoposterior direction


23.6 Complications
○ Higher recurrence rate due to lack of SMAS suspension ● Hematoma
– Correlated with high blood pressure
– Low threshold for immediate operative evacuation
23.4.2 Neck Rejuvenation ● Skin necrosis
● Platysmaplasty with or without platysmal myotomy – Local wound care with or without scar revision when
– Access through a submental incision or laterally through healed
facelift dissection – Avoid temptation to revise early as tissues have already
○ Platysma dissected and reapproximated in midline been stretched and are not likely to have improved outcome
– Open liposuction of preplatysmal fat may be a beneficial after revision
adjunct ● Nerve damage
○ Closed liposuction of the neck is controversial – More common with sub-SMAS dissection due to additional
dissection
– The great auricular nerve is the most commonly injured
23.5 Ethical Considerations nerve that gets noticed
● Avoid patients with comorbidities that place them at higher – The buccal branch of facial nerve is the most commonly
risk for complications injured nerve, but it often goes unnoticed because of
– Mitigate risk by offering nonsurgical options that may offer functional redundancy with other facial nerve
substantial improvement (see Chapter 22) branches

97
Face, Cosmetic

● Patient dissatisfaction ● Failure to take back a patient immediately for hematoma,


– Does not imply surgeon's error in judgment or technique which can cause flap necrosis
– It is important to have a standard response for accusation, ● Management of skin necrosis with premature surgical
requested revisions, or refunds intervention

23.7 Critical Errors


● Failure to recognize preoperative risk factors and control peri/
postoperative hypertension

98
Case 24 Aging Upper Face
Karen Leong

Case 24 (a, b) A 72-year-old female comes to your office complaining of looking tired. She is primarily concerned about her appearance around the
eyes and upper face.

99
Face, Cosmetic

Lower eyelid analysis


24.1 Description ●

– Lower eyelid/Iris relationship: Minimal to no scleral show,


● Fitzpatrick I skin type ideally covers 0.5 mm of the lower limbus
● Lateral brow ptosis with tattooed brows above supraorbital – Tear trough deformity
rim – Malar support
● Dermatochalasis of the upper eyelids; pseudoherniation of ○ Positive vector

lower eyelid fat pads ○ Negative vector—relative scarcity of skin, hemi

● Midface descent with a prominent arcus marginalis/ exophthalmos


infraorbital rim ● Ocular examination (▶ Table 24.1)
– Eyelid ptosis
○ Patient focuses on a light source with both eyes
24.2 Work-Up ○ Margin reflex distance (MRD1): The distance from the

pupillary light reflex to the upper eyelid margin


24.2.1 History (▶ Fig. 24.1)
○ Normal MRD1: 4–4.5 mm
● Identify medical conditions that may increase the risk of
complications – Levator excursion
○ Hold brow in resting position. Ask patient to look up and
– Blepharochalasis, Graves' disease, glaucoma
– Previous periorbital and facial procedures measure excursion of the upper eyelid with a ruler
○ Recent laser-assisted in-situ keratomileusis (LASIK) or (normal > 10 mm)
cataract surgery: Should not undergo blepharoplasty for – Dermatochalasis: Visual field test to document
at least 6 months following procedure – Visual acuity
– History of dry eyes/seasonal allergies – Dry eyes: Schirmer's test, Bell's phenomenon
○ Hormone replacement therapy: 70% higher risk of dry eye (see Chapter 25)
– Ectropion/lower eyelid laxity
○ Lower eyelid distraction < 7 mm, lower eyelid snap test
24.2.2 Physical Examination (Analysis of ○ MRD2: Measure the distance from the pupillary light

the Upper Third of the Face) reflex to the lower eyelid margin (▶ Fig. 24.2)
○ Normal MRD2: 5–5.5 mm
● Forehead analysis: Hairline (brow height), transverse and
glabellar rhytids
● Eyebrow analysis: “Ideal brow” 24.3 Patient Counseling
– Location: Relation between hair-bearing brow and
supraorbital rim ● Potential complications should be reviewed. Ensure
– Peak: Should be located at or just lateral to the lateral reasonable expectations and encourage patient compliance
limbus with postoperative recovery regimen.
– Evaluate for brow ptosis and compensation
● Upper eyelid analysis Table 24.1 Ptosis classification
– Upper eyelid/Iris relationship: Covers 2–3 mm of superior
Ptosis Mild Moderate Severe
limbus
– Upper eyelid crease: Female (7–10 mm), male (6–8 mm), MRD1 2–2.5 mm 1–1.5 mm < 1 mm
Asians (variable but low) Levator Good Fair Poor
– Lateral extension of the eyelid crease onto the lateral Excursion > 10 mm 5–10 mm < 5 mm
portion of the periorbital region is a marker of forehead
Abbreviation: MRD, margin reflex distance.
ptosis (Connell's sign)

Fig. 24.1 Margin reflex distance (MRD1):


Distance from light reflex to upper eyelid. Smaller
number is worse.

100
Aging Upper Face

Fig. 24.2 Margin reflex distance (MRD2):


Distance from light reflex to lower eyelid. Larger
number is worse.

Transconjunctival fat removal/redistribution


24.4 Treatment ●

● Transcutaneous approaches
● Stop anticoagulants, nonsteroidal anti-inflammatory drugs – Skin only
(NSAIDs), pertinent vitamins, and herbal supplements – Skin–muscle flap: Preserve 4–5 mm of the pre-septal
preoperatively orbicularis to preserve spontaneous eye blink
● Anesthesia: Patient safety and comorbidities should be the
primary determinant of type of anesthetic chosen
– General anesthesia, moderate/light sedation, local 24.5 Ethical Considerations
anesthetic ● Patients should have reasonable expectations and be able to
comply with postoperative instructions.
24.4.1 Brow Lift ● If the complication is due to another surgeon, obtain any
previous operative reports and do not speak negatively of
● Multiple techniques available other providers. Patients should be encouraged to seek
● Coronal incision—lengthens forehead second opinions if you sense they may be difficult.
● Pretrichial incision—shortens forehead ● Do not refer to oculoplastics as a first-line response; you
● Endoscopic should be able to treat and manage these patients and their
● Transcutaneous/Direct—scar may not be acceptable complications.
● Transpalpebral—upper blepharoplasty incision
● Temporal brow lift—lateral brow repositioning
24.6 Complications
24.4.2 Upper Blepharoplasty 24.6.1 Brow
● Multiple techniques; concurrent ptosis repair, when indicated ● Temporal branch of the frontal nerve paresis—usually
● Transconjunctival removal of medial fat pad
temporary, advise watchful waiting
● Skin excision only, with or without fat removal ● Alopecia
● Skin and muscle excision, with or without fat removal

24.4.3 Ptosis Repair 24.6.2 Upper Eyelid


● A-frame deformity
● Mild ptosis/Good levator: Fasanella Servat or Müllerectomy ● Dehiscence of the levator attachments may create post-
● Moderate ptosis: Levator advancement (good levator) or
blepharoplasty ptosis
resection (moderate levator) ● Lagophthalmos and corneal exposure from anterior lamellar
● Severe ptosis/Poor levator: Frontalis sling
shortage or excessive orbicularis removal
● Hering's law of equal innervation: If you correct unilateral
ptosis, the opposite side might develop new ptosis due to a
decrease in innervation 24.6.3 Lower Eyelid
● Retrobulbar hematoma: Return to operating room (OR)
24.4.4 Lower Blepharoplasty immediately or bedside lateral canthotomy and cantholysis to
prevent vision loss; consult ophthalmologist
● Multiple techniques available ● Ectropion: Combination of lower eyelid laxity with scarring
● Fillers—hyaluronic acid, fat transfer
of the capsulopalpebral fascia–septum interface
● Pinch blepharoplasty—skin only

101
Face, Cosmetic

– If unable to go to OR, must perform a lateral canthotomy


24.7 Critical Errors and cantholysis at the bedside
● Inadequate preoperative assessment: Failure to evaluate for ● Excessive fat removal
brow ptosis, upper eyelid ptosis, lower eyelid laxity/ ● Overzealous resection of the upper eyelid skin and/or muscle
ectropion, and dry eye during upper eyelid blepharoplasty leading to lagophthalmos
● Failure to address brow ptosis at the time of upper eyelid ● Transection of inferior oblique muscle during lower eyelid
ptosis repair transconjunctival approaches
● Not taking patient immediately to the OR when signs of
retrobulbar hematoma are evident

102
Case 25 Lower Eyelid Ectropion (Cicatricial)
Jason Chow and Michael E. Migliori

Case 25 A 72-year-old male patient presents for evaluation of persistent tearing and ocular irritation of the left eye after Mohs surgery and
reconstruction of the left lower eyelid.

103
Face, Cosmetic

25.1 Description 25.4 Treatment


● Cicatricial ectropion of the left lower eyelid: Evidence of 25.4.1 Medical Treatment
previous Mustarde flap
– Scarring of the anterior lamella ● Ocular lubrication to treat or prevent ocular surface
– Loss of apposition of the eyelid and lower punctum to the involvement
eye – Artificial tears during day
– Tearing of the lower eyelid and conjunctival injection – Lubricating ointment at night
● Right side with senile (involutional) ectropion and scleral ● Discontinuation of topical medications that may be causing
show (see Chapter 26) dermatitis
● Treatment of underlying inflammatory condition can lead to
stabilization or partial reversal of condition before surgical
25.2 Work-Up intervention

25.2.1 History
25.4.2 Surgical Intervention
● History of ocular irritation, conjunctival irritation (eye
redness), keratopathy, epiphora (excessive tearing), ocular
● Surgically release vertical cicatricial traction
trauma, eye or eyelid surgery, or facial paralysis
● Anterior lamella (Skin coverage)
● Past medical history: History of inflammatory/rheumatic – Z-plasty for mild ectropion
conditions, dermatitis, zoster, skin malignancy, or eyedrops – Placement of a local flap (e.g., Mustarde flap, Tripier flap)
● History of eyelid or facial surgery – Full-thickness skin graft (see ▶ Fig. 25.1) from opposite
eyelid, pre-/post-auricular region, or supraclavicular skin
● Horizontal eyelid laxity
25.2.2 Physical Examination – Canthopexy to tighten mild horizontal laxity
– Lateral tarsal strip procedure (see ▶ Fig. 25.2)
● Evaluate the lower eyelid and assess for sites of scarring
○ Inferior limb of the lateral canthus is released
– Assess the mobility and adequacy of the lower eyelid skin
○ Skin and mucosa overlying the lateral end of the tarsus is
● Snap back test for eyelid laxity
removed to shorten the width of the eyelid
– Pull lower eyelid down and away from globe
○ Tarsal strip is then anchored to lateral orbital rim
– Observe length of time to return to original position
○ 4–0 Proline placed into periosteum along inner border of
– Ectropion is significant if:
○ Eyelid does not immediately snap back
orbital rim
○ Eyelid does not return to original position

● Careful examination of the eyelid margin to evaluate for


blepharitis, rosacea, and any inflammatory etiologies
25.5 Ethical Considerations
– Also evaluate for signs of eyelid malignancy such as loss of ● Surgical correction should be offered to symptomatic patients
eyelashes (madarosis) and the risk and benefits should be discussed thoroughly
● Assess orbicularis oculi strength before proceeding
● Bell's phenomenon (palpebral oculogyric reflex) ● Smoking should be stopped perioperatively and
– Ask patient to close eyes while holding eyelids open anticoagulants should be stopped in conjunction with the
– Observe for upward and outward movement of eye upon primary care provider
eye closing
– This reflex protects the cornea when the eyes are closed
● Schirmer's test (Dry eye test): 25.6 Complications
– Tetracaine drops are applied in eyes for numbing
– Filter paper is placed in inferior fornix and eyes are closed 25.6.1 Ocular Complications
for 5 minutes ● Palpebral conjunctival keratinization
– Normal > 15 mm ● Corneal epithelial disease (including punctuate epithelial
● Slit lamp examination with fluorescein to evaluate for corneal erosions, corneal ulceration, and corneal perforation) from
epithelial disease inadequate ocular lubrication and exposure

25.3 Patient Counseling 25.6.2 Surgical Complications


● Patient should be counseled on the importance of ocular ● Infection
lubrication in order to prevent or treat corneal epithelial ● Wound dehiscence
disease which can be vision-threatening ● Graft failure

104
Lower Eyelid Ectropion (Cicatricial)

Fig. 25.1 (a–h) Full-thickness skin grafting for


correction of anterior lamellar defect.

● Incorrect position of the lateral canthal angle ● Not recognizing perioperative orbital compartment syndrome
● Retrobulbar hematoma in the setting of retrobulbar hematoma
– Rare complication leading to orbital compartment – Leads to irreversible vision loss
syndrome, a true surgical emergency ● Inadequate release and lengthening of the anterior lamella
– Signs include eye pain, decreased vision, afferent papillary ● Use of split thickness, instead of a full thickness, graft leading
defect, and elevated intraocular pressure to contracture and recurrence of cicatricial ectropion
– Treatment is emergent lateral canthotomy and cantholysis ● Failure to address coincidental horizontal laxity
to prevent irreversible vision loss ● Missing signs of eyelid malignancy

25.7 Critical Errors


● Forgetting to initiate ocular lubrication leading to corneal
injury

105
Face, Cosmetic

Fig. 25.2 (a–d) Lateral tarsal strip procedure for


lower eyelid tightening to correct horizontal
eyelid laxity.

106
Case 26 Lower Eyelid Ectropion (Involutional and Paralytic)
Jason Chow and Michael E. Migliori

Case 26 A 68-year-old male patient presents for evaluation of left eye irritation and persistent tearing.

107
Face, Cosmetic

– Plication of the lateral canthal tendon through a skin


26.1 Description incision, sparing the canthal angle
● Sagging and eversion of the left lower eyelid margin ● Lateral tarsal strip (see ▶ Fig. 25.1)
● Loss of apposition of the eyelid and lower punctum to the eye – The lateral end of the tarsus is dissected free
● Tearing and some evidence of ocular irritation – It is used to reattach the lower eyelid to the lateral orbital
rim
● Bick procedure
26.2 Work-Up – Full-thickness wedge excision of the lower eyelid at the
lateral canthal angle
26.2.1 History – Allows for shortening of the lower eyelid
● History of ocular irritation, conjunctival infection (eye ● Medial spindle procedure
redness), keratopathy, epiphora (excessive tearing), ocular – Horizontal excision of conjunctiva and retractors inferior to
trauma, eye or eyelid surgery, or facial paralysis the punctum to correct punctal eversion
● History of chronic eye irritation – Repair or reattachment of the eyelid retractors can be
combined with horizontal tightening of the eyelid to correct
retractor laxity or disinsertion
26.2.2 Physical Examination
● Evidence of sagging or eversion of the lower eyelid margin
– Search findings to classify etiology of ectropion
26.5 Ethical Considerations
– Involutional, cicatricial, paralytic, mechanical ● Surgical correction should be offered to symptomatic patients
● Evaluate lateral canthal tendon and the risk and benefits should be discussed thoroughly
– Can be lax or detached before proceeding.
– Occasionally associated with medial canthal tendon laxity ● Smoking should be stopped perioperatively and
and/or dehiscence or detachment of the lower eyelid anticoagulants should be stopped in conjunction with the
retractors primary care provider.
● Snap back test ● If anticoagulation cannot be stopped preoperatively or patient
– Pull lower eyelid down and away from globe and observe is otherwise a poor surgical risk, consideration should be
length of time to return to original position given to medical management with lubrication.
– Ectropion is significant if the eyelid does not immediately
snap back or does not return to original position
● Eyelid distraction test 26.6 Complications
– Laxity: Lower eyelid can be pulled away from the eye by
● Ocular complications
more than 6 mm
– Palpebral conjunctival keratinization, corneal epithelial
● Medial canthal laxity test
disease including punctate epithelial erosions, corneal
– Pull lower eyelid laterally
ulceration, and corneal perforation from inadequate ocular
– Punctum should not move laterally by more than 1 mm
lubrication and exposure
● Lateral canthal laxity test
● Surgical complications: Infection, wound dehiscence,
– Pull lower eyelid medially
incorrect position of the lateral canthal angle
– Lateral canthal angle should not move medially by more
● Retrobulbar hematoma is a rare complication leading to
than 2 mm
orbital compartment syndrome, a true surgical emergency
● Assess orbicularis oculi strength
– Signs include eye pain, decreased vision, afferent papillary
● Bell's phenomenon (palpebral oculogyric reflex)
defect, elevated intraocular pressure
– Ask patient to close eyes while holding eyelids open
– Treatment is emergent lateral canthotomy and cantholysis
– Observe for upward and outward movement of eye upon
to prevent irreversible vision loss
eye closing
– This protects the cornea when the eyes are closed
● Slit lamp examination with fluorescein to evaluate for corneal
epithelial disease
26.7 Critical Errors
● Forgetting to initiate ocular lubrication leading to corneal
injury
26.3 Patient Counseling ● Not recognizing perioperative orbital compartment syndrome
in the setting of retrobulbar hematoma leading to irreversible
● Patient should be counseled on the importance of ocular
vision loss
lubrication prior to repair in order to prevent corneal
● Failure to diagnose the correct type of ectropion (senile,
exposure/dryness which can threaten vision
paralytic, cicatricial, congenital) for the correct type of
surgical management
26.4 Treatment ● Error in positioning attachment of the tarsal strip, leading to
residual eyelid laxity or canthal dystopia necessitating
● Lateral canthopexy: Mild cases additional surgery

108
Case 27 Rhinoplasty
Raman Mehrzad and Albert S. Woo

Case 27 (a, b) A 20-year-old female presents to your office with concerns about the appearance of her nose.

109
Face, Cosmetic

– Nasal septum: Midline, C- or S-shaped deviation


27.1 Description – Inferior turbinates: Consider reduction/outfracture if
● Young female with prominent dorsal hump and absence of a enlarged/protuberant
supratip break ● Surrounding facial assessment
● The nose is deviated to the left; the nasal tip is well-defined – Hypoplastic/retruded chin: May affect overall balance of
● A ring in the columella is present and should be removed to face; can consider concurrent genioplasty
minimize distraction – Upper lip: Long or short, dynamic depressor septi muscles,
● The nasolabial angle (columella to upper lip) is between 95 upper lip crease
and 100 degrees

27.2.3 Pertinent Imaging or Diagnostic


27.2 Work-Up Studies
27.2.1 History ● Photographic documentation during preoperative
consultation is advocated
● Identify the patient's interest in surgery (function, ● Take pictures in the anteroposterior, lateral, worm's eye, and
appearance, or both?) bird's eye views
– Difficulty with breathing or history of snoring
– Specific concerns with appearance of nose
● Prior history of allergic disorders: Asthma, hay fevers, 27.3 Patient Counseling
sinusitis, and vasomotoric rhinitis. Inferior turbinate
● An honest discussion about what can be and cannot be
hypertrophy with secondary nasal obstruction is typically
achieved with the surgery should be discussed. This will help
found in patient with long-standing history of allergic
set realistic expectations to avoid postoperative patient
rhinitis.
dissatisfaction
● Prior nasal surgeries or trauma
● Discuss about long-term postoperative edema
● Social history: Smoking, alcohol consumption, and illicit drug
– General rule: First surgery (1 year); Revision (2 years)
use, especially cocaine
● Medications: Anticoagulation, nonsteroidal anti-
inflammatory drugs (NSAIDs), or fish oil may increase risk of
bleeding and postoperative ecchymosis
27.4 Treatment
● Open approach: Most common technique
– Access entire nose with a transcolumellar (e.g., stairstep)
27.2.2 Physical Examination incision extending into inferior margins of the lower lateral
● Skin quality: Identify Fitzpatrick skin type (▶ Table 22.1) and nasal cartilage
skin thickness – Allows for direct access to the septum (for cartilage harvest/
– Thin skin will show underlying framework better than thick septoplasty) by separating the lower lateral nasal cartilages
● Appearance of the nose and accessing the septum via the anterior septal angle
– Assess overall shape and symmetry of dorsum and tip ● Closed approach: Less commonly used and criticized for
○ Deviation, dorsal hump, and contour irregularities decreased visualization and control during the procedure
– Bony vault: Width, symmetry, and length of nasal bones – Access to the cartilages may be established through
– Mid-vault: Width, possible collapse, inverted-V deformity, intercartilaginous, transcartilaginous, or marginal incisions
and hump without the transcolumellar component
– Dorsal aesthetic lines: Straightness, symmetry, and well or – Septum may be approached with transfixion, hemi-
ill defined transfixion, or Killian incision
– Tip assessment: Evaluate bulbosity, tip-defining points, and ● Techniques to consider
size and shape – Dorsum/radix
– Nostrils: Evidence of notching or retraction and alar base ○ Reduction: Dorsal humpectomy with rasping of bone and

width excision of redundant cartilage


● Facial angles ○ Augmentation: Cartilage graft (diced or crushed cartilage,

– Nasolabial angle: 90 to 100 degrees in women, 90 degrees which may be wrapped in deep temporal fascia or
in men Surgical); rib graft remains ultimate option
– Nasofrontal angle: 115 to 130 degrees – Cartilage grafting: Donor sites include nasal septum, ear,
● Nasal function and rib
– Cottle maneuver to assess internal nasal valve competence – Spreader grafts: Improve airflow by widening internal
○ Compress opposite nostril and pull cheek laterally to open valve; also close open roof deformity and add support to
nostril the nasal dorsum
○ Test is positive if breathing improves as finger pulls cheek – Septal resection: Must leave an L-strut of at least 1 cm to
laterally preserve adequate septal support; perform only after hump
○ Indicates benefit for spreader grafting on affected side reduction to prevent over-resection of the cartilage
– Breathe Right® strip test: Similar to Cottle maneuver in – Lower lateral cartilages: Cephalic trim may be performed,
testing leaving 8 mm of cartilage to preserve support

110
Rhinoplasty

– Tip shaping: Columellar strut graft (prevents loss of tip


projection), transdomal and interdomal sutures (to narrow
27.6 Complications
the nasal tip), and tip graft (for additional nasal tip ● Hemorrhagic: Epistaxis
projection) – Septal hematoma: Needs immediate drainage to prevent
– Osteotomies: Lateral “low to high” osteotomies typically septal necrosis
performed to narrow a wide nasal base (with infracture); ● Infectious: Cellulitis, septal abscess
medial osteotomies as needed for wide nasal bones ● Aesthetic
○ Performed internally through nasal mucosa with – Malpositioned cartilage grafts
protected tip osteotomes, or externally with a 2 mm – Open roof deformity: Aggressive over-resection/rasp of
osteotome through the skin bony dorsum
– Inferior turbinates: May be outfractured, crushed, or – Saddle nose deformity: Aggressive septal resection leading
resected to improve nasal airway to nasal collapse
● Postoperative splinting – Pollybeak deformity: Inadequate reduction of the dorsum
– Internal: Petroleum gauze nasal packings, Doyle (silicone) or decreased nasal tip projection due to loss of support
splint – Over-reduction of nasal tip structures can lead to alar
– External: Denver (aluminum) splints, Aquaplast mold pinching, lateral nasal wall collapse, retracted ala, and/or
retracted columella
– Inverted V deformity: Disruption of the upper lateral
27.5 Ethical Considerations cartilages from the nasal bones and septum without
● When patients do not have reasonable expectations, surgeons restabilization
should forgo operating on these individuals.
● Functional: Vestibular stenosis and septal perforation
– Watch out for body dysmorphic disorder
● Soft tissue: Skin necrosis and prolonged edema
– “I'm sorry but I do not have the skills to give you the result
that you are hoping for”
● Be prepared to discuss management if the patient has had
27.7 Critical Errors
prior rhinoplasty by another surgeon and is either dissatisfied ● Inadequate preoperative assessment and planning
or experiencing a complication. ● Lack of appreciation of patient's physical characteristics
● Have a revision policy prepared, including conditions for ● Absence of coherent operative plan
revision and expectations for payment. ● Failure to evaluate other structures of the face other than
nose (e.g., retruded chin)

111
Case 28 Gender Transition (Male-to-Female)
Angie M. Paik and Daniel Kwan

Case 28 (a–c) A 22-year-old asserted female, natal male, presents to discuss top surgery for gender confirmation. She has been taking hormone
replacement for over a year.

113
Face, Cosmetic

– Male chest is wider with more pectoral muscle


28.1 Description development and stronger pectoral fascia
● Some breast development noted, well-defined lateral pectoral – Male areolae are smaller and more laterally positioned
border, no significant volume discrepancy, laterally – The male chest has a shorter distance between nipple and
positioned nipple-areolar complexes (NACs), and pink scars inframammary crease
overlying sternum and left shoulder ● Goals of surgery: Achieve a more feminine profile, cleavage,
● Tuberous breast deformity with herniation of breast tissue and desired breast volume
into the NAC, constricted base, and deficient lower pole ● Smoking cessation encouraged

28.2 Work-Up 28.5 Treatment


28.2.1 History ● Criteria to proceed with gender reassignment surgery (World
Professional Association for Transgender Health [WPATH]
● Duration of social transition as female Standards of Care)
● Use and duration of feminizing hormone therapy and – One referral letter from a qualified mental health
stability of breast development professional involved in patient's treatment confirming
● Use of padded brassiere or volume-enhancing prosthetic persistent well-documented gender dysphoria
● History of hypertrophic scarring or keloids – Capacity to provide informed consent
● Weight stability – Age of majority for a given country
● Medical comorbidities – Medical and mental health concerns are well-controlled
● Psychiatric history and any established relationship with a – Although not a requirement, it is recommended that patients
mental health professional undergo feminizing hormone therapy for at least 12 months
● History of smoking prior to surgery to optimize hormone-induced mammogenesis
● Family history or personal history of breast cancer ● Preoperative sizing sessions with prosthetics can help the
patient choose the desired volume and visualize the effect of
augmentation
28.2.2 Physical Examination
● Volume of native breast tissue
● Breast measurements: Sternal notch to nipple distance, 28.5.1 Surgical Techniques
inframammary fold (IMF) toNAC distance, breast width, ● Tissue expansion
height, projection, soft tissue pinch, and NAC dimensions – For larger volume augmentations, until the desired volume
● Skin quality is achieved
● Grade of breast ptosis ● Breast implant: Similar principles to normal breast
● Masses, discharge, or lymphadenopathy augmentation in cis-females
● Chest wall contour – Saline versus silicone
● Skin rashes – Smooth versus textured
– Anatomic versus round
– Subglandular versus submuscular
28.2.3 Pertinent Imaging or Diagnostic
– Axillary, inframammary, or periareolar approach
Studies – The IMF will be lowered and incisions in the fold should be
● Breast imaging if any abnormality is noticed on examination planned accordingly
● Fat grafting
– Provides a modest augmentation
28.3 Consultations – Forgoes need for an implant
– May need multiple rounds to obtain desired volume
● Mental health professional: Should be involved throughout the – Can be used as an adjunct to implant augmentation
preoperative and postoperative process ● Case scenario: Possible staged reconstruction with initial
● Endocrinologist or primary physician:Coordinating hormone tissue expansion and correction of tuberous breast deformity
therapy and follow-up (dual plane dissection and radial glandular release). Any fat
grafting or pocket revisions could be performed during the
implant exchange.
28.4 Patient Counseling ● Postoperative management
● Effects of feminization hormone therapy on the chest – If staged augmentation is performed, tissue expansion can
– Mammogenesis (similar to pubertal development) to be performed on a weekly basis until the desired volume is
varying degrees achieved
● Inherent differences between the male and female chest – Bandeau to encourage lower pole expansion and prevent
relevant to top surgery cephalic migration

114
Gender Transition (Male-to-Female)

28.6 Ethical Considerations 28.8 Critical Errors


● A 24-year-old asserted female with breast hypoplasia and a ● Failure to include the WPATH Standards of Care for gender
tight skin envelope is adamant that she wants a single-stage confirmation surgery
augmentation with 650 cc silicone implants. ● Ignoring need for multidisciplinary planning in counseling
– Counsel the patient that such a drastic increase in volume and treatment
could lead to complications including soft tissue ● Not considering staged augmentation for large volume
attenuation, ischemia, and implant extrusion, necessitating augmentation or not accounting for tuberous breast
multiple revisions. deformity

28.7 Complications 28.9 Other Considerations


● Hematoma: Meticulous hemostasis, surgical hematoma ● Bottom surgery (not likely to be tested)
evacuation – Criteria for orchiectomy
● Infection: Possible need for intravenous (IV) antibiotics and ○ Well-documented gender dysphoria, capacity to give

implant removal if there is progression despite outpatient informed consent, age of majority, well-controlled
therapy medical/mental health comorbidities, 12 continuous
● Capsular contracture: Risk factors should be discussed months of hormone therapy
preoperatively when choosing implant type and approach – Criteria for vaginoplasty
● Decreased nipple sensitivity: Avoid sharp dissection lateral to ○ Criteria for orchiectomy and 12 continuous months of

the pectoralis social transition


● Implant malposition: Deliberate pocket dissection – Surgical options
● Implant rupture: Obvious for saline implants ○ Orchiectomy, penectomy, creation of neovagina and

– Magnetic resonance imaging (MRI) for silicone implants if lining, reconstruction of urethral meatus, labiaplasty, and
rupture is suspected clitoroplasty
● Galactorrhea ○ Penile inversion vaginoplasty is most common. However,

– Hormonal evaluation to rule out pituitary gland etiology other techniques include nongenital skin flap
but could be normal side effect of hormone therapy vaginoplasty, intestinal vaginoplasty, and peritoneal
– Prolactin secretion may increase in the setting of breast vaginoplasty
manipulation and implant placement

115
Case 29 Gender Transition (Female-to-Male)
Angie M. Paik and Daniel Kwan

Case 29 (a–c) A 22-year-old asserted male, natal female, presents to discuss top surgery for gender confirmation.

117
Face, Cosmetic

Goals of surgery
29.1 Description ●

– Removal of breast tissue, address excess skin, reduction and


● Grade II breast ptosis, oval and moderately enlarged nipple- proper positioning of the NAC, elimination of the
areolar complexes (NACs), bilateral lateral chest adiposity inframammary fold (IMF), and aesthetic chest wall scars
● Bilateral chest and abdominal hair growth ● Expectations for surgery
● Asymmetry in shoulder height – Clear understanding of the limitations and benefits of each
surgical technique
– Counseling on smoking cessation, if appropriate
29.2 Work-Up
29.2.1 History 29.5 Treatment
● Duration of social transition as male ● Criteria to proceed with gender reassignment surgery: World
● Use and duration of masculinizing hormone therapy Professional Association for Transgender Health (WPATH)
● Use of breast binding standards of care
● Importance of nipple sensation – One referral letter from a qualified mental health
● History of hypertrophic scarring or keloids professional involved in the patient's treatment that
● Weight stability confirms persistent well-documented gender dysphoria
● Medical comorbidities – Capacity to provide informed consent
● Psychiatric history and any established relationship with a – Age of majority for a given country
mental health professional – Medical and mental health concerns are well-controlled
● History of smoking – For adolescents, they must also have had an ample social
● Family history or personal history of breast cancer transition and at least one year of testosterone treatment

29.2.2 Physical Examination 29.5.1 Surgical Techniques


● Perform chest examination to evaluate ● Peri-areolar, “Keyhole mastectomy”
– Volume of glandular tissue – Appropriate for patients with minimal breast tissue,
– Skin quality and amount of excess skin minimal excess skin, and good skin elasticity
– Grade of breast ptosis, position of NAC in relation to the IMF – Small and well-concealed scar
– Dimensions of the NAC – Little surgical exposure and higher risk for complication
– Masses, discharge, or lymphadenopathy – Peri-areolar mastopexy to re-size and/or re-position the
– Chest wall contour and axillary tissue excess NAC may be needed. This can possibly be staged if
mastectomy skin flaps are thin.
● Trans-areolarmastectomy
29.2.3 Pertinent Imaging or Diagnostic
– Appropriate for patients with small volume breast tissue
Studies and prominent nipple as it allows for nipple reduction
● Breast imaging, if any abnormality is noticed on examination ● Concentric circular mastectomy/mastopexy
– Appropriate for patients with moderate skin envelope or
small volume breasts with poor skin elasticity
29.3 Consultations – Allows for NAC reduction and re-positioning
– Permanent purse-string suture needed to prevent widening
● Mental health professional: Should be involved throughout the ● Inferior pedicled mammoplasty (can be with double incision
preoperative and postoperative process or inverted-T)
● Endocrinologist or primary physician: Coordinating hormone – Appropriate for patients with moderate skin elasticity and
therapy and follow-up grade II ptosis
– Inferior dermal pedicle positioned behind a superior skin
flap
29.4 Patient Counseling – Potential for postoperative return of nipple sensitivity
● Effects of masculinizing hormone therapy on the chest – Allows for NAC reduction and re-positioning
– Testosterone increases muscle mass, decreases fat mass, ● Double incision with free nipple grafts: Best option for the
and increases hair and acne case scenario
● Inherent differences between male and female chests – Good exposure to achieve the desired chest contour
relevant to top surgery – Ability to re-size and re-position the NAC
– The male chest is wider with more pectoral muscle – Low revision rate and good satisfaction profile
development, areolae are smaller, and distance between the – More visible scarring
nipple and inframammary crease is shorter ● Postoperative management
● Effects of binding: Loss of skin elasticity – Circumferential elastic compression bandage for 4 to 6 weeks

118
Gender Transition (Female-to-Male)

– Surgical drain ● Hypertrophic scarring, keloid formation


– Local wound care in the setting of a free nipple graft – Routine scar care, silicone sheets, and steroid injection
– Activity limitations
– Routine breast cancer screening
29.8 Critical Errors
29.6 Ethical Considerations ● Failure to include the WPATH Standards of Care for gender
confirmation surgery
● A 15-year-old asserted male patient comes to the office ● Ignoring the need for multidisciplinary planning in
seeking top surgery counseling and treatment
– It would be appropriate to proceed as long as the patient ● Poor choice of appropriate surgical technique based on the
meets WPATH Standards of Care adolescent criteria case scenario
– Joint discussions with the patient and his guardian/parents
to ensure informed consent and complete understanding of
the proposed surgical procedure 29.9 Other Considerations
● An asserted male interested in gender confirmation surgery ● Bottom surgery (not likely to be tested)
endorses suicidal ideation
– Criteria for salpingo-oophorectomy and hysterectomy
– Acute mental health issues should be addressed first by the ○ Well-documented gender dysphoria, capacity to give
patient's therapist or psychiatrist prior to proceeding with
informed consent, age of majority, well-controlled
surgical planning
medical/mental health comorbidities, and 12 continuous
months of hormone therapy
– Criteria for metoidioplasty or phalloplasty
29.7 Complications ○ Criteria for salpingo-oophorectomy/hysterectomy and 12

● Hematoma or seroma continuous months of social transition


– Meticulous hemostasis – Surgical options
– Surgical drain ○ Hysterectomy, salpingo-oophorectomy, vaginectomy,

– Compression garment postoperatively metoidioplasty, scrotoplasty, urethroplasty, insertion of


● Partial or full nipple necrosis testicular prosthetic, and phalloplasty
– Local wound care initially with debridement after full ○ Phalloplasty using the radial forearm free flap is most

demarcation common. Other options include pedicled or free


– Nipple reconstruction anterolateral thigh flaps and latissimus free flap
● Excess skin or contour irregularities
– Secondary procedures for skin reduction, dog-ear revision,
or re-excision

119
30 Open Wound: Upper Third of Leg 123
Section V
31 Open Wound: Middle Third of Lower Leg 127
Foot and Lower Extremity 32 Open Wound: Lower Third of Lower Leg 131
Reconstruction
33 Foot and Ankle Reconstruction 135

V
Case 30 Open Wound: Upper Third of Leg
Reena A. Bhatt

Case 30 A 55-year-old female presents after being hit by a car with open fracture and soft tissue loss of the right lower leg. Orthopedic surgeon has
placed a spacer in the bony defect and requests coverage.

123
Foot and Lower Extremity Reconstruction

30.1 Description 30.2.3 Pertinent Imaging or Diagnostic


● Gustilo IIIB (see ▶ Table 30.1) injury with open wound Studies
involving the upper third of the right lower extremity ● Plain films: Evaluation of bony injuries and prior
● The proximal third of the anterior tibia is exposed and a bony fixation
defect with a spacer is in place ● Computed tomography angiography (CTA)/Angiography
● There is a comminuted fracture and significant soft tissue indications: Preoperative planning for free flap
loss, without evidence of vascular injury reconstruction in patients with risk factors for peripheral
vascular disease or concerning physical exam

30.2 Work-Up
30.3 Consultations
30.2.1 History
● Vascular surgery: If vascular repair is required and surgeon
● Etiology
does not have microvascular expertise
– Traumatic: Mechanism of injury ● Orthopedic surgery: For management of bony injury and
– Tumor resection: Extent of resection
fixation
– Chronic: Etiology of wound and history of previous ● Infectious disease: For complex infectious processes such as
management (e.g., history of osteomyelitis—cultures,
osteomyelitis
antibiotics, debridement)
● Age and comorbidities (diabetes, peripheral vascular disease,
coronary artery disease, smoking history, steroid use, history
of or planned radiation, malnutrition)
30.4 Patient Counseling
● Input from multiple specialists, patient, and family is needed
in complex lower extremity trauma or oncologic defects with
30.2.2 Physical Examination reconstruction requirement
● In acute trauma cases, evaluate ABCs ● Risk-benefit analysis of reconstruction versus amputation:
● Assess wound Complex reconstruction, need for multiple surgeries,
– Soft tissue injury: Size, depth, and zone of injury duration of non-weight-bearing status, and potential major
– Degree of contamination and exposure of vital structures complications require consideration
– Vascular supply to lower extremity, bony defect, and ● Control risk factors such as smoking, poorly controlled
periosteal injury diabetes, and hypertension
– Tendon exposure: Paratenon intact versus damaged ● Lower extremity tumors: Counsel on potential complications
● Vascular examination such as delay in chemotherapy and radiation-induced
– Evaluate pulses, temperature, color, and turgor changes
– Ankle-Arm Index (AAI) measurements and Doppler
examination
● Neurologic examination: Check for peroneal or tibial nerve 30.5 Treatment
injuries
● Rule out Compartment Syndrome 30.5.1 Initial Management
– Tenderness over compartments involved (in Trauma)
– Pain out of proportion to injury on flexion/extension of
extremity ● Stabilizing fracture may require temporary external fixation
– Compromised neurovascular status (late finding) converted to internal fixation at time of reconstruction
– Absolute compartment pressures > 30 mm Hg or difference ● Restore vascular inflow, if required
between diastolic pressure and compartment ● Assess compartments and perform fasciotomies when
pressure < 30 mm Hg required

Table 30.1 Gustilo classification of open tibial fractures


Grade Wound Bony injury

I < 1 cm, clean, minimal soft tissue injury Simple, with minimal comminution

II > 1 cm, moderate contamination, moderate soft tissue injury Moderately comminuted fracture

IIIA < 10 cm, crushed tissue and/or contamination;local coverage Significant contamination or segmental bone loss; possible vascular
usually possible injury; highly contaminated wound; high velocity injury

IIIB > 10 cm, crushed tissue and/or contamination;inadequate soft As above


tissue; requires regional or free flap

IIIC Major vascular injury requiring repair for limb salvage;in some As above
cases, amputation is necessary

124
Open Wound: Upper Third of Leg

● Serially debride and washout wound until ready for definitive – Nonvascularized bone graft: Frequently cancellous, for
reconstruction defects < 6 cm
– Negative pressure dressing frequently used as interim ○ Requires healthy reconstruction and stable fixation

therapy ○ Can be performed at initial reconstruction or delayed

● Ensure appropriate antibiotic selection and duration in the – Vascularized bone graft: Defects > 6 cm; free fibula, iliac
setting of infection crest, scapula
– Distraction osteogenesis: Defects > 10 cm, can take up to
1 year
30.5.2 Timing of Reconstruction ○ Requires patient compliance; complications are frequent

● Early definitive reconstruction (< 72 hours) has the lowest flap – Masquelet technique: Temporary cement spacer followed
failure rates, postoperative infection rates, and fastest time to by staged bone grafting
bony union. Newer data suggests similar outcomes in flap
reconstruction within 10 days.
● Significant contamination and evolving demarcation of soft
30.5.5 Amputation
tissue injury are indications to delay definitive ● Non salvageable extremity: Complete disruption of posterior
reconstruction. tibial nerve; crush injuries with warm ischemia time
> 6 hours; serious associated life-threatening injuries;
medical comorbidities that preclude heroic measures for
30.5.3 Soft Tissue Reconstruction reconstruction
● Direct closure: Wound with limited contamination and ample ● Loss of sensation to plantar foot signifies poor prognosis for
local tissue functional recovery
● Skin graft: Requires clean vascularized bed ● Patient and surgeon decide risks of reconstruction outweigh
– Cannot be used in regions stripped of periosteum/ benefits
paratenon, over nerves, and over blood vessels
● Integra (bilayered dermal substitute): Followed 3 weeks later
by split thickness skin graft 30.6 Ethical Considerations
– Requires vascularized, healthy wound bed ● Patient compliance is a major factor for consideration of
– May be used over exposed structures complex reconstruction
– Potential salvage option if the wound is healthy and well- – In patient with multiple medical comorbidities and
vascularized; there should be no active infection inability to comply with perioperative regimen, the
● Pedicled muscle flap morbidity of the procedure may not be warranted
– Gastrocnemius muscle flap
○ Work horse for upper one-third leg reconstruction

○ Two heads (medial and lateral) with raphe in between


30.7 Complications
○ Single head or both can be used; medial head is longer

and broader ● Chronic osteomyelitis: Aggressive debridement with removal


○ Blood supply: Popliteal artery branches to medial sural of involved bone, soft tissue, and hardware
artery supplying medial head, and lateral sural for lateral – Often reconstructed with muscle/fasciocutaneous flaps
head of gastrocnemius – Infectious disease consultation for appropriateness and
– Tibialis anterior duration of antibiotic treatment
○ Important for ankle dorsiflexion, not expendable ● Fracture nonunion/malunion: Flap elevation, debridement,
○ Has been described to be split and used as bipedicled flap bone grafting, and revision of fixation
(preserves function) ● Flap loss: Another flap, if morbidity is acceptable; amputation
○ Supplied by perforators from anterior tibial artery is an alternative
● Free tissue transfer
– If there is extensive soft tissue trauma or size of wound
precludes use of local muscle flaps or flaps
30.8 Critical Errors
– Common options include ● In free tissue transfer: Erroneously choosing vessels within
○ Muscle (latissimus, rectus, gracilis) with skin graft the zone of injury
○ Fasciocutaneous flaps (anterolateral thigh [ALT]) ● Failure to adequately resect nonviable/infected tissue
● Delay in treatment, if not warranted
● Inadequate flap choice: Flap too small for debrided wound,
30.5.4 Bony Reconstruction
vessel length not adequate, and vein graft not considered/
● Management of bone gaps planned

125
Case 31 Open Wound: Middle Third of Lower Leg
Marten N. Basta and Daniel Kwan

Case 31 A 35-year-old male presents to the Emergency Department after motor vehicle accident, resulting in tibial bone loss and extensive overlying
soft tissue damage.

127
Foot and Lower Extremity Reconstruction

31.1 Description 31.3 Patient Counseling


● Open wound of the middle third of the right leg ● Comprehensive treatment of lower extremity trauma
– Gustilo IIIB (see ▶ Table 30.1): Comminuted bony defect requires multidisciplinary approach, likelihood of multiple
with extensive overlying soft tissue defect (> 10 cm) operations, and long rehabilitation process
without vascular injury ● Faced with multiple surgical options, surgeon and patient
– 30 cm × 12 cm soft tissue defect; large area of comminuted must consider long-term goals, pain management, and
tibial shaft exposed and stripped of periosteum functional recovery
● Minimize any modifiable risk factors for reconstructive failure
in preoperative setting whenever possible, such as smoking
31.2 Work-Up cessation, tight glycemic control, and blood pressure
medication compliance
31.2.1 History
● Etiology
– Traumatic: Mechanism of injury; evaluate for concomitant 31.4 Treatment
injuries
– Tumor resection: Extent of resection
31.4.1 Initial Management
– Chronic: Etiology of wound and history of previous ● Stabilize fracture
management ● Restore vascular inflow, if necessary
● Comorbidities of prognostic significance ● Assess compartments and perform fasciotomies when
– Peripheral vascular disease, cardiovascular disease, required
diabetes, smoking history, nutritional status, steroid use, ● Debride and washout wound until ready for reconstruction
and radiation treatment – Serial debridement usually required
– Vascular structures require immediate coverage
– Inadequate wound bed may be temporized with negative-
31.2.2 Physical Examination pressure wound therapy (Vacuum-Assisted Closure [VAC])
● In trauma cases, evaluate ABCs
● Assess wound Timing of Reconstruction: There are three phases of wound
– Soft tissue injury: Size, depth, and zone of injury progression
– Degree of contamination and exposure of vital structures ● Acute (1–5 days): Contaminated but not infected, edematous,

– Vascular supply to lower extremity, bony defect, and and hemorrhagic


periosteal injury ● Subacute (1–6 weeks): Colonized, infected, and bony

– Tendon exposure: Paratenon intact versus damaged demarcation still not clear
● Vascular examination ● Chronic (> 6 weeks): Granulating, contracting wound;

– Evaluate pulses, temperature, color, and turgor infection limited to scar; and bony demarcation clear
– Ankle-Arm Index (AAI) measurements and Doppler
examination Early definitive reconstruction (< 72 hours) has the lowest flap
● Neurologic examination: Check for peroneal or tibial nerve failure rates, postoperative infection rates, and fastest time to
injuries bony union
● Rule out Compartment Syndrome ● Significant contamination and evolving demarcation of soft

– Tenderness over compartments involved and pain out of tissue injury are indications to delay definitive reconstruction
proportion to injury on flexion/extension of extremity until the wound is clean and stable
– Compromised neurovascular status (late finding)
– Absolute compartment pressures > 30 mm Hg or difference
between diastolic pressure and compartment
31.4.2 Soft Tissue Reconstruction
pressure < 30 mm Hg ● Surgical reconstruction of soft tissue wounds divides leg into
three zones
– Upper third (see Chapter 30)
31.2.3 Pertinent Diagnostic Imaging – Middle third
Studies – Lower third (see Chapter 32)
● Direct closure: Remains an option in simple injuries with
● X-ray films: Evaluation of bony injuries
adequate tissue
● Computed tomography angiography (CTA)/Angiography
● Skin graft: Healthy vascularized bed necessary for adequate
indications: Preoperative planning for free flap
take
reconstruction in patients with risk factors for peripheral
– Should not perform over vital structures such as nerves,
vascular disease or concerning physical exam
vessels, and bone
● Integra (bilayered dermal substitute)
Classification: When a fracture is involved with the injury
– Has been used successfully for coverage of clean, stabilized,
● Gustilo Classification System of Open Tibial Fractures
well-vascularized wounds with exposed vital structures;
(see ▶ Table 30.1)

128
Open Wound: Middle Third of Lower Leg

may serve as a salvage measure when other procedures – Distraction osteogenesis (Ilizarov method): Can be used for
have failed bone gaps of up to 12 cm; requires long duration and
● Local muscle flaps: Most common reconstructive approach to patient compliance
middle third lower extremity defects – Masquelet technique: Temporary cement spacer followed
– Soleus muscle flap by staged bone grafting
○ Work horse flap for middle third defects – Limb shortening
○ Type II: Dominant pedicles (branches of popliteal,

posterior tibial, and peroneal arteries), minor pedicles


supply distal flap (post-tibial/peroneal) 31.5 Ethical Considerations
○ Can be split longitudinally (hemi-soleus) in smaller
● Reconstructive surgeon's role in decision to pursue limb
defects or rotated as propeller flap/reverse rotational flap
salvage versus amputation requires frank discussion with
based on distal medial perforators
patient regarding prospects of functioning limb
– Gastrocnemius flap ● Relative contraindications to salvage: Tibial nerve loss,
○ May be useful in middle third defects, although most
significant joint injury requiring fusion, insensate plantar
commonly used for reconstruction of proximal third
foot, single vessel lower extremity runoff and defect requiring
(see Chapter 30)
free tissue, and life-threatening concomitant injuries
○ Muscle can be divided into medial (larger) and lateral
● Surgery is never delayed for surgeon's convenience: Busy
heads
clinics have to cancel elective patients, etc.
– Tibialis anterior: Can be taken as a whole or longitudinally ● Patient compliance, social factors, and prolonged
split for coverage of smaller defects
immobilization
– Additional flap options for small defects: Flexor digitorum
longus, extensor digitorum longus, extensor hallucis
longus/flexor hallucis longus 31.6 Complications
● Free tissue transfer (see Chapter 32)
– Remains an option for coverage of large or complex defects ● Infection (cellulitis, osteomyelitis, hardware infection)
which cannot otherwise be managed with local or regional ● Flap loss/Exposure of bony hardware
flaps ● Nonunion/Malunion of fracture
● Vascular compromise
● Compartment syndrome
31.4.3 Bony Reconstruction
● If there is a bony defect, bone reconstruction must be
performed/planned
31.7 Critical Errors
– Essential elements of osseous healing are good blood supply ● Inability to recognize and debride all devitalized tissue
and stabilization; preserve periosteum wherever possible ● Failure to obtain adequate stabilization
● Options for bone gap reconstructions ● Delay in obtaining stable soft tissue coverage
– Nonvascularized bone graft: Recommended for ● Failure to recognize compartment syndrome
defects < 6 cm, although successful reports of up to 10 cm ● Performing vascular or nerve reconstruction within the zone
have been described of injury
– Free osseous or osteocutaneous flap transfer: Indicated in
defects > 6 cm
○ Common choices: Fibula, iliac crest, and scapula

129
Case 32 Open Wound: Lower Third of Lower Leg
Marten N. Basta and Daniel Kwan

Case 32 The orthopedics service contacts you to assist in management of a 42-year-old male involved in a motor vehicle accident with an open
comminuted tibial fracture of distal left leg. He has visible soft tissue deficit and areas of moderate periosteal stripping of bone.

131
Foot and Lower Extremity Reconstruction

32.1 Description 32.3 Patient Counseling


● Gustilo grade IIIB (see ▶ Table 30.1 ) open comminuted distal ● Comprehensive treatment of lower extremity trauma
tibial fracture requires multidisciplinary approach, likelihood of multiple
● Full-thickness defect of anteromedial distal third of leg with operations, and long rehabilitation process
inadequate local soft tissue ● Faced with multiple surgical options, surgeon and patient
must consider long-term goals, pain management, and
functional recovery
● Minimize any modifiable risk factors for reconstructive failure
32.2 Work-Up in preoperative setting whenever possible, such as smoking
cessation, tight glycemic control, and blood pressure
32.2.1 History medication compliance
● Mechanism of injury/Etiology of wound (traumatic,
oncologic, vascular insufficiency, etc.)
● Comorbidities of prognostic significance
32.4 Treatment
– Peripheral vascular disease, cardiovascular disease, ● Treatment of emergent and life-threatening issues during
diabetes, smoking history, nutritional status, steroid use, primary survey
radiation treatment, and preoperative functional status
● Support network
32.4.1 Acute Management
32.2.2 Physical Examination ● Fracture stabilization
● Restoration of vascular inflow (if indicated)
● In trauma cases, evaluate ABCs ● Fasciotomies (if indicated): Maintain high suspicion
● Assess wound ● Wound irrigation and debridement: Thorough debridement
– Soft tissue injury: Size, depth, and zone of injury is paramount to successful reconstruction; may require
– Degree of contamination and exposure of vital structures multiple sessions to achieve clean wound bed
– Vascular supply to lower extremity, bony defect, and ● Coverage of vital structures (nerves, vessels, bone)
periosteal injury ● Infection prevention: Aggressive debridement of devitalized
– Tendon exposure: Paratenon intact versus damaged tissue and appropriate antibiotic prophylaxis
● Vascular examination ● Inadequate wound bed may be temporized with negative-
– Examine pulses, temperature, color, and turgor pressure wound vacuum therapy
– Ankle-Arm Index (AAI) measurements and Doppler
examination
● Neurologic examination: Check for peroneal or tibial nerve 32.4.2 Bony Reduction and Stabilization
injuries ● Immobilization with cast; internal or external fixation
● Rule out Compartment Syndrome ● Management of bony gaps
– Compromised neurovascular status – Defects up to 8 to 10 cm: May be managed by
– Tenderness over involved muscular compartments and pain nonvascularized bone graft under muscle flap
out of proportion to injury on flexion/extension of extremity – Defects > 6 cm: May require vascularized osseous (or
– Absolute compartment pressures > 30 mm Hg or difference osteocutaneous) flap (e.g., free fibula)
between diastolic pressure and compartment – Distraction osteogenesis (Ilizarov method): Can be used for
pressure < 30 mm Hg bone gaps of up to 12 cm; requires long duration and
● Potential donor sites patient compliance
● Fractures: Gustilo Classification System of Open Tibial – Masquelet technique: Temporary cement spacer followed
Fractures (see ▶ Table 30.1) by staged bone grafting
– Limb shortening
32.2.3 Pertinent Diagnostic Imaging
Studies 32.4.3 Soft Tissue Reconstruction
● Plain films: Evaluation of bony injuries ● Goal: Stable soft tissue coverage with minimal donor
● Computed tomography angiography (CTA)/Angiography morbidity
indications: Preoperative planning for free flap ● Donor tissues and any vascular anastomoses must be outside
reconstruction in patients with risk factors for peripheral the zone of injury
vascular disease – Consider donor pedicle length requirement and possible
interpositional vein grafting
● Eradicate infection prior to reconstruction
32.2.4 Consultations ● In distal third wounds, free tissue transfer is frequently the
● Trauma surgery, orthopedic surgery, and vascular surgery (if choice among the options as few good local or regional flap
vascular injury) options exist

132
Open Wound: Lower Third of Lower Leg

– Primary closure/Secondary intention: Only for simple and ● Relative contraindications to salvage in acute setting: Tibial
small defects nerve loss, significant joint injury requiring fusion, insensate
– Skin grafting: If recipient bed is appropriate, potentially plantar foot, single vessel lower extremity runoff and defect
applicable after negative pressure wound therapy (Vacuum- requiring free tissue, and life-threatening concomitant
Assisted Closure [VAC]) injuries
– Integra (bilayered dermal substitute) ● Decision should be made through multidisciplinary approach
○ Has been used successfully for coverage of clean, ● Reconstructive cases must be scheduled in timely fashion
stabilized, well-vascularized wounds with exposed vital regardless of surgeon's busy schedule or issues of
structures inconvenience
○ Outer silicone layer is removed and a thin skin graft is ● Patient compliance, social factors, and prolonged
used to cover the newly vascularized material roughly immobilization are critical factors to consider with regards to
3 weeks following initial application amputation versus reconstruction
○ May serve as a salvage measure when other procedures

have failed
– Local/Regional flaps: Few good options for reconstruction 32.6 Complications
○ Consider dorsalis pedis flap, flexor hallucis longus,
● Infection (cellulitis, deep space soft tissue infection,
extensor digitorum longus, tibialis anterior
osteomyelitis, hardware infection)
○ Perforator/Propeller flaps: Reverse sural artery flap,
● Flap loss: Partial or complete
posterior tibial artery perforator/propeller (consider flap ● Lower extremity vascular compromise
delay, venous supercharge) ● Bony nonunion/malunion
– Free tissue transfer (gold standard for lower third wounds)
○ Muscle flap with split thickness skin graft for coverage

(e.g., latissimus, rectus abdominis, serratus anterior, 32.7 Critical Errors


gracilis)
○ Musculocutaneous (e.g., latissimus, parascapular, ● Inadequate debridement or delay in recognizing devitalized
transverse rectus abdominis myocutaneous [TRAM]) tissues
○ Fasciocutaneous (e.g., anterolateral thigh [ALT], radial ● Failure to provide appropriate immediate coverage of vital
forearm) structures
○ Osteocutaneous (e.g., fibula) ● Reconstruction within zones of injury
● Failure to recognize compartment syndrome
● Failure to obtain adequate stabilization
32.5 Ethical Considerations ● Exacerbation of tenuous vascular status (avoid by utilizing
end-to-side anastomoses, optimize vascular inflow prior to
● Limb salvage versus amputation: Frank discussion with
definitive reconstruction)
patient is necessary regarding prospects of functioning limb

133
Case 33 Foot and Ankle Reconstruction
Reena A. Bhatt

Case 33 (a–c) A 59-year-old female presents to the clinic. A car tire ran over her foot resulting in multiple metatarsal fractures that have been fixated.

135
Foot and Lower Extremity Reconstruction

Orthopedic surgery: Management of bony injury and fixation


33.1 Description ●

● Infectious disease: For complex infectious processes such as


● Roughly 17 cm × 6 cm region of necrosis of the plantar foot osteomyelitis
wrapping dorsally with obvious gangrene of the fourth and
fifth digits, and evolving region of demarcation dorsally
● Involves the weight-bearing region, with loss of soft tissue 33.4 Patient Counseling
support and coverage for the plantar second and third ● Input from multiple specialists, patient, and family is needed
metatarsals in case of complex lower extremity trauma or oncologic
● Large region of soft tissue injury with tendinous and bony defects with reconstruction requirement
involvement ● Reconstruction versus amputation: Complex reconstruction,
need for multiple surgeries, duration of non-weight bearing
status, difficulty in wearing shoe, and potential major
33.2 Work-Up complications require consideration
33.2.1 History ● Control risk factors such as smoking, poorly controlled
diabetes, and hypertension
● Etiology ● Lower extremity tumors: Counsel on potential complications
– Traumatic: Mechanism of injury such as delay in chemotherapy and radiation-induced
– Tumor resection: Extent of resection changes
– Chronic: Etiology of wound and history of previous
management—fixation, debridement, history of
osteomyelitis (cultures, antibiotics, debridements) 33.5 Treatment
● Age, Comorbidities (diabetes, peripheral vascular disease,
coronary artery disease, smoking history, steroid use, history 33.5.1 Initial Management (in Trauma)
of or planned radiation, malnutrition, functional status) ● Stabilize fracture: May require temporary external fixation
converted to internal fixation at time of reconstruction/joint
33.2.2 Physical Examination fusion
● Restore vascular inflow, if required
● In acute trauma cases, evaluate ABCs ● Serially debride and washout wound until ready for definitive
● Assess wound reconstruction
– Soft tissue injury: Size, depth, and zone of injury – Negative pressure dressing frequently used in interim
– Degree of contamination and exposure of vital structures ● In the setting of infection, ensure appropriate antibiotic
– Vascular supply to lower extremity, bony defect, and selection and duration of therapy
periosteal injury
– Tendon exposure: Paratenon intact versus damaged
– For foot injuries: Glabrous versus nonglabrous skin loss, 33.5.2 Timing of Reconstruction
weight bearing versus non-weight bearing ● Early definitive reconstruction (< 72 hours) has the lowest flap
● Vascular examination failure rates, postoperative infection rates, and fastest time to
– Evaluate pulses, temperature, color, and turgor bony union. Some data that suggests similar outcomes in flap
– Ankle-Arm Index (AAI) measurements and Doppler reconstruction is < 10 days.
examination ● Significant contamination and evolving demarcation of soft
● Neurologic Examination: Check for sensibility, especially of tissue injury are indications to delay definitive
plantar surface reconstruction.
● Definitive closure is done only after wound is clean and bony
33.2.3 Pertinent Imaging or Diagnostic stabilization is obtained.

Studies
● Plain films: Evaluation of bony injuries and prior fixation, and
33.5.3 Soft Tissue Reconstruction
for foreign bodies ● If plantar surface involved: Requires ability to withstand
● Computed tomography angiography (CTA)/Magnetic direct pressure and shear forces
resonance angiography (MRA)/Angiography indications: – Consider ability to wear normal footwear in the long term
Preoperative planning for free flap reconstruction in patients – Requires durable soft tissue with attachment to deeper
with concern for peripheral vascular disease structures
● Direct closure often not feasible
– Requires minimal contamination and adequate adjacent
33.3 Consultations tissue
● Skin graft: Cannot be used in regions stripped of periosteum/
● Trauma surgery: Depending on mechanism, acuity, and
paratenon, and over nerves and blood vessels
additional injuries
– For plantar surfaces: Traditional skin grafts often do not
● Vascular surgery: For vascular inflow concerns
have longevity

136
Foot and Ankle Reconstruction

● Integra (bilayered dermal substitute) – Chopart's amputation: Removes the forefoot and midfoot,
– May be successfully used over exposed structures preserving the talus and calcaneus
– Potential salvage option; there should not be active – Syme's amputation: Includes ankle disarticulation and
infection removal of malleoli
● Local flaps
– Limited by size and donor site morbidity; often cannot
approximate donor site 33.6 Ethical Considerations
– Length to width ratio should not exceed 1.5:1 in the lower ● Patient compliance is a major factor for consideration of
extremity
complex reconstruction
– Flaps should be designed outside the zone of injury ● In patient with multiple medical comorbidities and inability
● Pedicled flaps
to comply with perioperative regimen, the morbidity of
– Most muscles in this region have segmental blood supply
reconstruction may not be warranted
and can reconstruct only smaller defects
– Small defects (< 6 cm2): Can be reconstructed with type II
muscles such as abductor hallucis brevis, abductor digiti 33.7 Complications
minimi, and flexor or extensor digitorum brevis flaps
– Fasciocutaneous flaps: Frequently used on plantar surfaces ● Chronic osteomyelitis: Aggressive debridement with removal
– Pedicled flaps: Medial plantar, lateral calcaneal, dorsalis of involved bone, soft tissue, and hardware; often
pedis, reverse sural, and filet of toe flaps reconstructed with muscle/fasciocutaneous flaps; infectious
● Free microvascular flaps disease consultation for appropriateness of antibiotics and
– Given paucity of local/regional options, these flaps are often duration
utilized ● Fracture nonunion/malunion: Flap elevation, debridement,
– Consideration for maintenance of inflow to the foot and bone grafting, and revision of fixation
proper recipient vessel selection is important ● Partial/complete flap loss: Treatment with another flap if
– Free muscle flaps with split-thickness skin graft (STSG) morbidity acceptable versus amputation
○ Gracilis, rectus abdominis, serratus anterior, latissimus

dorsi (consider difficulty with crutch use)


– Fasciocutaneous flaps 33.8 Critical Errors
○ Radial forearm, anterolateral thigh, scapular, parascapular, ● In free tissue transfer: Choosing vessels within the zone of
and superficial circumflex iliac artery perforator flap injury
– Postoperative elevation secondary to concerns for venous ● Failure to adequately resect nonviable/infected tissue
outflow is critical ● Delay in treatment, if not warranted
○ Dangling protocols used, starting at 7 to 14 days ● Inadequate flap choice: Flap too small for debrided wound,
● Amputation vessel length not adequate, and vein graft not considered/
– Option in the setting of poor candidates, loss of sensibility planned
in plantar surface, or inability to comply with prolonged
rehabilitation
– Transmetatarsal amputation (TMA): Maintains significant
limb functionality by preserving the midfoot, distal to the
ankle joint
○ May provide adequate skin for direct closure

137
34 Breast Cancer Reconstruction 141
Section VI
35 Breast Augmentation 145
Breast 36 Secondary Breast Deformities
(Reconstructive and Aesthetic) 149

37 Tuberous Breast Deformity 153

38 Mastopexy/Augmentation 157

39 Breast Reduction 161

40 Gynecomastia 165

VI
Case 34 Breast Cancer Reconstruction
Victor A. King and Glyn E. Jones

Case 34 A 55-year-old woman with history of left-sided ductal carcinoma in situ. She is planned for a left-sided mastectomy and seeks reconstructive
options.

141
Breast

34.1 Description 34.4 Patient Counseling


● A 55-year-old female with evidence of left-sided breast ● Overall, 89.7% 5-year survival rate; 98.7% 5-year survival for
biopsy and large C-cup breasts localized breast cancer
● Image shows breast asymmetry with the left side higher than ● BRCA mutations: Lifetime risk > 50%, with increased risk of
the right, suggestive of possible previous procedure versus contralateral disease
baseline asymmetry ● Breast reconstruction often requires multiple operations
● Notable laxity of the lower abdomen; reasonable candidate
for autologous reconstruction
34.5 Treatment
34.2 Work-Up ● The most important consideration is oncologic safety and
prevention of recurrence
34.2.1 History ● Surgery, chemotherapy, and radiation are mainstays of
treatment
● Breast cancer diagnosis: Ductal carcinoma, lobular carcinoma,
and inflammatory breast carcinoma are most common
– Noninvasive versus invasive 34.5.1 Preoperative and Surgical
– Location, tumor size, node, and metastasis (TNM) Considerations
classification, and contralateral disease
● Previous procedures or surgeries: Oncologic or aesthetic
● Planned lumpectomy versus mastectomy
breast procedures, history of surgery, or radiation elsewhere
● Immediate reconstruction: Psychosocial benefit, with
in the body reduced number of operations
● Breast screening and diagnostic testing: Mammography,
● Delayed reconstruction
ultrasound, magnetic resonance imaging (MRI), computed – Overall lower rate of complications
tomography (CT) scan, pathology, and molecular and genetic – Preferred if medical comorbidities are significant,
testing tissue quality is poor, oncologic safety of primary
● Age, pre- versus post-menopausal, pregnancy plans, and procedure is not certain, reconstruction risks are
breastfeeding delaying adjuvant therapies, patient is not ready
● Family history of breast and other cancers to make an informed decision, or breasts are large
● History of smoking/tobacco use and comorbid conditions and ptotic
● Patient's reconstructive desires, expectations, and desired
● No reconstruction at all remains an option for treatment,
timing especially in very sick patients

34.2.2 Physical Examination 34.5.2 Partial Mastectomy/


● Body habitus and body mass index (BMI): Weight (kg)/
Lumpectomy Reconstruction
Height (m2) ● Lumpectomy with postoperative radiation
● Current breast characteristics (including contralateral breast) – Breast conservation = lumpectomy coupled with
– Measurements (see ▶ Fig. 36.1, ▶ Table 36.2) postoperative radiation
– Breast masses and presence of lymphadenopathy – Equivalent survival outcomes compared to mastectomy
– Quality and quantity of skin; presence and location of scars ● Oncoplastic surgery
and deformities – Oncoplastic reduction reduces breast size to help reduce
● Donor site availability and presence of scars: Abdomen, back, postradiation morbidity and reshapes breast to reduce
buttocks, and thighs localized deformity
● Lymphedema – Contraindications: Inflammatory breast cancer,
multicentric malignancies, diffuse malignancy, history of
irradiation, scleroderma, or persistently positive
34.2.3 Pertinent Imaging or Diagnostic margins
Studies
● Mammogram or other diagnostic imaging (especially if
operating on unaffected breast)
34.5.3 Prosthetic (Implant-based)
● Consider CT or MR angiography, if planning autologous free Reconstruction
flap ● Ideal patients: Small- and moderate-sized breasts and
bilateral reconstructions
● Reduced operative time, recovery time, cost, and no tissue
34.3 Consultations donor site morbidity
● Surgical (breast) oncology, medical oncology, and radiation ● Prior radiation therapy is a relative contraindication
oncology (if needed) ● Two-stage (expander-based) procedures in the immediate or
● Tumor board delayed setting

142
Breast Cancer Reconstruction

– Expander placed in submuscular plane with inferior ○ Good option if abdominal wall morbidity is not
border typically supported by acellular dermal matrix acceptable to patient
(i.e., Alloderm) ○ Advantages: Reliable blood supply, easy to harvest,

– Expander may be partially filled at time of surgery but functional loss often well tolerated, and provides
volume should not be at the expense of tension free closure nonradiated tissue to cover implant
and ischemia of the flap ○ Disadvantages: Flap volume small and usually combined

– Patient seen postoperatively and expanded on a weekly with implant/expander; axillary tunnel may injure
basis lymphatics
– Adjuvant radiation therapy may commence after final fill ● Free tissue transfer
– Permanent implant placed after 6 to 12 weeks after – This is often the go-to operation for many reconstructive
expansion is completed microsurgeons
● Single-stage (direct-to-implant) reconstruction – Identification of reliable perforators and muscle sparing
– Higher risk of complications relative to two-stage procedure techniques have allowed muscle integrity to be preserved,
– Assess mastectomy flap intraoperatively with sizer in place reducing donor site morbidity
– Consider avoiding in smokers or if adjuvant radiation – Free TRAM and DIEP flaps
therapy is planned ○ Based on deep inferior epigastric vessels

– Unpredictability of oncologic defects has a greater bearing ○ Recipient vessels: Internal mammary (preferred),

on final outcome thoracoacromial, and thoracodorsal vessels


● Submuscular versus prepectoral coverage ○ Advantages: Muscle sparing results in minimal abdominal

– Expanders typically placed in submuscular plane as wall morbidity


mastectomy skin flaps are too thin and cannot provide ○ Disadvantages: Technically demanding; BMI > 35 are poor

stable coverage candidates


– Acellular dermal matrices used for complete expander
coverage
– Prepectoral device placement eliminates postoperative 34.6 Ethical Considerations
animation deformity and may be appropriate in select ● Never embark on a new or complex procedure without
circumstances
appropriate counseling and consent
● Provide impartial evidence-based counseling regarding
34.5.4 Autologous Reconstruction autologous versus implant-based reconstruction
● Provide equal access to all patients
● Natural feel; provides additional skin if envelope is
inadequate or for radiation
● Implant may be added for additional volume 34.7 Complications
● Flaps and donor sites include:
– Abdomen: Pedicled transverse rectus abdominis ● Mastectomy skin flap necrosis: Full thickness/risk for implant
myocutaneous (TRAM), free TRAM, deep inferior epigastric exposure
perforator (DIEP), superficial inferior epigastric artery – Debride/excise early and reclose or cover with salvage flap
(SIEA), and deep circumflex iliac artery (DCIA) flaps (e.g., latissimus)
– Posterior trunk: Latissimus dorsi flap, inferior gluteal artery ● Seroma: Conservative management, aspiration in office,
perforator (IGAP), and superior gluteal artery perforator catheter placement, or incision and drainage depending on
(SGAP) flaps size of collections
– Thigh: confirmed anterolateral thigh (ALT), transverse ● Expander infection
upper gracilis (TUG), and profunda artery perforator (PAP) – Cellulitis: Start oral or intravenous (IV) antibiotics;
● Pedicled flaps ultrasound to exclude seroma
– Pedicled TRAM flap – Explant if patient fails IV antibiotic therapy or purulence
○ Based on ipsilateral or contralateral rectus abdominis within the pocket is suspected
muscle; tunneled through subcutaneous tissue to inset on ● Fat necrosis: Both skin and fat necrosis should be debrided
chest wall early and closed
○ Contraindications: Previous subcostal incision transecting ● Vascular compromise (free flap): Survival associated with
the superior epigastric vessels or internal mammary early recognition and time to revision of anastomosis
artery cardiac revascularization or internal mammary
artery is harvested with cardiac revascularization
○ Advantages: Safe, quick, and supplies large amounts of
34.8 Critical Errors
tissue ● Recommending prosthetic reconstruction for an irradiated
○ Disadvantages: Sacrifices rectus abdominis muscles, breast
which may result in reduction of abdominal wall strength ● Discussing or performing a reconstructive option that you do
and integrity not know very well
– Pedicled latissimus dorsi myocutaneous flap ● Inability to manage complications
○ Based on thoracodorsal artery ● Failure to screen remaining breast for cancer preoperatively

143
Case 35 Breast Augmentation
Zachary Okhah and Richard Zienowicz

Case 35 (a, b) A 21-year-old female presents to your office to discuss breast augmentation.

145
Breast

– When counseling on implant options, patients should be


35.1 Description informed that BIA-ALCL is a predominantly textured implant-
● Hypomastia: Small A cup breasts with mild asymmetry associated disease, although it is premature to conclude at this
● Right inframammary fold is slightly higher than the left side point that it cannot occur in smooth implants.
● Right nipple is slightly higher than the left nipple – Both silicone (57%) and saline (43%) implants have been
linked to BIA-ALCL with no specific predilection.

35.2 Work-Up
35.4 Treatment
35.2.1 History
● Standard perioperative care
● Age, medical comorbidities, anticoagulant use, and smoking – A single preoperative dose of a cephalosporin is indicated
history – Sequential compression devices should be used before
● Pregnancy/breastfeeding history; plans for future induction if general anesthesia is administered
childbearing
● Personal history of breast disease and/or procedures, prior
mammography or ultrasound 35.4.1 Implant Selection
● Family history of breast cancer ● Implant size
● Current bra size and desired breast size – Saline implants available up to 1,000 mL
● Motivation for surgery – Silicone implants available up to 800 mL
– Silicone implants: Only FDA-approved for primary
35.2.2 Physical Examination augmentation in women at least 22 years of age
– Silicone implant rupture: The FDA recommends magnetic
● Evaluate breast shape, skin quality, and adequacy of tissue resonance imaging (MRI) screening for silicone implant
envelope (e.g., upper pole pinch thickness) rupture at 3 years after implantation, then every 2 years
● Identify any asymmetries (volume, nipple–areola complex, thereafter
inframammary fold position) and thoracic wall abnormalities ● Implant placement
● Palpate for breast masses or axillary lymphadenopathy; – Subglandular: Can have pleasing aesthetic results, but has
identify skin dimpling or nipple discharge higher contracture rates and implant palpability
○ May complicate future mammography because the

implant is present adjacent to glandular tissue


35.2.3 Diagnostic Studies
– Submuscular: Placement is completely under pectoralis
● American Cancer Society guidelines for clinical breast exam major muscle; decreased capsular contracture rates; breast
(CBE) and mammography should be followed may distort with muscle contraction
– Clinical breast examination every 3 years for women aged – Dual plane: Placement under pectoralis major superiorly,
20 to 39 years but subglandular inferiorly; decreased contracture rates,
– Breast examination annually for women aged 40 and older but allows expansion of the inferior pole
– Yearly mammograms for women aged 40 and older ● Incision/Approach: Periareolar, inframammary, axillary, and
● Breast masses or lymphadenopathy discovered on physical transumbilical
examination should be evaluated before augmentation – Current planning involves a tissue-based approach that
acknowledges individual soft-tissue limitations rather than
a strictly volumetric approach
35.3 Patient Counseling
● An effective consultation establishes an accurate 35.4.2 Cancer Screening
understanding of the patient's goals from the surgeon's
perspective, and establishes realistic expectations of the final ● Breast augmentation does not increase the risk for breast
result from the patient's perspective. cancer, but implants may interfere with mammographic
● Informed consent must include management of patient screening
expectations and thorough discussion of existing ● Submuscular implants interfere significantly less than
asymmetries, potential complications, rupture screening subglandular implants
recommendations, rates of revisional surgery (approximately ● Silicone gel implants do not affect the incidence of connective
20%), responsibility for cost of revisions, and eventual need tissue diseases
for implant exchange or removal
● Breast Implant Associated Anaplastic Large Cell Lymphoma
(BIA-ALCL)
35.5 Ethical Considerations
– All patients being evaluated for breast implants should ● Avoid patients with comorbidities that place them at higher
be informed of the risks and common presentations of risk for complications.
BIA-ALCL. ● Patients may have unreasonable demands regarding breast
– Clinical experience has shown that this process is generally size or expectations. The surgeon is not mandated to operate
well received and is not disruptive to patient interactions. on such patients.

146
Breast Augmentation

● Be prepared to discuss management if the patient has had Table 35.1 Baker capsular contracture classification
prior augmentation by another surgeon and is either
Grade Severity Findings
dissatisfied or experiencing a complication.
● Have a revision policy prepared, including conditions for I Normal Natural feel; normal in size and shape
revision and expectations for payment. II Minimal Slightly firm; normal appearance

III Moderate Firm; appears abnormal and notable to patient


35.6 Complications IV Severe Hard; painful to the touch;appears abnormal
● High revision rate (about 20%)
● Capsular contracture
– Baker capsular contracture classification (▶ Table 35.1)
35.7 Critical Errors
– Inframammary incision < Periareolar incision ● Failing to recognize common abnormalities complicating
– Submuscular placement < Subglandular placement augmentation (e.g., tuberous breast deformity, thoracic wall
– Subglandular placement: Textured < Smooth abnormalities, significant asymmetries)
– Submuscular placement: No advantage to texturing in ● Performing a procedure outside the standard of care (e.g.,
capsular contracture silicone implant in patient younger than 22 years, extra-large
● Infection volume augmentation, transumbilical approach)
● Rupture ● Inability to handle common postoperative complications
– Saline: Risk approximately 1% per year ● Failure to obtain preoperative screening mammogram in
– Silicone: Risk approximately 0.5% at 3 years, 5.5% at 6 years women > 40 years of age
● Rippling, palpable
● “Double-bubble” deformity
– Type A: Implant sits above breast mound
– Type B: Implant sits below breast mound

147
Case 36 Secondary Breast Deformities (Reconstructive and
Aesthetic)
Elizabeth Kiwanuka and Karl H. Breuing

Case 36 (a–d) A 53-year-old female presents after 2 years of bilateral mastectomy and implant reconstruction without chemotherapy or radiation.
She is not happy with the reconstructive outcome and requests revision.

149
Breast

36.1 Description 36.3 Patient Counseling


● Breast mounds are large and out of proportion to her body ● Discuss risks and benefits of available operative techniques to
habitus address patient's concerns
● Irregular contour of the breasts in repose ● Establish an accurate understanding of the patient's goals from
● Breast asymmetry with the right breast larger than the left the surgeon's perspective and realistic expectations of the
● Nipple-areola complexes (NACs) are not in the center of the final result from the patient's perspective
breast mounds – If the patient's expectations are unrealistic, do not be afraid
● Dynamic animation deformity of breasts with activation of to communicate this
pectoralis muscle – If necessary, recommend getting a second opinion
● Discuss other treatment options/alternatives, if available
● Thorough discussion of existing asymmetries and potential
36.2 Work-Up complications
● If implants are involved: Rupture screening
36.2.1 History recommendations, rates of revisional surgery (approximately
● Age, medical comorbidities, anticoagulant use, and smoking 20%), responsibility for cost of revisions if the procedure will
history not be covered by insurance, and eventual need for implant
● Pregnancy and breastfeeding history, and plans for future exchange or removal
childbearing – Inform the risks and common presentations of Breast
● History of breast disease (e.g., fibro-adenomatosis, nipple Implant Associated Anaplastic Large Cell Lymphoma
discharge, mastodynia) (BIA-ALCL)
● Surgical history of all breast procedures performed and – BIA-ALCL is a predominantly textured implant–associated
postoperative courses disease, although it is premature to conclude at this point
● Prior pertinent imaging studies (mammography, ultrasound, that it cannot occur in smooth implants
computed tomography [CT], magnetic resonance imaging – Both silicone (57%) and saline (43%) implants have been
[MRI]) linked to BIA-ALCL with no specific predilection
● Family history of breast cancer ● Discuss potential need for subsequent additional surgeries
● Current bra size and desired breast size (e.g., repeat fat grafting, at the earliest 3 months later)
● Motivation for surgery (i.e., chief complaints) ● Estimated time needed for recovery and postoperative
– Itemize patient's chief complaints (list each deformity activity restrictions
described during your consultation)
– Establish timeline when deformity is first noted and
whether stable or still progressing 36.4 Treatment
● Establish and document a detailed operative treatment plan
36.2.2 Physical Examination (see ▶ Table 36.1, ▶ Table 36.2, ▶ Table 36.3)
– Match each deformity with the intended technique to
● Perform static and dynamic examination: Breast animation address it
deformity (BAD) – Sequence of operative steps to save time and prevent
– Examine with the patient in standing, sitting, and supine mistakes
positions ○ For example: Harvest fat first so that processing can be
● Perform qualitative exam (▶ Fig. 36.1) and establish done on the back table while actual surgery can proceed;
quantitative measurements (▶ Table 36.1, ▶ Table 36.2). unpack implant/acellular dermal matrix (ADM) right
● Evaluate breast shape, skin quality, and adequacy of tissue before using it, etc.
envelope ● Preoperative markings with the patient in an upright sitting
● Identify any asymmetries of the breast (volume, NAC, position:
inframammary fold [IMF] position), thoracic wall, etc. – Standard reference lines (i.e., breast meridian, jugular fossa
● Palpate for breast masses or axillary lymphadenopathy nipple, IMF)
– Identify skin dimpling or nipple discharge – Foot print of breast mound/implant pocket (present and
– Any findings must be evaluated before surgery desired)
– IMF positions (present and desired)
– Position of the NAC (present and desired)
36.2.3 Pertinent Imaging or Diagnostic
– Areas that require volume enhancement (fat grafting)
Studies versus volume reduction (lipo)
● Order diagnostic studies (ultrasound, CT, MRI) as indicated if – Areas identified as adhesion zones that require adhesiolysis
needed to establish operative plan and/or onlay grafting
– Rule out implant rupture, evaluate for fat necrosis, and ● Display written operative plan (sequence of steps involved)
assess chest wall abnormality for all to review (▶ Fig. 36.1)
– American Cancer Society guidelines for clinical breast exam – Printout or write on white board
(CBE) and mammography should be followed (see Chapter 35)

150
Secondary Breast Deformities (Reconstructive and Aesthetic)

Table 36.1 Breast examination—descriptive findings Table 36.2 Breast measurements—objective findings
Right Left Right (cm) Left (cm)

Asymmetry in shape Yes/No Yes/No Sternal notch to nipple


distance
Asymmetry in volume/ Excess/Deficit Excess/Deficit
size Mid-clavicle to nipple

Soft tissue deficit Yes/No Yes/No Mid-clavicle to IMF

Foot print violation Yes/No Yes/No Areolar width/length

Bottoming out Yes/No Yes/No Nipple to mid-sternum


(skin surface)
Upper pole over Yes/No Yes/No
projection Nipple to mid-sternum
(air)
Symmastia Yes/No Yes/No
Base width
Base width Too large/Small Too large/Small
Vertical breast height
Vertical breast height Too high/Low Too high/Low
Arcus (IMF to NAC) Neutral/Stretched Neutral/Stretched
Animation deformity Yes/No Yes/No
IMF Higher/Lower Higher/Lower
IMF Too high/Low Too high/Low
Overhang (lowest
Capsular contracture
breast to IMF)
(Grade 1–3)
Ptosis grade (1–3)
Double Bubble Yes/No/A/ Yes/No/A/
(Type A/B) Costo-cartilaginous Prominent/Normal Normal/Prominent
junction
Costo-cartilaginous Prominent/Normal Prominent/Normal
junction Anterior chest wall Prominent/Normal Normal/Prominent

Anterior chest wall Prominent/Normal Prominent/Normal Spinal deformity Present (Yes/No)

Abbreviation: IMF, inframammary fold. Implants in place Yes/No/Details


Volume, style,
manufacturer

Volume to be added Yes/No


– Display preoperative photos illustrating the deformities to
be addressed Volume to be reduced yes / no
● Assess the capsule: Re-operative surgery most often involves Fat grafting needed Yes/No/Where
capsulotomy, capsulectomy, or capsulorrhaphy
● Volume reduction or enhancement: Also includes the ADM needed Yes/No
contralateral breast for symmetry Present cup size
● NAC positioning: Mastopexy (peri-areola only or (pre-OP)
pedicled) Desired cup size
● Animation deformity: Change of pocket positioning versus (post-OP)
selective denervation of the lateral pectoral muscle involved
Abbreviations: ADM, acellular dermal matrix; IMF, inframammary fold;
NAC, nipple-areola complex.

36.5 Complications
● Persistent deformity and asymmetry
● Residual animation deformity 36.6 Critical Errors
● Infection (repeat surgery)
● Operating on patient with unrealistic expectations regarding
● Bleeding and hematoma
postoperative outcome
● Seroma (capsule work and use of ADM)
● Failing to diagnose/identify each individual deformity
● Skin necrosis (in areas of thin skin envelope after fat grafting)
(usually more than one)
● Fat necrosis and oil cysts (after fat grafting)
● Missing coexisting thoracic wall abnormalities
● Radiographic changes in mammogram after fat grafting (to be
● Forgetting to order backup implants in case of accidental
expected but can be differentiated from malignancy)
puncture
● Capsular contracture

151
Breast

Table 36.3 Treatment plan


Intended operative steps Right Left

Capsulotomy/Capsulectomy Yes/No Yes/No

Capsulorrhaphy Yes/No Yes/No

Volume reduction/Enhancement Yes/No Yes/No

Breast tissue reduction Yes/No Yes/No

Implant exchange Yes/No Yes/No

Skin envelope reduction Yes/No Yes/No

NAC position (Mastopexy) Yes/No Yes/No

Soft tissue support needed Yes/No Yes/No

ADM/mesh needed Yes/No Yes/No

Fat grafting Yes/No Yes/No

Animation deformity Yes/No Yes/No Fig. 36.1 Recording of breast measurements.

Implant pocket change Yes/No Yes/No


(retro vs. pre-pectoral)
● Performing a procedure outside the standard of care
Selective denervation Yes/No Yes/No ● Inability to handle common postoperative complications
(lateral pectoral muscle)
● Failure to obtain preoperative screening mammogram in
Abbreviations: ADM, acellular dermal matrix; NAC, nipple-areola women > 40 years of age and (younger if personal/family
complex. history of breast cancer)

152
Case 37 Tuberous Breast Deformity
Lauren O. Roussel and Karl H. Breuing

Case 37 A 28-year-old female requesting correction of breast appearance and asymmetry.

153
Breast

37.1 Description 37.2.3 Pertinent Imaging or Diagnostic


● Bilateral tuberous breast deformities with ptosis Studies
– Left breast slightly larger and more ptotic appearing than ● Mammogram (if clinically indicated)
right
● Skin and soft tissue deficiency in inferior pole of both breasts
with short nipple–areola complex (NAC) to inframammary 37.3 Consultations
fold (IMF) distance
● Protuberant areolar complexes bilaterally ● Psychiatry/Developmental psychology: Some patients,
especially adolescents, may require intervention
given the psychological morbidity associated with
37.2 Work-Up this condition

37.2.1 History
● Patient's age/pubertal maturity
37.4 Patient Counseling
● Patient's goals and expectations ● It is essential to help the patients understand that special
● History of prior breast surgeries techniques are needed to address their anatomy
● History of smoking/tobacco use – Mastopexy or augmentation alone is insufficient in the
● Plan for future childbirth setting of tuberous breast deformity
● Anticipated changes in weight ● Staged or delayed reconstruction may be the best option for
● Personal or family history of breast cancer treatment of some patients

37.2.2 Physical Examination


37.5 Treatment
● Evaluate size and shape of bilateral breasts
– Note breast asymmetries 37.5.1 Staging of Reconstruction
● Examine for presence of ptosis or pseudoptosis (see
● Single-stage procedure
▶ Table 37.1)
– Release of constricting bands
– Pseudoptosis: The nipple is at or above IMF with lower pole
– Permanent implant placed to correct breast asymmetry
breast ptosis
○ May involve unilateral or bilateral implant placement
● Important preoperative measurements include IMF level, NAC
with or without contralateral symmetry surgery
to IMF distance, sternal notch to nipple distance, and breast
– NAC reduction when there is an enlarged NAC or
and chest width
pseudoherniation of the NAC
● Characteristics of Tuberous Breast Deformity
– Most commonly performed as single procedure or single
– Constricted skin envelope in vertical and horizontal
stage procedure
dimensions
● Two-stage procedure: Useful in severe asymmetry and when
○ Constricted or narrowed breast diameter
significant deficiency of inferior pole skin envelope exists
○ Elevated IMF
– Tissue expander placed initially to allow for correction of
○ Short NAC to IMF distance
significant volumetric asymmetry
○ Deficient skin envelope in inferior pole
– Permanent implant
– Hypoplasia of breast parenchyma
– Herniation or pseudoherniation of breast parenchyma into
NAC 37.5.2 Components of Surgery
● Grolleau's classification of tuberous breast deformity
– Type I: Lower medial quadrant deficiency ● Surgical goals: Improving breast symmetry, increasing breast
– Type II: Lower medial and lateral quadrant (entire lower base width, lowering the IMF, and reduction of areolar size if
pole) breast deficiency necessary
– Type III: Deficiency of all quadrants ● Surgical approaches
– Infra-areolar approach: Allows access to fibrous,
constricting bands requiring release and potential implant
Table 37.1 Regnault classification of breast ptosis
placement
Grade Findings – Periareolar approach: Can be used if a circumareolar
I (Mild) Nipple at IMF mastopexy is performed concurrently
● Surgical principles
II (Moderate) Nipple below IMF, above most dependent part of breast
– Lower the IMF: Undermine the lower pole of the breast
III (Severe) Nipple at inferior-most aspect of breast subcutaneously to the desired position of the IMF
Pseudoptosis Nipple at or above IMF with lower pole breast ptosis
– Release subareolar constriction ring: May be performed by
radial scoring or dividing the inferior pole of the breast
Abbreviation: IMF, inframammary fold. vertically or horizontally

154
Tuberous Breast Deformity

Volume augmentation: If volume needs to be added, a


permanent implant or temporary tissue expander is placed


37.6 Complications
● Implant placement: Commonly placed in dual plane ● Residual asymmetry
– Allows soft tissue coverage of the implant superiorly ● Persistent deformity
– Encourages preferential expansion in the inferior pole at ● Complications of implant or tissue expander
the implant-parenchyma junction
● Can consider autologous fat grafting for improved symmetry
37.7 Critical Errors
37.5.3 Contralateral Breast Procedures ● Failure to correctly identify the tuberous breast deformity
● Augmenting the breast without addressing the tuberous
● Reduction/mastopexy or augmentation may be performed to
qualities
improved symmetry

155
Case 38 Mastopexy/Augmentation
Rachel R. Sullivan

Case 38 A 49-year-old female with chief complaint of loose skin and volume loss of the breasts after massive weight loss.

157
Breast

38.1 Description Table 38.1 Comparison between saline and silicone implants
Pros Cons
● Patient with severe deflation of bilateral breasts and grade 3
ptosis Saline Easily adjustable size Rippling
● Breasts appear to have a very long sternal notch to nipple- Low contracture rates Less natural feel
Rupture identified clinically
areola complex (NAC) distance
● Notable sagging and laxity of skin Silicone More natural feel Higher contracture rates
Lighter weight than saline Larger incisions for placement
Lower notable rippling MRI necessary to evaluate for
38.2 Work-Up rupture

Abbreviation: MRI, magnetic resonance imaging.


38.2.1 History
● Determine patient's motivation for seeking surgery and ● Postoperative breast cancer surveillance
desired result: Breast lift, increased volume, correction of ● Expected postoperative course and return to function
asymmetry, or a combination ● Possible use of drains, postoperative garments, and
● Prior history of breast or thoracic procedures: May impact instructions
blood flow – Postoperative activity and return to work
● Identify and discuss comorbidities that may affect healing
(e.g., diabetes, cardiopulmonary disease)
● Smoking history 38.4 Treatment
● Personal risk/family history of breast cancer and prior
● All women considering breast surgery should undergo
mammography
screening for breast cancer
● Pregnancy and breastfeeding history, and plans for future
– American Cancer Society Guidelines
pregnancy
○ Age 40 to 44—women should have choice to start
● Current breast size and desired breast size
screening
○ Age 45 to 54—women should get mammogram yearly

38.2.2 Physical Examination ○ Age 55 and above—either yearly of every 2 years

screening
● Systematic evaluation of the breast (▶ Fig. 36.1, ▶ Table 36.1)
○ Higher risk patients should start screening earlier
– Relationship of nipple to inframammary fold (IMF)
– Relationship of breast tissue to IMF
– Overall size and surface area of the breast 38.4.1 Surgical Principles
– Quality of skin (elasticity, thickness, striae) and breast
● Mastopexy addresses ptosis
parenchyma
– Ptosis is defined by IMF
– Breast and/or chest wall asymmetries
– Regnault classification (see ▶ Table 37.1)
– Palpate for masses and/or nipple discharge
● Augmentation addresses volume loss
– Assess nipple–areola complex (NAC) sensation
– Addition of augmentation particularly common with
● Key measurements
massive weight loss or deflation after breastfeeding
– Sternal notch to nipple distance, nipple to IMF during
stretch, breast base width, superior and inferior pole pinch
thickness, anterior pull skin stretch, and estimated 38.4.2 Surgical Options
parenchymal fill
● Preoperative photographic documentation
● Augmentation alone: May get adequate lift in minimal ptosis
cases
● Mastopexy alone: Lifting the NAC and tightening the skin
38.3 Patient Counseling envelope without volumetric enlargement
● Combined Augmentation/Mastopexy
● Discussion of patient's expectations versus anticipated – Combined procedure can lead to increased morbidity
outcomes – May be staged to decrease complications rate
● Possible complications and/or recurrence of ptosis – If staged, augmentation usually first
● Potential need for revisions and how revision costs may be ● Preoperative markings (note measurements in Physical
handled Examination)
● Implant type – Sternal notch, midline, breast meridians, IMFs, tangential
– Saline versus silicone (see ▶ Table 38.1 for comparisons) line between IMFs, mastopexy markings, proposed nipple
– Round versus shaped position, nipple to midline
– Smooth versus textured ● Augmentation
– Discuss Breast Implant Associated Anaplastic Large Cell – Preoperatively, decision should have been made as to
Lymphoma (BIA-ALCL) (see chapter 35) silicone versus saline, round versus shaped, smooth versus
● Implant placement (subglandular, subpectoral, dual plane) textured, and size estimates of implant
● Possible negative effects on future breastfeeding – Can use sizers intraoperatively to assess final implant size

158
Mastopexy/Augmentation

Mastopexy

– Periareolar (Simple, Benelli): Allows for limited lift around


38.6 Complications
the NAC ● Complication rate higher for combined augmentation/
– Vertical scar (Lassus, Lejour, Hammond, Hall-Findlay) mastopexy than either procedure alone
– Inverted-T scar (Wise-pattern skin excision, other skin ● Early complications
incision patterns) – Infection
● Augmentation-Mastopexy: General principles – Hematoma/Seroma
– Place implants first, then tailor the skin envelope to new – Loss of tissue (partial vs. total nipple necrosis, skin necrosis,
breast volume fat necrosis)
– Tailor-tack skin intraoperatively with patient upright before – Incision dehiscence
committing to planned mastopexy pattern – Pneumothorax
– Employ different mastopexy patterns, if needed, for breast – Deep vein thrombosis (DVT)/Pulmonary embolism (PE)
asymmetries ● Late complications
– Conservative, superficial skin undermining only to decrease – Poor scarring (hypertrophic, widened, hyperpigmented)
risk of NAC necrosis and wound healing complications – Rippling and wrinkling
– Always maintain a pedicle to the NAC – Capsular contracture: Baker's classification (see ▶ Table 35.1)
● Aseptic technique – Implant displacement, extrusion, and rupture
– Preoperative methicillin-resistant staphylococcus aureus – Breast asymmetry/nipple malposition
(MRSA) swabs and use of chlorhexidine washes – Contour irregularities (e.g., double-bubble or Snoopy dog
– Perioperative antibiotics and antibiotic irrigation of implant deformities)
pocket
– No-touch technique for implant placement
38.7 Critical Errors
38.5 Ethical Considerations ● Failure to obtain screening mammogram in patient > 40 years
of age
● Key area of criticism with combined mastopexy/ ● Inappropriate use of implants according to FDA guidelines
augmentation procedures is the higher risk of complications – Saline implant in < 18 years old patient or silicone implant
and tissue necrosis. Ethically speaking, this must be highlighted in < 22 years old patient (for primary augmentation, no age
to the patient so that she understands that she is asking for a restrictions for reconstruction)
procedure that has higher risk. ● Discussing/performing surgical techniques with which you
● Contraindications to treatment are not familiar
– Body dysmorphic disorder, inappropriate motivation (e.g., ● Dissection of tissue without consideration for pedicle/blood
salvaging marriage, peer pressure), significant breast supply
disease, and significant comorbidities – Must understand blood supply and innervation of the
● Managing another plastic surgeon's postoperative patient breast/nipple
● Reoperation on your own patient ● Not portraying a sense of caution when undertaking a single
● Operating on family members stage augmentation/mastopexy
● See Code of Ethics of the American Society of Plastic Surgeons ● Poor patient selection

159
Case 39 Breast Reduction
Victor A. King and Glyn E. Jones

Case 39 A 40-year-old female presents with complaint of large, pendulous breasts which are causing back, neck, and shoulder pain.

161
Breast

39.1 Description 39.2.3 Pertinent Imaging or Diagnostic


● 40-year-old woman with mammary hypertrophy and Studies
Grade III ptosis ● Breast cancer screening, in average, risk females 40 years of
● Breasts have reasonable symmetry; no evidence of skin age or greater or younger if risk factors are higher
breakdown ● Mammogram: Preoperative (when indicated) and 6 to
9 months post reduction for new baseline

39.2 Work-Up
39.3 Consultations
39.2.1 History
● Psychiatric/psychological consultation in the setting of gender
● Age and symptoms
identity issues or suspected body dysmorphic disorder
– Back pain, neck pain, shoulder pain, shoulder grooving, skin
moisture, infections, maceration, and breakdown along
inframammary fold (IMF) 39.4 Patient Counseling
– Physical or sexual embarrassment, limitations of physical
activities secondary to pain or restricted range of motion, ● Clarify expectations (i.e., size, shape, chief concern, patient's
difficulty finding properly fitting clothing goals, scaring, sensation, etc.)
● Current and desired cup size ● Provide the patients with an overview of the procedure they
● Lactation potential: Attempt to preserve, or delay surgery, will undergo
until childbearing is complete ● Reduction is a treatment for symptoms of macromastia, not
● Past medical history, including: breast pain
– Collagen vascular, skin, scaring, autoimmune disorders, ● Significant discussion regarding complications, including
obesity, and diabetes nipple loss and skin necrosis
– Bleeding diatheses or anticoagulation
– Oncologic history
○ Personal and family history; risk factors for breast cancer
39.5 Treatment
○ Obtain baseline mammogram preoperatively in all ● Goals of treatment
women of average risk 40 years of age or older, or – Reduction of breast volume and re-draping of skin, with
younger in those at high risk correction of ptosis
– Social history: Smoking, alcohol, and illicit drug use – Relief of symptoms
– Developmental, menstrual, and obstetric history, degree of – Improvement of the breast aesthetics
breast involution, response to hormones, and future ● Markings: Standing or sitting upright
pregnancy plans – IMF, mid-clavicular and breast meridian, new location of
– History of previous breast surgery and pedicles NAC via anterior projection of IMF onto breast meridian,
midline, and desired incision pattern
● Breast pedicle designs
39.2.2 Physical Examination
– Inferior: Versatile, reliable, and most common; breast
● Current breast size, chest size, and symmetry "bottoms out" over time with this technique
– The larger the thoracic circumference, the larger the breast – Bipedicle: Commonly vertically oriented
per cup size (i.e., a size 40B breast is larger than a size 34B – Medial/Superomedial/Superior: Preserves superior fullness
breast) – Lateral: Weaker pedicle and limits ability to resect lateral
– Measurements ▶ Fig. 36.1, ▶ Table 36.1: Nipple-to-sternal fullness
notch, nipple-to-IMF, inter-nipple distances, and IMF – Free nipple graft: Required in patients with exceptionally
position and symmetry large breasts
● Location of fullness (e.g., lateral vs. pendulous) ○ Sternal notch to nipple > 40 cm
● Skin quality: Elasticity, thickness, striae, presence, and ● Incision patterns (Pick a technique and know how to draw it!)
location of scars – Standard inverted-T incision (Wise pattern)
● Breast quality: Glandular, fatty, or fibrous ○ Versatile technique: May be used with any pedicle; allows
● Nipple reduction of parenchyma and skin in lateral and vertical
– Size: Ideal nipple diameter for a woman is 4 to 5 cm, dimensions, but with longer scars
depending on breast size ○ Great mobility for final nipple placement
– The nipple is often larger in women with macromastia – Vertical and circumvertical incisions: Superior pedicle
– Position, projection, symmetry, and mobility ○ Eliminates IMF incision with reduction in the horizontal
– Nipple sensation: General and two-point sensation dimension
● Regnault classification of breast ptosis (see ▶ Table 37.1) – Short scar periareolar inferior pedicle reduction (SPAIR):
– Based on position of nipple–areola complex (NAC) relative Inferior pedicle
to IMF ○ Scar pattern similar to vertical or circumvertical patterns
● Overweight patients should lose weight to ensure – Circumareolar: Superior pedicle
appropriate breast size post reduction

162
Breast Reduction

○ Limited scars, but restricted to small breasts and limited ● Loss of NAC
reductions – Use caution with secondary breast reductions, especially if
– Liposuction only pedicle is unknown
○ Used in primarily fatty breasts – NAC should be removed and converted to free nipple graft
○ Does not address ptosis or skin laxity if there is concern over blood supply from the pedicle
intraoperatively
● Breast asymmetry, over/under-resection, nipple malposition,
39.6 Ethical Considerations and bottoming out
● Loss of sensibility
● Insurance documentation/billing of tissue to be reduced and
● Wide or hypertrophic scars and dog ears
adjuvant procedures
● Lactation is decreased
● Patient selection is key; do not operate on poor candidates
● Recurrent enlargement
(e.g., procedure to save ailing relationship, dysphoria without
appropriate work-up, body dysmorphia, dangerously thin or


obese)
Cannot promise or guarantee breast size
39.8 Critical Errors
● Inadequate preoperative counseling: Setting an achievable
goal and clarification of patient's expectations and
39.7 Complications preferences
● Over-reduction of breast tissue
● Hematoma
● Excessive tension
● Seroma
● Failure to appropriately manage an ischemic NAC leading to
● Infection
necrosis
● Delayed healing: Usually caused by excess tension
● Embarking on complex or new procedure without training or
● Flap/Fat necrosis and delayed wound healing
experience
– Distal lateral flap is most tenuous, with T-junction having
● Overlooking oncologic aspects of breast care
the point of highest tension

163
Case 40 Gynecomastia
Lauren O. Roussel and Rachel R. Sullivan

Case 40 (a, b) A 21-year-old male presents for correction of the size and appearance of his breasts.

165
Breast

Simon's classification, based on degree of tissue and skin


40.1 Description ●

excess
● Adult male with bilaterally symmetric, mildly enlarged – Type I: Minor enlargement of breast without skin excess
breasts – Type II: Moderate enlargement of breast
● Minimal excess skin ○ Type IIA: Without skin excess

● Protuberance of nipple-areola complexes (NACs) ○ Type IIB: With minor skin excess

● Normal body habitus, without evidence of obesity – Type III: Breast enlargement with skin excess
● Virilization: Feminizing characteristics, lack of normal male
hair distribution, voice changes
40.2 Work-Up – Testicular examination
– Thyroid examination
40.2.1 History – Abdominal examination
○ Evaluate for organomegaly, abdominal masses, and ascites
● Onset of breast development
● Changes to breasts over time
● Presence of breast pain, breast enlargement, and nipple 40.2.3 Pertinent Imaging or Diagnostic
discharge
● Recent weight changes Studies
● Personal history of diseases of the liver, adrenal glands, ● Laboratory studies
thyroid, or kidneys, HIV, and complete medical history; – Prepubertal males
family history of breast cancer ○ Serum luteinizing hormone (LH)
● Current and prior medications or drug use ○ Serum follicle-stimulating hormone (FSH)
– Certain medications have been associated with ○ Beta human chorionic gonadotropin (B-hCG)
gynecomastia (see ▶ Table 40.1) ○ Estradiol

– Pubertal males
○ Serum LH and FSH
40.2.2 Physical Examination ○ B-hCG
● Breast examination ○ Estradiol
– Identify if breast enlargement is due to fat or glandular ○ Testosterone
hypertrophy – Adult males
○ Pseudogynecomastia: Bilateral breast enlargement due to ○ Serum LH and FSH
fat deposition without an increase in glandular tissue ○ B-hCG
– Finding concerning for malignancy: Small, firm, ○ Estradiol
eccentricity, chest wall fixation, nipple discharge, and skin ○ Testosterone
dimpling ○ Serum thyroid stimulating hormone (TSH) and free
– Tenderness: > 70% of benign gynecomastia will have thyroxine
tenderness ○ Liver function panel
– Presence of dense fibrous tissue ○ Basic metabolic panel (BMP) to evaluate renal function
– Degree of skin excess, breast ptosis ● Additional imaging or adjunct consultations for specific
clinical situations
– Routine imaging not usually ordered; additional imaging
Table 40.1 Medications associated with gynecomastia development
guided by clinical situation
Medication/Drug Class Examples – When concerning breast examination findings are present
Antacid medications Cimetidine, omeprazole, ranitidine ○ Imaging: Mammogram and ultrasound are equally

sensitive and specific


Antiandrogens Finasteride, spironolactone
○ Consult: Surgical oncology
Antibiotics Fluconazole, isoniazid, ketoconazole – Feminizing characteristics
○ Imaging: Adrenal computed tomography (CT) scan and
Cardiovascular Calcium channel blockers, digoxin,
medications furosemide testicular ultrasound
○ Karyotype to evaluate for Klinefelter's Syndrome (47 XXY)
Chemotherapeutic Alkylating agents, methotrexate,
○ Consult: Endocrinology, genetics
agents vincristine
– Testicular mass
Drugs of abuse Alcohol, amphetamines, heroin, marijuana, ○ Imaging: Testicular ultrasound
methadone ○ Consult: Urology, endocrinology

Exogenous hormones Anabolic steroids, estrogen cream or oral – Small, firm testes
estrogens, testosterone ○ Karyotype to evaluate for Klinefelter's Syndrome (47 XXY)

○ Consult: Endocrinology
HIV medications
– Thyroid mass
Psychoactive medications Diazepam, haloperidol, risperidone,
○ Imaging: Thyroid ultrasound
tricyclic antidepressants
○ Consult: Endocrinology

166
Gynecomastia

Ultrasound-assisted liposuction (UAL)


40.3 Patient Counseling ●

– Can use higher energy settings to treat more fibrous tissue


● Weight stability prior to surgical intervention – Do not excise excess skin concomitantly, and assess at
● Potential nipple sensation changes follow-up for degree of skin retraction
● Potential need for drain placement ● Simple mastectomy with or without free nipple grafts
● Discuss importance of being honest with physician including – Reserved for the most severe cases of gynecomastia when
full disclosure of drugs of abuse, including marijuana (see more conservative options are inadequate
▶ Table 40.1) – Preservation of the NAC versus free nipple grafts
● Pathology evaluation: Generally not indicated
– < 1% risk of atypical ductal hyperplasia
40.4 Treatment ● Postoperative period
– Compression garment: Wear for at least 4 weeks
● Stop any contributing medications, treat underlying
condition, and weight loss
● Smoking cessation to prevent wound healing problems and
nipple necrosis
40.5 Ethical Considerations
● Timing of gynecomastia ● For adolescents < 18 years of age, discuss with patient and
– < 12 months: Observe, often regresses after 3 to 24 months parents/guardians about motivations for intervention. Even if
of enlargement not legally of age, young patients should be the primary
– > 12 months: Surgical management can be considered, driving force behind seeking surgical intervention.
unlikely to regress spontaneously
● Gynecomastia is not associated with an increased incidence of
male breast cancer 40.6 Complications
– Exception: Klinefelter's syndrome, 20 to 60 times increased ● Nipple depression or “crater deformity”: Due to over-
risk of breast cancer resection of tissue underlying NAC
● Surgical excision ● Hematoma/seroma
– Appropriate for fibrous lesions with accompanying excess ● Asymmetry
skin ● Breast contour irregularities
– Circumareolar excision ● Inadequate resection with liposuction-only approach
– Leave sufficient tissue below the NAC to prevent nipple
depression
– May require free nipple grafts when breasts are large and/ 40.7 Critical Errors
or ptotic
● Suction-assisted lipectomy (SAL) ● Failure to fully assess for underlying causes of gynecomastia
– Indicated for more glandular/fatty composition that require treatment
– Incisions can be lateral at inframammary fold or periareolar ● Incomplete assessment of fibrous versus fatty nature of the
– More difficult to remove the fibrous tissue below the NAC tissue to determine when liposuction is appropriate
○ Combination with ultrasonic techniques or open ● Failure to assess skin excess to determine need for skin
approaches may be helpful excision

167
41 Ischial Pressure Sores 171
Section VII
42 Body Contouring after Massive Weight
Trunk Loss 175

43 Major Liposuction 179

44 Abdominal Wall Defect 183

45 Sternal Wound Infection 187

46 Chest Wall Reconstruction 191

47 Perineal Reconstruction 195

48 Abdominoplasty 199

VII
Case 41 Ischial Pressure Sores
Elizabeth Kiwanuka, Albert S. Woo, and Paul Y. Liu

Case 41 A 16-year-old paraplegic male presents with a chronic wound on his right buttock.

171
Trunk

– Osteomyelitis: May show abnormal bone marrow signals


41.1 Description – May not accurately distinguish osteomyelitis from bone
● Stage IV pressure ulcer over the right ischium with possible remodeling
bone exposure ● The extent of radiographic involvement cannot be used in
● No evidence of gross contamination isolation to guide bony resection
● Image reveals signs of wound contracture, demonstrating – Resection is determined by the amount of unhealthy bone
chronicity and evidence of healing identified
● Bone biopsy: May be useful to rule out osteomyelitis
● Long-standing wounds should raise concern for Marjolin's
41.2 Work-Up ulcer
– Tissue biopsy: Sent for culture and pathology in chronic
41.2.1 History wounds
● Risk factor assessment: Age, mental status, nutritional status,
comorbid conditions (diabetes, vascular disease), mobility,
spinal cord injury, spasm, and urinary and fecal continence
41.3 Patient Counseling
● History of current wound: Duration, infections, and ● Many of these patients are paraplegic. Such patients must be
progression counseled that they are at continued risk of developing
● Current wound management pressure ulcers throughout their lifetime. They must be
● Previous wound management and interventions coached to maintain constant vigilance
● Prior offloading interventions ● Counsel patient regarding critical importance of
● Social history: Tobacco and substance abuse, support postoperative compliance with offloading and the risk of
network, and home environment ulcers developing elsewhere without regular position
● Etiology of the wound: Paraplegia in wheelchair, bed sore, changes
neglect, and severe contracture ● Potential complications should be reviewed
● Nutritional status ● Discuss potential need for diverting fecal stream temporarily
or permanently
41.2.2 Physical Examination
● General appearance, including signs of infection and soilage 41.4 Consultations
● Location of wound
● Internal medicine and nutrition: Optimize medical and
● Dimension: Depth, length, and tunneling
nutritional status
● Quality of surrounding tissue
● Physical therapy, physical medicine, and rehabilitation:
● Pressure ulcer staging (see ▶ Table 41.1)
Manage rehabilitation of patient and assist with offloading
devices
● Social work and case management: Optimize living situation
41.2.3 Pertinent Imaging or Diagnostic and services
● Orthopedic surgery, general surgery, and urology: May be
Studies involved in diversion procedures or debridement of bony
● Laboratory tests: Complete blood count (CBC), complete prominences
electrolyte panel, albumin/prealbumin, hemoglobin A1C, ● Infectious disease: Assistance in management of infection
erythrocyte sedimentation rate (ESR), C-reactive protein and antibiotic treatment
(CRP)
● Magnetic resonance imaging (MRI): Osteomyelitis is
suggested by the presence of T2 hyperintensity and low 41.5 Treatment
intensity on T1 images
● Patient and family education to allow them to participate in
care
Table 41.1 Pressure ulcer staging ● Optimize nutrition: Guided by nutrition labs and patient
Stage Findings status; consider adding supplements
● Control/eliminate spasm (e.g., baclofen, diazepam, and/or
I Intact skin with non blanching erythema, usually over a bony
dantrolene)
prominence
● Pressure offloading with frequent turns: Turning every
II Partial-thickness dermal loss; appears as a shallow open ulcer 2 hours and lifting for 10 seconds (seated) every 10 minutes;
with a red–pink wound bed without slough, or as a serum- regimen reduces cycles of ischemia and reperfusion, and
filled bullous lesion (intact or ruptured)
prevents breakdown
III Full-thickness tissue loss; subcutaneous fat may be visible ● Avoid shearing during transfers
Bone, tendon, or muscle is not exposed; these are prefascial ● Prevention with a pressure-relieving mattress; pressure-
wounds; may have undermining
mapped cushion for wheelchair
IV Full-thickness tissue loss through the fascia with exposed bone, ● Debride all nonviable tissues and bone; antibiotic treatment
tendon, or muscle; often includes undermining and tunneling when indicated

172
Ischial Pressure Sores

41.5.1 Wound Care ● Flap options by location


– Ischium
● Dressings: Can decrease bacterial burden ○ Hamstring musculocutaneous V-Y advancement flap
– Silver sulfadiazine, damp to dry, alginate, and collagen are (see ▶ Fig. 41.1)
some examples ○ Posterior thigh fasciocutaneous flap
– Dakin's solution (sodium hypochlorite): Short-term use for (see ▶ Fig. 41.2)
local Pseudomonas infection ○ Tensor fasciae lata V-Y advancement flap
– Negative-pressure wound therapy with or without ○ Gluteal rotation flap with or without muscle (see
irrigating solution (instill vacuum-assisted closure [VAC]) ▶ Fig. 41.3)
○ Lateral and anterolateral thigh fasciocutaneous flaps

○ Gracilis myocutaneous flap


41.5.2 Surgical Management ○ Pedicled rectus abdominis myocutaneous flap
● Stage I and II ulcers: Usually treated with medical – Sacrum
management only ○ Gluteus maximus rotation flap
● Stage III and IV ulcers: Require surgical intervention after ○ V-Y advancement
optimization of patient to prevent recurrence
– Always consider future procedures in operating room planning
– Very high level of recurrence in patients with pressure sores 41.6 Complications
– Flaps should be larger than needed to allow re-elevation
● Hematoma and seroma
and advancement
● Surgical site infection
● Exposed bone
● Wound dehiscence
– If MRI and bone biopsy negative for osteomyelitis: Change
● Recurrence
dressing and schedule for coverage
– Recurrence rates after flap closure: 13 to 82% over
– If MRI confirms osteomyelitis: Resection of the affected
5 years
bone, potentially 6-week course of intravenous antibiotics,
and definitive closure

Fig. 41.2 Posterior thigh fasciocutaneous flap. This may be superiorly


based (as shown) or based medially/laterally to allow advancement in a
Fig. 41.1 Hamstring musculocutaneous V-Y advancement flap. superior direction.

173
Trunk

41.7 Critical Errors


● Failure to recognize wound etiology
– Biopsy chronic wound
– Evaluate for Marjolin's ulcer
● Inadequate debridement prior to surgical closure
– Control bacterial colonization of wound
– Recognize osteomyelitis
– Treat the infection
● Failure in flap design
– Designing a flap which is too small for closure
– Failure to anticipate need for re-elevation and advancement
● Failure to appropriately plan for postoperative outpatient
needs
– Services, turning, offloading devices, and transportation to
appointments

41.8 Ethical Considerations


● Goals of care
– Aggressive treatment of advanced pressure sores is often
inconsistent with the overall goals of therapy
● Stigmatized by the occurrence of a pressure sore even though
accountabilities may lie in the natural history of the disease
Fig. 41.3 Gluteal rotation flap. This may be performed with ● Inability of patient to comply with postoperative regimen
fasciocutaneous or musculocutaneous tissue.
(i.e., patient socially dependent on wheelchair with ischial
pressure ulcer and unable to decrease duration of time in
chair)

174
Case 42 Body Contouring after Massive Weight Loss
Jonathan P. Brower and Rachel R. Sullivan

Case 42 (a-d) A 51-year-old female requests body contouring after massive weight loss following gastric bypass.

175
Trunk

Psychological status/Expectations
42.1 Description ●

– Ensure psychological comorbidities are stabilized


● Loose, excess skin circumferentially noted across trunk – Establish patient's priorities to determine most appropriate
– Notable abdominal skin laxity extending to posterior trunk procedure(s)
● Significant sagging of skin across buttocks and medial thighs – Discuss patient's willingness to trade improved contour for
long/visible scars

42.2 Work-Up
42.2.2 Physical Examination
42.2.1 History ● Comprehensive assessment of body contour, skin, and tissue
● Obesity classification (see ▶ Table 42.1) quality, and degree of ptosis and/or deflation
● Original and current body mass index (BMI) calculated by ● Presence of scars and hernias
weight/height ● Signs of nutritional deficiency (e.g., pale mucous membranes,
– Should be within 10 to 15% of goal weight brittle nails/hair)
● Weight loss timeline (i.e., time from surgery or from start of
weight loss)
42.2.3 Preoperative Work-Up
● Length of time weight has been stable
– Weight should be stable for at least a 3-month period prior ● Laboratory analysis: Complete blood count (CBC), electrolytes
to surgery with albumin and blood urea nitrogen (BUN)/creatinine, liver
– May consider earlier intervention if there is a resulting function tests (LFTs), prothrombin time (PT)/partial
functional issue (e.g., activities of daily living affected, thromboplastin time (PTT), with or without micronutrients
unable to exercise) (e.g., iron, B12, thiamine)
● Method of weight loss, including bariatric procedures ● Guided by physical examination, medical history, and type of
– Be aware of key procedures and their physiologic bariatric procedure
consequences/nutritional deficiencies
○ Restrictive: Lap-Band, vertical banded gastroplasty

○ Malabsorptive: Biliopancreatic diversion with or without 42.3 Patient Counseling


duodenal switch ● Expected scars: Many are long, with some visible when
○ Combination of restrictive and malabsorptive: Roux-en-Y
patient is clothed
gastric bypass – Possible dog ears
● Current diet and exercise habits – Inability to achieve “perfection”
– Nutritional supplementation: Consider micronutrient ● Patient's priorities and staging of procedures for safety
deficiency (e.g., fatigue, hair loss, poor wound healing, – Limit outpatient surgery to < 6 hours
neuropathy) – Overnight observation recommended for > 6 hours long
– Assess protein intake (goal 70–100 g/day preoperatively) surgery
– Refer back to bariatric surgeon if there is weight loss ● Revision policy: Discuss costs and timing after initial
plateau at unacceptably high BMI procedure
● Medical comorbidities
– Ensure excellent glycemic control for diabetics (target
HbA1c < 6.5%) 42.4 Treatment
– Personal or family history of thrombosis or bleeding
diathesis
● General sequence for reconstruction: Correct answer varies
– Ensure cessation of nicotine exposure for 1 month prior to – Tailor to patient's priorities, anatomy, safety, and budget
surgery – Wait a minimum of 3 to 6 months in between stages
● Deep vein thrombosis (DVT) prophylaxis
– Caprini Risk Assessment Model: Preoperative risk
Table 42.1 Obesity classification
stratification (see ▶ Table 42.2)
Description BMI ○ Treat body contouring patients as “moderate risk” or

higher, depending on additional comorbidities


Overweight 25–30
– Strategies for minimizing venous thromboembolism risk
Obesity 30–35 ○ Discontinuation of hormones

Severe obesity 35–40 ○ Limit duration of surgery (< 6 hours preferred)

– A hematology consultation should be obtained for


Morbid obesity 40–50
perioperative guidance if there is any personal or family
Super obesity > 50 history of venous thrombosis
Abbreviation: BMI, body mass index.
● Intraoperative management
– Perioperative antibiotics are generally recommended

176
Body Contouring after Massive Weight Loss

Table 42.2 Guidelines for perioperative venous thromboembolism prophylaxis


Caprini score Caprini score Caprini score Caprini score
<2 3–4 5–6 >7
(low risk) (moderate (high risk) (highest risk)
risk)

Early ambulation x x x x

Pneumatic compression boots x x x x

Heparin 5,000 U subcutaneous 8–24h post-op, discontinued on discharge x x x

Heparin 5,000 U pre-op, discontinued on discharge x x

LMWH x 5 days after discharge x

Abbreviation: LMWH, low molecular weight heparin.

– Positioning (padding pressure points, pillow under knees)


– Patient's temperature (warm fluids, warming devices)
– Hemodynamic status (monitor blood loss, intravenous [IV]
hydration, urine output)

42.4.1 Surgical Procedures


● Trunk, abdomen, buttocks, and lateral thighs
– Abdominoplasty (see Chapter 48)
○ Mark with patient standing and optimize position of scars

in relationship to anatomic landmarks (e.g., within bikini


line)
○ Liposuction can be safely undertaken laterally, avoid

medially
○ Fleur-de-lis component can be added to address vertically

oriented laxity and further contour the waist


– Belt lipectomy: Can be used to address circumferential rolls
○ Avoid posterior over-resection: Will elevate gluteal cleft

and flatten buttock


– Lower body lift: Often performed in concert with
abdominoplasty
○ Addresses buttock deflation and saddle bag deformity,

especially when liposuction is performed concurrently


● Upper thorax, breasts, and arms
– Upper body lift: Addresses back rolls and contours
the waist
– Mastopexy or breast reduction (see Chapters 38 and 39,
respectively)
○ Mastopexy may be undertaken with or without

autoaugmentation (dermal suspension, parenchymal


plication)
○ Special attention to breast borders (i.e., lateral border,

inframammary fold [IMF]) as these landmarks are Fig. 42.1 Brachioplasty incision.
frequently lost in massive weight loss (MWL) patients
○ Avoid breast implants: Susceptible to malposition in these

patients ● Medial thighs


– Brachioplasty alone versus staged – Vertical medial thigh lift (more effective than horizontal)
liposuction + brachioplasty with or without liposuction
○ Upper arm excess in MWL crosses the axilla onto the ○ Avoid injury to saphenous vein and lymphatics of femoral

chest wall triangle


○ Excision must also extend to lateral chest wall ○ Suspend flap to deep (Colles) fascia

○ Incision design should prevent linear scar contracture ○ Avoid a horizontal scar technique to reduce risk for labial

(e.g., Z-plasty at axilla) (see ▶ Fig. 42.1) spreading

177
Trunk

● Face: Standard rhytidectomy techniques (see Chapter 23) ● Procedure-specific complications


with modifications – Abdominoplasty: Injury of lateral femoral cutaneous
– Skin redundancy exceeds superficial musculoaponeurotic nerve
system (SMAS) redundancy – Medial thigh lift: Disruption of lymphatic drainage and
– Typically requires more skin undermining and less SMAS labial spreading
elevation – Brachioplasty: Injury of medial antebrachial cutaneous or
ulnar nerves
● Over-resection: Avoid with careful tailor-tacking prior to
42.5 Ethical Considerations definitive excision
– Progressive tension sutures can be useful if closure is too
● Patients with unrealistic expectations or body dysmorphic
tight
disorder (BDD)
– Vacuum assisted closure (VAC) or skin graft may be used as
● Avoid excessive number of procedures grouped together
temporizing measure
● Comorbidities or suboptimal weight loss may affect
suitability
● Revision surgery/management of complications for patients
of other surgeons
42.7 Critical Errors
● Questionable or unsafe practices, compromising safety
● Inadequate DVT prophylaxis
42.6 Complications ● Combining too many procedures (> 6 hours)
● Inappropriate patient selection (e.g., before weight
● Common: Hematoma, seroma, wound dehiscence, skin
stabilization or when nutritional deficiencies exist)
necrosis, infection, and persistent laxity or laxity within a
● Unable to discuss the markings, positioning, and surgical
year
steps of any technique you mention
● Deep veinous thrombosis (DVT), pulmonary embolism
– Must discuss appropriate prophylaxis (see ▶ Table 42.2)
– Early recognition, diagnosis, and treatment

178
Case 43 Major Liposuction
Dardan Beqiri and Rachel R. Sullivan

Case 43 (a, b) A 46-year-old female presents to the clinic to discuss possible surgical options to improve the appearance of the “saddlebags” on her
thighs.

179
Trunk

43.1 Description Table 43.1 Wetting solutions for liposuction


Technique Infiltrate EBL (% volume)
● Significant, diffuse lipodystrophy noted bilaterally in
saddlebag regions of upper lateral thighs Dry None 20–45

Wet 200–300 mL/area 4–30

43.2 Work-Up Superwet 1 mL infiltrate: 1 mL aspirate 1

Tumescent 2–3 mL infiltrate: 1 mL aspirate 1


43.2.1 History (or to skin turgor)
● Weight stability Abbreviation: EBL, estimated blood loss.
● Medical comorbidities
● Patient’s concerns and goals of treatment

– Lidocaine, epinephrine, and bicarbonate solution added to


43.2.2 Physical Examination saline or Ringer’s Lactate
● Identify regions of suboptimal contour, asymmetry, and – Provides anesthesia and hemostasis
lipodystrophy – Maximum lidocaine with epinephrine: 35 mg/kg
● Evaluate skin quality and tone (thickness and elasticity): ● Liposuction modality
Pinch test – Suction-assisted liposuction (SAL): Traditional liposuction
● Abdomen: Examine for hernias and diastasis technique
– Power-assisted liposuction (PAL): Motorized oscillating
hand piece
43.3 Patient Counseling – Ultrasound-assisted liposuction (UAL): Ultrasonic energy
applied after wetting solution to emulsify fat before
● Discuss risk/benefit profiles of each procedure aspiration
● Ensure the goals of the patient align with the goals of the ○ Ideal for fibrous regions: Buttock, lumbar, and
procedure gynecomastia
– Liposuction is a contouring procedure ○ Precautions to avoid cutaneous thermal injury
○ Best in areas of thick, elastic skin with underlying contour
– Laser-assisted liposuction (LAL)
fat irregularity ○ Possible skin tightening effect due to heating of
○ Does not address cellulite or obesity: Cellulite thought
subdermal tissue
to be due to hypertrophy of superficial fat within septa ● Fluid management for large-volume liposuction (Critical
and/or increased skin laxity safety issue)
○ Does not resect skin
– Replace preoperative deficits
● Set expectations in regards to pain and swelling during the – Employ superwet or tumescent technique
postoperative period – Administer maintenance fluids + replacement of 0.25 mL
per 1 mL of aspirate over 5L
– Titrate intravenous fluid (IVF) to patient’s clinical picture
43.4 Treatment (e.g., urine output, vital signs)
● Make preoperative markings with patient upright: To – Maintain intraoperative fluid ratio: [(IVF + infiltrate)/
determine treatment areas and asymmetries, and outline aspirate] ≈ 1.2
zones of adherence ○ Older technique (Pitman): (IVF + infiltrate) = 2 x (aspirate)

– Distal iliotibial tract – With aspirate, 25 to 30% of infiltrate is removed


– Gluteal crease ● If large-volume liposuction (≥4–5L) is performed, it must be
– Lateral gluteal depression done in acute-care hospital or accredited facility
– Middle medial thigh – Monitor vitals and fluid balance with Foley catheter;
– Distal posterior thigh overnight inpatient observation
● Preoperative considerations – Warm patient, fluids, and operating room (OR) to avoid
– Complete blood count (CBC) if expecting to perform large hypothermia
volume procedure – Dilute lidocaine further if greater volume of infiltration is
– Deep vein thrombosis (DVT) prophylaxis (see Chapter 42) necessary
– Hypothermia and necessary precautions ● DVT prophylaxis (see ▶ Table 42.2 for risk stratification)
– Positioning – Mechanical: Sequential compression devices
● Target deep fat layer and cross tunnel to prevent contour – Ambulate on postoperative day 0 (day of surgery)
irregularities – Chemoprophylaxis not routinely required, but should be
● Wetting solution technique (see ▶ Table 43.1) considered
– Four types of liposuction techniques described based on the ● Postoperative care
volume of infiltration or wetting solution injected: Dry, – Early ambulation
wet, superwet, and tumescent techniques – Compression garments 4 to 6 times in a week

180
Major Liposuction

Leg symptoms / Clinical suspicion for DVT

Low probability Moderate or high probability

D-dimer test Venous ultrasound examination

Negative Positive or not available Negative Positive

Venous ultrasound examination D-dimer test DVT confirmed

DVT excluded Negative Positive Negative Positive Treat

DVT excluded DVT confirmed DVT excluded Follow-up studies (eg, second
venous ultrasound, venography)

Treat

Fig. 43.1 Algorithm showing work-up for DVT.

Lidocaine toxicity
43.5 Ethical Considerations ●

● Seroma
● Inappropriate patient selection (e.g., medically inappropriate/ ● Skin necrosis
high volume liposuction but patient refuses to be admitted as ● Thermal injury (UAL)
does not want to pay out of pocket) ● Contour deformities
● Ensuring the patients have adequate follow-up in proximity ● Prolonged paresthesias
to their place of residence (in cases where patients travel far ● Infection
for procedure) ● Perforation of abdominal viscera

43.6 Complications 43.7 Critical Errors


● Excessive ecchymosis, discoloration, hematoma, or blood loss ● Ignoring key issues of safety in this patient population
related to disruption of vasculature ● Lidocaine toxicity (dosing > 35 mg/kg)
● Fat embolus, DVT, or pulmonary embolus: Any concern should ● Inadequately monitoring fluid balance
warrant immediate admission and work-up (e.g., computed ● Failure to require inpatient monitoring after large-volume
tomography [CT] angiogram, anticoagulation, etc.) (see liposuction
▶ Fig. 43.1) ● Forgetting to take precautions against thermal injury with
– If patient has high pre-test probability for pulmonary UAL
embolism (PE), obtain CT angiography (CTA) immediately ● Adding significant liposuction to an already extensive
and then institute appropriate anticoagulation plan reconstruction involving other procedures
accordingly. If CTA returns negative, consider having an ● Trying to manage critical complications (e.g., PE/DVT) in the
angiography performed. outpatient setting
● Fluid shifts and pulmonary edema

181
Case 44 Abdominal Wall Defect
Marten N. Basta and Karl H. Breuing

Case 44 A 75-year-old female enters with complaints of abdominal discomfort from worsening ventral abdominal hernia. She had originally
undergone an open cholecystectomy, which was complicated by small bowel obstruction requiring emergent exploratory laparotomy and eventual
skin graft.

183
Trunk

44.1 Description 44.3 Consultations


● Large ventral abdominal hernia: Full-thickness defect ● General surgery
traversing all tissue layers of abdominal myofascial wall – May need to excise skin present on bowel, lyse adhesions,
– Herniation of peritoneal contents with skin directly over and repair any iatrogenic bowel injury during the initial
bowel process to obtain access for reconstruction
● Loss of domain: > 50% of abdominal contents lie outside of the
abdominal cavity
– Rectus abdominus muscles migrate laterally as a 44.4 Patient Counseling
consequence ● Acknowledge disabling nature of abdominal hernia and
● There is no evidence of visceral incarceration
diminished quality of life
● Frank discussion regarding substantial risk of recurrence
44.2 Work-Up ●
depending on individual surgical history
Identify modifiable risk factors and develop clear plan for
44.2.1 History addressing them
● Review risks of surgical complications → ACS NSQIP Universal
● Etiology of defect: Congenital, previous surgery, trauma, and Risk Calculator
resection
● History of hernia repair: Description of attempts, techniques
used, presence of mesh, location, type, and concurrent intra 44.5 Treatment
abdominal procedures
● Duration defect has been present
● Preoperative preparation should include weight loss, smoking
● Associated symptoms: Chronic pain, bowel obstruction, cessation, and correction of malnutrition
physical mobility, and psychosocial impact
● It is best to delay abdominal reconstruction for at least
● Nutritional status 6 months after initial injury or surgery to minimize
● History of smoking or pulmonary disease inflammation and edema
● Steroids or immunosuppressive medications and
autoimmune disease 44.5.1 Reconstructive Options
● Decision influenced by the tissue defect, size of defect, and
44.2.2 Physical Examination location
● Body mass index (BMI): Weight (kg)/[Height (m)]2
● Gold standard reconstruction: Primary fascial closure with
● Abdominal wall defect description prosthetic reinforcement
– Location
● Skin and subcutaneous defects
○ Midline or lateral
– Primary repair
○ Upper, middle, or lower abdomen
– Local tissue rearrangement
– Tissue defect – Skin graft
○ Skin and subcutaneous tissue
– Tissue expansion can be performed for large defects, with a
○ Myofascial
skin graft over the defect as a temporizing measure
○ Full thickness
● Myofascial defects
– Size of defect – Primary repair: Only for small defects with significant skin
– Condition of existing tissues laxity
● Pre-existing incisions – Prosthetic material for reconstruction of fascia
○ Polypropylene mesh (nonabsorbable)
● Evidence of loss of domain
○ Acellular dermal matrix (biologic)

○ Selection based upon biomechanical characteristics of

44.2.3 Pertinent Imaging or Diagnostic defect and degree of contamination; avoid permanent
Studies mesh in contaminated/dirty wounds
– Components separation: Useful for central defects up to
● Computed tomography (CT) of the abdomen with contrast 20 cm in width (see ▶ Fig. 44.1 and ▶ Fig. 44.2)
may be helpful to delineate the extent of the defect, the ○ The hernia is taken down and separated from the
related anatomy, and other relevant information (e.g., bowel abdominal flaps.
adhesions, abscesses) ○ Skin flaps are elevated lateral from the abdominal
● Pulmonary function testing should be performed if there is musculature to the anterior axillary line.
pre-existing respiratory compromise or suspicion for loss of ○ The external oblique aponeurosis is incised vertically
domain from a large hernia 1 cm lateral to semilunar line from costal margin to

184
Abdominal Wall Defect

Fig. 44.1 Planes of separation: Indicated by dashed line as it proceeds


posteriorly to the posterior axillary line between the external and
internal obliques.

Fig. 44.2 Components separation technique: External oblique is


inguinal ligament. The external oblique is raised off the incised lateral to the semilunar line and released from the internal
internal oblique in a relatively avascular plane, up to the oblique (dashed lines 1 and 4). Posterior rectus sheath is released
midaxillary line. (dashed line 3). The resulting myofasciocutaneous flaps are advanced
○ If additional advancement is needed, the posterior rectus toward the midline.
sheath may be incised and the rectus muscle may be
separated from this layer.
– Tissue expansion ● Peak airway pressures are measured by the anesthesia team
○ Components separation is not possible in all cases (e.g., during closure. If peak airway pressure increases do not allow
lateral defects) closure, mesh interposition may be necessary.
○ An expander can be placed between external and internal ● If the abdominal wall defect is in a radiated field, nonradiated
oblique muscles tissue will be needed (distant pedicled or free flap) for
● Full-thickness defects reconstruction.
– Primary repair: Small defects with adequate tissue laxity only
– Pedicled flaps
○ Rectus abdominus: Superiorly-based (upper abdominal 44.6 Ethical Considerations
defects) or inferiorly-based (lower abdominal defects) ● Noncompliant patient (failure to participate in preoperative
○ External oblique: Defects of the upper two-thirds of the
risk-reduction strategies)
abdomen ● Management of chronic debilitating pain for patient with
○ Latissimus dorsi: Lateral defects of the upper two-thirds
high surgical risk
of the abdomen
○ Tensor fascia lata: Defects of the lower two-thirds of the

abdomen 44.7 Complications


○ Rectus femoris: Defects of the lower two-thirds of the

abdomen ● Wound infection


– Free flaps: For very large defects (e.g., anterolateral thigh ● Delayed wound healing
[ALT], latissimus dorsi) ● Chronic pain
○ Most commonly used recipient vessels: Superior ● Recurrence
epigastric, deep inferior epigastric, deep circumflex iliac, ● Acute respiratory/renal failure
internal thoracic, and saphenous vein loop grafts ● Enterocutaneous fistula

185
Trunk

Inability to describe in detail the surgical option chosen


44.8 Critical Errors ●

● Inadequate preparation for complications (e.g., bowel


● Not optimizing medical status before surgery perforation)
● Not eliminating risk factors for recurrence (e.g., weight loss, ● Failure to consider backup or salvage measures when closure
smoking cessation, etc.) is not possible

186
Case 45 Sternal Wound Infection
Marten N. Basta and Karl H. Breuing

Case 45 Preoperative (a) and intraoperative (b) views of 64-year-old male status post median sternotomy for coronary artery bypass with harvest of
right internal mammary artery. The patient presents with suppurative sternal wound infection and dehiscence 2 weeks postoperatively.

187
Trunk

45.1 Description 45.4 Patient Counseling


● Anterior midline chest wall wound measuring roughly 45.4.1 Multidisciplinary Goals of Care
20 cm × 6 cm in size
● The sternal edges and mediastinum are exposed; upon Assessment
presentation, the image demonstrates gross purulence and ● Mortality risk of acute mediastinitis down from 50 to 5–20%
necrotic edges ● Palliative care consultation (if necessary)
● With serial debridement, an extensive underlying dead ● Regular multidisciplinary collaborative rounds to assist with
space is evident (blue dotted line); no vascular grafts are decision-making
visible

45.4.2 Optimize chances of healing


45.2 Work-Up prior to reconstruction
45.2.1 History ● Make effort to optimize nutrition
● Assist with local wound care when feasible
● Etiology: Sternal wound infection (following median ● Pulmonary rehabilitation
sternotomy), tumor resection, and radiation (ulcers,
osteoradionecrosis)
● Duration wound has been present 45.5 Treatment
● Current wound care regimen
● Comorbidities: Respiratory insufficiency, sepsis, and cardiac 45.5.1 Goals
disease ● Treat and eradicate infection
● Review previous operative reports (e.g., what vessels have ● Establish clean vascularized wound bed
been used and ribs resected) ● Restore stability and mechanics of chest wall
● Protect vital structures and provide durable coverage
● Obliterate dead space
45.2.2 Physical Examination ● Achieve cosmetically acceptable result
● Vital signs: Is the patient stable?
● Size and depth of defect
● Presence of infected or necrotic tissue 45.5.2 General Approach
● Sternum stability with respirations ● Stabilize patient if acutely ill
– Sternal wires/plates/other hardware present ● Broad work-up, imaging, and culture data
● Exposed grafts, vascular devices, or mediastinum ● Serial debridement
(e.g., pericardium, pleura) ● Consider wound vacuum assisted closure (VAC) therapy once
● Prior surgical scars on chest or abdomen infection is cleared
● Congenital abnormalities: Poland syndrome and pectus ● When ready, attempt sternal fixation and vascularized flap
excavatum/carinatum coverage

45.2.3 Pertinent Imaging or Diagnostic 45.5.3 Pattern of Presentation


Studies ● Acute: First postoperative week (typically sterile, culture
negative)
● Operative culture data, consider fungal infection for
– Sternal instability, serosanguinous drainage, and dehiscence
late-presenting mediastinitis
– Bony nonunion but no osteomyelitis or cellulitis
● Chest X-ray (CXR): For presence of sternal wires and
● Sub-acute: Second to fourth postoperative weeks (culture
evaluation of lung fields
positive)
● Computed tomography (CT) scan: Evaluation for deep
– Purulent mediastinitis, fever/sepsis, and leukocytosis
abscesses in case of persistent fevers and sepsis
– Osteomyelitis and cellulitis present
● Magnetic resonance imaging (MRI): Most useful in chronic
● Chronic: Months to years post sternotomy
sternal defects for evaluation of extent of infection and/or
– Typically confined to sternum (osteomyelitis) and overlying
osteomyelitis
soft tissue (chronic draining sinus tracts, localized cellulitis)
● Angiogram: Allows study of available vessels and their
– May be due to osteonecrosis and persistent foreign body
patency

45.5.4 Flap Coverage Options


45.3 Consultation ● Pectoralis major
● Cardiac or thoracic surgery necessary to assist with – Workhorse flap for sternal coverage, appropriate for
debridement, particularly if there is risk of exposure of the superior and mid-sternal defects of modest size, and
heart unreliable for inferior sternal coverage

188
Sternal Wound Infection

– Type V muscle flap (thoracoacromial pedicle, internal – Sternal wiring


mammary segmental perforators) – Sternal plate fixation
○ Advancement muscle/musculocutaneous flap (may be – Other hardware systems (e.g., Sternal Talon)
bilateral)
○ Turnover muscle flap (detach from humeral insertion):

Cannot be performed if internal mammary artery (IMA) 45.6 Complications


sacrificed on that side ● Hematoma
● Omentum
– Most commonly due to early start of anticoagulation
– Based on left or right (preferable) gastroepiploic vessels
– Platelets and fresh frozen plasma (FFP) can be transfused if
– Tunneled into the thoracic cavity over the costal margin or
indicated
through the diaphragm
– Usually requires operative evacuation
– Large surface area or dead space obliteration coverage ● Seroma: Minimized with liberal use of drains
– Added antimicrobial activity of rich lymphatic supply ● Infection
– Excellent for coverage of exposed grafts and vascular
– Most likely due to incomplete initial debridement or wound
devices and the inferior part of the sternum
preparation
– Cons: Donor site complications, requires intra-abdominal
– Decompress and institute local wound care
access for harvest ● Flap failure
● Rectus abdominus muscle
– Ensure that the dominant vessel to the flap has not been
– Muscle or vertically oriented musculocutaneous flap
sacrificed in a previous operation
(VRAM)
– May need another flap for coverage
– Excellent coverage of the inferior sternum ● Mortality (5–20%)
– Turnover or pedicled rotation flap is first option:
Dependent on superior blood supply from IMA
– Bipedicled flap consisting of pectoralis major continuous 45.7 Critical Errors
with ipsilateral rectus abdominis (based on
thoracoacromial and deep inferior epigastric artery (DIEA) ● Failure to cover exposed vascular grafts
pedicles respectively) may be used if bilateral IMA pedicles ● Inadequate debridement of necrotic/infected tissue
and omentum are unavailable ● Not ensuring intact vascular supply to proposed flap
● Latissimus dorsi – Do not base flaps on IMA vessels (e.g., superiorly based
– Can be used for limited defects rectus, pectoralis turnover flap) if they have been sacrificed
– As a free flap, it provides wide coverage ● Not leaving drains or taking drains out too soon
● Bony reconstruction

189
Case 46 Chest Wall Reconstruction
Marten N. Basta, Albert S. Woo, and Karl H. Breuing

Case 46 (a–c) A 17-year-old male with history of Ewing's sarcoma of the right lateral chest wall. He presents after en bloc resection with an open
chest wound with exposed pleura, segmental loss of several ribs, and overlying soft tissue defect.

191
Trunk

46.1 Description 46.4 Patient Counseling


● Large lateral chest wall defect including the lateral portion of ● Chest wall defects often result from cancer; oncologic
the pectoralis major, segmental loss of serratus, and several resection may restrict our reconstructive options leading to a
ribs less than optimal aesthetic outcome
● Defect involves loss of structural support and overlying skin/ ● Postoperative recovery is challenging and may require an
soft tissue extended stay in the intensive care unit (ICU), with
hospitalization of 15 to 25 days on average
● For chest wall tumors, 5-year survival is approximately 50%,
46.2 Work-Up but morbidity is acceptable; surgery leads to increased
survival and better quality of life
46.2.1 History
● Previous history of chest, back, or abdominal surgery/trauma:
Potential compromise of specific flaps 46.5 Treatment
– History of coronary artery bypass grafting: Possible absence ● Complete resection and debridement of all tumor/nonviable
of internal mammary arteries (IMAs), which can affect flap tissues/infection
viability ● Reconstruction of potential missing layers: Pleural cavity,
● Pulmonary disease (chronic obstructive pulmonary disease thoracic skeleton, and soft tissues
[COPD], asthma): Increased risk of respiratory compromise in ● Obliteration of intrathoracic dead space with soft tissue
the absence of chest wall skeletal reconstruction (e.g., pedicled muscle or omental flaps over chest tube)
● Other comorbidities – Latissimus dorsi, pectoralis major, serratus anterior, rectus
● Tobacco use abdominis, and omentum
● Nutritional status – May require new thoracotomy incision to inset: Ensure
● Etiology of chest wall wound/deformity pedicle is not strangled or twisted
– Traumatic, oncologic, infectious, radiation, congenital, or ● Skeletal defect reconstruction
postsurgical – Skeletal support generally needed in defects involving four
– If oncologic, benign versus malignant: History of (or plan or more consecutive ribs or defect larger than 5 cm
for) radiation therapy, extent of surgical resection planned, ○ Smaller defects may require reconstruction if severe
and surgical approach (spare locoregional flaps, if possible) pulmonary disease present
○ Larger defects may not require reconstruction in the

46.2.2 Physical Examination setting of radiation-induced chest wall fibrosis


(i.e., stiffened chest wall, resulting in greater stability and
● Define defect/mass: Location, depth, and fixed/mobile less paradoxical motion) or location under scapula
● Perform lymph node examination – Reconstructive options
● Assess muscle involvement in the chest: Is the pectoralis ○ Autologous: Pectoralis major, latissimus dorsi, rectus
major involved? abdominis, serratus anterior, and omentum
● Assess abdomen for hernias and diastasis recti ○ Alloplastic: Mesh (polypropylene, expanded
● Evaluate back musculature and soft tissue laxity polytetrafluoroethylene [e-PTFE]) and methylmethacrylate
● Assess for chest wall, back, or abdominal scars ○ Bone grafts: Split-rib, iliac crest, or fibula grafts

○ Hardware

● Soft tissue coverage


46.2.3 Pertinent Imaging or Diagnostic
– Locoregional or free flaps
Studies – Latissimus flap
○ Can reach contralateral axillary fold
● Computed tomography (CT) to evaluate extent of mass/
○ If thoracodorsal pedicle is injured/unavailable, may be
wound/deformity and any involved or absent structures
● Angiogram (or magnetic resonance [MR]/CT angiography) if pedicled on serratus branch of the lateral thoracic artery
○ With thoracotomy, latissimus muscle may be compromised
uncertain about vascular anatomy
● Pulmonary function tests (PFTs) may be indicated: Help – Serratus anterior flap
○ Good for small anterior and posterior chest coverage
determine need for reconstruction in small defects
○ Risk of scapular winging if entire muscle is removed

○ May be compromised in case of prior thoracotomy

46.3 Consultations – Pectoralis major flap


○ Useful for superior-anterior chest
● Cardiac/Thoracic surgery for assistance in debridement and ○ Based on internal mammary artery (IMA) or pectoral
management of chest wall reconstruction branch of the thoracoacromial trunk
● Oncology for nonoperative plan (radiation, chemotherapy) ○ Prior coronary bypass (i.e., loss of IMA) may limit options

192
Chest Wall Reconstruction

– Omentum ● Infected mesh: Requires mesh removal


○ Useful for large surface area defects and coverage of – Inflammatory process may result in fibrosis of sufficient
exposed pericardium mechanical stability such that no mesh has to be replaced
○ Requires entrance to abdominal cavity; concomitant ● Hematoma/seroma: Requires prompt drainage
hernia risk ● Bronchopleural fistula: Devastating complication with high
○ Excellent back-up option as it is far from the zone of recurrence rates
injury and will not be injured in a primary reconstruction ● Donor site morbidity, including hernia, scar/contracture,
with more local flaps wound dehiscence, and hematoma/seroma
– Rectus abdominis flap
○ Good for anterior chest wall when significant volume is

required 46.8 Critical Errors


○ If IMA is not available the flap can be based on the 8th
● Failure to recognize gross lymphadenopathy or disease
intercostal vessels
progression on CT altering resectability of the primary lesion
● Inadequately assessing a patient’s cardiac history to
46.6 Ethical Considerations ●
determine if the IMA is intact
Failure to assess need for skeletal reconstruction
● Delay in chemotherapy secondary to delayed healing/flap loss ● Neglecting to discuss surgical approach with cardiothoracic
● Palliative surgical options in patients with end-stage surgeon and sacrificing locoregional flaps unnecessarily
malignancies ● Proceeding with reconstruction without adequate
– Surgery may improve quality of life even if it does not debridement
lengthen survival ● Placing nonautogenous materials (e.g., proline mesh) in a
potentially infected wound bed

46.7 Complications
● Flail segment: Avoid with adequate skeletal reconstruction/
stabilization

193
Case 47 Perineal Reconstruction
Lauren O. Roussel and Rachel R. Sullivan

Case 47 A 72-year-old woman with vulvar cancer underwent wide local excision of tumor and now requires perineal reconstruction.

195
Trunk

47.1 Description 47.3 Consultations


● Wide resection of perineal contents including mons pubis, ● Oncologic surgeon: Wide local excision with pelvic lymph
vulva, and clitoris node dissection, as indicated
● Resection area extends to urethral and vaginal orifices ● Radiation oncologist: Evaluation for postoperative or
intraoperative radiation therapy, including possible placement
of brachytherapy catheters at time of tumor resection
47.2 Work-Up ● Medical oncologist: Consider need for neoadjuvant or
postoperative chemotherapy
47.2.1 History ● May consider involvement of psychiatrist or sex therapist
● Personal history of cancer and prior related treatments or
surgeries
● Family history of cancer 47.4 Treatment
● Prior radiation treatment or exposure
● Prior urinary or fecal incontinence
47.4.1 Goals of Therapy
● Promote rapid wound healing
● Decrease pelvic dead space and restore support of the pelvic
47.2.2 Physical Examination floor
● Evaluate surgical defect and classify as partial (anterior,
● Reestablish normal sexual function and body image
lateral, posterior, upper two-thirds) or circumferential/total
loss of vagina 47.4.2 Reconstructive Options
● Examine abdomen and lower extremities to assess possible
reconstructive options ● Vertical rectus abdominis myocutaneous (VRAM) flap
● Evaluate for dead space, hernias, fistulae, prolapse, infection, – Access via midline laparotomy incision
devascularized tissue, and regional lymphadenopathy – Cover defects of anterior or posterior vagina by insetting
● Assess radiation effects to surrounding tissues flap along wound margins
● Evaluate need for ostomy after resection – Can be rolled for circumferential vaginal reconstruction
– Skin paddle can be used for perineal reconstruction
– Based on deep inferior epigastric artery
47.2.3 Pertinent Imaging or Diagnostic ● Gracilis myocutaneous flap
– Useful if laparotomy incision is not required for oncologic
Studies resection and in total vaginal reconstruction
● Assess nutritional status: Transferrin, albumin, and – Consider bilateral flaps for large defects and to create a
prealbumin levels neovagina
● Computed tomography (CT) scan to evaluate nodal spread in – Based on medial femoral circumflex artery
the pelvis ● Pudendal thigh (modified Singapore) fasciocutaneous flap
● Positron emission tomography (PET) scan has improved (see ▶ Fig. 47.1)
sensitivity to detect small nodal metastases – Used to reconstruct anterior or lateral vaginal defects

Fig. 47.1 (a–c) Pudendal thigh flap.

196
Perineal Reconstruction

– Can be designed anteriorly, posteriorly, laterally, or medially


○ Most frequently posteriorly-based

– Does not provide sufficient bulk for large defects


– Difficult to mobilize in obese patients
● Pedicled anterolateral thigh (ALT) flap
– Can be used for posterior and lateral vaginal defects and
defects of the perineum
– Favorable donor site
– Can be harvested as fasciocutaneous flap or myocutaneous
flap with vastus lateralis muscle
– Based on descending branch of lateral femoral circumflex

47.5 Complications
● Infection and pelvic abscess
● Partial or complete flap loss
● Wound dehiscence and delayed healing at site of
reconstruction or donor site
● Seroma accumulation in dependent areas
● Venous congestion due to pedicle kinking during inset
● Distal third of skin paddle for gracilis myocutaneous flap may
be unreliable
● Pudendal thigh flap may be damaged by irradiation and
suboptimal for reconstruction

47.6 Critical Errors


● Inadequately filling soft tissue defect in the perineal cavity
after resection
● Failure to discuss with patient postoperative goals for sexual
Fig. 47.2 Posterior thigh flap. function (creation of neovagina, sensation, etc.)
● Failure to account for previous abdominal and lower
extremity scars leading to injury to pedicles for flap
reconstruction
– Useful when considering postoperative sensation; ● Poor compliance with positioning postoperatively (no sitting
innervated by posterior labial branches of pudendal nerve allowed for at least 3 weeks)
and posterior cutaneous nerve of the thigh ● Attempts to reconstruct perineum with skin graft will result
– May be unilateral or bilateral in a high rate of failure in an irradiated bed
– Based on posterior labial vessels
● Thigh flap (see ▶ Fig. 47.2)
– Useful for defects confined to perineum, may not reach
vaginal defects

197
Case 48 Abdominoplasty
Angie M. Paik and Karl H. Breuing

Case 48 (a, b) A 46-year-old female with history of two previous pregnancies and a 40-pound weight loss is interested in improving her abdominal
contour.

199
Trunk

48.1 Description 48.3 Consultations


● Abdominal skin laxity with periumbilical and lower ● General surgery: Coordination for concurrent hernia repair
abdominal striae may be warranted
● Central abdominal protuberance ● Primary care physician: Coordinated care of concomitant
● Adiposity of bilateral flanks and hips medical issues
● Nutritionist: If there is any history of bariatric surgery or
nutritional deficiency
48.2 Work-Up
48.2.1 History 48.4 Patient Counseling
● Elicit the patient’s specific concerns regarding the appearance ● Smoking cessation: Minimum of 6 weeks prior to surgery
of her abdomen ● Weight stability: Stable for at least 12 months prior to surgery
● Concomitant medical issues ● Medical optimization prior to surgery: Strict glucose control
– General risk factors for surgery and wound healing if diabetic, nutritional supplementation as needed, and
○ Cardiac disease, pulmonary disease, diabetes, obesity, cardiac clearance if indicated
connective tissue disease, and steroid use ● Postoperative expectations
– Risk factors for thrombotic event – Drains, binder/compression garment, early ambulation, scar
○ History of deep venous thrombosis, pulmonary emboli, care, and hip flexion
multiple miscarriages, or known genetic hypercoagulable
state
● Obstetric history: Number of previous pregnancies and plans 48.5 Treatment
for future pregnancies
● Surgical history: Previous abdominal surgery 48.5.1 Preoperative Management
● Goal weight and weight stability ● Antimicrobial skin wash and no razor shaving around the
● History of smoking surgical site
● Compression stockings, sequential compression device, and
Caprini risk stratification
48.2.2 Physical Examination ● Photo-documentation
● Calculate body mass index (BMI): Higher incidence of wound ● Preoperative single dose of antibiotic prophylaxis
complications, deep vein thrombosis (DVT)/pulmonary ● Markings and surgical planning
embolism (PE) in obese patients – Lateral extent of incision is determined by soft tissue excess
● Examine excess skin and subcutaneous tissue – With the pannus elevated, midline is marked 6 to 8 cm
– Skin pinch to assess subcutaneous adiposity above the vulvar commissure
– Note any striae and chest/abdominal scars – Efforts made to keep the incision hidden under
● Examine for hernias or rectus diastasis undergarments and in bikini line
– Amount of skin excision can be simulated with a pinch test
of the abdominal apron
48.2.3 Pertinent Imaging or Diagnostic
Studies 48.5.2 Surgical Techniques
● Complete blood count (CBC), basal metabolic profile (BMP), ● Standard abdominoplasty (appropriate choice for this case)
prothrombin time (PT)/international normalized ratio (INR), – Indicated for skin and soft tissue excess of the upper and
and activated partial thromboplastin time (aPTT) may be lower abdomen
considered depending on medical comorbidities – Excellent access to address rectus diastasis and any hernias
● Hematologic testing may be considered if history is – Adjunct liposuction of flanks may be performed, staying
consistent with hypercoagulable process deep to the superficial fascia layer to maintain the
● Fingerstick glucose on the day of surgery, if diabetic subcutaneous vascular network
● Urine beta human chorionic gonadotropin level on the day of ● High lateral tension abdominoplasty (also an appropriate
surgery choice for this case)
● Nutrition labs may be considered for post bariatric surgery – Indicated for patients desiring lateral thigh/hip and
patients abdominal contouring
● Urine cotinine level 10 days prior to surgery (to confirm – Liberal liposuction can be performed due to preservation of
compliance with smoking cessation) lateral perforators
● Computed tomography (CT) of abdomen/pelvis if there is a – Limited undermining just over rectus fascia, and
questionable hernia that is difficult to characterize on discontinuous undermining as needed laterally to preserve
examination vascularity

200
Abdominoplasty

● Fleur-de-lis abdominoplasty – Frank discussion regarding the effects on wound healing


– Indicated for significant soft tissue excess of upper and and respiratory function
lower abdomen often after massive weight loss or with – Urine cotinine test 10 days prior to surgery
existing midline vertical scar – Referral to smoking cessation program
– Determine vertical and horizontal excess through skin
pinch
– Lateral flap mobilization with discontinuous undermining, 48.7 Complications
as needed ● Patient dissatisfaction
● Mini abdominoplasty
– Preoperative discussion on expectations and potential local/
– Indicated for mild to moderate skin laxity confined to the
systemic complications
infraumbilical abdomen with adequate or longer distance ● Local complications
between pubic symphysis and umbilicus
– Hematoma/seroma, infection, fat necrosis, wound
– Must maintain at least 9 cm between umbilicus and
dehiscence, numbness, unfavorable scarring, and contour
superior extent of excision
irregularity
● Reverse abdominoplasty ● Systemic complications
– Indicated for soft tissue excess of the upper abdomen
– DVT, PE, acute blood loss anemia, and systemic infections
● Technical pearls: Progressive retention sutures and lymphatic
(see ▶ Table 42.2 and ▶ Fig. 43.1)
preservation with superficial dissection

48.5.3 Postoperative Management 48.8 Critical Errors


● Encourage early ambulation ● Failure to address medical comorbidities and DVT prophylaxis
● Incentive spirometry ● Ignoring the need for a thorough discussion regarding
● DVT chemoprophylaxis postoperative care
– Risk stratification with Caprini score (see ▶ Table 42.2) ● Inadequate discussion on the possibility of poor or imperfect
● Maintenance of patient in a flexed position to minimize outcome and the possible need for revision
tension on wound margins ● Failure to consider previous abdominal and chest wall scars
● Pain control: Multi-modal with attempt to limit narcotics or abdominal hernias
– Consider intercostal blocks, transverse abdominis plane ● Excision of abdominal tissue without confirming that
blocks, pain pumps, liposomal bupivacaine, etc. for abdomen can be definitively closed causing excess tension on
postoperative analgesia wound margins or inability to close the wounds

48.6 Ethical Considerations


● A patient scheduled for abdominoplasty in 2 weeks who
reports smoking cessation presents for her preoperative
appointment smelling of tobacco smoke

201
49 Acute Burn Injury 205
Section VIII
50 Electrical Injuries 209
Burns 51 Upper Extremity Burns 213

52 Scalp Burn Reconstruction 217

53 Neck Burn Contracture 221

VIII
Case 49 Acute Burn Injury
Charles C. Jehle and Albert S. Woo

Case 49 A 25-year-old male presents to critical care bay of emergency department after sustaining burns from a house fire. Burn injury is partially
shown here and additionally involves the face, back, and both arms.

205
Burns

Inhalation injury
49.1 Description ●

– Examine for singed facial hair and/or soot in the airway


● Acute burn injury to portions of the face, chest, back, – History of asthma or chronic obstructive pulmonary disease
abdomen, bilateral arms and legs, and groin (COPD) can compromise oxygenation
– Superficial and deep partial-thickness burns – Have a low threshold to intubate patients with any
– Approximately 65% of body surface area evidence of inhalation injury
– Possible circumferential involvement of the forearms ● Extent of burn
● Concern for inhalation injury – Rule of Nines: Calculation of extent in percentage of total
body surface area (%TBSA) based on second- and third-
49.2 Work-Up degree burns; does not include first-degree burns
(see ▶ Fig. 49.1)
49.2.1 History and Physical – Assess depth of burn: Partial (superficial or deep) versus
full thickness
Examination – If electrical: Identify entry and exit wounds; determine
● Trauma evaluation path of injury
– ABCs (airway, breathing, circulation): Assess for critical ● Extremities involved
injuries during the primary survey – Circumferential burns: Assess need for escharotomies or
● Mechanism of injury fasciotomies
– Flame: Most common; can lead to superficial to deep burns, ○ Escharotomies for superficial burns to release tight,

depending on degree of exposure leathery burned skin


– Oil or grease burns: Must be careful not to underestimate ○ Fasciotomies when injuries are deep (as in electrical

the severity of such burns; oil will continue to burn over burns); fascial release to prevent necrosis of deeper
longer period of time if not washed off immediately structures
– Electrical (high or low voltage): Raises concern for deeper – Compartment syndrome
injury to underlying structures ○ Compartmental pressures may be measured with STIC

○ There is a greater concern for compartment syndrome pressure monitor (Stryker; Kalamazoo, MI)
and rhabdomyolysis (see Chapter 50) ○ Concern if pressures are > 30 mmHg

Fig. 49.1 Rule of nines for estimation of burn


injury in adults: Head (9%), Arm (9%), Trunk
(18% front, 18% back), Leg (9% front, 9% back),
Groin (1%). In children, the head is
proportionately larger and receives double the
surface area of an adult.

206
Acute Burn Injury

● Face involved ● Dressings and topical ointments: After debridement, multiple


– Eyes: Consider consulting ophthalmology options exist
– Cartilage exposure: Assess for presence of exposed tissues – Face: Antibiotic ointment 3 to 4 times per day
– Ears and nose: Mafenide (Sulfamylon; Mylan
Pharmaceuticals, Morgantown, WV) twice daily because of
49.2.2 Diagnostic Studies improved cartilage penetration
● Chest X-ray – Body and extremities: Silver sulfadiazine (Silvadene; Pfizer,
● Carboxyhemoglobin level: Treat with supplemental oxygen New York, NY) twice daily
● Electrocardiography, heart monitor: Especially in a patient ● Splinting of joints and early range of motion (ROM) with
with electrical burn occupational therapy
● Urine creatine kinase level: When concern for myoglobinuria ● Tangential excision and grafting
exists – As soon as patient is stable, multiple operations generally
required
– Surgery performed in warm environment
49.3 Patient/Family Counseling – Begin with largest areas first
– Address hand burns by 14 days and address the face last
● Patients with severe burn injury are sometimes critically ill
● Surgical end points: 2 to 3 hours of operating time or
and often intubated. Early and frank discussions regarding
transfusion of 10 units of packed red blood cells
the patient’s condition and prognosis should be had with the
● Graft options: Cadaver, xenograft, and autografts
family and, if possible, the patient.
– Split-thickness skin grafts (STSGs) versus full-thickness skin
● The patient and family should understand that acute
grafts (FTSGs)
injuries typically require many operations and long
○ STSGs are more readily available and can be re-harvested
hospital stays in the intensive care unit (ICU) followed by
over time
months to years of physical therapy and reconstructive
○ STSG is commonly used for extremity and trunk
operations.
reconstruction
○ FTSG has less contraction than STSG

49.4 Treatment – Meshed versus sheet grafts: Consider sheet grafts for face
and hands and full-thickness grafts if possible or sheet
● Secure the airway, if you suspect inhalation injury grafts across joints
– Supplemental oxygen if patient is not intubated
– Intravenous (IV) access for maintenance and resuscitation
fluids 49.5 Ethical Considerations
● Resuscitation: Adjust formula for adequate urine output
● Treatment of most large burns almost always requires
(1–2 mL/kg per hour in children and 0.5–1 mL/kg per hour in
resuscitation with blood transfusion. If a patient has ethical or
adults)
religious objections to use of blood products consulting the
– Parkland formula: 4 mL/kg per percentage burn per
hospital’s ethics committee may be necessary.
24 hours
○ Percentage burn: Total second-degree and deeper burns

○ Apply formula for burns greater than 20% TBSA

○ Give half of IV fluids in the first 8 hours, and the second


49.6 Complications
half over the next 16 hours ● Burn sepsis: Fevers, change in mental status, hypotension,
● Early excision and debridement: Remove dead skin and tissue and change in wounds
to fully assess extent of burns and coverage usually with ● Scar/contracture: Avoid with early excision and grafting,
xenografts initially splinting, ROM, and compression garments
● Escharotomies: Incisions designed to optimize function of ● Electrical burns: Cardiac arrhythmias, myoglobinuria, and
extremities and chest, and minimize morbidity and muscle necrosis
contracture
● Fasciotomies in the patient with an electrical burn: Bone
continues to conduct heat and may lead to continued soft- 49.7 Critical Errors
tissue necrosis ● Failure to recognize inhalation injury and secure airway
● Nutrition: Metabolic demands are increased; enteral feeds ● Missing other concomitant injuries and trauma
are ideal ● Not performing escharotomies with circumferential burns
– Place feeding tube if patient is not taking adequate and/or fasciotomies with electrical burns
nutrition orally or if intubated ● Failure to excise and graft deep partial- and full-thickness burns

207
Case 50 Electrical Injuries
Ean Saberski, David Tsai, and Adnan Prsic

Case 50 A 19-year-old man sustained a work-related injury on a high voltage power line. The patient's right shoulder on arrival to the emergency
department is shown.

209
Burns

– Orthopedic examination: Assess for any concurrent


50.1 Description orthopedic trauma
● Evidence of full-thickness burn to the right shoulder, ● Compartment syndrome assessment
suggesting this as a possible entry site – High index of suspicion for both upper and lower
● Exit site of electrical current is unclear extremities
– Disproportionately severe pain to active and passive
motion
50.2 Work-Up – Assess tenderness and firmness of extremity compartments
– Measure compartment pressures: Can use needle pressure
50.2.1 History gauge (STIC pressure monitor; Stryker, Kalamazoo, MI)
○ Pressure > 30 mmHg is an indication for compartment
● Age, gender, handedness, and occupation
● Past medical, surgical, and social history release
● Source of electricity: Power line, power outlet, lightning, – Pain, pallor, paresthesias, paralysis, and pulselessness are all
electrical arc, etc. indications for release and are late findings
● Intensity of electricity: High voltage (≥1,000 V) or low voltage
(< 1,000 V) 50.2.3 Pertinent Imaging or Diagnostic
● Points of contact with electricity: Grasp of power source, fall
onto power source, or nearby flash from power source Studies
● Context of injury: Concurrent with fall or other trauma ● Cardiac studies: Electrocardiogram (ECG), 24-hour cardiac
● Type of electrical current: Alternating current (AC) or direct monitoring
current (DC) – Serial creatine kinase (CK), troponin, lactate dehydrogenase
● Duration of contact with power source (LDH)
● Urine studies: Urinalysis and urine myoglobin
50.2.2 Physical Examination ● Bloodwork: Metabolic panel and complete blood count (CBC)
● Imaging studies
● Advanced Trauma Life Support (ATLS) Protocol: Primary – Computed tomography (CT) of head for all high voltage
survey injuries
– ABCs: Assess if airway is maintained and assess for – Magnetic resonance imaging (MRI) or CT if there is concern
inhalation injury for deep injury
– Expose patient for evaluation; remove all potentially
constricting clothing and jewelry
● Secondary survey: Complete ATLS secondary survey 50.3 Consultations
– % Total Body Surface Area burn: Rule of Nines (see
● Referral to a burn center: All high voltage injuries and criteria
▶ Fig. 49.1)
listed in ▶ Table 50.1
– Indications for transfer to a burn center (▶ Table 50.1)
● Trauma and other services as indicated for additional injuries
– Neurological examination: Establish baseline neurological
found on secondary examination (general surgery, trauma,
examination, assess sensation in all limbs, assess mental
orthopedic surgery, nephrology, etc.)
status, and evaluate cranial nerves
– Vascular examination: Assess perfusion of limbs
– Otoscopic and ophthalmologic examination assessing
extent of injury
50.4 Patient Counseling
● Injuries may evolve over time and require multiple
reconstructive efforts
Table 50.1 Indications for referral to a burn center
● Electrical burns lead to deeper injuries and may threaten
Partial-thickness burns greater Burns that involve the face, hands, limbs and cause kidney injury
than 10% total body surface area feet, genitalia, perineum, or major ● Physical therapy (PT) and rehabilitation are critical to
(TBSA) joints
functional recovery
Third degree burns in any age Electrical burns, including lightning
group injury

Chemical burns Inhalation injury


50.5 Treatment
Burn injury in patients with pre- Any patient with burns and con- ● Low voltage injury with no loss of consciousness and no
existing medical disorders that comitant trauma (such as dysrhythmia can be discharged to home
could complicate management, fractures) in which the burn injury ● Reconstruction should follow principles of burn management
prolong recovery, or affect poses the greatest risk of morbidity (see Chapter 49)
mortality or mortality ● Fluid and electrolyte management
Burned children in hospitals with- Burn injury in patients who will – Crystalloid resuscitation to maintain urinary output of
out qualified personnel or equip- require special social, emotional, or 0.5 mL/kg/hr
ment for the care of children rehabilitative intervention – Continue fluid resuscitation until myoglobinuria clears

210
Electrical Injuries

– Alkalinize urine to prevent precipitation of myoglobin in ● Free tissue transfer: Indicated only for wounds with loss of
renal tubules vascularized surface (exposed tendons, vessels, nerves, and
– Mannitol infusion if myoglobinuria persists bones)
● Fasciotomy and compartment release – Not recommended for other indications in acute period
– Indicated following high voltage electrical injury (> 1,000 V) – High risk of thrombosis and failure during high
– Upper and lower extremity compartments to be released, if inflammatory acute burn state
necessary (see ▶ Fig. 51.1 and ▶ Fig. 51.2) ● Amputation: Only when indicated
– Decompression of the carpal tunnel should also be
considered
– Consider 48 to 72 hours period of observation for tissues to 50.7 Complications
demarcate prior to escharotomy (see Chapter 51) ● Late neurologic symptoms including weakness, numbness,
● Peripheral nerve decompression
and pain
– Controversial, but may consider at the time of compartment ● Cataracts may develop through unknown mechanism
release ● Burn scar contracture
● Debridement ● Joint contractures
– Myonecrosis shows 3 to 5 days following injury ● Infection
– Decide for early versus late debridement on individual basis
● Temporary skin replacement
– Cadaver allograft 50.8 Ethical Considerations
● Permanent skin replacement
– Split-thickness skin graft (sheet or meshed) ● Autonomy, beneficence, nonmaleficence, and justice must be
○ Meshed grafts: Trunk, upper arm, and forearm respected
○ Sheet grafts: Dorsum of the hand and fingers ● Informed consent
– Full-thickness skin graft: Palmar surface – Risks, benefits, and alternative treatments
(glabrous skin) – Possible intraoperative decisions
● Nonautogenous materials – Risk to life, limb, and eyesight must be emphasized when
– Alloderm: Human cadaveric acellular allogeneic dermal appropriate
matrix (LifeCell, Bridgewater, NJ)
– Integra: Bovine collagen dermal regeneration scaffold
(Integra, Plainsboro, NJ) 50.9 Critical Errors
– Apligraf: Bovine collagen + human-derived fibroblasts
● Failure to perform Advanced Trauma Life Support (ATLS) and
bilayered living skin equivalent (Organogenesis, Canton,
evaluate for additional injuries
MA)
● Failure to perform early escharotomy or fasciotomy as needed
– Dermagraft: Human-derived fibroblast bilayered
(see Chapter 51)
bioabsorbable living skin equivalent (Advanced BioHealing,
● Poorly designed escharotomy or fasciotomy leading to
New York, NY)
exposure of critical structures
– Cytal/Micromatrix: Bovine urinary bladder matrix (Acell,
● Inadequate assessment of all compartments of extremities
Columbia, MD)
● Insufficient debridement of wounds and colonized/infected
tissue
50.6 Secondary Procedures – Graft failure secondary to infection/inadequate
debridement
● Contracture release ● Incorrect graft choice leading to poor functional outcome
● Compression garments/silicone sheets (e.g., meshed grafts on the hand instead of sheet grafts to
● Intralesional steroids minimize contracture)
● Early occupational therapy (OT)/ PT ● Incorrect splinting and delayed PT and OT, leading to
● Local flaps or pedicled flaps: If skin grafting is inadequate contractures and requirement for avoidable secondary surgery

211
Case 51 Upper Extremity Burns
W. Kelsey Snapp, Albert S. Woo, and Adnan Prsic

Case 51 A 1-year-old boy presents with burns to the left upper extremity after dipping his hand in a pot of boiling water.

213
Burns

Intracompartmental pressures > 30 mmHg require


51.1 Description ○

intervention
● Circumferential burns of variable thickness to the left upper ● Secondary reconstruction
extremity in a pediatric patient – Status of soft-tissue coverage (thickness, durability,
● Blistering suggestive of second degree burn, with the sensibility, and elasticity)
possibility of third degree injury – Contractures, and active and passive range of motion (ROM)
● Burn of this variety in 1-year-old patient suspicious for abuse of each joint
or neglect

51.2.3 Pertinent Imaging or Diagnostic


51.2 Work-Up Studies
51.2.1 History ● Standard radiography (three views) of the hand, elbow, and
humerus
● Age, gender, handedness, and occupation of the patient ● Bloodwork: Complete blood count (CBC), electrolytes, blood
● Timing and mechanism of burn injury urea nitrogen (BUN), creatinine, international normalized
– Thermal: Type of burn (flame, contact, scald, steam, or ratio (INR), partial thromboplastin time (PTT), glucose, and
grease); associated injuries; suspicion of abuse (especially blood type
in elderly and children) ● Consider arterial blood gases for associated inhalational
– Chemical: Type of chemical (alkali, acid, or organic injuries or certain chemicals
compound); any attempts at neutralization ● Cardiac enzymes, urine myoglobin, creatine kinase, and
– Electrical: Type of current (alternating current [AC] or direct 12-lead electrocardiography (hyper K + , electrical burns)
current [DC]), voltage, and location of initial contact
● Past medical and surgical history
● Social history, including tobacco, alcohol, and substance 51.3 Consultations
abuse
● Indications for referral to a burn center (see ▶ Table 50.1)
● General surgery or critical care team if burns are
51.2.2 Physical Examination extensive
● Poison control center if chemical is involved and
● Primary survey: Advanced Trauma Life Support (ATLS)
management is unclear
protocol and ABCDs.
● Hand is scrubbed of any soot, dirt, or debris
– Potentially constricting clothing, jewelry, and watches are
removed 51.4 Treatment
● Acute injury
– Location and total body surface area of burn injury (see
51.4.1 Acute Burn Management
Chapter 49) ● Advanced Burn Life Support (ABLS)/Advanced Trauma Life
– Depth of burn injury: First degree, superficial or deep Support (ATLS) guidelines
second degree, or third degree; note exposure of deep ● Fluid resuscitation with Parkland Formula (see Chapter 49)
structures ● Escharotomy (see ▶ Fig. 51.1)
– Perfusion of upper extremity, hands and fingers – Indications
– Motor and sensory function ○ Full-thickness, circumferential burns

– Compartment syndrome: Limb-threatening condition ○ Consider in partial-thickness circumferential burns at risk

○ Circumferential burns; associated crush injury or trauma for compartment syndrome with onset of post-
○ Pain out of proportion with movement (finger extension, resuscitation edema
flexion) ○ Evidence of compartment syndrome

○ Five P’s (late signs): Pain, pallor, paresthesias, paralysis, ○ To prevent further soft-tissue death due to vascular

and pulselessness compromise

a b c

Fig. 51.1 (a–c) Escharotomies of the hand.

214
Upper Extremity Burns

○Incisions should minimize morbidity and optimize future – Consider a 48 to 72 hours period of observation for tissues
hand function to demarcate
– If fingers are threatened, mid-axial incisions over non- – Advantages of early excision: Improved hand
dominant sides may be performed function, reduced risk of abnormal scarring, reduced
● Fasciotomy number of reconstructive procedures, decreased
– Highvoltage electrical injury (> 1,000 V) or severe burn length and cost of hospital stay, and reduced pain and
injury (see Chapter 50) complications associated with prolonged
– Release fascial compartments immobilization
○ Forearm (see ▶ Fig. 51.2): Superficial and deep volar,

dorsal, and mobile wad


○ Hand (see ▶ Fig. 51.3): Dorsal interossei (four
51.4.2 Skin Resurfacing Options
compartments), palmar interossei (three compartments), ● Temporary skin replacement with cadaver allograft
adductor pollicis, thenar, and hypothenar ● Split-thickness skin graft (sheet or meshed)
– Decompression of the carpal tunnel should also be – Meshed grafts: Trunk, upper arm, and forearm
considered – Sheet grafts: Dorsum of the hand and fingers
● Burn eschar excision ● Full-thickness skin graft: Palmar surface (glabrous skin)

Fig. 51.2 Fasciotomies of the forearm.

A B C D E

Fig. 51.3 (a, b) Fasciotomies of the hand.

Dor. interossei
B C
Dor. interosseous
fascia
II III
IV
C B V
I D
A

Thenar Hypothenar
muscle Ad. pollicis Vol. interossei muscle
a

Trans. ret. lig. Cleland’s lig.


b

215
Burns

Fig. 51.4 Reconstruction of first web space


contracture with four-flap Z-plasty.

A C A
B D C
B D

Nonautogenous materials: AlloDerm, Integra, Apligraf,


Dermagraft, etc.
51.5 Ethical Considerations
● Cultured epidermal autograft ● Informed consent must be sought along the way for all
– Cultured epidermal sheets from skin biopsy of the patient interventions
– Expensive, require 2 to 3 weeks to culture, and are thin and ● Futility of care/reconstruction will occasionally arise and
unstable must be carefully discussed with the patient and surrogates
● Flap reconstruction options in the context of beneficence, non-maleficence, and justice
– Local and pedicled flaps
– Free flaps: Not recommended in acute period due to high
risk of thrombosis and failure during high inflammatory 51.6 Complications
state of acute burn
● Infection
● Inadequate graft take secondary to shear forces, and
51.4.3 Prevention of Secondary Injury inadequate wound bed or infection
● Wound breakdown, partial or total flap loss
● Edema control: Early motion and elevation ● Hypertrophic scarring
● Occupational therapy for wrist and hand rehabilitation with ● Burn scar and joint contractures
splinting to prevent joint contractures (static, static–
– Claw hand deformity: Inadequate splinting, therapy, or
progressive, or dynamic)
early operative management
● Surgical immobilization with Kirschner wires across joints
– Upper arm: Elbow contractures and axillary contractures
(increased risk for infection)

51.4.4 Secondary Burn Management 51.7 Critical Errors


● Hypertrophic scarring and contracture bands ● Failure to perform early escharotomy or fasciotomy, as
– Compression garments and silicone sheet therapy needed
– Early active motion therapy ● Poorly designed escharotomy or fasciotomy leading to
– Intralesional steroid injections exposure of critical structures
– Scar band release with local tissue rearrangement ● Inadequate assessment of all compartments of the hand or
(V-Y advancement, Z plasty) or skin graft extremity, especially in circumferential burns
● First web space contracture release – Incomplete release of fascia or eschar leading to
– Skin and adductor pollicis fascia contracture release neurovascular compression
– First dorsal interosseous and adductor pollicis muscle ● Inadequate debridement of wounds and colonized/infected
releases, if needed tissue leading to systemic illness and subsequent graft failure
– Local flaps: Four-flap Z-plasty (see ▶ Fig. 51.4), V-M plasty, ● Incorrect graft choice leading to poor functional outcome
and five-flap Z-plasty (e.g., meshed grafts on the hand instead of sheet grafts [to
– Regional flap: Reverse radial forearm and reverse posterior minimize contracture])
interosseous artery flaps ● Incorrect splinting and delayed physical and occupational
– Free flap: Lateral arm flap and radial forearm flap from therapy leading to contractures and requirement for
contralateral arm avoidable secondary surgery

216
Case 52 Scalp Burn Reconstruction
Lauren O. Roussel and Albert S. Woo

Case 52 (a, b) A 45-year-old man with history of burns to the scalp after a work-associated explosion presents for scalp reconstruction.

217
Burns

52.1 Description 52.3 Treatment


● Evidence of skin grafting overlying the vertex of the scalp ● Establish patient’s expectations for reconstruction
● Notable alopecia on vertex extending to right side – Correction of alopecia (bring in new hair-bearing tissue)
● Defect involving > 50% of normal hair-bearing scalp – Improvement in hairline
surface – Optimization of facial appearance (excise grafted regions
and replace with local tissue, if possible)

52.2 Work-Up
52.3.1 Flap Coverage
52.2.1 History ● Viable option for smaller defects of the scalp
● Etiology of injury, including mechanism and depth of burns ● Scalp tissue has less mobility than other parts of the body
● Time interval since injury ● Large flaps should be designed to optimize result and
● History of prior reconstruction minimize tension
● Medical comorbidities ● Common flap options:
– Wound healing problems – Rotation
– Smoking history – Advancement (V-Y)
– Bleeding disorders – Transposition
– Orticochea (see ▶ Fig. 52.1)
– Pinwheel (see ▶ Fig. 52.2)
52.2.2 Physical Examination
● Assess size of scar
52.3.2 Tissue Expansion
● Evaluate for areas of scalp laxity
● Assess directionality of remaining hair (Defects up to 50% of scalp can be reconstructed)
● Assess for other scars or affected body regions ● Preferred technique for scalp reconstruction

Fig. 52.1 Orticochea three-flap technique for


Flap 3
coverage of large defects of the scalp.
Relaxing
incisions

1 2
Flap 1

Flap 2

Bone
grafts

2
1

218
Scalp Burn Reconstruction

Fig. 52.2 Pinwheel flaps for closure of a circular


defect.

● Multiple expanders may be used for single defect ● Facilitate flap advancement and tension-free closure with
– Use largest expander(s) possible galeal scoring at 1 cm intervals
● More than one expansion may be performed – Score in a direction perpendicular to the direction of
● Incisions made perpendicular to axis of expansion desired tissue gain
– Incisions can be placed within lesion to allow future – Use caution as scoring may compromise vascularity of
excision overlying skin
● Expander placed in subgaleal plane – Approximately 1 mm additional lengthening is achieved per
● Internal ports are less convenient but have a lower risk of score
infection compared to external ports ● Do not excise dog ears: These usually settle over time on the
convex scalp

52.3.3 Expansion Technique


● Begin expansion about 2 weeks after tissue expander (TE)
52.3.4 Reconstruction by Region
placement; continue at weekly intervals ● Anterior scalp defects: Goal is to recreate anterior hairline
● With each session, fill until patient feels discomfort or – Tissue expansion
notable skin changes occur – Advancement with or without rotation, V-Y advancement,
● Continue until there is sufficient skin to cover the defect and rhomboid flaps
– Available tissue = Dome of expander – Base width of – Orticochea flaps (see ▶ Fig. 52.1)
expander ○ Two flaps based on superficial temporal vessels to fill

– Create flap approximately 20% larger than the defect to defect


account for recoil of tissue ○ One large occipital flap to fill donor site defect

– Second stage surgery should be performed once adequate ● Parietal scalp defects: More scalp mobility present than
expansion is achieved anteriorly
● Design flaps with a combination of advancement and rotation – Tissue expansion
based on at least one named vessel as the pedicle – V-Y advancement and rhomboid flaps for sideburns

219
Burns

– Rotation with or without advancement and bipedicled


fronto-occipital flaps
52.4 Complications
● Occipital scalp defects ● Expander exposure
– Tissue expansion – Exposed implants must be removed
– Rotation with or without advancement flaps – If significant expansion has already taken place, the
– Orticochea three-flap technique expander should be removed and the tissues should be
● Vertex scalp defects: Limited scalp mobility here advanced as much as possible
– Tissue expansion ● Infection: When an expander is involved, this should be
– Pinwheel (see ▶ Fig. 52.2), rhomboid, and rotation with or removed and tissues advanced
without advancement flaps ● Hematoma and seroma
– Large rotation flaps requiring near-complete scalp ● Expander puncture/deflation
undermining ● Skin necrosis
● Near-complete scalp defects: Complete coverage with hair ● Insufficient tissue to reconstruct the entire defect: Advance
bearing skin may not be possible; goal may simply be scalp flaps and re-expand
coverage with tissue
– Free tissue transfer
○ Muscle flaps create nice contour with atrophy (e.g., 52.5 Critical Errors
latissimus dorsi)
○ Omentum, radial forearm, and anterolateral thigh (ALT)
● Incision for TE placement in the hair-bearing scalp meant to
– Integra and split-thickness skin graft (STSG) be expanded
● Hair transplantation ● Incision made directly over expander or parallel to axis of
– Remains an option if adequate hair-bearing scalp is available expansion
– Requires excision of hair-bearing regions of the scalp with ● Failure to assess skin when filling expander
direct closure of donor site ● Failure to advance the flaps when removing an infected or
– Transplantation can be performed with separation of hair exposed expander
into naturally occurring follicular units. These are separately ● Excision of entire defect prior to advancing the flaps and
inserted into regions of alopecia: determining if there is sufficient tissue for coverage
○ Micrograft: One to two hair follicles
● Poor design of flaps (e.g., flaps that are too small to correct
○ Minigraft: Three to six hair follicles the defect)

220
Case 53 Neck Burn Contracture
Sarah A. Frommer and Renata S. Maricevich

Case 53 (a–c) A 13-year-old female with extensive burn from a fire 6 years ago. She had undergone previous skin grafting to the neck and torso. She
is now complaining of tightness when moving her neck.

221
Burns

– Should be addressed before intrinsic facial contractures


53.1 Description because neck release can change lower facial reconstruction
● Extensive burn contracture involving the neck and anterior plan
trunk – Goals are to establish improved range of motion and restore
– Evidence of previous split- and full-thickness grafts to the natural cervico mental angle
neck and torso – If pediatric patient, early intervention should be performed
– Hypertrophic scars and multiple tethering bands to reduce restriction of skeletal maturation and speech
– Notable limitation in extension of neck ● Retrognathia and mental retrusion
– Obtuse cervico mental angle – Bilateral sagittal split osteotomies versus distraction
osteogenesis
– Sliding genioplasty versus implant
53.2 Work-Up
53.2.1 History and Physical 53.4.2 Surgical Management
Examination (Neck Focused) ● Scar excision
– Respect aesthetic subunits
● Mechanism, depth, and extent of burns ○ Lower lip and chin
● Time interval since injury and treatments performed
○ Submental
● History of drooling, difficulty eating, speech deficiencies,
○ Anterior neck unit
abnormal posture, limitation of range of motion of neck, and
– Scoring the platysma or full excision may be needed for
occlusion
adequate contracture release
● Patient’s aesthetic concerns
– Z-plasty of individual contracture bands may not provide
● Identify contracture bands
sufficient functional improvement
● Assess for donor site availability
● Graft coverage
● Medical comorbidities
– Each aesthetic subunit should be grafted individually
● Speech and swallow evaluation
– Integra
● Airway assessment ○ No donor site, but needs additional skin grafting in about

2 weeks
53.2.2 Pertinent Imaging or Diagnostic ○ Higher risk of contracture recurrence and poor aesthetic

outcome
Studies – Split-thickness grafting
● Computed tomography (CT) scan: Obtain cephalometric ○ Performed as sheet grafts to minimize contracture

measurements to assess mandible and chin position; useful ○ Higher risk of contracture recurrence and poor aesthetic

to assess recipient vessels (with contrast) if planning for free outcome


flap reconstruction – Full-thickness grafting
● Preoperative anesthesia assessment: Airway needs to be ○ Most common method for skin grafting for neck

assessed; possible need for direct laryngoscopy, or even neck contractures


release under sedation/tracheostomy in extreme cases ○ Improved aesthetic outcome and reduced risk of

contracture over split grafting


○ Donor site limitations for adequate skin coverage
53.3 Patient Counseling ● Flap coverage with or without tissue expansion
– Local pedicled flaps
● Often require multiple surgeries to achieve reasonable range
○ Appropriate options: Supraclavicular flap, occipital artery
of motion and aesthetics
flap, and dorsal scapular flap
● Surgical outcomes may never meet patient’s hopes in regard
○ May need debulking
to aesthetics
– Free tissue transfer
● With grafting, postoperative period with long-term splinting;
○ Thinner flaps are favorable
patient needs to be able to demonstrate compliance and
○ Flaps may be pre-expanded to get enough tissue to cover
understanding of postoperative plan
entire neck aesthetic subunit
● Recurrent contractures are possible and need to be discussed
○ Appropriate options: Radial forearm flap, anterolateral
● Airway may become an emergent issue
thigh flap, parascapular flap, and groin flap
○ Good aesthetic outcome and low risk for contracture

53.4 Treatment ○ May need debulking

○ Donor site can be associated with high morbidity

53.4.1 Surgical Planning for Neck ● Postoperative management


– Neck brace/splinting to minimize shear with movement for
Contracture grafting
● Consider need to stage reconstructions and plan for available – Physical therapy protocol
donor site use – Fibrin glue and/or negative pressure wound therapy may be
● Neck contracture helpful adjunctive measures to maintain graft adherence

222
Neck Burn Contracture

– Immediate postoperative and long-term feeding ● Flap loss


considerations ● Poor aesthetic outcome
○ Possible need for nasal feeding tube or even G-tube

53.7 Critical Errors


53.5 Ethical Considerations ● Lack of early anesthetist involvement for airway evaluation
● Parent and patient compliance is a major consideration for and plan for safe intubation
postoperative care and outcome. Lack of patient involvement ● Poor operative plan (e.g., full-thickness grafts yield better
could lead to worse morbidity than current function. results than split thickness and should be used if there is an
appropriate donor site)
● Absence of postoperative plan to prevent graft loss (e.g., neck
53.6 Complications brace, possible feeding tube)
● Failure to recognize and counsel patient that multiple
● Contracture recurrence
procedures may be required to restore good range of motion
● Loss of graft secondary to inadequate prevention of shearing
and aesthetic appearance
and/or infection
● Inadequately managing patient’s expectations
● Infection

223
54 Flexor Tendon Laceration 227
Section IX
55 Degloving Injury 231
Hand 56 Traumatic Amputation 235

57 Peripheral Nerve Injury 239

58 Dupuytren’s Contracture 243

59 Syndactyly 247

60 Metacarpal and Phalangeal Fractures 251

61 Carpal Tunnel Syndrome 255

62 Adult Brachial Plexus Injury 259

IX
Case 54 Flexor Tendon Laceration
Reena A. Bhatt

Case 54 (a, b) A 50-year-old female presents to the emergency department with a laceration sustained while cutting an avocado. The patient is
attempting to flex in the second image.

227
Hand

54.1 Description 54.2.3 Pertinent Imaging or Diagnostic


● Abnormal resting cascade of the hand involving the left small Studies Laboratory investigations

finger ● Standard three-view X-rays (anteroposterior, lateral, and


● Laceration over the volar distal palm, distal to the distal oblique) of the hand should be obtained to assess for fracture
palmar crease, likely in proximal zone 2 cbc, chemistry,
or foreign bodies coagulation profile
● Images show lack of active flexion of the distal abo blood group
interphalangeal joint (DIP) and proximal interphalangeal
(PIP) joint and loss of cascade 54.3 Patient Counseling informed consent
● Rule out injuries to nerves, lumbricals, vessels, or tendons to
● Patient should be counseled on critical importance of
small and ring fingers as well
postoperative compliance with splinting and hand therapy.
Potential complications should be reviewed benefits
54.2 Work-Up
54.2.1 History 54.4 Treatment
● Mechanism of injury (sharp, blunt, or avulsion) ● Antibiotics and tetanus prophylaxis as indicated
● The position of the hand at the time of injury (flexed vs. ● If unable to repair immediately: Irrigate wound, repair
extended position) laceration, and splint with wrist and MCPs in flexion
● Numbness or tingling (question whether patient had local ● Timing
anesthetic injected) – Explore immediately for threatened/devascularized finger
● Significant bleeding at time of injury – Ideal operative treatment within 72 hours for zone 2
– Suggests arterial and digital nerve injury repairs, up to 14 days for delayed primary repair
duration ● Time elapsed since injury (e.g., 2 days vs. 2 months) – Delayed presentation (i.e., injury > 6 weeks old): Patient
of injury ● Right hand dominant versus left hand dominant should be counseled for staged tendon reconstruction,
● Any additional or associated injuries tendon graft, or salvage/fusion
○ If FDS is intact, staged repair of FDP can be deferred if DIP
● Occupation and hobbies
● Prior injuries or associated medical problems especially to is stable (FDS-only finger)
hands surgical history, drug history, psychological history ● In the operating room, traditionally flexor tendons are
● Comorbidities and social history (drug/alcohol abuse) repaired with tourniquet and general anesthesia. However,
smoking wide awake local anesthesia no tourniquet (WALANT) is
quickly gaining favor
54.2.2 Physical Examination ● Prepare for digital nerve repairs with or without artery
● Resting cascade of the hand: Interphalangeal joints should (microscope/loupes)
maintain flexion with arc increasing as examination moves
ulnarly 54.4.1 Flexor Tendon Repair Technique
● Tenodesis (especially important to check with child/obtunded
patient) ● Utilize existing lacerations and extend to appropriate Bruner
– Passive wrist extension results in flexion of the fingers (zig-zag flaps) or Bunnell (midaxial incision) to improve
– Lack of flexion of the lacerated finger demonstrates exposure
discontinuity of flexor tendons ● Proximal tendon(s) often retracted into palm (sometimes into
● Level of laceration and likely zone of injury wrist)
(e.g., laceration over center of middle phalanx, – Flex wrist and milk tendon distally while retrieving with
likely zone 1 and flexor digitorum profundus [FDP] only tendon passer
repair) – Alternatively, tendon can be found through proximal
● Separately assess active flexor action for all fingers incision at a level proximal to A1 pulley
– Flexor digitorum superficialis (FDS): The examiner – Pulley or portions of pulleys can be vented if needed for
maintains other digits in extension while attempting repair or gliding; A2 pulley should be preserved
flexion of injured finger to isolate the FDS ● Distal tendon retrieved by flexing digit passively; retrieve
– FDP: The examiner maintains the PIP in extension while with tendon passer
attempting flexion of the DIP ● Repairs are performed with core sutures and epitendinous
– Flexor pollicis longus (FPL): In the setting of an injured sutures
thumb/palm, the examiner maintains the – 3–0 or 4–0 permanent suture for core sutures: Braided
metacarpophalangeal joint (MCP) in extension (Ethibond), braided looped (Supramid), and monofilament
while attempting flexion of the interphalangeal (Prolene)
joint (IP) – Strength of repair correlates with number of core suture
● Sensibility: Radial and digital nerve injuries strands and size of suture
● Perfusion of the digit: Absence of perfusion would prompt – Recommend ≥4 strand core sutures to allow for early active
emergent intervention protocol postoperatively

228
Flexor Tendon Laceration

Fig. 54.1 The modified Kessler two-strand core


suture repair.

Cruciate type
Fig. 54.2 The cruciate-type four-strand core
suture repair.

– Multiple types of core suture repairs small finger while flexing wrist), plantaris, toe extensors,
○ Two-strand modified Kessler (see ▶ Fig. 54.1) extensor indicis proprius, and extensor digiti minimi
○ Four-strand cruciate repair (see ▶ Fig. 54.2) ● If there is significant sheath scarring or need for pulley
– Epitendinous sutures provide up to 20% of repair strength reconstruction, two-stage reconstruction can be performed
(see ▶ Fig. 54.3) – Resect flexors within sheath, and leave distal FDP stump at
○ 5–0 or 6–0 monofilament placed in running fashion least 1 cm
Little finger – May consider single tendon repair (i.e., FDP only, especially – Hunter rod (silicone implant) threaded from level of mid-
zone 2
only FDP repair in zone 2 small finger, given small size of sheath and high lumbrical FDP to distal insertion through remnant sheath
adhesion rate) and secured
– Pulley reconstruction can be performed as needed
– Patient allowed to perform passive motion with hand
54.4.2 Postoperative Care therapy instruction
● Dorsal blocking splint with wrist in slight flexion (20 – Approximately 3 months later, Hunter rod replaced by
degrees), MCPs (50 to 70 degrees) flexion, and IPs neutral; all tendon graft from level of mid-lumbrical to stump;
fingers need to be blocked dorsally except thumb (common proximal repair of graft with Pulvertaft weave
muscle belly)
● Children may require finer sutures; patients who are unable
to comply with postoperative therapy protocol will require 54.4.4 Postoperative Treatment
casting for 4 to 6 weeks
● Prompt involvement of hand therapist for early motion
protocol as well as orthoplast splint (1–5 days postoperatively)
54.4.3 Flexor Tendon Reconstruction/ ● Improved tendon gliding and diminished adhesions as well as
Secondary Repair increased strength of repair with early controlled
Belfast
mobilization protocols (passive or active)
● In the absence of significant tendon sheath scarring or
destruction, tendon grafting can be performed from the level
of the palmar FDP to the insertion
– Need to resect the remainder of flexors within the sheath
54.5 Ethical Considerations
except for distal 1 cm stump ● A patient’s ability to comply with the postoperative regimen
– Common grafts include palmaris longus (examine patients is a major factor for favorable outcome. Significant
preoperatively by having them actively oppose thumb to complications can occur in the injured digit and adjacent

229
Hand

Fig. 54.3 (a–c) Various epitendinous suture


techniques for flexor tendon repair.

digits if the patient is unable to comply. A patient with a lumbrical plus deformity (paradoxical PIP extension with
lacerated FDP in the setting of intact FDS and inability to attempted flexion secondary to FDP retraction or long tendon
comply with postoperative regimen may be more suitable for graft), pulley rupture with bowstringing, and swan-neck
FDS only finger. deformity secondary to FDS transection

54.6 Complications 54.7 Critical Errors


● Stiffness/Adhesions/Contractures: Surgical tenolysis may be ● Missed injuries: Other tendons, nerve, vascular, and joint
considered as early as 3 months if the patient plateaus ● Excessive pulley division with bowstringing
● Ruptures: Re-repair, tendon grafting, or staged tendon ● Inadequate immobilization
reconstruction ● Failure to start hand therapy early
● Hematoma and soft tissue loss ● Inadequately addressing soft tissue deficits
● Quadriga effect (weak or limited flexion of other digits ● Starting aggressive hand therapy with inadequate strength of
secondary to excess advancement of FDP tendon repair), repair (i.e., two-strand technique)

230
Case 55 Degloving Injury
Charles C. Jehle and Adnan Prsic

Case 55 A 55-year-old man presents with a wound after degloving injury of the dorsal right upper extremity caused by a motorcycle collision. Image
demonstrates appearance after initial operative debridement.

231
Hand

55.1 Description 55.3 Consultations


● Dorsal hand soft tissue defect secondary to avulsion ● General surgery/trauma in the setting of other injuries, if
mechanism indicated
– Dorsal: Zones V, VI, VII, and part of VIII ● Orthopedics in the setting of upper extremity long bone
– Full-thickness soft tissue defect down to extensor tendons fractures
– Paratenon has been stripped and extensor retinaculum is ● Social work for adjustment to illness; approximately 90% of
intact patients with hand injuries display at least one sign or
– Segmental loss of ring finger extensor tendon(s) symptom of post-traumatic stress disorder (PTSD) or acute
● No vascular compromise distally stress disorder (ASD) within 2 months of initial injury

55.2 Work-Up 55.4 Preoperative Patient


55.2.1 History Education
● Age, gender, and social history, paying particular attention to ● Setting expectations for recovery and discussion of possible
handedness, occupation, smoking status, and substance abuse complications
of the patient ● Critical importance of active participation in rehabilitation
● Functional status of hand prior to injury
● Previous injury or surgery of the hand in question
● Manual demands of daily living and overall lifestyle 55.5 Treatment
● Past medical and surgical history
● Timing and mechanism of injury
55.5.1 Key Principles
– Trauma: Associated injuries, underlying fractures, ● Establishment of clean and well-vascularized wound bed and
dislocations, and neurovascular insult coverage
– Infection (if subacute or chronic) ● Critical structures such as vessels, nerves, and tendons must
○ Type of infection (bacterial, fungal, other) be covered with vascularized soft tissue
○ Operative management to date (debridements, incision, – Reconstruction must take tendon glide into consideration
and drainage) ● Grafting of ring finger extensor tendon
○ Antimicrobial treatment and any resistance ● Consider patient’s overall medical condition, manual
○ Local versus systemic signs and symptoms demands, and patient-centered priorities for reconstruction
● Carefully consider duration of immobilization after surgery
– Adequate time for skin graft/dermal substitute take and
55.2.2 Physical Examination tendon graft healing
● Advanced Trauma Life Support (ATLS) protocol – Adequate time for bone healing, if not treating with rigid
● Complete upper extremity examination starting with the fixation
shoulder, elbow, and hand – Patient should begin movement of joints and tendons as
– Brachial plexus injury is most common in motorcycle soon as it is safe, to avoid adhesions and joint stiffness (as
collisions early as 2 days to 2 weeks post operation)
● Location and size of soft tissue deficit
● Specific deficits: Tendon, nerve, muscles, and skin
● Wound bed status: Vascularity, exposed structures, and
55.5.2 Treatment Options
nonviable skin ● Dermal substitutes
● Motor function: Delineate musculotendinous deficit from – Scaffolds composed of extracellular matrix components:
neurologic deficit Collagen and glycosaminoglycan or hyaluronic acid
● Sensory function – Generally followed by split-thickness skin graft (STSG) over
scaffold after incorporation
● Skin graft (split or full thickness)
55.2.3 Pertinent Imaging or Diagnostic – When possible, use full-thickness grafts to decrease
Studies secondary contracture, optimize graft durability, and
improve aesthetics
● Plain radiography: Three views of hand, one joint above and
– Formation of tendon adhesions high in both instances
below level of injury
● Local flaps: Good for small defects
● Computed tomography (CT): In the setting of comminuted
– Transposition flaps: Z-plasty and rhomboid flap
fractures requiring detailed evaluation
– Advancement flaps: V-Y advancement
● Angiography: Evaluate vascular patency for soft tissue
– Axial pattern flaps: First dorsal metacarpal artery flap and
transfer, when needed
neurovascular island flap

232
Degloving Injury

● Regional flaps ● Interventions which may require considerable postoperative


– Cross-finger flaps: Standard, innervated, or reverse cross therapy should be matched to patient’s level of motivation
finger, and cross-thumb flaps and ability to comply with therapy
– Radial forearm flap: Fasciocutaneous, or fascia only
– Reverse radial forearm flap
– Posterior interosseous artery flap 55.7 Complications
● Distant flaps: Pedicled ● Stiffness/joint contractures
– Abdominal/epigastric flaps ● Tendon adhesions
– Groin flap ● Free flap failure: Infection, ischemia, embolic/thrombotic
● Free tissue transfer: Commonly used flaps
events, and technical failure
– Temporoparietal fascia flap
– Typically within the first 72 hours postoperation
– Lateral arm flap
– Arterial occlusion presents as a pale, cool flap
– Anterolateral thigh (ALT) flap ○ Arterial occlusion by thrombus warrants operative
– Scapular and parascapular flaps
exploration
– Serratus anterior muscle flap ○ Dressing may cause external compression
– Latissimus dorsi muscle flap
– Venous insufficiency presents as a swollen, congested (blue/
– Superficial circumflex iliac artery flap
purple) flap with brisk cap refill
● “Spare Parts” from nonviable segments of injured tissue may ○ Venous congestion should be treated with operative
be used for skin grafts, fillet flaps, bone or tendon graft, etc.
exploration
● Immediate or delayed grafting of extensor tendon at the time
of soft tissue coverage
55.8 Critical Errors
55.5.3 Secondary Procedures ● Inadequately vascularized wound bed (e.g., skin graft on bone
● Flap revision: Reduce bulk, trim dog ears, etc. or exposed tendon)
● Scar contracture: Z-plasty and local tissue rearrangement ● Inadequate debridement and increased risk of infection
● Joint contracture release ● Failure to consider functional demands of patient when
● Tenolysis selecting reconstructive options
● Inadequate medical work-up and clearance prior to free
tissue transfer
55.6 Ethical Considerations ● Negligence in informing patient of donor site morbidity and
additional surgical stages which may be required for
● Patients must be informed that poor cooperation with hand definitive reconstruction (e.g., contracture release, tenolysis,
therapy and/or postoperative instruction often preclude a flap thinning) and time necessary before return to work
favorable outcome

233
Case 56 Traumatic Amputation
W. Kelsey Snapp and Adnan Prsic

Case 56 A 67-year-old male with right hand dominance sustained a sharp amputation of the distal phalanx of his left thumb with an axe 1 hour ago.
The amputated segment has been brought on ice.

235
Hand

Social work for adjustment to illness


56.1 Description ●

– Approximately 90% of patients with hand injuries display at


● Sharp amputation of distal phalanx through interphalangeal least one sign or symptom of post-traumatic stress disorder
joint of the left thumb (PTSD), or acute stress disorder (ASD) within 2 months of
– Falls within T1 zone of flexor tendon initial injury
● No other visible soft tissue deficits or segmental injury

56.2 Work-Up 56.4 Patient Counseling


● Discuss replantation versus complete amputation
56.2.1 History ● Anticipate extended operative time and a multi-day hospital
admission after replantation
● Age, gender, and social history
● Discuss complications such as replant failure, blood loss,
– Particular attention to handedness, occupation, smoking/
prolonged immobilization, long-term stiffness, nonunion,
nicotine use, and substance abuse
malunion, infection, neuroma formation, and complex
● Medical history, surgical history, and comorbidities
regional pain syndrome
● Mechanism of injury
● Systemic complications such as deep vein thrombosis (DVT)/
– Trauma type: Sharp, avulsion, crush, ballistic, or animal and
pulmonary embolism (PE), myocardial infarction (MI),
human bites
respiratory failure in patients with previous pulmonary
– Associated injuries, underlying fractures, dislocations, and
history, and death
neurovascular insult
● Functional outcomes of the hand are related to zone of injury
● Time since injury and time period of ischemia of amputated
and postoperative rehabilitation
segment
● Handling of amputated segment
– Should be wrapped in moist gauze, placed in a plastic bag,
and then placed over ice 56.5 Treatment
● Replantation versus revision amputation
56.2.2 Physical Examination – Patient factors (age, occupation, and patient’s goals) and
injury-specific factors should be considered
● Advanced Trauma Life Support (ATLS) protocol: ABCDs ● The thumb is always the top priority in attempted salvage
● Complete upper extremity examination evaluating the ● In multi-digit amputations, when not all digits are
shoulder, elbow, and hand salvageable, heterotopic replantation can be used
– Be aware of distracting injuries ● Operative indications
● Number of digits involved – Thumb amputations at any level, multi-digit amputations,
● Location of the injury amputations in pediatric patients, and amputations through
– Zone 2 flexor tendon injuries adversely affect outcome and the wrist, forearm, or elbow
may impact the operative plan – Relative indications: Single-digit amputation distal to the
● Size of soft tissue deficit, if present flexor digitorum superficialis (FDS) insertion
● Examine the amputated segment for segmental injury ● Relative operative contraindications
● Assess perfusion and sensibility of all digits of the injured hand – Single-digit amputations through flexor tendon zone 2
● Specific deficits: Tendon, nerve, muscles, skin, and bone – Multiple segmental amputations through a single digit
● Wound bed status: Vascularity, exposed structures, nonviable – Avulsion injuries and prolonged ischemia time
skin, and contamination ● Absolute contraindications: Medically unstable patients with
life-threatening trauma
56.2.3 Pertinent Imaging or Diagnostic
Studies 56.5.1 Operative Techniques
● Standard three-view X-rays of the hand and amputated parts ● Irrigation and debridement of nonviable tissue prior to
● A complete set of labs prior to the operation
replantation
– Complete blood count (CBC), basic metabolic panel (BMP), ● Operative sequence
and coagulation factors (prothrombin time [PT]/partial
– Bony fixation (with or without bone shortening); Kirschner
thromboplastin time [PTT]/international normalized ratio
wire, 90–90 wire, or plate for early motion
[INR])
– Tendon repair
– Type and cross for 2 units as pre-hospital blood loss can be
– Veins, arteries (consider vein graft), and lastly nerves
profound
– For proximal amputations, an arterial shunt should be
considered prior to any bony fixation due to the sensitivity
56.3 Consultations ●
of muscle during ischemia
Artery, vein, and nerve repairs should be performed without
● General surgery/Trauma/Orthopedics in the setting of other tension and with the use of loupe magnification and/or
injuries, if indicated operating microscope

236
Traumatic Amputation

56.5.2 Postoperative Treatment ○ Check for external compression from the dressing
○ Arterial occlusion by thrombus warrants operative
● Patients should be admitted to a critical care unit exploration
postoperatively with microsurgical precautions, frequent – Venous insufficiency presents as a swollen and congested
neurovascular checks, elevation, and edema control (blue/purple) digit with brisk cap refill
● There is no good evidence to support systemic ○ Venous congestion can often be alleviated by leeching or
anticoagulation heparin nail-bed rub. Hemoglobin levels should be
● The replanted finger should be immobilized until bony routinely monitored while undergoing leeching.
healing ○ If venous congestion does not respond to leeching,

operative exploration should be considered.


– Infection
56.5.3 Secondary Surgery ○ Due to contaminated wounds, vascular insufficiency, or
● Joint contracture and appropriate release leech exposure
● Tendon adhesion treated with tenolysis ○ Aeromonas infection (from leech therapy) can be avoided

by prophylactic ciprofloxacin or sulfamethoxazole/


trimethoprim
56.6 Ethical Considerations ● Nonunion and malunion
● The prolonged operating room time and subsequent stress on ● Extensor/flexor tendon adhesions and joint stiffness
the patient should not be discounted. ● Flexor tendon ruptures
● In patients with extensive comorbidities or life-threatening ● Neuroma formation
injuries, replantation should be deferred for a revision
amputation.
● Interventions which may require considerable postoperative
56.8 Critical Errors
therapy should be matched to the patient’s level of ● Not understanding indications and contraindications for
motivation and ability to comply with therapy. Poor replantation
cooperation with hand therapy and/or postoperative ● Replantation performed in poor sequence (i.e., bone usually
instruction often preclude a favorable outcome. first)
● Failure to detect early arterial or venous circulatory failure
and threatened flap
56.7 Complications ● Not recognizing early signs of infection
● Replant failure ● Missing the need for temporary vascular shunts in proximal
– Typically, within the first 72 hours postoperation amputations
○ Arterial occlusion: Pale, cool digit with decreased turgor

237
Case 57 Peripheral Nerve Injury
W. Kelsey Snapp and Reena A. Bhatt

Case 57 A 28-year-old female presents to the clinic after sustaining a laceration to her right forearm 2 days ago. She was initially seen in the
emergency department after punching a mirror, where the wound was washed out and skin repaired. She describes numbness in her thumb, and
index and middle fingers as well as weakness in the hand.

239
Hand

57.1 Description 57.3 Patient Counseling


● Laceration repair over the volar aspect of the mid-forearm ● Nerve recovery after closed or open injury takes time. Axonal
with decreased sensation to the median nerve distribution regeneration occurs at a rate of approximately 1 mm/day.
– Concerning for sharp injury to the median nerve – Recovery time can be estimated by measuring the distance
from the site of injury to the distal target.
● Nerve recovery after repair is unpredictable. Successful
57.2 Work-Up functional recovery after peripheral nerve repair has been
reported to be between 50 and 80%, depending on the nerve
57.2.1 History and the location of injury.
● Mechanism of injury: Closed versus open
– Closed injuries: Best treated initially with observation to
distinguish between neuropraxia and true neurotmesis 57.4 Treatment
– Open injuries with nerve deficits: One must assume there is ● Indications for operative exploration of a nerve
a nerve transection, requiring operative exploration and – Paralysis/sensory deficits of a major nerve after an open
repair wound
● Time since injury – Paralysis/sensory deficits affecting a nerve after a nearby
– Early repair of injured nerve is associated with improved iatrogenic surgery
recovery – A nerve lesion associated with arterial injury
● Any additional or associated injuries and limitation in motion – A nerve lesion associated with a fracture or dislocation
● Significant bleeding at the time of injury requiring open reduction and internal fixation (e.g., radial
● Hand dominance, occupation, smoking history, and nerve exploration after open humeral fracture)
comorbidities – Deterioration/failure of recovery of a closed nerve injury
while under observation
57.2.2 Physical Examination – Exploration for other injuries (i.e., tendon lacerations)
● Initial treatment plan should be based on type (open vs.
● Assess for associated lacerations, scars, presence of open closed) and chronicity of nerve injury
injury, and viability of soft tissue coverage – Muscle denervation begins to take place soon after nerve
● Complete sensory examination, including two-point injury and the potential for reinnervation decreases as time
discrimination of ulnar, median, and radial nerves; assess passes (approximately 1% per week)
motor nerve function in a controlled manner – The time needed for nerve regeneration (1 mm/day) should
– Median nerve at this level innervates the flexor pollicis be weighed against the potential for muscle reinnervation
brevis, opponens pollicis, abductor pollicis brevis, and when considering nerve repair
radial two lumbricals (index and long) ● All patients will benefit from hand therapy to maintain
○ Ask patient to oppose thumb or flex at
passive range of motion, manage pain, edema, and other
metacarpophalangeal (MCP) joints of index and long problems associated with the inciting injury
fingers
– Ulnar nerve innervates the hypothenar musculature and
interossei (abduct and adduct fingers, assess for clawing, 57.4.1 Acute Open Injuries with Nerve
Wartenberg’s sign, Froment’s sign) Dysfunction
● Vasomotor paralysis is a subtle yet reliable sign of early nerve
● Require nerve exploration: Must assume the nerve was
injury; the skin in the area of the affected nerve is often
transected
warm and dry
● The prognosis of nerve recovery is directly related to the
● Tinel’s sign: May help in identifying the site and level of
interval between injury and nerve repair; clean injuries to
injury, as well as the presence of neuroma or neuroma-
motor nerves are best treated within a few days
in-continuity
● Local contamination should be considered when considering
● Palpation and Doppler evaluation of ulnar and radial arteries
nerve repair; nerve repair should be delayed until the wound
● Examine tenodesis and cascade
bed is clean
● Independently examine wrist flexors and finger flexors
● Direct repair or repair with an interposition graft is
appropriate
57.2.3 Pertinent Imaging or Diagnostic ● Nerve transfers and babysitter procedures can be considered
Studies in high (proximal) motor nerve injuries

● Standard three-view x-rays should be obtained to assess for


fracture or foreign body 57.4.2 Acute Closed Injuries with Nerve
● For a closed injury, electrodiagnostic studies at 3 months may Dysfunction
be helpful in ascertaining prognosis of nerve recovery
● Determine classification of nerve injury (see ▶ Table 57.1);
– Fibrillation: Indicate some degree of motor injury
obtain a baseline electromyography (EMG) study 3 to 4 weeks
– Motor unit potentials (MUPs): Indicate recovery
after nerve injury

240
Peripheral Nerve Injury

● If there is no clinical improvement within 3 months, repeat 57.4.4 Operative Techniques


EMG
– Fibrillations and MUPs present: Indicate nerve recovery; ● Direct repair of the injured nerve in acute injuries
defer operative intervention – Debridement to healthy nerve tissue in the zone of injury
– Fibrillations present, no MUPs: Requires operative – Nerve approximation without tension
intervention to regain function; options include direct repair – Alignment of individual fascicles
and repair with nerve graft or nerve transfer procedures; – Atraumatic yet secure coaptation of nerve ends
distal tendon transfers may be considered as well ● Nerve conduits can be used to bridge nerve gaps of up to 3 cm
– If recovery slows, even with MUPs noted on EMG, consider ● Allograft and autograft can be used to reconstruct nerve gaps
neurolysis or release of nerve at a known compression point greater than 3 cm.
– Autograft remains the gold standard in reconstructing gaps
in peripheral nerve repair. Donor sites include the medial
57.4.3 Chronic Injuries with Nerve and lateral antebrachial cutaneous nerves and sural nerve
Dysfunction
● If reinnervation of motor nerve target muscles cannot be 57.4.5 Tendon Transfers
achieved within 18 months, alternative methods should be (see ▶ Table 57.2)
considered for restoration of function
● Tendon transfer should be considered to improve function ● Tendon transfers can recreate a specific motion that was lost,
and restore critical movements, such as pinch and grip restore function across a joint, or improve function while
● Nerve transfers can be considered at an earlier point of time awaiting recovery of a nerve injury

Table 57.1 Classification of nerve injury


Seddon classification Sunderland classification Fibrillations Motor unit potentials Prognosis without
intervention

Neurapraxia I – +

Axonotmesis II + + +

III + + + /–

IV + – –

Neurotmesis V + – –

VI + Depends on injury Depends on injury

Table 57.2 Common tendon transfers


Function Donor Recipient

Radial nerve palsy Elbow extension Deltoid, latissimus dorsi, or biceps Triceps

Wrist extension Pronator teres ECRB

Finger extension FDS, FCR, or FCU EDC

Thumb extension PL or FDS EPL

Low median nerve palsy Thumb opposition/abduction FDS (ring) Base proximal phalanx or APB

EIP APB

High median nerve palsy Thumb IP flexion BR FPL

Index and long finger flexion FDP of ring and small fingers FDP of index and middle

Ulnar nerve palsy Thumb adduction FDS or ECRB adductor pollicis

Finger abduction APL, ECRL, or EIP 1st dorsal interosseous

Reverse clawing effect FDS, ECRL Lateral bands of ulnar digits

Abbreviations: APB, abductor pollicis brevis; APL, abductor pollicis longus; BR, brachioradialis; ECRB, extensor carpi radialis brevis; ECRL, extensor Carpi
radialis longus; EDC, extensor digitorum communis; EIP, extensor indicis proprius; EPL, extensor pollicis longus; FCR, flexor carpi radialis; FCU, flexor carpi
ulnaris; FDP, flexor digitorum profundus; FDS, flexor digitorum superficialis; IP, interphalangeal joint; PL, palmaris longus.

241
Hand

Passive range of motion must be established and maintained


across a joint for tendon transfers to be successful


57.6 Complications
● The SEACOAST mnemonic for the principles of tendon repair ● Incomplete or absent nerve recovery, hypersensitivity, and
– Synergistic transfers (S) neuroma or neuroma-in-continuity
– Expendable donor muscle (E) ● Complex regional pain syndrome
– Adequate strength, Adequate amplitude (length) (A) ● Hand stiffness and joint contracture while awaiting
– Contractures need releasing (C) functional recovery
– One tendon, one function (O) ● Loss of excursion over time in tendon transfers
– Adequate excursion (A) ● Infection and wound dehiscence
– Straight line of pull (S)
– Tissue equilibrium (T)
57.7 Critical Errors
57.5 Ethical Considerations ●


Failure to explore an open injury with nerve deficit
Unnecessary exploration of a closed injury with a nerve palsy
● The decision to explore closed injuries should not be taken ● Failure to recognize that the time of injury and the time
lightly. All factors should be carefully weighed when needed for nerve regeneration are critical in predicting
considering exploration. outcome
● Tendon transfers should only be considered in compliant – After more than 1 to 2 years following injury, muscle
patients. Noncompliant patients are at higher risk of wound reinnervation is not possible and attempt at nerve repair is
healing problems with minimal functional benefit. futile
● Reconstructing motor function with a tendon transfer across
a joint with limited passive range of motion

242
Case 58 Dupuytren’s Contracture
Charles C. Jehle and Reena A. Bhatt

Case 58 A 67-year-old Caucasian male presents with painless progressive loss of motion of the left ring finger as well as palmar nodules. He is unable
to straighten his metacarpophalangeal (MCP) and proximal interphalangeal (PIP) joints actively or passively.

243
Hand

58.1 Description 58.4 Treatment


● Flexion contracture of the left ring finger that includes the ● Observation of disease that does not affect function and/or
metacarpophalangeal (MCP) and proximal interphalangeal quality of life
(PIP) joints ● Relative indications for intervention
● Palmar cords and nodules visible over middle, ring, and small – MCP joint flexion contracture of 30 degrees or greater
rays consistent with Dupuytren’s contracture – Any flexion contracture of the PIP joint
– Adduction contracture of the thumb that interferes with
activities of daily living or leisure
58.2 Work-Up ● Treatment of joint contractures
– MCP joint: Collateral ligaments are stretched in flexion.
58.2.1 History Therefore, these joints return to their normal position after
● Symptoms and degree of impairment of activities release of Dupuytren’s cord.
● Age at presentation and duration of the disease – Interphalangeal (IP) joint: Joint contracture at this level
● Family history of Dupuytren’s disease may require additional procedures to release joint level
– Dupuytren’s diathesis: Young age at onset, positive family scarring from long-standing disease.
history, bilateral disease, and ectopic lesions – PIP joint: Fixed joint contractures may occur in patients
– More prevalent in northern Europeans and Japanese with severe, long-standing disease. Capsuloligamentous
● Sex: More common in males release may be required, although its efficacy has not been
● May have history of plantar fascia (ledderhose disease) or proven.
Penile (peyronie disease) involvement
● Risk factors: Alcohol, epilepsy medications, diabetes mellitus,
58.4.1 Surgical Options
and smoking
● Previous surgical treatment for this condition ● Aim of surgery is release: Dupuytren’s disease cannot be
● Trauma to the palm can result in traumatic palmar fasciitis totally excised
● Needle fasciotomy/aponeurotomy
– The cord is percutaneously sectioned with 23- to 25-gauge
58.2.2 Physical Examination needle
● Observe presence and location of pits, modules, and cords – Closed manipulation can further break up the cord
– Palpate for symptomatic nodules and cords ● Enzymatic fasciotomy
● Note digits and joints involved, and measure the degree of – Collagenase from Clostridium histolyticum (Xiaflex; Endo
contracture Pharmaceuticals, Malvern, PA) is injected at the point of
– Tabletop test: Patient is unable to lay the palm flat on a rigid maximum bowstringing of the palpable cord
surface – After 24 hours, the patient returns for closed digit
– Adduction contracture of the thumb manipulation to rupture the cord
● Assess integrity of extensor mechanism ● Segmental aponeurectomy
– Flex wrist and MCP joint to create tenodesis effect – Segments of the diseased cord are removed through a series
– An extensor lag indicates that the central slip is attenuated of small incisions
and postoperative extension splinting may be required; the ● Local fasciectomy
patient should be cautioned that full extension of the – A portion of the diseased cord is removed
affected finger will likely not be regained ● Regional fasciectomy: Must be familiar with this technique
● Sensory examination of all digits (see ▶ Fig. 58.1)
● Sites of ectopic disease – Diseased cords and fascia are excised
– Garrod’s nodes (nodules on dorsum of PIP joint) and knuckle – Options exist for skin incision: Zig-zag incision with linear
pads (fibrosing lesions on dorsum of PIP joint) extension to palmar flexion crease; Littler-Brunner incision
– Ledderhose disease (plantar fibromatosis) which can be closed with a “V-Y” plasty or left open in
– Peyronie’s disease (penile fibromatosis) honeycomb fashion; longitudinal incision closed by
Z-plasties; or transverse incisions of McCash’s open palm
technique
58.3 Patient Counseling ● Dermofasciectomy
– Diseased cords and fascia are excised along with overlying
● Dupuytren’s contracture is a benign fibromatosis of the
skin
palmar and digital fascia of the hand.
– Wounds are closed with full-thickness skin graft or allowed
● Surgical treatment usually improves function but cannot cure
to granulate in
this disease. While recurrence is common, the need for repeat
– Performed in recurrent disease, for replacing flaps with
surgery in patients over 50 years old is uncommon. Younger
uncertain viability and in patients with Dupuytren’s
patients are more likely to need reoperation later in life.
diathesis

244
Dupuytren's Contracture

● The patient must have an obtainable goal in mind. Full


correction of the contracture is unlikely and complication
rates are higher because surgery is more difficult for
recurrent disease.

58.6 Complications
● Wound healing problems
● Hematoma
● Vascular and nerve injury
– Laceration can occur at the time of release
– Straightening a severely contracted joint may cause traction
injury
● Flare reaction: Stiffness, pain, and edema
● Complex regional pain syndrome (formerly reflex
sympathetic dystrophy): Stiffness, pain, edema, and
vasomotor changes
– Management includes pain control and/or stellate
sympathetic ganglion block
● Tendon rupture, particularly with enzymatic fasciotomy

58.7 Critical Errors


● Inattention to critical structures during intervention
– Neurovascular structures are often displaced proximally,
superficially, and toward the midline of the digit by the
spiral cord and should not be assumed to be in their normal
position
– If transected, the digital neurovascular structures should be
repaired
– Critical ischemia may occur if both digital arteries are
Fig. 58.1 Common incisions used for exposure. Different pattern types damaged or if severe PIP joint contracture is forced to stay
are demonstrated on each finger. in extension (i.e., pinned)
● Failure to inform the patient about risks of intervention and
chance of recurrence, need for vigorous therapy
58.5 Ethical Considerations postoperatively, possible need for prolonged wound care and
dressing changes, and risk for neurovascular and/or tendon
● Surgery for recurrent disease requires special consideration
injuries
and may be best performed by a surgeon with subspecialty
certificate in surgery of the hand.

245
Case 59 Syndactyly
Angie M. Paik and Reena A. Bhatt

Case 59 (a, b) A 1-year-old male presents to the clinic with fusion of the left ring and small fingers.

247
Hand

Indications for surgery


59.1 Description ●

– Surgery is indicated for most cases


● Simple, complete syndactyly involving the ring and small – Mild, incomplete syndactyly without impaired function can
fingers of the left hand be considered cosmetic
– Contraindications
○ Severe medical comorbidities
59.2 Work-Up ○ Complex/complicated cases where separation risks

worsened function/necrosis
59.2.1 History ● Goals of surgery
● Gestational history and issues during pregnancy – Reconstruct the web space to improve function and
● Medical comorbidities and any previous medical work-up appearance
● Family history of syndactyly (autosomal dominant with – Separate and resurface the fused digits
variable expressivity and incomplete penetrance) or
associated syndromes (e.g., Poland or Apert syndromes)
● Patient’s current handedness and hand function
59.5 Treatment
● Timing of surgery: Variable among surgeons but generally
12 to 18 months of age
59.2.2 Physical Examination
– Border digit syndactyly should be released earlier (about
● Perform total body examination 6 months of age) to prevent progressive deviation
– Evaluate for any craniofacial or chest wall anomalies of fingers
– Examine feet and contralateral upper extremity to rule out – For multiple web syndactylies, release is staged to only one
additional syndactylies side of an affected digit at a time to prevent vascular
● Complete upper extremity examination compromise of digits
– Assess for extent and location of webbing as well as the – Aim to have all releases completed by school age
number of digits involved ● Surgical markings: One representative technique
– Look for other concurrent deformities (i.e., polydactyly, (▶ Fig. 59.1)
clinodactyly, brachydactyly, and symphalangism) – Design a broad, proximally based dorsal skin flap to line the
– Examine contralateral hand for comparison new web space
– In children, assess for hand function by evaluating the ○ The flap should extend at least two-thirds of the way
patient during play from the metacarpal heads to the proximal
● Classification interphalangeal (PIP) crease to ensure adequate length to
– Simple/Complex avoid tension and possible contracture
○ Simple syndactyly: No bony fusion ○ Goal is to recreate the natural dorsal to volar slope of the
○ Complex syndactyly: Bony fusion webspace of 40 to 45 degrees
– Complete/Incomplete – Design interdigitating, opposing, and zig-zag pattern flaps
○ Complete syndactyly: Fused web space extends to fingertips for digital separation
○ Incomplete syndactyly: Web space involvement but – Use a template of the remnant skin loss (frequently the
fingertip and nail are spared dorsal region at the base of the proximal phalanx)
– Complicated: Associated with a syndrome to design full-thickness skin grafts from the groin,
antecubital fossa, hypothenar eminence, abdomen, or
volar wrist
59.2.3 Pertinent Imaging or Diagnostic – Some incomplete syndactyly cases may be amenable to
Studies other techniques to deepen the web space including Z-
● Hand three-view X-rays: Image bilateral hands to assess for plasty, four-flapZ-plasty, or double-opposing Z-plasty
underlying bony fusion and additional anomalies (see ▶ Fig. 59.2)
● Postoperative management
– Dissolvable sutures used for closure
59.3 Consultations – Long-arm cast immobilization in immediate postoperative
period for 3 weeks
● Occupational therapy
● If indicated: Genetics and cardiology

59.6 Ethical Considerations


59.4 Patient Counseling ● The patient with simple, incomplete syndactyly but has good
● Implications of syndactyly function of the hand
– Inhibited individual digit function – Thorough discussion with the parents regarding the
– Reduced span of the hand due to limitations of abduction indications for surgical intervention and the potential risks
– Size discrepancy between fused digits can lead to deviation and complications of surgery
of the involved digits with growth, as well as flexion – Decision to proceed with surgical release should be made
contracture (especially in the setting of complete syndactyly) jointly with the parents after this discussion

248
Syndactyly

Fig. 59.1 (a, b) Representative surgical


markings for syndactyly release.

Fig. 59.2 Double-opposing Z-plasty for web


space deepening.

● The patient has a complication following surgery – Reassure that these issues are within the realm of expected
(e.g., scar contracture, partial graft loss, hypertrophic complications of syndactyly release
scarring, or web creep) and the parents are distraught – Discuss next steps to take to address the complication
and accusatory (conservative measures versus surgery)

249
Hand

Joint instability or skeletal deformity in complex syndactyly


59.7 Complications ●

cases
● Digital ischemia – Potential arthrodesis once skeletal maturity is reached
– Ensure dressing is not compressive ● Hypertrophic scarring and keloid formation
– Careful preservation of neurovascular bundles
intraoperatively and staged reconstruction for adjacent
webspace releases 59.8 Critical Errors
● Scar contracture: Scar release/excision and re-grafting, ● Failure to evaluate and work-up for additional congenital
splinting, and therapy anomalies
● Skin graft loss: Debridement and re-graft for significant loss ● Performing syndactyly release at an inappropriate age
● Web space abnormalities (“web creep”) ● Failure to stage surgery in cases of multiple digit syndactyly
– Design flaps to minimize scars within webspace ● Failure to revise skin graft loss, resulting in severe scar
– Make flaps deep and long enough to mitigate risks for web contracture
creep ● Using split-thickness skin grafts rather than full-thickness
● Nail deformity in complete syndactyly skin grafts for skin coverage

250
Case 60 Metacarpal and Phalangeal Fractures
W. Kelsey Snapp and Reena A. Bhatt

Case 60 (a–c) A 23-year-old male presents to the emergency department after striking a wall. X-rays of the hand are shown. The patient complains
of pain over the ulnar side of the hand and deformity of the hand.

251
Hand

Closed reduction should be considered for all displaced


60.1 Description ●

metacarpal and phalangeal fractures


● Transverse fractures of the ring and small finger metacarpal ● Open fractures should be thoroughly irrigated upon initial
shafts with angulation, rotation, and shortening of the presentation and treated with antibiotics. Depending on the
affected digits degree of contamination, soft tissue may be reapproximated
● Dorsal prominence of the hand secondary to dorsally after an adequate washout
angulated fractures ● After reduction, the injury should be immobilized one joint
above and below the fracture
– For immobilization of metacarpal fractures, the hand
60.2 Work-Up should be placed in the intrinsic plus position with the wrist
extended at 30 degrees, the metacarpophalangeal joint
60.2.1 History flexed 70 to 90 degrees, and the interphalangeal joints
● Mechanism of injury (e.g., assault, fall, motor vehicle collision, maintained in extension
“fight bite,” or gunshot) – Serial X-rays should be performed to ensure that reduction
● Time since injury is maintained for approximately 4 weeks
● Any additional or associated injuries – Casting should be considered in pediatric patients,
● Prior history of similar injury and hand surgery although it is often delayed until the initial swelling has
● Hand dominance stabilized
● Occupation
● Smoking history
● Comorbidities
60.4.1 Operative Indications
● Decision for operative intervention is based on the stability of
the fracture after reduction and the degree of deformity in
60.2.2 Physical Examination the affected digits
● Evaluate for open wound and additional injuries ● Transverse shaft and oblique fractures: Inherently unstable
– If open, determine degree of contamination from wound ● Fractures in consecutive metacarpals are unstable
● Examine resting cascade of the hand while in slight flexion ● Scissoring and rotational deformity are poorly tolerated
– The distal phalanges should all point toward the scaphoid ● The degree of angulation tolerated in metacarpal neck
tubercle fractures depends on the digit
● Assess for angulation, scissoring, or malrotation of the digits – In the small finger, 40 to 50 degrees of angulation is
● Evaluate for shortening of the digits relative to the tolerated
contralateral hand – In the ring finger, 30 to 40 degrees
● Assess perfusion and sensibility of the affected digits – In the index and long fingers, ≤ 15 degrees
– Viability and adequacy of soft tissue coverage ● Intra-articular involvement with > 30% of the articular surface
● Displacement more than 1 to 2 mm

60.2.3 Pertinent Imaging or Diagnostic


Studies 60.4.2 Operative Techniques
● Standard three-view X-rays (anteroposterior, lateral, and
● Closed reduction with percutaneous pinning
oblique) of the hand should be obtained to assess the fracture – Crossed K-wires across fracture site hold reduction and
or presence of foreign bodies prevent rotation
● Consider computed tomography (CT) scan in cases of severe – Advantages: Minimal soft tissue dissection and adhesions
comminution/suspected articular disruption compared to open reduction
– Disadvantages: Nonrigid fixation often requires more than
4 weeks of immobilization after surgery; pins may migrate
60.3 Patient Counseling or become dislodged; reduction access is limited
● Open reduction with internal fixation (ORIF)
● Patient should be counseled on critical importance of – Required for fractures that are not reducible by other means
compliance with splinting/casting, surgery if indicated, and – Dorsal longitudinal incision is made over the metacarpals
hand therapy for fracture exposure (or between metacarpals for adjacent
● Smoking cessation should be included in preoperative fractures)
counseling ○ Retract and protect extensor tendons and sensory nerves
● Potential complications should be reviewed including but not in the process
limited to stiffness, infection, pain, nonunion, and malunion – Internal fixation can be performed in a variety of ways:
K-wires, low profile plates, lag screws (spiral fractures),
cannulated screws, and intramedullary fixation
60.4 Treatment – Advantages: Ensures near-anatomic reduction and allows
● Nonoperative versus operative management should be for rigid fixation, and potential for immediate range of
reviewed motion after surgery

252
Metacarpal and Phalangeal Fractures

– Disadvantages: Increased risk of adhesions and postoperative


stiffness due to scarring, and risk of nonunion secondary to
60.6 Complications
overaggressive stripping of the periosteum ● Malunion: May result in pseudoclawing due to excessive apex
dorsal angulation or scissoring due to malrotation
● Nonunion: Increased risk of nonunion with aggressive
60.4.3 Postoperative Treatment periosteal stripping in ORIF, smoking, and noncompliance
● After nonoperative management, fractures should remain ● Extensor tendon adhesions from open approach and/or
immobilized for a minimum of 3 to 4 weeks tendon attrition due to prominent plate
● K-wires are usually removed at 4 to 6 weeks ● Infection (e.g., osteomyelitis and pin-site infection)
– Passive and active range of motion can be initiated once
K-wires are removed and patient is nontender over the
fracture site 60.7 Critical Errors
● Range of motion exercises can be initiated immediately after ● Failing to recognize an open fracture
open reduction with rigid fixation
– “Fight-bites” should always be identified upon initial
evaluation and the patient should be treated with washout
60.5 Ethical Considerations ●
and antibiotics
Not correcting rotation/scissoring with operative reduction
● A patient’s ability to comply with the postoperative regimen ● Failing to recognize loss of reduction in fractures that are
is a major factor for favorable outcome. Significant treated nonoperatively, resulting in malunion
complications can occur in the injured digit and adjacent
digits if patient is noncompliant

253
Case 61 Carpal Tunnel Syndrome
W. Kelsey Snapp and Reena A. Bhatt

Case 61 A 73-year-old right-hand-dominant male presents with a 2-year history of increasing numbness of the thumb, index, and middle fingers of
the right hand. He also reports worsening weakness and clumsiness.

255
Hand

– Carpal tunnel syndrome (CTS) is a clinical diagnosis: EDSs


61.1 Description are useful to verify the site of compression and predict the
● Paresthesias reported in the median nerve distribution of the likelihood of recovery
right hand – NCSs demonstrate increased latencies in early CTS,
● Significant thenar atrophy of the hand relative to the specifically in sensory testing
contralateral side – EMG may demonstrate fibrillation potentials and
fasciculations, if severe
● Ultrasound: Can identify increased cross-sectional area of the
61.2 Work-Up median nerve in affected patients
● Magnetic resonance imaging (MRI): Controversial to use in
61.2.1 History patients with standard CTS; useful if mass is suspected
● Duration and evolution of symptoms ● X-ray: Pertinent if wrist pathology is a concern (e.g., trauma)
● Prior treatments and their effectiveness
● Pertinent symptoms consistent with nerve compression
– Nocturnal pain, numbness, and tingling in the thumb and
61.3 Patient Counseling
one or more radial fingers ● Median nerve recovery from CTS takes time. Depending on
– Daytime paresthesias elicited with activities involving the degree and duration of the patient’s CTS, reversible
prolonged wrist flexion and/or extension symptoms and signs may take from months to years to
– Need for shaking and wringing the hand to alleviate resolve.
symptoms – There is no guarantee of recovery as irreversible injury may
– Gritty sensation or numbness in fingers, grip and pinch have already occurred, especially in the cases of severe
weakness, and diminished finger dexterity with a history of symptoms and evidence of marked thenar atrophy. These
dropping objects patients may require tendon transfer at the time of carpal
– Cold intolerance, dryness, and unusual textures in the radial tunnel release for opposition.
digits ● Patients with a mixed clinical presentation: Difficult to
● History of neck injuries or pain should be elicited (evaluate determine whether improvement will occur after release
for more proximal issues) (i.e., those patients with findings of cervical radiculopathy
and CTS on EMG/NCSs)
61.2.2 Physical Examination
● Tinel sign: Examiner percusses over the carpal tunnel. 61.4 Treatment
Tingling over distribution of the median nerve indicates
● Nonsurgical: Nonsteroidal anti-inflammatory drugs (NSAIDs),
positive result.
nighttime splinting with wrist extension (cock-up splint),
● Phalen maneuver: Patient maintains full flexion of wrists for
activity modifications, and steroid injections
30 to 60 seconds. Result is positive if tingling, burning, or
– Steroid injections can be diagnostic and therapeutic. Failure
numbness of the thumb, index, long, or ring finger is elicited.
of symptoms to improve after injection can be a poor
● Durkan’s (carpal compression) test: Examiner applies pressure
prognostic factor for surgical treatment.
with thumbs over the carpal tunnel for 30 seconds. Onset of
● Surgical: Recommended after nonsurgical treatments have
pain/paresthesias in the median nerve distribution within
been tried and failed or with significant progression of signs
30 seconds is a positive result.
or symptoms.
● Diminished (increased) two-point discrimination compared
– Open versus endoscopic carpal tunnel release remains
to the ulnar nerve and contralateral hand. The normal
controversial. Results are equivocal at 1 year. However,
distance is 4 to 6 mm. Semmes-Weinstein monofilament
some studies have demonstrated a slightly faster return to
testing or vibrometry.
work and decreased perioperative pain for endoscopic
● Weakness and/or atrophy of the thenar musculature and
release.
radial lumbrical; atrophy and burning are later signs
– For the board examination, an open approach is
● Examination of soft tissues for skin and muscle atrophy,
encouraged.
manual muscle strength testing, grip and pinch testing,
percussion of all major peripheral nerves, assessment of
deep tendon reflexes, and assessment of blood flow to 61.4.1 Surgical Technique
each hand
● For open carpal tunnel release, a longitudinal incision is made
● Cervical spine and entire upper extremity examination to
just proximal to the intersection of Kaplan’s cardinal line and
rule out cervical radiculopathy or thoracic outlet syndrome
the radial border of the fourth ray (see ▶ Fig. 61.1).
– Kaplan’s cardinal line: Runs from the ulnar base of the
61.2.3 Pertinent Imaging or Diagnostic abducted thumb toward the hook of the hamate
– This corresponds to an area just proximal to the superficial
Studies arch
● Electrodiagnostic studies (EDSs): Nerve conduction studies – Alternatively, a curvilinear incision may be made within the
(NCSs) and electromyography (EMG) confluence of the thenar and hypothenar eminences

256
Carpal Tunnel Syndrome

● The superficial surface of the TCL is carefully incised under


direct visualization. The surgeon should confirm that the
proximal and distal edges of the TCL are fully released into
the forearm.
● After complete decompression is confirmed, the wound is
closed.

61.5 Ethical Considerations


● The decision to proceed with surgery should not be taken
lightly. The surgeon should be confident in the diagnosis and
nonoperative management should be tried prior to surgery.
● The patient should fully understand the risks of surgery as
well as the expected recovery time after surgery.

61.6 Complications
● Iatrogenic injury during surgery
– Injury to the median nerve
– Transection of the recurrent motor branch and/or palmar
cutaneous branches of the median nerve
○ The anatomy of the recurrent motor branch relative to the

carpal tunnel is often unpredictable


– Injury to the superficial palmar arch
– Transection of the ulnar nerve
– Laceration of flexor tendon(s)
● Hypertrophic scar and scar sensitivity
● Incomplete release of the TCL
● Infection
● Hematoma

Fig. 61.1 Standard markings for open carpal tunnel release.


61.7 Critical Errors
● Failure to recognize pathology that mimics CTS, such as
– The incision should be long enough to visualize the median nerve compression proximally, thoracic outlet
proximal and distal edges of the transverse carpal ligament syndrome, or cervical radiculopathy
(TCL) (approximately 3 cm) ● Incomplete release of the TCL
● Sharp dissection is used to cut through the subcutaneous fat ● Failure to recognize and address iatrogenic injury to median
and palmar fascia until the transverse fibers of the TCL are nerve and its branches or flexors during surgery
visualized. Palmaris brevis fibers are often visualized just
above the TCL and cauterized or brushed aside.

257
Case 62 Adult Brachial Plexus Injury
Timothy Fei and Kyle Chepla

Case 62 A 32-year-old right-hand-dominant male was involved in a high-speed motorcycle crash 3 months ago. He subsequently developed left-sided
unilateral loss of sensation, shoulder weakness, and inability to flex his elbow.

259
Hand

Joint mobility: Passive range of motion at shoulder, elbow,


62.1 Description ●

wrist, hand, and fingers


● Unilateral loss of upper extremity sensation with biceps and ● Vascular status: Pulses and capillary refill to evaluate for
shoulder weakness after high-energy injury and concern for associated arterial injury (10–25% incidence)
upper brachial plexus injury (C5–C6) ● Evaluate for Horner’s syndrome (ptosis, anhidrosis, and
● Clinical photograph demonstrates left-sided wasting of the miosis) which indicates a preganglionic injury involving the
deltoid and biceps with preserved triceps and wrist sympathetic chain
extensors; patient is unable to abduct and externally rotate ● Assess for common associated bony fractures: Cervical
his shoulder or flex his elbow fracture, clavicle fracture, rib fracture, scapula fracture, and
shoulder dislocation

62.2 Work-Up
62.2.3 Injury Type
62.2.1 History ● Preganglionic: Root avulsion
● Mechanism of injury: Penetrating (e.g., sharp laceration or ● Postganglionic: Stretch versus rupture
gunshot) versus traction; high-energy versus low-energy
injury
62.2.4 Injury Patterns
● Time since the injury occurred
● Age and hand dominance ● Pan-plexus palsy (C5–T1): Flail extremity—78 to 80%
● Occupation and hobbies
● Previous upper extremity injury or surgery
Table 62.1 British Medical Research Council Motor Grading Scale
● Any previous work-up of the injury
Grade Exam findings

62.2.2 Physical Examination 0 No muscle contraction

● Critical to take a full inventory of sensory and motor deficits 1 Visible muscle contraction, but no movement
– Localizes level of injury and helps in guiding treatment plan 2 Visible muscle contraction, active movement in plane, with
● Evaluate and grade motor function: British Medical Research gravity eliminated
Council (MRC) scale for muscle strength (see ▶ Table 62.1) of 3 Active movement against gravity, but not against resistance
all muscles innervated by the brachial plexus (see ▶ Fig. 62.1)
● Evaluate sensory function: Dermatome distribution and two- 4 Active movement against strong resistance, but not full strength
point discrimination (2PD) in fingers 5 Active movement, with full strength

Fig. 62.1 Brachial plexus anatomy. For reference (do not memorize).

260
Adult Brachial Plexus Injury

● Upper plexus (C5–C6/7): Loss of shoulder abduction (deltoid ● Recovery after surgery may take several years and may be
and supraspinatus) and external rotation (infraspinatus), and incomplete
elbow flexion weakness (biceps and brachialis)—20 to 25% ● Pain after root avulsion is difficult to treat and will not be
● Lower plexus (C8–T1): Intrinsic weakness of the hand, claw altered by surgery
hand, and Horner’s syndrome— < 5%

62.2.5 Imaging and Diagnostic Studies 62.4 Treatment


● X-ray: Cervical spine, shoulder, clavicle, and humerus to 62.4.1 Critical Principles
evaluate for associated fracture
● Computed tomography (CT) myelogram: Gold standard for ● Acute intervention: Sharp penetrating trauma should be
detecting pseudomeningoceles from root avulsion injuries. explored and repaired immediately with direct repair or
Should be done 3 to 4 weeks after injury to allow clot to grafting if necessary.
resolve and pseudomeningocele to form. ● Early intervention: Prevents irreversible changes and
● Magnetic resonance imaging (MRI): Difficult to interpret loss of motor end plates. Nerve transfers or nerve
secondary to edema but allows visualization of entire plexus. grafting should ideally be performed 6 to 9 months
In injury, T2 imaging demonstrates pseudomeningocele and after injury.
empty root sleeves on T1. ● Preganglionic injury (root avulsion) cannot be repaired by
● Electromyography (EMG)/Nerve conduction studies (NCS) nerve grafting.
– Timing: 3 to 4 weeks after injury ● Low-energy injury (e.g., low caliber gunshot wounds)
– Loss of sensation with intact sensory nerve action potentials may cause neuropraxia and may recover spontaneously.
(SNAPs) is pathognomonic for root avulsion (preganglionic It should be followed with serial examination and
injury) because dorsal root body in ganglia is preserved EMG.
– Signs of recovery: Presence of active motor units, nascent
potentials, and decrease in fibrillation potentials
62.4.2 Primary Repair with/without
62.3 Patient Counseling Graft (see ▶ Fig. 62.2)
● Patients should be educated on severity of injury, prognosis, ● Cable grafting with autograft (sural nerve) or allograft, if
and possible need for multiple surgical procedures direct repair is not possible

Fig. 62.2 Postganglionic C7 stretch injury.


Injured segment resected and reconstructed with
sural nerve cable graft.

261
Hand

62.4.3 Alternative Reconstructive ○ Shoulder abduction: Triceps motor branch to axillary nerve
○ Elbow flexion: ICN to musculocutaneous nerve; Oberlin:
Options ulnar nerve flexor carpi ulnaris (FCU) fascicles to biceps
● Performed when there is no evidence of ongoing spontaneous motor branches; and Double Oberlin: ulnar FCU to MSC
neurologic recovery biceps and median flexor carpi radialis (FCR) fascicles to
● Nerve transfer brachialis motor branches
○ Elbow extension: Axillary nerve branch to triceps branch
– Performed 6 to 9 months after injury; nerve transfer after
○ Contralateral C7 transfer: Only performed for flail arm
12 months likely unsuccessful secondary to irreversible loss
of recipient motor endplates (see ▶ Fig. 62.3) after pan-plexus injury; high donor site morbidity with
– Common donors mixed outcomes; not commonly performed in the US
○ Intraplexal: Uninjured ipsilateral peripheral nerves ● Free muscle transfer: Can be performed at any time after
○ Extraplexal: Spinal accessory nerve (SAN), intercostal injury (with extraplexal nerve donor) with ICN as donor if
nerve (ICN), and contralateral C7 joints are supple
– Common transfers – Most commonly used to restore elbow with or without
○ Shoulder stabilization: SAN to suprascapular nerve finger flexion

Fig. 62.3 Transfer of the ulnar nerve flexor carpi


ulnaris (FCU) motor fascicles to the biceps motor
branches to restore elbow flexion.

262
Adult Brachial Plexus Injury

Tendon transfers: Performed at any time after injury; follow


principles of tendon transfer


62.7 Critical Errors
● Amputation and glenohumeral arthrodesis: Salvage option for ● Making an incorrect diagnosis
flail extremity ● Failure to perform and document complete motor and
sensory examination
● Neglecting to rule out associated injury (e.g., spinal cord or
62.5 Ethical Considerations intracranial injury)
● Patient compliance, experienced surgeon, and therapy team
● Not understanding contraindications for grafting (e.g.,
are critical to optimize success. These are complex injuries preganglionic injury)
and proper evaluation and planning are key. Patients need to
● Failure to understand timing of nerve transfer or grafting for
be aware of prognosis, potential risks of surgery, and reconstruction
expected surgical outcomes.

62.6 Complications
● Donor site morbidity from nerve transfer, tendon transfers,
and graft harvest
● Failure of free flap or nerve transfer reconstruction

263
63 Ethics 267
Section X
Ethics

X
Case 63 Ethics
Karel-Bart Celie, Sabrina Khalil, and Michael A. Harrington

– The AMA Code of Ethics (Opinion 9.031) cites an ethical


63.1 Introduction obligation to report impaired, incompetent, or unethical
Medical ethics lies at the heart of medicine. The mission state- colleagues.2 This obligation stems from the responsibility to
ment of the American Board of Plastic Surgery (ABPS) reads, protect patients from harm. Depending on the situation, the
“The mission of the American Board of Plastic Surgery, Inc. is to reporting is usually escalated in a stepwise fashion.
promote safe, ethical, efficacious plastic surgery to the public by “Impaired” is defined as conditions that interfere with the
maintaining high standards for the education, examination, cer- ability to engage safely in professional activities.2 With
tification and continuous certification of plastic surgeons as regard to incompetence, the Code recommends that
specialists and subspecialists.”1 This chapter will aim to review incompetence that poses an immediate threat to the safety
important principles in medical ethics and apply them to clini- of patients should be reported directly to the state licensing
cal cases pertaining to plastic and reconstructive surgery. board.
The American Board of Plastic Surgery strives to certify com- – Likewise, the code of ethics of the American Society of
petent, ethical, and safe surgeons. Some may even argue that Plastic Surgeons (ASPS) states that members “should
ethical standards are one of the most important testing princi- expose, without hesitation, illegal or unethical conduct of
ples of the Oral Board Examination. It is a guarantee that there fellow Members of the profession.”3
will be ethical scenarios in the Oral Board Examination, so you – In the case above, if the new surgeon does not believe that
will need to be prepared to work through these. Just like the the patient experienced impaired, incompetent, or
clinical scenarios of the examination, we recommend working unethical treatment, then the best course of action is to
through the problems in a stepwise fashion (see ▶ Fig. 63.1) focus on developing a therapeutic relationship with the
patient and to avoid engaging in speculation regarding
prior care.
63.2 Application of Medical Ethics ○ Tell the patient that you will not focus on what happened

in the past, but will view the problem with a fresh set of
to Plastic and Reconstructive eyes and will focus on the resolution of the problem.
Surgery ○ Even with blatant signs of malpractice, discussions about

other physicians’ practices should be avoided until risk


● Case 1: A patient sees a new surgeon for revision of a
management and/or legal counsel is involved.
previous rhinoplasty that had some postoperative ○ It is a good idea to consider multiple preoperative
complications and a subjective poor result. The patient
consultations with this patient to ensure that he or she is
expresses the desire to sue his or her previous surgeon and
fully aware of what procedure will be performed and that
would like you to revise the previous surgeon’s “mess ups.”
the expectations are reasonable. The patient also needs to

What are the medical facts?


Identify the issue What are the ethical
considerations?

What is the effect on a patient's health?


Consider What laws come in to play?
What are the moral obligations?

How does this help frame the


Which ethical problem?
principles are at Have there been similar cases in
stake? the past?

Evaluate the resolution


Formulate a plan and predict future
issues.

Fig. 63.1 Suggested workflow for ethical assessment

267
Ethics

understand the possible risks and benefits of the ● Case 3: A surgical procedure is declined by insurance. The
proposed procedure. patient requests that the surgeon to adjust his or her
○ The consulting plastic surgeon is not obligated to operate on assessment and document different findings in order to get
this patient. If you decide that you do not want to perform insurance approval.
the procedure, give the patient the names of other plastic – Accurate documentation is not only an important
surgeons who may be better suited to treat the issue. component of good patient care, but an ethical obligation
– A word on malpractice law: There are criminal law and civil on the part of the surgeon. Adjusting findings in order to
law.4 To be found guilty in a criminal case is to be found obtain insurance coverage could be considered insurance
guilty of crimes against society and is associated with fraud and is strongly discouraged.
prison and monetary damages. To be found guilty in a civil – The physician should politely decline to falsely adjust
case is to be found liable to the individual plaintiff and is findings and instead focus on developing an alternative
associated with monetary damages. Most medical strategy with the patient.
malpractice cases are civil lawsuits.4 However, there is a ○ One way to approach the conversation is to say that

possibility of “criminal negligence” if the opposition can falsification of the medical records could result in the loss
prove that the physician acted so recklessly that there was of your medical license. When patients hear this, they
“implied intent” of harm. Thankfully, allegations of criminal tend to understand your hesitation and comfort level with
negligence against physicians are uncommon. The most what they are asking you to do for them.
expensive medical malpractice allegations include4: ○ Focusing on alternative methods to approach the problem

○ Improper performance helps build trust with the patient that you are a sound
○ Errors in diagnosis and ethical surgeon.
○ Failure to supervise or monitor—especially important to ● Case 4: A patient’s insurance has approved a panniculectomy.
note for surgeons in teaching hospitals The patient would like an abdominoplasty but cannot afford
○ Medication errors to pay for it out-of-pocket. The surgeon considers doing the
● Case 2: During preoperative consultation for a breast abdominoplasty work for free. What are the ethical
reconstruction, it appears to the surgeon that a patient has considerations regarding this decision?
unreasonable expectations or an incomplete understanding – There are several considerations here:
of the risks and benefits of surgery. ○ Physicians have a responsibility to help ensure that the

– The ethical consideration at stake in this case is the ability needs of the poor are met (see Opinion 9.065 of the Code
of the patient to participate in informed consent, which of Ethics).7
falls under the principle of Respect for Personhood. ○ An abdominoplasty is not medically necessary and thus

– If the patient is incorrectly or incompletely informed about conveys no substantial moral obligation on the physician.
the procedure, the surgeon has an ethical obligation to ○ The physician is able to provide services free of charge but

inform the patient adequately prior to obtaining consent. is under no obligation to do so.
– If the patient is incapable of giving consent (i.e., lacking ○ If the physician chooses to perform the abdominoplasty

capacity), then attempts must be made to obtain proxy portion of the procedure for no charge, the patient has to
consent in order to respect the patient. Traditionally, the understand that he or she will still be required to pay for
capacity for giving consent is considered include the the extra anesthesia required for prolonging the case.
following four elements5: ○ Another consideration is for potential complications that

○ The patient is able to communicate a choice. might occur from the abdominoplasty. Insurance will
○ The patient understands the relevant information. most likely not cover the fees required to fix the
○ The patient appreciates the medical condition and/or complications.
consequences of treatment. ● Case 5: A healthy patient comes in requesting to have his
○ The patient can manipulate information rationally. facial appearance changed to better resemble the features of
– A physician’s duty to respect personhood goes beyond the an animal. The surgeon’s own assessment and that of a
delivery of information.6 Physicians must attend to how the psychiatrist deems him to have capacity. What are the
information is received and understood. A physician should surgeon’s ethical obligations in this situation?
aim to understand the rationality behind a patient’s beliefs – This hypothetical case raises the topic of the limits of
and help a patient to deliberate more effectively. patient’s self-determination. As described above, the clinical
– This scenario is very common in a plastic surgeon’s practice. relationship should not be understood as patient’s
There are many strategies that you can employ in your “autonomy” versus physician’s “paternalism.” Rather, the
discussions with the patient, but the end goal is that you patient-physician relationship is a joint venture with a
and the patient are “on the same page” and the common goal: the patient’s well-being.
expectations are completely understood. – In a case like this, the physician should attempt to present
○ The patient should return multiple times for a specialized information in a simplified manner and engage
preoperative consultation to have ongoing discussions the patient in a dialogue to ensure patient’s understanding
about the proposed procedure and the potential of the immediate and long-term sequalae of the request.6
complications and/or deformities that could occur. The surgeon should investigate the rationale behind the
○ The patient should be shown photos of the proposed request but is under no ethical obligation to perform a
procedure so that she is fully aware of the potential scars surgery to accommodate a request that he or she deems
and cosmetic outcome. neither necessary nor reasonable.

268
Ethics

– The principle of beneficence presupposes that persons – With regard to the case at hand, the patient’s wishes are
other than the patient himself/herself is able apprehend known. Amputating one or more limbs to sustain his or her
what ought to be good for the patient. Hence, due diligence life would amount to extraordinary means. Withholding
and discernment are an ethical obligation on the part of the this would not—legally or ethically—amount to wrongdoing
surgeon when entertaining requests by patients. A request by the physicians involved in this care. However, there are
should not be fulfilled simply on the basis that it is what the many social complexities. Physicians should always seek
patient wants. consensus with and among family members. Involving the
– Ethicists have attempted to show that, even if individual institution’s ethics committee early on is certainly
choice is given absolute value (a dubious position), warranted in this situation.
paternalism in some forms can still be justified.8 ● Case 7: A patient requests an elective surgical procedure. The
Furthermore, Opinion 2.19 of the AMA Code states: surgeon feels that performing this procedure would conflict
“Physicians should not provide, prescribe, or seek with his or her personal beliefs. What are the surgeon’s
compensation for medical services that they know are ethical obligations to the patient in this case?
unnecessary.”9 – Principle VI of the AMA Code states, “A physician shall, in
– The stronger moral imperatives in this case include the provision of patient care, except in emergencies, be free
exposing the patient to unnecessary risk of harm to choose whom to serve, with whom to associate, and the
(nonmaleficence) and pursuing the patient’s well-being environment in which to provide medical care.”12
(beneficence), which may require more inquiry as to the – Most states have legal provisions (so-called “conscience
reasons behind the request. clauses”) allowing physicians to refuse provision of certain
– This patient can be referred to another psychiatrist to types of care based on moral/religious grounds. The
obtain a second opinion about capacity. Department of Health and Human Services (HHS) has in
– Risk management can also be involved prior to any surgical recent years put forth regulations that do not require
intervention. physicians to provide referrals to providers who will
● Case 6: A patient is admitted to the hospital in fulminant provide the care the patient is seeking.
septic shock due to an infected limb. The patient is not – Legal requirements aside, the surgeon in this case does
conscious but has on multiple occasions expressed that he or have ethical obligations to the patient which include
she would rather die than live without a limb. The patient’s beneficence and justice. Furthermore, professionalism
family is present and requests amputation of the limb. The requires courtesy that extends beyond one’s personal
multidisciplinary care team agrees that, medically, beliefs. In most cases, it is in the best interest of the patients
amputation of the infected limb would be an important step that they be referred to an accommodating provider or that
to controlling the infection. What are the ethical they be counseled regarding the options that are available
considerations? to them in this situation. As one bioethicist put it,
– If faced with this situation, the hospital’s Risk Management, “Accepting a collective obligation does not mean that all
Ethical Committee, and Psychiatry teams need to be members of the profession are forced to violate their own
involved immediately. This is not a decision that should be consciences. It does, however, necessitate ensuring that a
made solely on the plastic surgeon’s assessment and plan. genuine system for counseling and referring patients is in
– The distinction between withholding life-sustaining place, so that every patient can act according to his or her
treatment and physician-assisted death/suicide is one own conscience just as readily as the professional can.”13
around which there is now a significant consensus in law ● Case 8: What are some of the ethical considerations of a
and ethics. The consensus is that it is not a moral violation plastic surgeon who is active on social media?
to withdraw or withhold medical treatment that has been – The ASPS Code of Ethics explicitly prohibits any form of
refused in a valid manner.10 This consensus also accepts a public communication that contains “a false, fraudulent,
distinction between withholding/withdrawing life-saving deceptive, or misleading statement or claim.”3
care and physician-assisted suicide. According to the – This includes images that have been altered—post-
Opinion 2.20 of the AMA Code, “There is no ethical production or by use of photographic techniques such as
distinction between withdrawing and withholding life- lighting and perspective—so as to misrepresent a condition
sustaining treatment.”11 or improvement. The Code condemns any public
– The debate was initially spurred by the case of Karen Ann communication that “is intended or is likely to attract
Quinlan in 1976, a young woman who remained in a patients by use of exaggerated claims.”3 Surgeons should
persistent vegetative state and whose parents argued, also disclose if the results shown are not typical.
against the wishes of her husband, for the removal of – Informed consent should be acquired, and part of this
life-sustaining treatment (i.e., ventilator). The concept of requires that the patient understands that images or videos
“extraordinary means,” which has roots in Catholic moral posted online are potentially irrevocable, even if deleted by
theology, played a big role in this case and essentially the person who uploaded them.
elaborates that patients and families are under no ethical – Social media posts that do not honor the profession of the
burden to pursue unduly burdensome treatment in order to surgeon and which, most importantly, do not honor the
prolong life. The legal consensus on this topic was formed patient-physician relationship, are considered
around several decisions by the U.S. Supreme Court, “medutainment” and should be avoided.14 Some authors
especially after the Cruzan Case of 1990. have shown that public perception of the importance of a

269
Ethics

plastic surgeon’s role in patient care is extremely low, and [4] Choctaw WT. Avoiding Medical Malpractice: A Physician’s Guide to the Law.
Verlag, New York: Springer; 2008:5–8
that “medutainment” further undermines the professional
[5] Appelbaum PS. Clinical practice. Assessment of patients’ competence to
reputation of plastic surgery.14 Plastic surgeons must be consent to treatment. N Engl J Med. 2007; 357(18):1834–1840
especially cognizant of “sexualization” of the body, which is [6] Boylan M. Medical Ethics. Prentice Hall; 2000:64–65, 128–130
much more likely to occur in the context of social media as [7] Council on Ethical and Judicial Affairs. Code of Medical Ethics of the American
compared to journal articles. Medical Association. 2014–15 ed. American Medical Association. Opinion
9.065, pp. 359–361
– Any public communication that “appeals primarily to
[8] White J. Dworkin on paternalism. In: Contemporary Moral Problems. 9th ed.
layperson’s fears, anxieties, or emotional vulnerabilities” is Wadsworth Publishing Company; 2005:402
also condemned by the ASPS Code.3 [9] Council on Ethical and Judicial Affairs. Code of Medical Ethics of the American
– Naturally, interacting with social media requires Medical Association. 2014–15 ed. American Medical Association. Opinion
2.19, p. 92
corresponding compliance with the Health Insurance
[10] Beauchamp T, Zalta EN. The principle of beneficence in applied ethics. In: The
Portability and Accountability Act (HIPAA). It also requires
Stanford Encyclopedia of Philosophy. Spring 2019 ed. Metaphysics Research
familiarity with institutional policies, which may vary. Lab, Stanford University; 2016 https://plato.stanford.edu/archives/spr2019/
entries/principle-beneficence
[11] Council on Ethical and Judicial Affairs. Code of Medical Ethics of the American
References Medical Association. 2014–15 ed. American Medical Association. Opinion
2.20, p. 94
[1] What Is Maintenance of Certification? ABPlasticsurgery.org. http://www. [12] Council on Ethical and Judicial Affairs. Code of Medical Ethics of the American
abplasticsurgery.org/public/what-is-maintenance-of-certification/ Medical Association. 2014–15 ed. American Medical Association. Principle IV,
[2] Council on Ethical and Judicial Affairs. Code of Medical Ethics of the American p. xv
Medical Association. 2014–15 ed. American Medical Association. Opinion [13] Charo RA. The celestial fire of conscience—refusing to deliver medical care. N
9.031, pp. 345–347 Engl J Med. 2005; 352(24):2471–2473
[3] Code of Ethics of the American Society of Plastic Surgeons (ASPS). Updated [14] Bennett KG, Vercler CJ. When is posting about patients on social media
2017. https://www.plasticsurgery.org/documents/governance/asps-code-of- unethical “medutainment”? AMA J Ethics. 2018; 20(4):328–335
ethics.pdf

270
Index
Note: Page numbers set bold or italic indicate headings or figures, respectively.

A Bell's reflex 56 Chest X-ray (CXR) 188 Cruciate-type four-strand core suture
Belt lipectomy 177 Chestwall Reconstruction 191 repair 229
Abbe flap 33 Bick procedure 108 Chondrocutaneous transposition Crystalloid resuscitation 210
Abdominal wall defect 183 Bigonial compression 11 flaps 44 Cultured epidermal autograft 216
Abdominal/epigastric flaps 233 Bilateral bicortical osteotomies 74 Chopart's amputation 137 Cytal/Micromatrix 211
Abdominoplasty 199 Bilateral Cleft Lip Deformity 65, 66 Chronic Injuries with Nerve Dysfunc-
Acquired Facial Paralysis 51 Bilateral cleft lip repair advocated by tion 241
Acrylic Dental Splints (Gunning Mulliken 67 Chronic obstructive pulmonarydisease D
Splints) 22 Bilateral sagittal split osteotomies dis- (COPD) 206 Dakin's solution 173
Acute Burn Injury 205 traction osteogenesis 222 Chronic osteomyelitis 125, 137 Davis flap 44
Acute Burn Management 214 Bilateral subcondylar fractures 11, 22– Cicatricial ectropion of left lower eye- Deep vein thrombosis (DVT) prophy-
Acute Closed Injurieswith Nerve Dys- 23 lid 104 laxis 176, 180
function 240 Bilobed flap 37 Circum-orbital wires 23 Degloving Injury 231
Acute Management 132 Bluntly tunnel 56 Circum-piriform wires 23 Delayed reconstruction 142
Acute Open Injurieswith Nerve Dys- Body Contouring after Massive Weight Circum-zygomatic wires 23 Delayed surgicalreconstruction 53
function 240 Loss 175 Circumareolar 162 Dermagraft 211
Acute repair nerve injury 53 Body dysmorphic disorder (BDD) 178 Circumferential burns 206 Dermal substitutes 232
Adult Brachial Plexus Injury 259 Body fractures 11 Circummandibular wires 23 Dermatochalasis 100
Advanced Burn Life Support (ABLS) Body mass index (BMI) 184 Cleft Lip Adhesion appliance 66 Dermofasciectomy 244
214 Bone biopsy 172 Cleft lip and palate deformity 62 Design flaps 219
Advanced Trauma Life Support (ATLS) Bony Reconstruction 125, 129 Cleft Lip Repair 66 Design similar to cervicofacial flap 49
protocol 4, 18, 210, 232, 236 Bony Reduction andstabilization 132 Cleft nasal deformity 62 Diplopia 7
Advancement flap 64 Bottom surgery 115 Cleft nasal/Septal reconstruction 62 Displaced anterior table fracture 14
Advancement flaps 232 Botulinum toxin 52 Cleft Palate 69 Displaced Fractures 6, 10, 19
Aging Face and Neck 95 Botulinumtoxin 93 Cleft palate repair 62 Dissected deep to platysma 49
Aging Upper 99 Brachial plexus anatomy 260 Cleft Palate repairtechnique 70 Dissected deep to platysma 49
Airway obstruction 71 Brachioplasty incision. 177 Closed approach 110 Distal iliotibial tract 180
Albinism 28 Brain/epidural abscess 15 Closed nasal reduction 4 Distal posterior thigh 180
Alloderm 211 Breast Augmentation necrosis 145 Combined anterior/posterior table frac- Distant flaps 233
Alveolar bone grafting 62 Breast cancer diagnosis 142 tures 14 Distraction osteogenesis 125
American Cancer Society guidelines for Breast Cancer Reconstruction 141 Combined augmentation/Mastopexy Donor 53
clinical breast exam 146 Breast examination—descriptive find- 158 Dorsal blocking splint 229
Amputation and glenohumeral ings 151 Commissure defects 33 Dorsal hand soft tissue defect 232
arthrodesis 263 Breast Implant Associated Anaplastic Compartment Syndrome 128 Dorsal longitudinal incision 252
And Paralytic 107 Large celllymphoma (BIA-ALCL) 146 Compartment syndrome 206 Double-opposing Z-plasty Furlow 70
And silicone implants 158 Breast pedicle designs ptosis 162 Compartment syndrome assessment Dry eyes 100
Angle fractures 11 Breast Reduction 161 210 Dupuytren’s Contracture 243
Ankle-Arm Index (AAI) 124, 128, 136 Bridle wire 10 Complete amputation/avulsion 44 Durkan’s (carpal compression) test
Anotia 82 British Medical Research Council Motor Complete blood count (CBC) 180 256
Anterior lamella 40 Grading Scale 260 Complete left-sided facial paralysis 52
Anterior open bite 18 Brow lif 53 Complete trauma evaluation 22
Anterior scalp defects 219 Brow Lift 101 Components separation technique 185 E
Anterior table fractures 14 Burn eschar excision 215 Composite graft nasal septal cartilage–
Anterolateral thigh (ALT) flap 233 Ear Reconstruction 43
Burow's triangle 49 Mucosa 41
Antia-Buch flap 44, 46 Early definitive reconstruction 125,
Buttresses of the face. 7 Computed tomography (CT) myelo-
Apligraf 211 136
gram 261
Apraclonidine drops 93 Ectropion 42, 49
Computed tomography (CT) scan 188
Assess for midface instability 18 C Computed tomography angiography
Ectropion/lower eyelid laxity 100
Assess mobility 10 Edentulous mandibles 11
C Flap used 64 (CTA) 124, 132, 136
Assisted liposuction (PAL) 180 Elbow extension 262
Cadaver allograft 211 Computed tomography angiography
Audiologist 82 Elbow flexion 262
Cancer Screening 146 CTA 128
Augmentation-Mastopexy 159 Electrical Injuries 209
Capsular contracture 115 Concomitant injuries 10
Autologous Reconstruction 143 Electrodiagnostic studies 256
Carpal Tunnel Syndrome 255 Condylar fractures 23
Axial pattern flaps 232 Endocrinologist 114
Carpal tunnel syndrome (CTS) 256 Congenital Facial Paralysis 55
Endocrinologist primary physician
Cartilage graft 4 Continue fluid resuscitation 210
118
Converse flap 44
B Cartilage grafting 110
Corneal abrasion 42
Enophthalmos 7
Cephalic or midline neck 88 Enophthalmos, hypoglobus zygomati-
Baker capsular contracture Cervical spine evaluation 22 Corneal epithelial disease 104
cofrontal (ZF) suture 6
classification 147 Cervical spine precautions 10 Corneal protection 52
Entropion 42
Banner flap 44 Cervicofacial advancement flap 49 Correction of prominent ears using
Enzymatic fasciotomy 244
Bardach two-flap palatoplast 72 Cervicofacial flap 49, 49 Mustarde Suture 79
Erich Arch Bars Maxillomandibular Fix-
Basal cell carcinoma 32 Cervicopectoral flap 49–50 Cosmetic/elective 78
ation 22
Bell's palsy 52 Champy technique noncomminuted Cottle maneuver 110
Escharotomies 207
Bell's phenomenon (palpebral oculogy- fractures 11 Cross-facial nerve graft 53
Escharotomies of the hand 214
ric reflex 108 Cheek Reconstruction 47 Cross-facial nerve grafting 53
Escharotomy 214
Bell's phenomenon palpebral oculogy- Chemical lipolysis (deoxycholic acid) Cross-facialsural nerve graft 56
Estlander flap 33, 34
ric reflex 104 93 Cross-finger flaps 233
Expansion Technique 219

271
Index

Exposed bone 173 Intra-oral approach (Keen) 6 Magnetic resonance imaging (MRI)
Extent of burn 206
G Intraoperative management 176 188
External Auditory Atresia Reconstruc- Galactorrhea 115 Intraoral (vestibular) approach 11 Major Liposuction 179
tion 83 Gastrocnemius flap 129 Intraoral approach 23 Malignant Skin Lesion 27
External auditory canal 44 Gastrocnemius muscle flap 125 Intraoral approach 11 Malocclusion 11, 19
External oblique 185 Gender Transition 113 Intraoral lacerations 10 Malpractice law 268
External Risdon (submandibular) Gender Transition (Female-to-Male) Intravelar veloplasty 70 Mandible series 10
approach 11 117 Ischial Pressure Sores 171 Mandibular Fractures 9
Extracellular matrix components 232 Giant Hairy Nevus 87 Ischium 173 Mandibular hypoplasia 74
Extraoral (submental) approach 11 Glogauphotoaging classification 92– Isolated fracture 14 Mannitol, acetazolamide 7
Eyebrow analysis 100 93 Isolated Zygomatic Arch Fracture 6 Margin reflex distance MRD1 100
Eyelid distraction test 108 Gluteal crease 180 Margin reflex distance MRD2 101
Eyelid ptosis 100 Goals of Care Assessment 188 Masquelet technique 125, 132
Eyelid Reconstruction 39 Gold standard reconstruction 184 J Mastectomy skin flap necrosis 143
Gorlin's (nevoid basal cell) syndrome Mastopexy 159
Joint (MCP) 228
28 Mastopexy addresses ptosis 158
F Gracilismyocutaneous flap 196 Mastopexy/Augmentation 157
Facelift 96
Grade II breast ptosis 118 K Maxillofacial computed tomography 4
Facelift procedures 53 Grades of ear hypoplasia 82 Maxillomandibular fixation (MMF) 10,
Kaplan’s cardinal line 256
Graft coverage 222 19
Facial artery musculomucosal (FAMM) Karapandzic flap 33, 33
flap 32 Groin flap 233 Medial canthal laxity test 108
Keyhole mastectomy 118
Guillain–Barre 52 Medial spindle procedure 108
Facial fractures 6
Gunshot wounds 10 Median nerve 240
Fasciocutaneous flaps 137
Fasciotomies 132, 207
Gustilo classification of open tibial frac-
L Melanoma 32
tures 124 Mental health 114
Fasciotomies of the forearm 215
Gustilo grade IIIB 132 Large ventral abdominal hernia 184 Mental health professional 118
Fasciotomy 211, 215
Gustilo IIIB 124, 128 Laser-assisted liposuction (LAL) 180 Meshed grafts 211, 215
Final excision margins 29
Gynecomastia 165 Lateral canthal laxity test 108 Micrograft 220
First web space contracture release
Gynecomastia development 166 Lateral canthopexy 108 Microtia 81
216
Lateral gluteal depression 180 Microvascular reconstruction 49
Fisher anatomic subunit technique 63
Lateral orbital rim (zygomaticofrontal) Mid-facial and periorbital edema 6
Fitzpatrick classification 92
Fitzpatrick I skin type 100
H 6 Middle lamella 40
Lateral orbital wal Middle medial thigh 180
Fitzpatrick skin type II Glogau class III Hair transplantation 220
(zygomaticosphenoid) 6 Midface fractures 19
92 Hamstring musculocutaneous V-Yad-
Lateral tarsal strip 108 Mild instability or displacement 10
Flail segment 193 vancement Flap. 173
Lateral tarsal strip procedure 104 Millard rotation-advancement 64
Flap 196 Hard palate repair 70
Latham appliance 62, 66 Millard rotation-advancement flap 63
Flap Coverage 218 Harvest gracilis from 56
Latissimus dorsi 185, 189 Millard rotation-advancement repair
Flap coverageoptions 188 Health [WPATH] 114
Latissimus dorsi muscle flap 233 63
Flap reconstruction options 216 Helical rim advancement 44, 46
Latissimusflap 192 Mini abdominoplasty 201
Fleur-de-lis abdominoplasty 201 Hematoma 49, 97, 189
Le Fort fracture patterns 18 Minigraft 220
Fleur-de-lis component 177 Hematoma/seroma 193
Le Fort Fractures 17 Moderate hypoplasia 82
Flexor digitorum superficialis (FDS) Hemi-coronal incision 57
Le Fort I 19 Modified Kessler two-strand core
228 Hemifacialmicrosomi 82
Le Fort I transverse fracture of the max- suture repair 229
Flexor digitorumprofundus [FDP] 228 Herpes simplex virus activation zoster
illa 18 Mohs micrographic surgery 28–29
Flexor Tendon Laceration 227 52
Le Fort II 19 Mohs surgery referral 32
Flexor Tendon Reconstruction/Secon- High lateral tension abdominoplasty
Le Fort II pyramidal fracture 18 Mucocele/mucopyocele 15
dary Repair 229 200
Le Fort III 19 Mucosal advancement 32
Flexor tendonrepair Technique 228 High resolution maxillofacial CT scan
Le Fort III craniofacial disjunction 18 Multiple types of core suture repairs
Foot and Ankle Reconstruction 135 6
Ledderhose disease (plantar fibromato- 229
Forehead analysis 100 High-resolution maxillofacial com-
sis) 244 Myofascial defects 184
Forehead flap 37 puted tomography (CT) 10
Levator excursion 100
Four-stage Brent technique 83 High-resolution maxillofacial CT 22
Lip Reconstruction 31
Four-strand cruciate repair 229 Hunter rod (silicone implant) 229
Fracture stabilization 19 Hybrid MMF 10
Lip switch 33 N
Lip taping 62
Fractures 251 Hypernasal speech 69 Nager syndrome 82
Liposuction modality 180
Free flaps 185 Hypertrophic scarring 216 Nasal deformity 4
Local fasciectomy 244
Free muscle transfer 262 Hypomastia 146 Nasal Fractures 3
Local flaps 137
Free tissue transfer 125, 133, 143, 220, Nasal intubation 19
Local flaps 37
222 Nasal lining options 37
Fresh frozen pathologic evaluation 36 I Local muscle flaps 129
Nasal or septal surgery 4
Local pedicled flaps 222
Frontal Sinus Fractures 13 Implant-Based Reconstruction 83 Nasal reconstruction 67
Lower Blepharoplasty 101
Frontal sinus fractures, algorithm for Incision patterns 162 Naso-orbito-ethmoid (NOE) fractures
Lower Eyelid 101
management of 15 Incomplete soft palate cleft (Veau I) 71 8
Lower eyelid analysis 100
Frontal sinus obliteration 14 Inferior orbital rim 6 Nasoalveolar molding (NAM) 62
Lower Eyelid Ectropion Cicatricial 103
Full-thickness defects 37 Infra-areolar approach 154 Nasofrontal angle 110
Lower eyelid incision 7
Full-thickness grafting 222 Infraorbital rim 7 Nasolabial flap 37
Lower lateral cartilages 110
Full-thickness skin graft 33, 211, 215 Inhalation injury 206 Nasopharyngealairway 74
Lower lid ectropion 8
Full-thickness skin grafts (ftsgs) 207 Integra (bilayered dermal substitute) Near-complete scalp defects 220
Lower lip full-thickness defects 33
Functioning gracilis free 56 137 Neck Burn Contracture 221
Furlow double-opposing Zplast 71 Integra bilayered dermal substitute Neck Rejuvenation 97
125 M Nerve damage 97
Internal fixation 252 Nerve injury, classification of 241
M Flap 64 Nerve transfer 262

272
Index

Neuromuscular retraining 52 Permanent skin replacement 211 Secondary Burn Management 216 Tensor fascia lata 185
Nipple-areola complexes (nacs) 150 Pertinent Imaging 14 Secondary reconstruction 214 Testosterone 118
Nipple-areolar complexes nacs 114 Peyronie’s disease (penile fibromato- Sentinel lymph node biopsy 29, 36 Thigh flap 197
Nonautogenous materials 211 sis) 244 Septal deformity 62 Tighten 106
Nondisplaced anterior table fracture Phalen maneuver 256 Septal hematoma 4 Timing of Immobilization 23
14 Philtral flap 66 Septal resection 110 Tinel sign 256
Nondisplaced Fractures 6, 10, 19 Philtral preservation 66 Septoplasty 4 Tinel’s sign 240
Noninvasive versus invasive 142 Pierre Robin Sequence 73 Serratus anterior flap 192 Tissue expansion 49
Nonsalvageable extremity 125 Place sutures 56 Serratus anterior muscle flap 233 Tongue flap 32
Nonsurgical Rejuvenation of the Face Planned lumpectomy mastectomy 142 Shave biopsyis 28 Tongue-lip adhesion 76
91 Pollybeak deformity 111 Sheet grafts 211, 215 Tongue–Lip Adhesion 74
Nonsurgical Treatment 28 Positive lymph nodes 36 Short scar periareolar inferior pedicle Total ear reconstruction 44
Nonvascularized bone graft 125 Posterior auricular flap 44 reduction (SPAIR) 162 Total lip reconstruction 33
Nose Reconstruction 35 Posterior interosseous artery flap 233 Short scar technique 96 Tracompartmental pressures 214
Posterior lamella 40 Shoulder abduction 262 Transfacial approaches comminuted
Preferred technique for scalp recon- Shoulder stabilization 262 fractures gunshot wounds 10
O struction 218 Simon's classification 166 Transmetatarsal amputation (TMA)
Obesity classification 176 Pressure sore staging Sores 172 Single-digit amputation distal flexor 137
Obesity classification 176 Primary cleft nasal reconstruction 63 digitorum superficialis (FDS) 236 Transposition flaps 232
Occipital scalp defects 220 Primary closure 36 Skeletal defect reconstruction 192 Traumatic Amputation 235
Ocular Complications 104, 108 Primary closure+/– Tanzer's excision Skin laxity 96 Treachercollins syndrome 82
Ocular examination 100 44 Skin necrosis 97 Treatment of joint contractures 244
Omentum 193 Prominent Ear Deformity 77 SMAS facelift 96 Trismus 6
Oncoplastic surgery 142 Prominent lobule 78 Snap back test 104, 108 Tubed pedicle flap 44
One-Stage Facial Reanimation 57 Prophylaxis 177 Soft palate repair 70 Tuberous Breast Deformity 153
One-Stage Temporalis Transfer 57 Proximal tendon(s) 228 Soft tissue coverage 192 Two-flap palatoplasty (Bardach 71
Open approach 110 Pseudogynecomastia 166 Soft Tissue Reconstruction 136 Two-Stage Facial Reanimation (Cross-
Open Reduction and Internal Fixation Ptosis classification 100 Soft Tissue Reconstruction 128, 132 Facial Nerve Graft withgracilis Free
(ORIF) 23 Ptosis Repair 101 Soleus muscle flap 129 Flap) 56
Open Reduction/Internal Fixation Pudendal thigh flap. 196 Split-thickness graft 88 Two-stage Nagata technique 83
(ORIF) 10, 14 Pulley reconstruction 229 Split-thickness grafting 222
Openwound Lower Third of lowerleg Pulmonary disease 192 Split-thickness skin graft 215
131 Split-thickness skin graft (STSG) 220 U
Openwound Middle Third oflower Leg Spreader grafts 110 Ulnar nerve innervates 240
127
R Squamous cell carcinoma 32 Ultrasound-assisted liposuction (UAL)
Openwound upper third of Leg 123 Radial forearm flap 233 Stable Fractures 19 180
Ophthalmology consultation 4 Ramus fractures 11 Standard abdominoplasty 200 Unilateral Cleft Lip Deformity 61
Ophthalmology consultation in 6 Reconstruction byregion 219 Standard anterior approach 7 Unilateral or bilateral cleft lip and pal-
Options for bone gap reconstructions Reconstruction of first webspace con- Standard inverted-T incision Wise pat- ate (Veau III and IV) 71
129 tracture 216 tern 162 Unstable Fractures 19
Orbicularis reconstruction 66 Reconstructive options 192, 196 Standard superficial musculoaponeur- Upper blepharoplasty 7, 101
Orticochea 44 Reconstructive Surgery 267 otic system (SMAS) 97 Upper Extremity Burn 213
Orticochea three-flap technique 218 Recording of breast measurements State of dentition 18, 22 Upper Eyelid 101
Osteotomies 111 152 Sternalwound Infection 187 Upper eyelid analysis 100
Otolaryngologist 82 Rectus abdominis flap 193 Subciliary incision 8 Upper eyelid ptosis 93
Oxymetazoline lidocaine injection 4 Rectus abdominus 185 Subcondylar fractures 11 Upper lip full-thickness defects 33
Rectus abdominus muscle 189 Subcutaneous facelift 97
Rectus femoris 185 Subcutaneous versus subplatysmal fat
P Regional flaps 37 96 V
Regional muscle transfers 53 Subglandular 146
Palatal fistula 71 Vascular occlusion/Embolism 93
Regnault classification of breast ptosis Submuscular versus prepectoral cover-
Palpate bony structures 10 Vascularized bone graft 125
154, 162 age 143
Panorex 10, 22 Vasomotor paralysis 240
Resuscitation 207 Subunit principle 37
Parasymphyseal fractures 23 Veau cleft classification system 70
Retrobulbar hematoma 7, 101, 108 Superficial circumflex iliac artery flap
Parietal scalp defects 219 Velopharyngeal insufficiency (hyper-
Retrognathia mental retrusion 222 233
Parkland formula 207 nasal Speech) 71
Retromandibular incision submandibu- Syndactyly 247
Partial flap loss 49 Vermilion advancement 32
lar (Risdon) 11 Synkinesis 52
Partial Mastectomy/Lumpectomy Vermilion lip switch 32
Reverse abdominoplasty 201 Systematic Facial Analysis 92
Reconstruction 142 Vertex scalp defects 220
Reverse radial forearm flap 233 Systemic complications 201
Pectoralis major flap 192 Vertical and circumvertical incisions
Pediatric Mandible Fractures 21 Rhinoplasty 4, 109 162
Ring test 14
Pedicled anterolateral thigh (ALT) flap
Risdon approach comminuted Frac-
T Visual acuity 100
197 Vomer flap 71
Pedicled flaps 137, 143, 185 tures gunshot wounds 11 Tabletop test 244 Von Langenbeck palatoplasty 70
Pedicled muscle flap 125 Robin sequence airway management Tarsoconjunctiv (Hughes) flap 41
Perforator/Propeller flaps 133 75 Temporal (Gillies) approach 6, 6
Peri-areolar mastopexy 118 Rotation flap 64 Temporalis advancement 57 W
Periareolar approach 154 Rule out Compartment Syndrome 124 Temporalis muscle turnover Proce-
Wedge resection 45
Pericranial flap 14 dure 57
Wetting solution technique 180
Perineal Reconstruction 195 Temporary skin replacement 211
Periorbital region 92
S Temporoparietal fascia flap 85, 233
Wetting solutions for liposuction 180
White roll and vermilion 67
Peripheral nerve decompression 211 Scalp Burn Reconstruction 217 Tendon repair 230
Wiring techniques 23
Peripheral Nerve Injury 239 Scapular and parascapular flaps 233 Tendon Transfers 241, 263
Wound bed status 232
Tension band 11

273
Index

Wrinkle severity rating scale (WSRS) Zones of the eyelid 40 Zygomaticomaxillary (ZM) nasomaxil-
92
Z Zones of the mandible 11 lary (NM) 18
Zona pellucid 70 Zygomatic arch 6 Zygomaticomaxillary buttress 6–7
Zone I (Upper Eyelid) 40 Zygomatic Fractures 5 Zygomaticomaxillary complex (ZMC)
X Zone II (Lower Eyelid) 41 Zygomatic reduction 7 fracture 6
Zone III (medialcanthus) 41 Zygomaticofrontal (ZF) suture 18
Xerodermapigmentosum 28
Zone IV (Lateral Canthus) 41 Zygomaticofrontal region lateral orbital
Zone V (Periorbital) or multiplezone rim 7
Defect 42

274

You might also like